Applied Mathematics Paper 2

Download as pdf or txt
Download as pdf or txt
You are on page 1of 475

Vectors

A vector is a quantity which has both magnitude and direction. Examples include, force,
displacement, acceleration, momentum and velocity.

Representation of a vector

A vector is represented by a line with an arrow to indicate the direction of the vector.
A
OA
O
where the order of the letters shows the direction

Vectors in dimensions

Vectors can be represented in three dimensions as I, j and k along the x, y and z- axes respectively

Resultant of vectors

When several vectors (V1, V2, V3… Vn) are acting on a point object, the net vector R, is calculated as
the vector sum

R = V1 + V2 + V3 + … + Vn = ∑𝑟=𝑁
𝑟=1 𝑉𝑟

Example 1

Find the resultant of the following vectors

(a) (2i + 3j + 3k) and (2i + 4j – 8k)


2 2 4
𝑅 = (3) + ( 4 ) = ( 7 ) 𝑜𝑟 4𝑖 + 7𝑗 − 5𝑘
3 −8 −5
(b) (7i – 4j + 3k), (5i -2j + 8k), (i –k)
7 5 1 13
𝑅 = (−4) + (−2) + ( 0 ) = (−6) 𝑜𝑟 13𝑖 − 6𝑗 + 10𝑘
3 8 −1 10
Example 2

The resultant of (5i -2j), (7i + 4j), (ai + bj) and (-3i + 2j) is (5i + 5j). Find the values of a and b.
5 7 𝑎 −3 5
𝑅= ( )+( )+( )+( )=( )
−2 4 𝑏 2 5
9+𝑎 5
=( )= ( )
4+𝑏 5
9 + a = 5; a = -4 also 4 + b = 5; b = 1

Example 3

The resultant of the forces (3i + (a-c)j)N, ((2a + 3c)I + 5j)N and (4i, 6j)N acting on a particle is
(10i + 12j)N. find

(i) Values of a and c


3 2𝑎 + 3𝑐 4 10 (i) + 3(ii)
𝑅=( )+ ( )+( )= ( )
𝑎−𝑐 5 6 12
2a + 3c + 7 = 10 5a = 6; a = 1.2
2a + 3c = 3 ……….. (i) from eqn. (ii)
a –c +11 = 12
a – c = 1 ………….. (ii) c = 0.2

(ii) magnitude of (2a + 3c)I + 5j

R = (2a + 3c)I + 5j = (1.2 x 2 + 3 x 0.2)i + 5j = 3i + 5j


|𝑅| = √32 + 52 = 5.831𝑁

Magnitude or modulus of a vector


This is the length of a vector

(i) Given R = xi + yj; |𝑅| = √𝑥 2 + 𝑦 2


(ii) Given R = xi + yj + zk; |𝑅| = √𝑥 2 + 𝑦 2 + 𝑧 2

Example 5

Find the magnitude of the following vectors

(a) 3i + 4j; |𝑅| = √32 + 42 =5


(b) 3i + 2j – 6k; |𝑅| = √32 + 22 (−6)2 = 7

Direction of a vector
Consider R = xi + yj

𝑥
𝜃 = tan−1 ( )
𝑦

Example 6

Find the magnitude and direction of the resultant of each of the following
(a) (2i + 3j)N, (5t – 2j)N, (-3i, 3j)
2 5 −3 4
𝑅 = ( )+ ( )+ ( )= ( ) 𝜃 = tan−1 ( )= 450
4
3 −2 3 4 4
|𝑅| = √42 + 42 = 5.6569N
2 −6 2
(b) ( ) 𝑁, ( ) 𝑁𝑎𝑛𝑑 ( ) 𝑁
4 −5 1
2 −6 2 −2
𝑅 = ( ) +, ( ) + ( ) = ( ) 𝜃 = tan−1 ( )= 1800
0
4 −5 1 0 −2
2
|𝑅| = √(−2) + 0 = 2N 2

Example 7

Four forces of ai + (a-1)j, 3i + 2aj, 5i -6j, and –i -2j act on a particle. The resultant forces make an
angle of 450 with horizontal. Find a. Hence determine the magnitude of the resultant force.
𝑎 3 5 −1
𝑅= ( )+( )+ ( )+ ( ) 𝑎+7
𝑎−1 2𝑎 −6 −2 = tan−1 (450 ) = 1
3𝑎−9
𝑎+7
=( ) a + 7 = 3a – 9, a = 8
3𝑎 − 9
𝑎+7 8+7 15
𝑅=( )= ( )=( )
3𝑎 − 9 3𝑥8−9 15
|𝑅| = √152 + 152 = 21.21N

Unit vector
This a vector whose magnitude is unit (1)
𝑟
Unit vector of r denoted by r = |𝑟|

Example 8

Find the unit vector of a = 6i -2j + 3k

Solution
6i −2j + 3k 1
a= = (6i − 2j + 3k)
√62 + (−2)2 +32 7

Parallel vectors

If vectors a and b are parallel, then one of them is a scalar multiple of the other.
xi + yj + zk
If a vector r of magnitude |𝑟| moves in direction xi + yj + zk then, r =|𝑟| ( )
√𝑥 2 +𝑦 2 +𝑧 2

Example 9

Find the vector, V which has a magnitude of 15 units and is parallel to 16i + 12j
16i + 12j 16i + 12j
V = 15 x = 15 x = 12𝑖 + 9𝑗
√162 +122 20

Example 10

A body of velocity v and of magnitude 20m/s moves in the direction 6i + 8j. Find V.
6i + 8j 6i + 8j
V = 20 x = 20 x = 12𝑖 + 16𝑗
√62 +82 10

Example 11

A force of magnitude 12N acts on a body in the direction 2i + j + 2k. Find the force
2i + j + 2k 2i + j + 2k
V = 12 x = 12𝑥 = 8𝑖 + 4𝑗 + 8𝑘
√22 +12 + 22 3

Example 12
1
The force A of magnitude 5N in the direction with unit vector (3𝑖 + 4𝑗) and force B of magnitude
5
1
13N in the direction with unit vector (5𝑖 − 12𝑗). Find the resultant forces of A and B.
13
1 3 8
FA = (3𝑖 + 4𝑗)𝑥5 = 3𝑖 + 4𝑗 5
5 F = ( )+ ( )=( )
4 −12 −8
1
FB = (5𝑖 − 12𝑗) 𝑥 13 = 5𝑖 − 12𝑗 |𝐹| = √82 + (−8)2 = 11.3137N
13

Example 13

A particle P moves through a displacement of 2m when acted o by two forces F1 and F2. Find the
work done by the resultant force, if F1 = i – j and F2 = 10 N and acts in the direction 4i + 3j

Solution
1 8 9
F =( )+( )=( )
F1 = i – j −1 6 5

4i + 3j |𝐹| = √92 + 52 = 10.2956N


F2 = 10 𝑥 = 8i + 6j
√4 2 +3 2
W = |𝐹| 𝑥 𝑑 = 10.2956 x 2 = 20.5912J

Revision exercise 1
1. Find the resultant of each of the following forces
(a) (6i + 2j)N, (-5i + j)N, (3i – 3j)N. [(4i)N]
(b) (2i + 4j)N, (3i -5j), (6i +2j)N, (-7i – 7j)N. [(4i -6j)N]
(c) (2i + 3j -7k)N, (2i + 5k)N, (3j + 4k)N. [(4i + 6j + 2k)N]
2. The resultant of forces (5i + 7j), (ai + bj) and (bi –aj)N is a force (11i + 5j)N. Find a and b.
[a = 4, b = 2]
3. Find the magnitude and direction of the resultant of each of the following;
(a) (-2i + 5j)N, (I + 2j)N. [ 7.07N at 98.10]
(b) (6i + 2j)N, (4i – 3j)N. [10.05N at 354.30]
(c) (3i + 2j), (-5i + j)N. [3.61N at 1240]
4. A force of magnitude 50N acts on a body in the direction 24i + 7j. Find the force. [(48i + 14j)]
5. Two forces F1 and F2 have magnitude αN and βN and act in the direction i -2j and 4i + 3j
respectively. Given that the resultant of F1 and F2 is (48i + 14j). Find the magnitude of αN and
βN. [α = 8√5N and β = 50N]
6. If a = 3i + 4j, b = 4i + 20j and c = 5i – 19j; find the
(i) resultant of a and b [(7i + 24j)]
(ii) resultant of a and c [(8i – 15j)]
(iii) vector is parallel to a and has magnitude of 15 unit [(9i + 12j)]
(iv) vector parallel to (a + b) and has a magnitude of 100 units [(28i + 96j]
7. If a = 2i + 5j, b = -7i + 7j and 14i. Find the;
(i) resultant of a and b [(-5i + 12j)]
(ii) resultant of a, b and c [(9i + 12j)]
(iii) |b| [7√2]
(iv) |a + b + c| [15units]
(v) vector is parallel to a and has a magnitude of 5√29 units. (10i + 25j)
(vi) Vector is parallel to (a + b + c) and has magnitude 90 units. [(54i + 72j)]
8. If a = i -3j + 2k, b = 5i + 4j and c = 3i + j + 4k. Find the
(i) resultant of a and b [(6i + j + 2k)]
(ii) resultant of a, b and c.[(9i + 2j + 6k)
(iii) |a| [√14]
(iv) |a + b + c|[11units]
5
(v) Vector parallel to (a + b + c) and has magnitude 5 units [ (9𝑖 + 2𝑗 + 6𝑘)]
11
9. If a = 2i + 7j + 7k, b = 6i -3j + 2k and c = -4j -3k. find the
(i) resultant a and b [8i +4jj + 9k]
(ii) resultant a and c [2i + 3j + 4k
(iii) |b| [7units]
(iv) |a + b + c| [10 units]
(v) vector is parallel to |a + b + c| and has magnitude of 50 units [40i + 30k]

Scalar products or dot products


The dot product of two vectors a and b inclined at angle θ is given by

a.b = |𝒂||𝒃| 𝒄𝒐𝒔𝜽

Note

If two vectors are perpendicular then the angle between them is 900 and

a.b = |𝒂||𝒃| 𝒄𝒐𝒔 𝟗𝟎 = 𝟎

Example 14

If a = i – 2k and b = 3i – 3j + k, find

(i) a.b (ii) the angle between a and b

Solution
1 3
(i) a.b = ( 0 ) . (−3)= 3 + 0 + -2 = 1
−2 1
𝑎.𝑏 1
(ii) 𝜃 = cos −1 (|𝑎||𝑏|) = cos −1 = 84.10
√12 + (2)2 √32 +(−3)2 +12

Example 15

If p = 2i – j + 3k and q = i + 4j + 3k; find the angle between p and q.

Solution
2 1
p.q = (−1) . (4) = 2 + -4 + 9 = 7
3 3
𝑝.𝑞 7
𝜃 = cos −1 (|𝑝||𝑞|) = cos −1 = 680
√22 + (−1)2 +32 √12 +4 2 +32

Example 16

If the angle between two vectors a = xi + 2j and b = 3i + j is 450.Find the two possible values of
constant x.

Solution
𝑥 3
( ) . ( ) = √𝑥 2 + 22 .√32 + 12 x2 + 3x – 4 = 0
2 1
√2 (x + 4)(x -1) = 0
3x + 2 = √𝑥 2 + 22 . √10.
2
x = -4 and x = 1
2
(3x + 2)2 = (x2 + 4). 10 x
4

Example 17

If p = 2αi + 7j – k and q = 3αi + αj + 3k.Find the value of the scalar α if the vectors are perpendicular

Solution
2𝛼 3𝛼
( 7 ).( 𝛼 ) = 0
−1 3
6α2 + 7α – 3 = 0
1 3
α = and α =
3 2

Revision exercise 2
1. Find the scalar products for each of the following pairs of vectors.
(i) a = 2i + j, b = i - 3j [-1] 0 −3
(ii) a = 3i, b = -2i + j [-6] (vii) ( 5 ) 𝑎𝑛𝑑 ( 2 ) [8]
(iii) a = 5i + j - 2k, b= 4i + 3j – 8k [39] −2 1
(iv) 2i + 4j – 15k and -8i + 2j – k [7] 5 1
2 3 (vi) ( 2 ) 𝑎𝑛𝑑 ( 2 ) [2]
(v) ( ) 𝑎𝑛𝑑 ( ) [4]
1 −2 7 −1
5 2
(vi) ( ) 𝑎𝑛𝑑 ( ) [6]
−1 4

2. Find the angles between each of the following pairs of vectors


(i) 3i + 4j and 5i – 12j [1210] 3 1
(ii) 3i and -2j [900] (v) ( ) 𝑎𝑛𝑑 ( ) [820]
1 −2
(iii) 2i + 3j – 6k and 2i + j + 2k [100]
6 5
(iv) i + 2j – k and –I + 2j – k [480] (vi) ( ) 𝑎𝑛𝑑 ( ) [920]
−8 4
0 −1 2 2
(v) (vii) ( 1 ) 𝑎𝑛𝑑 ( 0 ) [1200] (viii) (2) 𝑎𝑛𝑑 ( 1 ) [730]
−1 1 3 −1
3. If a = αi + 2j – k and b = 5i –αj + k. Find the value of the scalar α if the vectors are perpendicular
1
[ ]
3
4. If a = 2i + αj and b = -α – k. Find the value of the scalar α if the vectors are perpendicular [0]
5. If a = 4i + 5j and b = qi – 8j. Find the value of scalar q if the vectors are perpendicular. [10]
6. If a = 6i – j and b = 2i + pk. Find the value of scalar p if the vectors are perpendicular [12]
𝑞 −1
7. Given ( 2 + 𝑞 ) and ( 3 ) are perpendicular vectors. Find the value of q. [18]
3 4−𝑞
8. If a = qi + 8j +(3q + 1)k and b = (q+1)I + (q-1)j – 2k. Find the value of the possible values of
constant q if the vectors are perpendicular. [2 or -2]

Thank you
Dr. Bbosa Science
Vector mechanics
Crossing a river
There are two cases to consider when crossing a river

Case I: Shortest route

If the water s not still and boatman wishes to cross directly opposite to the standing point. In order
to cross from point A to point B directly opposite A (perpendicularly), then the course set by the
boat must be upstream of the river.

At point B: w = usinθ
; 𝑤
θ= ( )
𝑢

θ = is the direction to the vertical but the


direction to the ban is (90 – θ)0
𝐴𝐵
u = speed of the boat in still water Time taken = 𝑢𝑐𝑜𝑠𝜃

w= speed of running water

Example 1

A man who can swim at 6km/h in still water would like to swim between two directly opposite point
on the river banks of the river 300m flowing at 3km/h. Find the time taken to do this.

He must swim at 300 to AB in order to cross


directly and it takes him 3.46minutes.

Alternatively

Using Pythagoras theorem


AB = 0.3km
62 = 32 + VAB2
; ;
θ= ( ) (6) =300
VAB= 36 − 9 = 5.1962km/h

Time taken = =6 𝐴𝐵
Time = 𝑉 5 96
= 0.058hrs
𝐴𝐵

time = 0.058hrs = 3.46minute

digitalteachers.co.ug
Example 2
5 ;
Two points A and B are on opposite banks of a river flowing at 6 . A man who can swim at
5 ;
8
in still water would like to swim directly from A to B. Find the width of the river if he takes
2minutes to cross the river.
Alternatively: using Pythagoras theorem
5 5
( 8)2 = (6)2 + VAB2

VAB=1 1111𝑚
𝐴𝐵
Time taken =𝑉
5⁄ 𝐴𝐵
; ; 6
θ= ( ) ( 5⁄ ) =36.870
8 𝐴𝐵
2 x 60 =
Time taken = =
6 87 AB = 133.333m

AB = 133.333m

Case II: the shortest time taken/ as quickly as possible


If the boatman wishes to cross the river as quickly as possible, then he should steer his boat directly
from A to B as shown. The river pushes the boat down stream

Or
𝐴𝐵
Distance downstream = 𝑤 𝑢
𝑤 ; 𝑤
tanθ = 𝑢 or θ = ta 𝑢

The resultant velocity downstream VR


Time taken to cross the river, t =
𝑉𝑅 𝑤 +𝑢
Distance covered downstream = wt

Example 3

A man who can swim at 2ms-1 in still water wishes to swim across a river 120m wide as quickly as
possible. If the river flowsat0.5ms-1, find the time the man takes to cross far downstream he travels.

Solution
u = 2ms-1, w = 0.5ms-1, AB = 120m
𝐴𝐵
t= 𝑢
60𝑠

Distance = wt = 0.5 x 60 = 30m

digitalteachers.co.ug
Example 4

A boat can travel at 3.5ms-1 in still water. A river is 80m wide and the current flows at 2ms-1,
calculate

(a) the shortest time to cross the river and the distance downstream the oat is carried.

u = 3.5ms-1, w = 2ms-1, AB = 80m


𝐴𝐵 8
t= 𝑢 5
22 95𝑠

Distance, BC = wt = 2 x 22.95 = 45.8m

(b) the course that must be set to point exactly opposite the starting point and time taken for
crossing
; 𝑤 ;
course, θ = 𝑢 5
34 8
8
Time for crossing = = 27.8s
5𝑐𝑜𝑠 4 8

u = 3.5ms-1, w = 2ms-1, AB = 80m

Revision exercise 1
1. A man who can row at 0.9ms-1 in still water wishes to cross a river of width 1000m as quickly as
possible. If the current flows at a rate of 0.3ms-1. Find the time taken for journey. Determine the
direction in which he should point the boat and position of the boat where he lands. [111.11s,
71.570 to the bank, 333.33 downstream]
2. A man swims at 5kmh-1 in still water. Find the time it takes the man to swim across the
river250m wide, flowing at 3kmh-1, if he swims so as to cross the river
(a) the shortest route [225s]
(b) in the quickest time[180s]
3. A boy can swim in still water at 1ms-1, he swims across the river flowing at 0.6ms-1 which is
300m wide, find the time he takes
(a) if he travels the shortest possible distance [375s]
(b) if he travels as quickly as possible and the distance downstream, [300s, 180m]
4. A boy wishes to swim across a river 100m wide as quickly as possible. The river flows at 3kmh-1
and the boy can swim at 4kmh-1 in still water. Find the time that the boy takes to cross the river
and how far downstream he travels. [90s, 75m]

digitalteachers.co.ug
Relative motion
It is composed of

(a) relative velocity


(b) Relative path

(a) Relative velocity

This is the velocity a body would have as seen by an observer on another body. Suppose A and B are
two moving bodies, the velocity of A relative to B is the velocity of A as it appears to observer on B

It is denoted by VAB = VA – VB

Note that VAB VBA since VBA = VB – VA.

Numerical calculations

There are two methods used in calculations

- Geometric method and


- Vector method
(i) Geometric method.
If VA and VB are not given in vector form and the velocity of A relative to B is required, then
we can reverse the velocity of B such that VAB =VA +(-VB) and the vector triangle is drawn as
below.
VAB2 = VA2 + VB2 - 2VA x VB cosθ

and
𝑉𝐴𝐵 𝑉𝐵 𝑉𝐴
𝑠𝑖𝑛𝜃 𝑠𝑖𝑛𝛼 𝑠𝑖𝑛𝛽

(ii) Vector method


Find components of velocity for each separately

Example 5

Particle A is moving due north at 30ms-1 and particle B is moving due south at 20ms-1. Find the
velocity of A relative to B.

Solution 0 0 0
𝑉𝐴𝐵 ( )−( ) ( )
30 −20 50
↑VA = 30ms-1 and ↓VB = 20ms-1
𝑉𝐴𝐵 0 + 50 = 50ms-1 due north

Example 6

A particle A has a velocity (4i + 6j -5k)ms-1 while B has a velocity of (-10i – 2j + 6k)ms-1. Find the
velocity of A relative to B.


4 −10 14
;
( 6 ) − ( −2 ) ( 8 )
−5 6 −11

digitalteachers.co.ug
Example 7

A girl walks at 5kmh-1 due west and a boy runs 12kmh-1 at a bearing of 1500. Find the velocity of the
boy relative to the girl

Method I (geometrical)
VBG = VB – VG
12𝑠𝑖𝑛30 −5 11
=( ) − ( )= ( )
−12𝑐𝑜𝑠30 0 −10 39

𝑉𝐵𝐺 11 + −10 39

= 15.13ms-1

VBG2 = VB2 + VG2 - 2VB x VG cos1200

5 + 12 − 2 5 12 120 = 15.13ms-1
5 5

; 9
0 𝜃 ta = 43.40
θ = 16.63

The relative velocity is 15.13ms-1 at S46.630E The relative velocity is 15.13ms-1 at


S46.630E
Method II (Vector)

Example 8

Plane A is flying due north at 40kmh-1 while plane B is flying in the direction N300E at 30kmh-1. Find
the velocity of A relative B

The relative velocity is 20.53kmh-1 at N46.90W

Method II (vectors)

0 30𝑠𝑖𝑛30 −15
VAB2 = VA2 + VB2 - 2VA x VB cos300 VAB = VA – VB = ( ) − ( ) ( )
40 −30𝑐𝑜𝑠30 14 02
40 + 30 − 2 40 30 30 𝑉𝐵𝐺 −15 + 14 02 = 20.53kmh-1
= 20.53kmh-1
5
; θ = 46.940

digitalteachers.co.ug
; 4
θ = ta 43 1
5

The relative velocity is 20.53kmh-1 at N46.90W

Example 9

Ship P is steering 60kmh-1 due east while ship Q is steering in the direction N600W at 50kmh-1. Find
the velocity of P relative to Q.

Method I (Geometrical)
VPQ = VP – VQ
60 −50𝑠𝑖𝑛60 103 301
=( )−( ) ( )
0 50𝑐𝑜𝑠60 −25

𝑉𝐵𝐺 103 301 + −25 =106.3kmh-1

VPQ2 = VP2 + VQ2 - 2VP x VQ cos1500

60 + 50 − 2 60 50 150
= 106.28kmh-1 θ =ta ; 5
= 13.60
5 6 8
; θ = 13.60 Direction S(900 – 13.60)E
The relative velocity is 106.28kmh-1 at S76.40E The relative velocity is 106.28kmh-1 at S76.40E
Method II (Vector)

Finding true velocity


Example 10

To a cyclist riding due north at 40kmh-1, a steady wind appears to blow from N600E at 50kmh-1. Find
the true velocity of the wind

Solution

VWC = VW – VC
30 0
( ) –( )
0 40
; 4
θ =ta = 53.130
4

-1
Direction N (90 – 15.13)0E = N36.870E
30 + 40 = 50kmh

digitalteachers.co.ug
Example 11

To a motorist travelling due north at 40kmh-1, a steady wind appears to blow from N600E at 50kmh-1.

(a) find the true velocity of the wind


VWM = VW – VM
50 60 0 𝑉𝑊 −43 5 + 15 = 46kmh-1
( ) –( )
−50 60 40
; 5
;4 5 θ =ta 4 5
= 19.020
5
Direction: N710W

(b) If the wind velocity and direction remain constant but the speed of the motorist is increase,
find his speed when the wind appears to be blowing from the direction N450E.
VWM = VW – VM
− 45 46 71 0 j components: -bcos45 = -46cos71 – a
( ) ( )–( )
− 45 −46 71
i components : -bsin45 = 46sin71 a = 58.47kmh-1
b = 61.5096

Example 12

To a man travelling due north at 10kmh-1, a steady wind appears to blow from East. When he travels
in the direction N600W at 8kmh-1, it appears to come from south. Find the velocity of the wind.

VWM = VW – VM a = 8sin60 = 4 3
− 0
( ) –( ) 10 = b + 8cos60
0 10
− b=6
( ) …………………………….(i)
10
−𝑎
Also 𝑊 ( ) (−4 3)
10 10
VWM = VW – VM
𝑊 −4 3 + 10 = 12.17kmh-1
0 −8 60
( ) –( ) ….. (ii)
+ 8 60 ;
𝜃 ta ( )= 55.30
4
(i) and (ii)
Direction: N(90 -53.3)0W= N34.70W
− −8 60
( ) ( )
10 + 8 60
Example 13

To a cyclist riding due north at 40kmh-1, a steady wind appears to blow eastwards. On reducing his
speed to 30kmh-1 but moving in the same direction, the wind appears to come from southwest. Find
the velocity of the wind
Also
VWC = VW – VC
0 VWC = VW – VC
( ) –( )
0 40 𝑏𝑠𝑖𝑛45 0
( ) 𝑊– ( ) ….. (ii)
𝑏𝑐𝑜𝑠45 30
( ) …………………………….(i)
10

digitalteachers.co.ug
(i) and (ii) 𝑎 10
𝑊 ( ) ( )
45 40 40
( ) ( )
40 30 + 45
𝑊 10 + 40 = 41.23kmh-1
40 = 30 + bcos45 4
;
𝜃 ta ( )= 75.960
b = 10 2
Direction: N(90 -76)0E= N140E
a = bsin45 = 10 2sin45 = 10

Exercise 2
1. Car A moving Eastward at 20ms-1 and car B is moving Northward at 10ms-1. Find the
(i) velocity of A relative to B [10 5 ;
(ii) velocity of B relative to A [10 5 ;
2. A yatch and a trawler leave a harbour at 8am. The yatch travels due west at 10kmh-1 and
trawler due east at 20kmh-1
(i) what is the velocity of the trawler relative to yatch [30kmh-1 east]
(ii) how far apart are the boats at 9.30am [45km]
3. At 10.30am a car travelling at 25ms-1 due east overtakes a motor bike travelling at 10ms-1 due
east. What is the velocity of the car relative to the motor bike and how far apart are the vehicle
at 10.30am. [15ms-1 east, 900m]
4. Bird A has a velocity of (7i + 3j + 10k)ms-1 while bird B has a velocity (6i – 17k)ms-1. Find the
velocity of B relative to A [(-i -3j – 27k0ms-1]
5
5. Joe rides his horse with a velocity ( )kmh-1 while Jill is riding her horse with velocity
24
5
( )kmh-1
12
5 ;
(i) Find Joe’s velocity as seen by Jill *( ) +
24
0 ;
(ii) What is Jill’s velocity as seen by Joe. *( ) +
−124
-1
6. In EPL football match, a ball is moving at 5ms in the direction of N450E and the player is
running due north at 8ms-1. Find the velocity of the ball relative to the player.
[5.69ms-1 at S38.380E]
7. An aircraft is flying at 250kmh-1 in direction N600E and a second aircraft is flying at 200kmh-1 in
the direction N200W. Find the velocity of the first aircraft as seen by the pilot of the second
aircraft. [292kmh-1 at S77.90E]
8. A ship is sailing southeast at 20kmh-1 and a second ship is sailing due west at 25kmh-1. Find the
magnitude and direction of the velocity of the first ship relative to the second.
[41.62kmh-1 at S70.130E]
9. What is the velocity of a cruiser moving at 20kmh-1 due to north as seen by an observer on a
liner moving at 15kmh-1 in the direction N300W [10.3kmh-1 at N46.90E]
10. A car is being driven at 20ms-1 on a bearing of 0400. Wind is blowing from 3000 with speed of
10ms-1. Find the velocity of the wind as experienced by the driver of the car.
[48.13ms-1 at S18.130W]
11. An aircraft is moving at 250kmh-1 in direction N600E. The second aircraft is moving at 200kmh-1
in a direction N200W. Find the velocity of the first aircraft as seen by the pilot of the second
aircraft. [292kmh-1 at S77.90E]

digitalteachers.co.ug
12. To a pigeon flying with velocity of (-2i + 3j + k)ms-1, a hawk appears to have a velocity of
(i – 5j -10k)ms-1.Find the true velocity of the hawk[(-i -2j – 9k)ms-1]
13. To a cyclist riding at 3ms-1 due east, the wind appears to come from the south with the speed
3 3 ; . Find the true speed and direction of the wind. [6ms-1 from S300W]
14. To the pilot of an aircraft A travelling at300kmh-1 due south, it appears that an aircraft B is
travelling at 600kmh-1 in a direction N600W. Find the true speed and direction of the aircraft B.
[520kmh-1 west]
15. Jane is riding her horse at 5kmh-1 due north and sees Suzan riding her horse apparently with
velocity 4kmh-1, N600E. Find Suzan’s true velocity. *7.81kmh-1 N26.30E]
16. A eagle flying at 8ms-1 on a bearing of 2400 sees a chick apparently running at 5ms-1 on
bearing3000. Find true velocity of the chick. [11.4ms-1 at 262.40]
17. A train is travelling at 80kmh-1 in direction N150E. A passenger on the train observes a plane
apparently moving at 125kmh-1 in the direction N500E. Find the true velocity of the plane.
[196kmh-1 N36.50E]
18. To an athlete jogging at 12kmh-1 on a bearing of N100E, the wind seems to come from a
direction N200W at 15kmh-1. Find the true velocity of the wind. [7.57kmh-1 N72.50W]
19. To a passenger on a boat which is travelling at 20kmh-1 on a bearing 2300, the wind seems to be
blowing from 2500 as 12kmh-1. Find the true velocity of the wind [9.64kmh-1 N24.80E]
20. On a particular day wind is blowing N300E at a velocity of 4ms-1 and a motorist is driving at
40ms-1 in the direction of S600E.
(a) Find the velocity of the wind relative to the motorist. [40.2ms-1 at N54.280W]
(b) If the motorist changes the direction maintaining his speed and the wind appears to blow
due east. What is the new direction of the motorist [N85.030W]
21. A, B and C are three aircrafts. A has velocity (200i + 170j)ms-1. To the pilot of A it appears that B
has velocity (50i – 270j)ms-1. To the pilot of B it appears that C has a velocity (50i + 170j)ms-1.
Find the velocities of B and C [(250i -100j)ms-1, (300i + 70j)ms-1]
22. To a bird flying due east at 10ms-1, the wind seems to come from south. When the bird alters its
direction of flight to N300E without altering its speed, the wind seems to come from the north-
west. Find the true velocity of wind. [10.6ms-1 from S69.90W]
23. To an observer on a trawler moving at 12kmh-1 in the direction S300W, the wind appears to
come from N600W. To an observer on a ferry moving at 15kmh-1 in a direction S800E, the wind
appears to come from the north. Find the true velocity of the wind. [26.8kmh-1 N33.40W]

b. Relative position
Consider two bodies A and B moving with VA and VB from points with position vectors OA and OB
respectively.

Position of A at time t is RA(t=t) = RA(t=0) + t x VA

Position of A at time t is RBt=t) = RB(t=0) + t x VB

Position of A relative to B at time t is RABt=t) = RA(t=0) – RB(t=0)

RABt=t) = (RA(t=0) + t x VA) – (RB(t=0) + t x VB)

RABt=t) = (RA(t=0) - RB(t=0)) + (VAB)t

digitalteachers.co.ug
Example 14

The velocities of ships P and Q are (i +6j)kmh-1 and (-i + 3j)kmh-1. At a certain instant, the
displacement between the two ship is (7i + 4j)km. Find the

(a) Relative velocity of ship P to Q


VPQ = VP – VQ
1 −1 2 ;
( )−( ) ( )
6 3 3
(b) Magnitude of displacement between ships P and Q after 2hours.
RPQt=t) = (RP(t=0) – RQ(t=0)) + (VPQ)t
7 2
RPQt=t) =( ) + ( )
4 3
7 2 11
RPQt=2) =( ) + 2 ( ) = ( )
4 3 10
| | 11 + 10 14 87

Example 15

Two ship A and B move simultaneously with velocities 20kmh-1 and 40kmh-1. Ship A moves in the
northern direction while ship B moves in N600E. Initially ship B is 10km due west of A. Determine

(a) the relative velocity of A to B.


0 40𝑠𝑖𝑛60 −34 641
𝑉𝐴𝐵 ( )−( ) ( )
20 40𝑐𝑜𝑠60 0

𝑉𝐴𝐵 −34 641 + 0 = 34.641


VAB = VA – VB

(b) the position of A relative to B at any time t


10 0 −34 641
RAB(t=t) =*( ) − ( )+ + 𝑡 ( )
0 0 0
10 − 34 641𝑡
RAB(t=t) =( ) 𝑘𝑚
RAB(t=t) = (RA(t=0) – RB(t=0)) + (VAB)t 0

Distance and time of closest approach


(Shortest distance and time of shortest distance)
When two particles are moving simultaneously with specific velocities, time will come when they are
closest to each other without colliding.

There are three methods used for the distance and time of closest approach, i.e. Geometrical, vector
and differential method.

1. vector method
Consider particle A and B moving with velocities VA and VB from point with position vector
OA and OB respectively.
For minimum distance to be attained then < 0. This gives the time.
Then shortest distance, d = | < |
2. Differential method
The minimum distance is reached when | < | 0. This gives time
Then shortest distance, d = | < |

digitalteachers.co.ug
3. Geometrical method
If VA and VB are not given in vector form, then the velocity of B is reversed such that
+ − and the vector triangle is drawn as below.
The shortest distance, d will be perpendicular to VAB

𝑉𝐴𝐵 𝑉𝐵
𝑠𝑖𝑛𝜃 𝑠𝑖𝑛𝛼

Shortest distance, d =ABsinβ


𝐴𝐵𝑐𝑜𝑠𝛽
Time to the shortest distance, t = 𝑉𝐴𝐵

+ −2

Example 16

A particle P starts from rest from a point with position vector (2j + 2k)m with a velocity (j +
k)ms-1. A second particle Q starts at the same time from a point whose position vector is (–11i – 2j –
7k)m with a velocity of (2i + j + 2k)ms-1. Find
(i) the shortest distance between the particles
(ii)
(iii) how far each has travelled by this time.
0 2 −2
− (1) − (1) (0) |𝑅𝑃𝑄 𝑡<6 | −1 4 + 4 + 2 8 = 5.08m
1 2 −1
(ii) How far each has travelled
< < − < +
𝑅𝑃 𝑡<𝑡 𝑅𝑃 𝑡< + 𝑉𝑃 𝑡
0 −11 −2
< [(2) − ( −2 )] + ( 0 ) 0 0 0
2 −7 −1 𝑅𝑃 𝑡<6 (2) + 6 2 𝑥 (1) (8 2)
11 − 2 2 1 82
< ( 4 )
|𝑅𝑃 𝑡<6 | 0 +82 +82 11 6𝑚
9−
For minimum distance: 0. −11 2 14
<
𝑅𝑄 𝑡<6 ( −2 ) + 6 2 𝑥 (0) (4 2)
−2 11 − 2 −7 1 54
(0) ( 4 ) 0
−1 9− |𝑅𝑄 𝑡<6 | 14 +42 +54 6 8𝑚

-22 + 4t + 0 -9 + t = 0 Method II
𝑑
t= 62 |𝑅 | 0.
5 𝑑𝑡 𝐴𝐵 𝑡<𝑡

(i) Then shortest distance, d = | < | 11 − 2𝑡


|𝑅𝑃𝑄 𝑡<𝑡 | ( 4 )
11 − 2 9−𝑡
<6 ( 4 )
9− |𝑅𝑃𝑄 𝑡<𝑡 | 11 − 2𝑡 +4 + 9−𝑡
11 − 2 6 2 −1 4
<6 ( 4 ) ( 4 ) |𝑅𝑃𝑄 𝑡<𝑡 | 218 − 62𝑡 + 5𝑡
9−62 28

digitalteachers.co.ug
11 − 2𝑥 6 2 −1 4
| < | 218 − 62 + 5 𝑅𝑃𝑄 ( 4 ) ( 4 )
𝑡<6
9−62 28
| < | −62 + 10 0
|𝑅𝑃𝑄 𝑡<6 | −1 4 + 4 + 2 8 = 5.08m
t = 6.2s

Example 17

At 12 noon the position vectors r and velocity vectors v of ship A and ship B are as follows

rA = (-9i + 6j)km, vA = (3i + 12j) kmh-1 and rB = (16i + 6j), vB = (-9i+ 3j)kmh-1 respectively

(i) Find how far apart the ships are at noon

< < − <

−9 16 −25
< ( )−( ) ( )
6 6 0

| < | −25 +0 25

(ii) Assuming velocities do not change, find the least distance between the ships in the subsequent
motion
3 −9 12 4
− ( ) − ( )=( ) t= 𝑜𝑢𝑟𝑠
12 3 9

< < − < + Shortest distance, d = 𝑅𝑃𝑄 𝑡<


−25 12 −25 + 12
< ( )+( ) ( ) 4
0 9 9 −25 + 12 𝑥 −9
𝑅𝐴𝐵 𝑡< 4 = ( ) 𝑘𝑚
For minimum distance: < 0. 9𝑥 12

12 −25 + 12
( ) ( ) 0
9 9 𝑅𝐴𝐵 𝑡<
−9 + 12 = 15km
-300 + 144t + 81t = 0

(iii) Find when their distance of closest approach occurs and the potion vectors of A and B
4
It occurs at 12.00 + 60 = 1.20pm
𝑅𝐵 𝑡<𝑡 𝑅𝐵 𝑡< + 𝑉𝐵 𝑡
how far each travelled
< +
16 −9 4 4
< 𝑅𝐵 𝑡<
( ) + ( ) 𝑥 =( ) 𝑘𝑚
−9 3 4 −5 6 3 10
<
( )+( ) =( )
6 12 22
Example 18

At a certain time, the position vectors r and velocity vectors v of ship A and ship B are as follows

rA = (20j)km VA = (9i -2j)kmh-1 at 14.00hrs

rB = (i + 4j)km VB = (4i + 8j)kmh-1 at 15.00hrs

Assuming velocities do not change, find

(a) the position vector of A at 15.00hrs

digitalteachers.co.ug
< < +

0 9 9
At 16.00hrs: < ( )+( ) 1 ( )
20 −2 18
(b) the least distance between A and B in the subsequent motion

< < − < +


t = 0.8hrs
9 1 9 8
< *( ) − ( )+ + *( ) − ( )+t Shortest distance, d = |𝑅𝑃𝑄 |
18 4 −2 8 𝑡< 8

8+5 8+5𝑥08 12
< ( ) 𝑅𝐴𝐵 𝑡< 8 ( ) ( ) 𝑘𝑚
14 − 10 14 − 10 𝑥 0 8 6
For minimum distance: < 0. 𝑅𝐴𝐵 𝑡< 8 12 + 6 = 13.42km
5 8+5
( ) ( )=0
−10 14 − 10
(c) time at which this least separation occurs.
15.00 + 0.8 x 60 = 15.48hrs

Example 19

At a certain time, the position vectors r and velocity vectors v of ship A and ship B are as follows

rA = (-2i + 3j)km vA =(12i -4j)kmh-1 at 11.45am

rB = (8i + 7j)km vB = (2i – 14j)kmh-1 at 12.00 noon

Assuming velocities do not change, find

(a) The least distance between A and B in the subsequent motion


−2 12 ; For minimum distance: 𝑉𝐴𝐵 𝑅𝐴𝐵 0.
OA = ( ) and vA = ( ) 𝑡<𝑡
3 −4
10 −7 + 10𝑡
< < + ( ) ( ) 0
10 −5 + 10𝑡
−2 12 1 t = 0.6hrs
12.00: <
( )+ ( ) ( )
3 4 −4 2
−7 + 10 𝑥 0 6 −1
12 2 10 𝑅𝐴𝐵 𝑡< 6 ( ) =( ) 𝑘𝑚
− ( )−( ) ( ) −5 + 10 𝑥 0 6 1
−4 −14 10
− + |𝑅𝐴𝐵 𝑡< 6 | −1 + 1 =1.4142km
< < <

1 8 10
< *( ) − ( )+ + ( )
2 7 10
−7 + 10
< ( )
−5 + 10
(b) length of time for which A is within range, if ship B has guns within a range of up to 2km

−7 + 10𝑡
−7 + 10 ( ) 2
( ) −5 + 10𝑡
<
−5 + 10
−7 + 10𝑡 + −5 + 10𝑡 4
| < | 2

digitalteachers.co.ug
Time for which they are in range
100t2 -12t + 35 = 0
= 0.7 – 0.5 = 0.2h
t = 0.7hrs or t = 0.5hrs

Example 20

At 10am, ship A moves with a constant velocity (4i+ 20j) kmh-1 and ship B due north of A moves with
a constant velocity (-3i – 4j) kmh-1.

(a) Find the velocity of A relative to B


4 −3 7 ;
− ( )−( ) ( )
20 −4 24
(b) If the shortest distance between the two ships is 4.2km. Find the
(i) time to the nearest minute when they are closest together
(ii) original distance apart at 10am
(iii) the bearing of B from A when they are closest together.

Solution
49t – 24a + 576t = 0
(ii) Let a km be the distance apart at 10am
6 5𝑡
𝑎 …………………………………..(ii)
< < − < + 4

0 0 7 7 Substituting for a in eqn. (i)


< *( ) − ( )+ + ( ) ( )
0 24 − + 24 6 5𝑡 6 5𝑡
625t2 – 48( )t +( ) = 17.64
7 4 4
< ( )
− + 24 53.1684t2 = 17.67
| < | 42
t = ±0 57 0 576 𝑥 60 35𝑚𝑖𝑛𝑢𝑡𝑒
7 6 5𝑡 6 5𝑥 576
( ) 42 (ii) a = = = 15km
− + 24 4 4
7𝑥 0 576
7 + − + 24 17 64 (iii) 𝑅𝐴𝐵 𝑡< 576 ( ) 𝑘𝑚
−15 + 24 𝑥 0 576
4 032
625t2 – 48at + a2 = 17.64 ………….(i) 𝑅𝐴𝐵 𝑡< 576 ( )
−1 176
76
For minimum distance: < 0. 𝜃 ta ; ( ) 16 3
4
𝐷𝑖𝑟𝑒𝑐𝑡𝑖𝑜𝑛: 𝐸16 3 𝑆
7 7
( ) ( ) 0
24 − + 24
(c) length of time for which A is within range, if the visibility of ship B is within 12km
7 7
< ( ) ( ) t = 1.026h or t = 0.126h
− + 24 −15 + 24
| | 12 Time for which they are in range
<
= 1.026 – 0.126 = 0.9h
7 + − + 24 144
625t2 – 720t + 81 = 0

Example 21

At 12 noon a ship A is moving with constant velocity of 20.4kmh-1 in the direction Nθ0E where tan θ =

5
A second ship B is 15km due to north of A. Ship B is moving with constant velocity of 5kmh-1 in the

digitalteachers.co.ug
direction Sα0W, where tan α= ¼. If the shortest distance between the ships is 4.2km, find the time to
the nearest minute when the distance between the ships is shortest. (12mars)

Table of results
0 0
At t = 0, 𝑟𝐴 ( ) => c = ( )
Vector Magnitude Direction 0 0
VA 20.4kmh-1 NθE 4𝑡
Hence 𝑟𝐴 ( )
VB 5kmh-1 SαW 20𝑡
−3 −3𝑡
RB = ∫ 𝑉𝐵 𝑑𝑡 =∫ ( ) 𝑑𝑡 = ( )+𝑐
−4 −4𝑡
0 0
At t = 0, 𝑟𝐵 ( ) => c = ( )
15 15
−3𝑡
0 Hence 𝑟𝐵 ( )
tan θ = ; θ = 11.3 15 − 4𝑡
5
4𝑡 −3𝑡 7𝑡
4 ArB = rA – rB = ( )-( )=( )
( 20 4 11 3 ) ( ) 20𝑡 15 − 4𝑡 24𝑡 − 15
20 4 11 3 20
ds = ArB = 7𝑡 + 24𝑡 − 15 5

but ds = 4.2

 7𝑡 + 24𝑡 − 15 5 = 4.2
( 7𝑡 + 24𝑡 − 15 5 ) 42
0
tan α = ; α = 36.87 7𝑡 + 24𝑡 − 15 42 5
5
47𝑡 + 576𝑡 − 720𝑡 + 225 17 64
−3
; ( −5 36 87 ) ( ) 625𝑡 − 720𝑡 + 207 36 0
−5 36 87 −4
7 ± ;7 ;4 6 5 7 6
t= = 0.576hours
6 5
= 0.576 x 60 = 35minutes
Hence the time at which the distance is
shortest is 12:35pm

4 4
rA = ∫ =∫ ( ) =( )+
20 20

Example 22

At noon a boat A is 30km from boat B and its direction from B is 2860. A is moving in the North-East
direction at 16kmh-1 and B is moving in the north direction at 10kmh-1. Determine when they are
closest to each other. What is the distance between them?

0 30𝑠𝑖𝑛74
𝑅𝐴 𝑡< ( ) , 𝑅𝐵 𝑡< ( ) 𝑘𝑚
0 −30𝑐𝑜𝑠74
16𝑠𝑖𝑛45 0 11 314
𝑉𝐴𝐵 ( )−( ) ( ) 𝑘𝑚/
16𝑐𝑜𝑠45 10 1 314
𝑅𝐴𝐵 𝑡<𝑡 𝑅𝐴 𝑡< − 𝑅𝐵 𝑡< + 𝑉𝐴𝐵 𝑡

digitalteachers.co.ug
0 30 74 11 314 𝑅𝐴𝐵
< *( ) − ( )+ + ( ) 𝑡< 4
0 −30 74 1 314
−28 838 + 11 314 𝑥 2 43
−28 838 + 11 314 ( ) 𝑘𝑚
< ( ) 8 269 + 1 314 𝑥 2 43
8 269 + 1 314
−1 345
For minimum distance: < 0. 𝑅𝐴𝐵 𝑡< 4 ( ) 𝑘𝑚
11 462
11 314 −28 838 + 11 314
( ) ( ) 0 |𝑅𝐴𝐵 𝑡< 4 | −1 345 + 11 462
1 314 8 269 + 1 314
t = 2.43h = 11.54km

Alternatively

9
𝑠𝑖𝑛𝛼 𝑠𝑖𝑛45

α = 38.380

45 + α + β + 74 = 1800

β = 22.620

d = ABsinβ = 30sin22.62 = 11.54km


𝐴𝐵𝑐𝑜𝑠𝛽 𝑐𝑜𝑠 6
Time, t = 𝑉𝐴𝐵 9
= 2.43h
+ −2 45
Time is 2.43h from noon or 2hour and
16 + 10 − 2 16 10 45 25.8minutes

= 11.39km/h It occurs 2.26pm at a distance 11.54km

Example 23

Two planes A and B are both flying above the Pacific Ocean. Plane A is flying on a course of 0100at a
speed 300kmh-1 and plane B is flying on a course of 3400 at 200kmh-1. At a certain instant, plane B is
40km from A. Plane A is then on a bearing of 0600. After what time will they come closest together
and what will be their minimum distance apart.
−34 641 + 120 4985𝑡
𝑅𝐴𝐵 𝑡<𝑡 ( )
−20 + 107 5038𝑡
For minimum distance: 𝑉𝐴𝐵 𝑅𝐴𝐵 𝑡<𝑡 0

120 4985 −34 641 + 120 4985𝑡


( )( ) 0
107 5038 −20 + 107 5038𝑡
-6324.2645 + 26076.9555t = 0

t = 0.2425h
300 10 −200 20 120 4985
( )−( ) ( )
300 10 200 20 107 5038 Least distance = |𝑅𝐴𝐵 |
𝑡< 4 5
0 40 30
< ( ) , < ( ) 𝑅𝐴𝐵 𝑡< 4 5
0 40 30
−34 641 + 120 4985𝑥0 2425
− + ( )
< < < −20 + 107 5038𝑥0 2425
0 40 30 120 4985
< *( ) − ( )+ + ( )
0 40 30 107 5038

digitalteachers.co.ug
5 4202
< 4 5 ( )
6 0692

| < 4 5 | 5 4202 + 6 0692 8 14

Alternatively
= 161.484km/h
6 484
𝑠𝑖𝑛𝛼 𝑠𝑖𝑛

α = 38.260

α + β =50

β = 11.740

d = ABsinβ = 40sin11.74 = 8.14km


𝐴𝐵𝑐𝑜𝑠𝛽 4 𝑐𝑜𝑠 74
Time, t = 𝑉𝐴𝐵 6 484
0 2425
+ −2 30

300 + 200 − 2 300 200 30

Example 24

At a given instant two cars are at a distance 300m and 400m from a point of intersection O of two
roads crossing at right angles and are approaching O at uniform speeds of 20m/s and 40m/s
respectively. Find

(i) Initial distance between the two cars


(ii) shortest distance between the cars
(iii) time taken to reach this point

0 500𝑐𝑜𝑠53 1 40
𝑅𝐴𝐵 𝑡<𝑡 *( ) − ( )+ + 𝑡 ( )
0 500𝑠𝑖𝑛53 1 −20
−399 842 + 40𝑡
𝑅𝐴𝐵 𝑡<𝑡 ( )
300 21 − 20𝑡
For minimum distance: 𝑉𝐴𝐵 𝑅𝐴𝐵 𝑡<𝑡 0
40 −399 842 + 40𝑡
300 + 400 500 ( ) ( ) 0
−20 300 21 − 20𝑡
4
ta ;
( ) 53 1 -21,997.88 +2000t = 0

0 50 53 1 t = 11s
< ( ) , < ( )
0 50 53 1 −399 842 + 40𝑥11 40 158
𝑅𝐴𝐵 𝑡< ( ) ( )𝑚
− 300 21 − 20 𝑥 11 80 21

0 −40 40 ; 𝑅𝐴𝐵 𝑡< 40 158 + 80 21 =89.701m


( )−( ) ( )
−20 0 −20

< < − < +

digitalteachers.co.ug
Example 25

A road running north-south crosses a road running east-west at a junction O. Initially Paul is on the
east-west, 1.7km west of O and is cycling towards O at15mk/h. At the same time John is at O cycling
due north at 8km/h. If Paul and John do not alter their velocities, find the

(i) relative velocity of Paul to John


(ii) shortest distance between Paul and John

−1 7 + 15𝑡
𝑅𝑃𝐽 𝑡<𝑡 ( )
−8𝑡
For minimum distance: 𝑉𝑃𝐽 𝑅𝑃𝐽 𝑡<𝑡 0

15 −1 7 + 15𝑡
( ) ( ) =0
−8 −8𝑡
0 17
< ( ) , < ( ) -25.5 +289t = 0
0 0
− t = 0.088h
15 0 15 ; Least distance = |𝑅𝑃𝐽 |
( )−( ) ( ) 𝑡< 88
0 8 −8
−1 7 + 15 𝑥 0 088 −0 38
| | 15 + −8 = 17km/h 𝑅𝑃𝐽 𝑡< 88 ( ) ( )
−8𝑥 0 88 −0 704

< < − < + |𝑅𝑃𝐽 | −0 38 + −0 704 = 0.8km


0 17 15
< *( ) − ( )+ + ( )
0 0 −8

Example 26

Two airship P and Q are 100km apart, P being west of Q. Two Helicopters A and B fly simultaneously
from P and Q respectively, at 11.00a.m. Helicopter A is flying with a constant speed of 400km/h in
the direction N500E. Helicopter B flying at a constant speed of 500km in the direction N700W. Find
the

(i) Time when the helicopters are closest together.


(ii) closest distance between the helicopters

𝑅𝐴𝐵 𝑡<𝑡 𝑅𝐴 𝑡< − 𝑅𝐵 𝑡< + 𝑉𝐴𝐵 𝑡

0 100 776 264


𝑅𝐴𝐵 𝑡<𝑡 *( ) − ( )+ + 𝑡 ( )
0 0 86 105
−100 + 776 264𝑡
𝑅𝐴𝐵 𝑡<𝑡 ( ) 𝑘𝑚
0 100 86 105𝑡
< ( ) , < ( )
0 0 For minimum distance: 𝑉𝐴𝐵 𝑅𝐴𝐵 0
𝑡<𝑡

776 264 −100 + 776 264𝑡
( )( )
400 50 −500 70 86 105 86 105𝑡
( )−( )
400 50 500 70 609999.869t =77624.4
776 264 ;
( ) t = 0.1273h = 0.1273 x 60= 8minutes
86 105

digitalteachers.co.ug
−1 182
𝑅𝐴𝐵 𝑡< 7 ( ) 𝑘𝑚
10 961
Time when they are closest
|𝑅𝐴𝐵 𝑡< 7 | −1 182 + 10 961
= 11.00 + 8= 11.08am
=11.0247km
Least distance = | < 7 |
Closest distance =11.025km
−100 + 776 264 0 1273
< 7 ( )
86 105 0 1273
Alternatively
α = 33.670

α + β + 50 =90

β = 6.330

d = PQsinβ = 100sin6.33 = 11.025km


𝑃𝑄𝑐𝑜𝑠𝛽 𝑐𝑜𝑠6
Time, t = 𝑉𝐴𝐵 78 5

+ −2 120 t = 0.1273h = 0.1273 x 60= 8minutes

Time when they are closest


400 + 500 − 2 400 500 120
= 781.025km/h = 11.00 + 8= 11.08am

5 78 5

Example 26

Car A is 80m North-West of point O, Car B is 50m N300E of O. Car A is moving at 20m/s on a straight
road towards O. Car B is moving at 10m/s on another straight road towards O. Determine the

(i) Initial distance between the cars


(ii) Velocity of A relative to B
(iii) shortest distance between the two cars as they approach O

𝑉𝐴𝐵 𝑉𝐴 − 𝑉𝐵
20𝑠𝑖𝑛45 −10𝑠𝑖𝑛30
𝑉𝐴𝐵 ( )−( )
−20𝑐𝑜𝑠45 −10𝑐𝑜𝑠30
19 1421
𝑉𝐴𝐵 ( )
−5 4819
𝑉𝐴𝐵 19 1421 + −5 4819

−80 45 50 30 = 19.9116m/s
< ( ), < ( )
80 45 50 30
−80 45 50 30 −81 5685
< ( )−( ) ( )
80 45 50 30 13 2673

[ < ] −81 5685 + 13 2673 = 82 .6404m

20 45 −10 30
(ii) ( ), ( )
−20 45 −10 30

digitalteachers.co.ug
5 48 9 19 1421 −81 5685 + 19 1421𝑡
θ=ta ;
=15.980 ( ) ( ) 0
9 4 −5 4819 13 2673 − 5 4821𝑡
Direction: E15.980S t = 4.1216s

(iii) Least distance = |𝑅𝐴𝐵 𝑡<4 6𝑠 |

− + −81 5685 + 19 1421 𝑥 4 1216


< < < 𝑅𝐴𝐵 𝑡<𝑡 ( )𝑚
13 2673 − 5 4821 𝑥 4 1216
−81 5685 19 1421
< ( )+ ( ) −2 672
13 2673 −5 4819 𝑅𝐴𝐵 𝑡<𝑡 ( )𝑚
−9 328
−81 5685 + 19 1421
< ( ) |𝑅𝐴𝐵 | −2 672 + −9 328 =9.728m
13 2673 − 5 4821 𝑡<𝑡

For minimum distance: < 0

Alternatively

Method II: Using geometric approach

𝑉𝐴𝐵 20 + 10 − 2 𝑥 20 𝑥10 75

𝑉𝐴𝐵 19 912𝑚𝑠 ;
𝑉𝐴𝐵
750 𝛼

𝛼 29 02

45 + 29 02 74 02
The velocity of A relative to B is 19.912ms-1
Using cosine rule due S74.020E

The shortest distance between the two cars as they approach O (04marks)

Using geometrical approach


5 8 64
; θ = 35.760
𝜃 750
< 𝐵𝐴𝐷 35.76 – 29.02 = 6.740
𝑟𝐴𝐵
6 74 𝐴𝐵
ArB = 82.64sin 6.740 = 9.699m

The shortest distance between the two cars they


approach O is 9.699m

Revision exercise 3
1. 1.At 8am ship A and ship Bare 11km apart with B due west of A. a and B move with a constant
velocities (-4i +3j)kh/h and (2i + 4j)km/h respectively. Find the the

(i) least distance between the two ship in the subsequent motion[1.81km]

digitalteachers.co.ug
(ii) time to the nearest minute at which this situation occurs[9.47am]

2. 2. At 7.30am, two ship A and B are 8km apart with B due north of A. A and B move with constant
velocities (12j)km/h and (-5i)km/h respectively. Find the

(i) least distance between the two ship in the subsequent motion[3.08km]

(ii) time to the nearest minute at which this situation occurs[8.04am]

3. A and B are two tankers at 13.00hrs, tanker B bas position vector of (4i+8j)km relative to A. A
and B move with constant velocities (6i + 9j)km/h and (-3i + 6j)km/h respectively. Find the
(i) least distance between the two ship in the subsequent motion[6.32km]
(ii) time to the nearest minute at which this situation occurs[13.40hrs]
4. At 12 noon the position vectors r and velocity vectors v of two ship A and B are as follows
rA = (5i + j)km, VA = (7i + 3j)km/h and rB = (8i + 7j)km, VB = (2i –j)km/h
(i) Assuming velocities do not change , find the least distance between the ships in
subsequent motion [2.81]
(ii) Find the time when their distance of closest approach occur [12.57pm]
5. At a certain time, the position vectors r and velocity vectors V of two ship A and B are as follows

rA = (3i + j)km, VA = (2i + 3j)km/h at 11.00am

rB = (2i - j)km, VB = (3i + 7j)km/h at 12.00noon

Assuming velocities do not change; find the

(i) The position vector of A at noon [5i + 4j]


(ii) Distance between the ships at 12.00 noon [5.83km]
(iii) The least distance between A and B in the subsequent motion[1.7km]
(iv) Time at which the least separation occurs [1.21pm]

6. At 12 noon, the position vectors r and velocity vectors V of two battle ship A and battle B are as
follows

rA = (13i + 5j)km, VA = (3i - 10j)km/h

rB = (3i - 5j)km, VB = (15i + 14j)km/h

(i) Assuming the velocities do not change, find the least distance between the ships in
subsequent motion [4.47km]
(ii) The battle ship has guns with a range of up to 5km, find the length of time during which
the cruiser is within range of the battle ships [10minutes]
7. At tine t = 0 the position vectors r and velocity vectors V of two battle ship A and battle B are as
follows
1 −6 4 −5
(2) , (0) / (−14) , (1) /
3 1 1 7
Assuming velocities do not change, find
7 9
(i) The potion vectors of B relative to A at time t seconds[ (2) + ( 13 ) ]
2 −7
(ii) The least distance between the ships in the subsequent motion [15.9m]
5
(iii) The time taken to the closest distance * +

digitalteachers.co.ug
8. At time t = 0 the position vectors r and velocity vectors V of two battle ship A and battle B are as
follows
rA = (3i + j + 5k)m, VA = (4i + j – 3k)m/s
rB = (i - 3j+ 2k) m, VB = (i + 2j + 2k)m/s
Assuming velocities do not change, find
2+3
(i) The position vector of B relative to A at time t second[( 4 − ) ]
3−5
(ii) The value of t when A and B are closed ( 5)
(iii) Least distance between A and B [4.917m]
9. At tine t = 0 the position vectors r and velocity vectors V of two battle ship A and battle B are as
follows
2 1
, (1) / 2 , (−5) /
−5 1
Where β is a constant, assuming velocities do not change show that the least distance between
6
the ships in the subsequent motion is 7 and their distance of closest approach is 7
.
65
10. A lizard on a wall at point A, has a position vector (40) . At time t= 0 seconds a fly has
0
37 5
a position vector (16) and velocity vector (2) /
22 −1
If the fly were to continue with this velocity, find the closest distance it would come to the lizard
and the value of t when it occurs [ 374 , 7s]
11. A particle P move with constant velocity (2i + 3j + 8k)m/s passes a point with position vector
(6i – 11j + 4k)m. At the same instant particle Q passes through a point whose position vector is
(i -2j + 5k)m moving at constant velocity of (3i + 4j- 7k)m/s. Find
(a) Position of Q relative to P at that instant. [10.344m]
(b) Shortest distance between the particles [10.32m]
(c) Time that elapses before the particles are nearest to each other [0.0485s]

12. Two particles P and Q move with constant velocities (4i + j – 2k)m/s and (6i + 3k)m/s
respectively. Initially P is at a point whose position vector is (i – 20j + 21k)m and Q is at a point
whose position vector is (i + 3k)m. find
(a) Time for which the distance between P and Q is least [2.2s]
(b) Distance of P from the origin at the time when the distance between P and Q is least
[28.8m]
(c) Least distance between P and Q [24.14m]

Course of closest approach


If A is to pass as close as possible to B, then velocity of A must ne perpendicular to the relative
velocity

digitalteachers.co.ug
Example 27

Two particles P and Q initially at positions (3i + 2j)m and (13i + 2j)m respectively begin moving.
Particle P moves with a constant velocity (2i + 6j)m/s. A second particle Q moves with a constant
velocity of (5j)m/s

(a) Find
(i) Time when the particles are closest together.
(ii) Bearing of particle O from Q when they are closest to each other.
(b) Given that half the time, the particle are moving closest to each other, particle P reduces its
speed to half its original speed, in the direction to approach particle Q and the velocity of
particle Q remains unchanged, find the direction of particle P.

Solution
; 4
𝛽 ta =63.430
3 13
< ( ) , < ( )
2 2 The bearing of P from Q = (270 + 63.43)
2 0 2
− ( )−( ) ( ) / =333.430
6 5 1
1
< < − < + (b) At t = 2s, 𝑉𝑃 ( ) 𝑚/𝑠
3
3 13 2 −10 + 2 Let P move at angle θ to x-axis
< ( )−( )+( ) ( )
2 2 1
For minimum distance =0 𝑐𝑜𝑠𝜃 0 10𝑐𝑜𝜃
< 𝑉𝑃𝑄 10 𝑠𝑖𝑛𝜃 − ( ) ( )
5 10𝑠𝑖𝑛𝜃 − 5
2 −10 + 2
( ) ( ) 0 If P is to approach Q; 8𝑠𝑖𝑛𝜃
1
-20 +4t +t = 0 10𝑐𝑜𝜃 𝑐𝑜𝑠𝜃
( ) ( ) =0
10𝑠𝑖𝑛𝜃 − 5 𝑠𝑖𝑛𝜃
t = 4s
10cos2θ + 10sin2θ -5 10𝑠𝑖𝑛𝜃 =0
−10 + 2 4 −2
(ii) <4 ( ) ( )
4 4 ;
P θ= 5
= 39.20

N Direction: N50.80E
4

β
-2 Q

Example 28

A motor boat B is travelling at a constant velocity of 10m/s due east and motor boat A is travelling at
a constant speed of 8m/s. Initially A and B are 600m apart with A due south of B. Find

(a) course that A should set to get close as possible to B

Let A move at an angle θ to x-axis 8𝑐𝑜𝑠𝜃 8𝑐𝑜𝑠𝜃 − 10


( ) ( ) =0
8𝑠𝑖𝑛𝜃 8𝑠𝑖𝑛𝜃
8 10 8 − 10
( )−( ) ( ) ; 64
8 0 8 θ= 8
=36.90
If A is to approach B; 0 Direction: N53.10E or E36.90N

digitalteachers.co.ug
(ii) Closest distance and time taken for the situation to occur
0 0 t = 80s
< ( ) , < ( )
0 600
−3 603 𝑥 80
< < − < + 𝑟𝐴𝐵 𝑡<8 ( )
−600 + 4 803𝑥80
0 0 8 36 9 − 10 −288 24
< *( ) − ( )+ + ( ) ( )
0 600 8 36 9 −215 76
−3 603 Least distance,d = |𝑟𝐴𝐵 |
< ( ) 𝑡<8
−600 + 4 803
For minimum distance: < 0 |𝑟𝐴𝐵 𝑡<8 | −288 24 + −215 76

−3 603 −3 603 d = 360m


( ) ( ) =0
4 803 −600 + 4 803

Example 29

A motor boat B is travelling at constant velocity of 14km/h due north and a motor boat A is travelling
at constant speed 12km/h. Initially A and B are 5.2km apart with A due west of B. Find

(i) Couse that A should set in order to get as close as possible to B

Let A move at an angle θ to x-axis


12𝑐𝑜𝑠𝜃 12𝑐𝑜𝑠𝜃
12 0 12 ( ) ( )=0
( )−( ) ( ) 12𝑠𝑖𝑛𝜃 − 13 12𝑠𝑖𝑛𝜃
12 13 12 − 13
;
θ= =67.40
If A is to approach B; 0
Direction: N22.60E or 67.40N
(ii) Closest distance and time taken for the situation to occur
0 52
< ( ) , < ( ) t = 0.961h
0 0
− + −5 2 + 4 612 𝑥 0 961
< < < 𝑟𝐴𝐵 𝑡< 96 ( )
−1 921 𝑥 0 961
0 52 12 67 4
< *( ) − ( )+ + ( ) −0 7725
0 0 12 67 4 − 13 ( )
−1 8442
−5 2 + 4 612
< ( ) Least distance, d = |𝑟𝐴𝐵 |
−1 921 𝑡< 96

For minimum distance: < 0 |𝑟𝐴𝐵 𝑡< 96 | −0 7725 + −1 8442


4 612 −5 2 + 4 612 d = 2km
( ) ( ) =0
−1 921 −1 921

Revision exercise 4
1. A ship A is moving with constant velocity of 18km/h in a direction N550E and is initially 6km from
a second ship B, the bearing of A from B being N250W. If B moves with a constant speed of
15km/h. Find

digitalteachers.co.ug
(a) Course that B should set in order to get as close as possible to A [N21.40E]
(b) Closest distance and time taken for the situation to occur.[4.135km, t=0.437h]
2. Two aircraft A and B are flying at the same altitude with A initially 10km due north of B and
flying at constant speed of 300m/s on a bearing of 0600. If B flies at constant speed of 200m/s,
find
(a) Course that B should set in order to get as close as possible to A [E78.40N]
(b) Closest distance and time taken for the situation to occur.[9.79km, t= 9.12s]
3. At 8am two boats A and B are 5.2km apart with A due west of B, and B travelling due north at a
steady speed 13km/h. If A travels with a constant speed of 12km/h, show that for A to get as
5
close as possible to B, A should set a course of Nθ0E where sinθ = . Find the closest distance
and time at which it occurs. [2km, 8.57 am]
4. Two aircraft A and B are flying at the same altitude with A initially 5km due north of B and B
flying at constant speed of 300m/s on bearing of 0600.If A flies at constant speed of 200m/s, find
(a) Course that A should set in order to get as close as possible to B [108.20]
(b) Time taken for the situation to occur.[5.4min]

5. A ship A moving with a constant speed of 24km/h in the direction N400E and is initially 10km
from a second ship B, the bearing of A from B being N300W. If B moves with a constant speed of
22km/h; find
(a) Course that B should set in order to get as close as possible to A [N16.40E]
(b) Closest distance and time taken for the situation to occur.[6.89km, 45min]

Interception and collision


Consider two bodies A and B moving with VA and VB from points with position vectors OA and OB

Position of A after time t is For collision to occur 𝑟𝐴 𝑡<𝑡 𝑟𝐵 𝑡<𝑡

< < + 𝑟𝐴 𝑡< + 𝑡 𝑥 𝑉𝐴 𝑟𝐵 𝑡< + 𝑡 𝑥 𝑉𝐵


Position of B after time t is 𝑟𝐴 − 𝑟𝐵 + 𝑡 𝑉𝐴𝐵 =0
𝑡< 𝑡<

< < + Hence 𝑟𝐴𝐵 0


𝑡<𝑡

Example 30

The position vectors 5 −3 +4 and 7 +5 −2 are for two particles with


velocities 2 +5 +3 / and −3 − 55 + 18 / respectively. Show that if the
velocities remain constant, a collision will occur

Solution
Along i direction: -2 = -5t; t = 0.4s
< + < +
Along j direction: -8= -20t; t = 0.4s
5 2 7 −3
(−3) + (5) ( 5 ) + (−15) Along k direction: 6 = 15t; t = 0.4s
4 3 −2 18
Since t is the same in all directions, collision
−2 −5 occurred
(−8) (−20)
6 15

digitalteachers.co.ug
Example 31

At 12 noon the position vectors r and velocity vectors V of two ships A and B are as follows

+7 , 6 +2 / , 6 +4 −4 + 8 /
Assuming velocities do not change

(i) Show that collision will occur


(ii) Find the time at which collision occurs
(iii) Find the position vector of the location during collision

Solution
(ii) time it occurred = 12:00 + 0.5 x 60
< + < +
= 12:30pm
1 6 6 −4
( )+ ( ) ( )+ ( ) (iii) How far each had travelled
7 2 4 8
−5 −10 𝑟𝐴 𝑡<𝑡 𝑟𝐴 𝑡< + 𝑡 𝑥 𝑉𝐴
( ) ( )
3 6
1 6 4
Along the i direction: -5 = -10t; t = 0.5h 𝑟𝐴 𝑡< 5 ( ) + 0 5( ) ( ) 𝑘𝑚
7 2 8
Along the j direction: 3 = 6t; t = 0.5h

Since t is the same in all directions

Collision occurred

Example 32

At 11:30am a battle ship is at a place with position vector (-6i + 12j)km and is moving with velocity
vector (16i – 4j)km/h. At 12:00 noon a cruiser is at a place with position vector (12i – 15j) and is
moving with velocity vector (8i + 16j)km/h. Assuming velocities do not change

(i) Show that collision will occur


(ii) Find the time at which collision occurs
(iii) Find the position vector of the location of collision

< < + Along the j direction: 15 = 20t; t = 1.25h


−6 16 2 Since t is the same in all directions collision
At 12:00: < 5 ( )+ 05( ) ( )
12 −4 10
occurred
For collision to occur
(ii) time it occurred = 11:30 + 1.25 x 60
< + < +
= 12:45pm
2 16 12 8
( )+ ( ) ( )+ ( ) (iii) How far each had travelled
10 −4 −15 18
−10 −8 𝑟𝐴 𝑡<𝑡 𝑟𝐴 𝑡< + 𝑡 𝑥 𝑉𝐴
( ) ( )
25 20
2 16 22
Along the i direction: -10 = -8t; t = 1.25h 𝑟𝐴 𝑡< 5 ( ) + 1 25 ( ) ( ) 𝑘𝑚
10 −4 5

digitalteachers.co.ug
Example 33

At 12:30 noon two ships A and B are 10km apart with B due east of A. A is travelling N600E at a
speed of 12km/h and ship B is travelling due north at 10km/h. Show that, is the two ships do not
change their velocities, they collide and fins to the nearest minute when collision occurs.
N
600 20km/h 10km/h Along the i direction: 10 3𝑡 = 10;
A 10km B t = 0.5774h
For collision to occur
Along the j direction: 10t = 10t
< + < +
t =0.5774 x 60 = 35minute
0 20 60 10 0
( )+ ( ) ( )+ ( ) Collision occurred at 12:35minutes
0 20 60 0 10
10 0
(10 3) ( )+ ( )
10 0 10
Example 34

Two aircraft P and Q are flying at the same height. P is flying due north at 500km/h while Q is flying
due west at 600km/h. When the aircrafts are 10km apart, the pilots realize that they are about to
collide. The pilot of P changes Course to 3450 and maintains the speed of 500km/h. The pilot Q
maintains his course but increases speed. Determine the

(i) Distance each aircraft would have travelled if the pilots had not realized that they were about
to collide

Solution
600km/h 0 0 100𝑠𝑖𝑛50 2 −600
O Q (100sinθ, 100cosθ) (0) + 𝑡 (500) (
100𝑐𝑜𝑠50 2
)+𝑡(
0
)
500km/h
θ Along i direction: 0 = 100sin50.2 – 600t
θ 100km
P t = 0.128h
; 6
θ = ta 5
= 50.20 Distance moved by P

For collision to occur 0 0 0


( ) + 0 128 ( ) ( ) 64𝑘𝑚
0 500 64
< + < +
Distance moved by Q
0 0 100 −600
( )+ ( ) ( )+ ( ) 100𝑠𝑖𝑛𝜃 −600 0 0284
0 500 100 0 ( ) + 0 128 ( ) =( )
100𝑐𝑜𝑠𝜃 0 64 011
=64.011km

(ii) New speed beyond which the aircraft Q must fly in order to avoid collision

Vkm/h
Q (100sinθ, 100cosθ)
0 θ
500km/h 15
θ 100
P

digitalteachers.co.ug
For collision to occur
t = 0.1325h
< + < +
Along i direction:
0 −500 15 100 −
( )+ ( ) ( )+ ( ) -500sin15t = 76.8284 – Vt
0 500 15 100 0
0 −500 15 100 50 2 − V x 0.1325 = 76.8284 + 500sin15 x 0.1325
( )+ ( ) ( )+ ( )
0 500 15 100 50 2 0
V =709.2837km/h
Along j direction: 500cos15t = 64.011

Course of interception
Suppose particle A with speed VA is to intercept particle B with speed VB, then

 Draw a sketch diagram showing the initial position and velocities of the two particles
 For interception to occur, the relative velocity must be in the direction of the initial
displacement of the particles.

Example 35

At an instant a body A travelling south at 10 3m/s is 150m west of B. Show that B will intercept A if
B is travelling S300W at 20m/s and find the time that elapses before collision occurs.

Solution
𝑉𝐴𝐵
A B Bearing S300W
θ
10 3𝑚/𝑠 90-θ 5 5
𝑉𝐵 i: t = 𝑐𝑜𝑠𝜃 co 6
15𝑠

Alternatively
0 150
( ) ( ) 𝑠𝑖𝑛9 𝑠𝑖𝑛𝜃
0 0
0 −20
( ) / ( ) / θ = 600: bearing S300W
−10 3 −20
𝑠𝑖𝑛9 9 ;𝜃
OA + t x = OB + t x Also,
𝑉𝐴𝐵

0 0 150 −20 𝑉𝐴𝐵 10𝑚/𝑠


( )+( ) ( )+( )
0 −10 3 0 −20
𝐴𝐵 5
t = 𝑉𝐴 15𝑠
j: 20sinθ =10 3 𝐴𝐵

;
θ= = 600

Example 36

At 9:00am two ships A and B are 15km apart with B on a bearing of 2700 from A. Ship A moves at
5km/h on a bearing of 3300. If the maximum speed of B is 10km/h. Find the

(i) Direction B should set in order to intercept A as soon as possible


(ii) Time taken for the interception to occur.

digitalteachers.co.ug
Solution

Bearing E25.70N
120 - θ 5km/h
5 5
0
i: t = 𝑐𝑜𝑠𝜃: 5 5: co 57
1 303
60
θ
B A t = 1.303 x 60 = 78mins
𝑉𝐴𝐵
0 15 Alternatively
( ) ( )
0 0
𝑠𝑖𝑛6 𝑠𝑖𝑛𝜃
10 −5 60
( ) / ( ) / 5
10 5 60
θ = 25.70: bearing E25.70N
OB + t x = OA + t x
𝑠𝑖𝑛9 ;𝜃
0 10 15 −5 60 Also, 𝑉𝐴𝐵
( )+( ) ( )+( )
0 10 0 5 60
𝑉𝐴𝐵 11 515𝑘𝑚/
j: 10sinθ =5 60
𝐴𝐵 5
t = 𝑉𝐴 𝑥 60 78𝑚𝑖𝑛𝑢𝑡𝑒𝑠
; 5 6 0 𝐴𝐵 5 5
θ= = 25.7

Example 37

At 12:00 noon two ship A and B are 12km apart with B on a bearing of 1400 from A. Ship A moves at
30km/h to intercept B which is travelling at 20km/h on a bearing of 3400. Find the

(i) Direction A should set in order to intercept B (ii) time taken for the interception to occur.

A α
θ
20km/h 231.39sinα – 275.79cosα = 82.078
200
𝑇 ;𝑇
500 But sinα= :𝑇
and cosα= :𝑇
B
0 12 50 𝑇 ;𝑇
( ) ( ) 231.39( ) – 275.79(𝑐 ) = 82.078
0 −12 50 :𝑇 :𝑇

30 −20 20 357.868T2 + 462.78T – 193.712 = 0


( ) / ( ) /
30 20 20
T = -1.626 or T = 0.333
OA + t x = OB + t x
T = 0.333
0 30 12 50 −20 20
( )+( ) ( )+( )
0 30 −12 50 20 20 𝑇
sinα= :𝑇
i: 30 =12 50 + −20 20
; 𝑥
77
α= : 0 =36.80
:6 84
………. (i)
Bearing: E36.80S
J: 30 =−12 50 + 20 20
77 77
9 9
𝑡
𝑐𝑜𝑠𝛼:6 84 𝑐𝑜𝑠 6 8:6 84
: 8 794
………. (ii)
T = 0.25h=0.25x 60 =15minutes
77 9 9
(i) = (ii):
:6 84 : 8 794

digitalteachers.co.ug
Alternatively
𝑠𝑖𝑛 8 ; :
Also, 𝑉𝐴𝐵

𝑉𝐴𝐵 48 03𝑘𝑚/
θ = 13.20:
𝐴𝐵
Bearing (50 – 13,2)0S t = 𝑉𝐴 48
𝑥 60 15𝑚𝑖𝑛𝑢𝑡𝑒𝑠
𝐴𝐵

E36.80S or S53.20E

Revision exercise 5
1. At 12:00 noon two ships A and B are 12km apart with B on a bearing of 2500 from A. Ship A
moves at 4km/h on a bearing of 3200. If the maximum speed of B is 7km/h, find the
(a) Direction B should set in order to intercept A [N37.60E]
(b) Time taken for interception to occur. [99minutes]
2. Initially two particles A and B are 48m apart with B due north of A. A has a constant velocity of
(5i + 4j)m/s and B a constant speed of 13m/s. Find the velocity of Y if it is to intercept A and find
the time taken to do so [5i – 12j)m/s, 3s]
3. At 12 noon the position vectors, r and velocity vectors, V of two ships are
5 15 7 9
( ) , ( ) / ( ) , ( ) /
2 10 7 −5
Show that if the ships do not alter their velocities, a collision will occur and find the time at
6
which it occurs and the position vector of its location [12.30pm, (10i + )km]
4. At 11.30 a jumbo jet has position vectors (-100i + 220j)km and it is moving with velocity vectors
(300i + 400J)km/h. At 11:45am a cargo plane has a position vectors (-60i + 355j)km and is moving
with velocity vectors (400i + 300j)km/h. Assuming velocities do not change
(a) Show that the planes will crash
(b) Find the time of the crash. [12.06pm]
(c) Find the position vector of the crash [ (80i + 460j)km]
5. At 2pm the position vectors, r and velocity vectors, V of three ships are as follows
5 + 9 + 18 /
12 + 5 −12 + 6 /
13 − 3 9 + 12 /
Assuming velocities do not change
(a) Show that Ship A and B will collide and fins when and where collision occur.
[2.20pm, (8i + 7j)km]
(b) Find the position vector of C when A and B collide and find how far C is from the point of
collision. [(16i + j)km, 10km]
(c) When the collision occurs, C immediately changes its course but not its speed and streams
directly to the scene. When does C arrive[ 3.00pm]
6. At 12 noon the position vectors, r and velocity vectors, V of three ships A, B and C are as follows
10 5 + 6 9 + 18 /
7 + 20 12 + 6 /
10 + 15 6 + 12 /
Assuming velocities do not change
(a) Show that Ship A and B will collide and fins when and where collision occur.
[1:10pm, (21i + 27j)km]
(b) When the collision occurs, C immediately changes its course but not its speed and streams
directly to the scene. When does C arrive[ 1:300pm]

digitalteachers.co.ug
7. In gulf water, a battleship streaming at 16km/h is5km southwest of a submarine. Find the course
which the submarine should set in order to intercept the battle ship, if its speed is 12km/h.
[N150W]
8. A boy hits a ball at 15m/s in a direction S800W. A girl 45m and S650W from the boy run at 6m/s
to intercept the ball. Find in what direction the girl must rum to intercept the ball as quickly as
possible and how long does it take her. [N24.70E, 2.35s]
9. A helicopter sets off from its bas and flies at 50m/s to intercept a ship which, when the
helicopter sets off, is at a distance of 5km on a bearing 3350 from the base. The ship is travelling
at 10m/s on a bearing 0950. Find the course that the helicopter pilot should set if he is to
intercept the ship as quickly as possible and the time interval between the helicopter taking off
and its reaching the ship. [N150W, 92.2s]
10. A life boat sets out a harbour at 9:10pm to go for assistance of a yatch which is, at the time,
5km due north of the harbour and drifting due west at 8km/h. If the life boat travels at 20km/h
find:
(a) Course the life boat should set so as to reach the yatch as quickly as possible [S23.60W]
(b) Time when the boat arrives [9:27pm]
11. A coast guard vessel wishes to intercept a yatch suspected of smuggling. At 1am the yatch is
10km due east of the coast guard vessel and travelling due north at 15km/h. If the coast guard
vessel travels at 20km/h,
(a) In which direction should it steer in order to intercept the yatch? [N41.40E]
(b) When would this interception occur. [1:45am]
12. The driver of a speed boat travelling at 75km/h wishes to intercept a yatch travelling at 20km/h
in a direction N400E. Initially the speed boat is 10km from the yatch on a bearing S300E. Find
(a) Course the speed boat should set so as to reach the yatch as quickly as possible. [N15.50W]
(b) Time when the interception occurs[9minutes and 7 seconds]
13. A jet fighter travelling at 30km/h wishes to intercept a plane travelling at 20km/h in a course of
2000. Initially the plane is 40km away on a bearing of 110 from the jet fighter. Find
(a) Course the jet fighter should set so as to reach the plane as quickly as possible. [S50E]
(b) Time taken for interception to occur. [48minutes and 24 seconds]
14. A batman hits a ball at 15m/s in a direction S800W. A fielder, 45m and S650W from the batsman,
runs at 6m/s to intercept the ball. Assuming the velocities remain unchanged,
(a) Find what direction the fielder must take to intercept the ball as quickly as possible.
[N24.70E]
(b) How long did it take him. [2.4s}

Thank you
Dr. Bbosa Science

digitalteachers.co.ug
Vertical motion under gravity
When a body is projected vertically downwards, it is subjected to an acceleration of 9.8ms-2. i.e.
a = g = 9.8ms-2

Equations of motion become


𝟏
v = u + gt; h = ut + 𝒈𝒕𝟐 ; v2 = u2 + 2gh
𝟐

When a body is projected vertically upwards, it is subjected to a retardation of 9.8ms-2. i.e.


a = g = 9.8ms-2

Equations of motion become


𝟏
v = u - gt; h = ut - 𝒈𝒕𝟐 ; v2 = u2 - 2gh
𝟐

Maximum /greatest height

When a particle is projected vertically upwards, the final velocity is 0ms-1 at its maximum height

v2 = u2 - 2gh 𝑢2
hmax =
2𝑔
0 = u2 - 2ghmax

Time to reach maximum height

v = u – gt 𝑢
t=
𝑔
0 = u – gt

Time of flight
2𝑢
T=
𝑔

Example 1

A stone is dropped from a point which is 40m above the ground. Find the time taen for the stone to
reach the ground
1
h = ut + 𝑔𝑡 2 40
2
t=√ = 2.857s
1 9.8
2
40 = 0 x t + x 9.8 x t
2

digitalteachers.co.ug
Example 2

A ball is thrown vertically upwards with an initial speed of 30ms-1. Calculate

(i) Time taken to reach thrower


2𝑢 2 𝑥 30
T= = = 6.12s
𝑔 9.8
(ii) maximum height reached
𝑢2 302
hmax = = = 45.92m
2𝑔 2 𝑥 9.8

Example 3

A particle is projected from the ground level vertically upwards with velocity of 19.6ms-1. Find

(i) greatest height reached


𝑢2 19.62
hmax = = = 19.6m
2𝑔 2 𝑥 9.8
(ii) time taken by the particle to reach maximum height
𝑢 19.6
t= = 2s
𝑔 9.8
(iii) Time of flight
T = 2t = 2 x 2 = 4s

Example 4

1. A stone is thrown vertically upwards with velocity 16ms-1 from a point H meters above the
ground level. The stone hits the ground 4 seconds later.
Calculate the
(a) Value of H (03marks)

Using s = ut + ½ at2; s = -H(below point of projection), u = 16ms-1, a = -g, t = 4s


1
-H = 16 x 4 - 𝑥 9.8 𝑥 42
2
H = 14.4m
(b) Velocity of the stone as it hits the ground (02marks)
Using v = u + at; v = -v( below point of projection), a = -g, t = 4s
-v = 16 -9.8 x 4
v = 23.2ms-1
∴ the velocity of the stone as it hits the ground is 23.2ms-1

Example 6

A stone is thrown vertically upwards with a velocity of 21ms-1. Calculate the

(a) Maximum height attained by the stone (03marks)


𝑢2 212
H= = = 225𝑚
2𝑔 2 𝑥 9.8
(b) Time the stone takes to reach the maximum height. (02marks)
𝑢 21
t= = = 2.143𝑠
𝑔 9.8

digitalteachers.co.ug
Example 7

A particle is projected vertically upwards with velocity ums-1. After t seconds another particle is
projected vertically upwards from the same point of projection and with the same initial velocity.
2 𝑢 𝑢2 −(𝑔𝑡)2
Prove that the particles collide after( + ) 𝑠. Hence show that they meet at a height of .
2 𝑔 8𝑔

Solution
2𝑢+√4𝑢2 −8𝑔ℎ 2𝑢−√4𝑢2 −8𝑔ℎ
t1 = time taken by 1st particle − =t
2𝑔 2𝑔

t2 = time taken by 2nd particle


√4𝑢2 − 8𝑔ℎ = 𝑔𝑡 …… (ii)
t1 – t2 = t ………….. (i)
From eqn (ii)
t1 and t2 are roots of the equation 4𝑢2 −(𝑔𝑡)2
ℎ=
1 8𝑔
h = ut - 𝑔𝑡 2 or 𝑔𝑡 2 – 2ut + 2h = 0
2
2𝑢+√4𝑢2 −8𝑔ℎ
𝑡1 = putting eqn. (ii)
2𝑢+√4𝑢2 −8𝑔ℎ 2𝑔
𝑡1 =
2𝑔
2𝑢 + 𝑔𝑡 2 𝑢
2𝑢−√4𝑢2 −8𝑔ℎ
𝑡1 = = ( + )𝑠
𝑡2 = 2𝑔 2 𝑔
2𝑔

Example 8

A particle is projected upwards with velocity of 10ms-1. After 2s another particle is projected
vertically upwards from the same point of projection with the same initial velocity. Find the height
above the level of projection where the particle meet and time taken by the first particle before they
meet.

t1 = time taken by 1st particle


20+√400−8𝑔ℎ 20−√400−8𝑔ℎ
− =t
nd
t2 = time taken by 2 particle 2𝑔 2𝑔

t1 – t2 = t ………….. (i) √400 − 8𝑔ℎ = 𝑔𝑡 …… (ii)

t1 and t2 are roots of the equation From eqn (ii)


1 400−(2 𝑥 9.8)2
h = ut - 𝑔𝑡 2 or 𝑔𝑡 2 – 2ut + 2h = 0 ℎ=
8 𝑥 9.8
= 0.202𝑚
2

20+√400−8𝑔ℎ 2𝑢+√4𝑢2 −8𝑔ℎ


𝑡1 = 𝑡1 = putting eqn. (ii)
2𝑔 2𝑔

20−√400−8𝑔ℎ 2𝑢+𝑔𝑡 2 𝑢 2 𝑥10+9.8 𝑥 2


𝑡2 = 𝑡1 = =( + )= = 2.02 𝑠
2𝑔 2𝑔 2 𝑔 2 𝑥 9.8

Revision exercise

1. A particle is projected vertically upwards with a velocity of 21ms-1. How long it takes to reach a
point 280m below the point of projection. [10s]
2. A particle is projected vertically upwards with a velocity of 17.5ms-1. Find
(i) how high the particle goes. [15.6m]

digitalteachers.co.ug
5 22
(ii) what time elapse before it’s at a height of 10m [ 𝑠; 𝑠
7 7
3. A particle is projected vertically upwards with velocity of 24.5ms-1. Find
(a) when its velocity is 4.9ms-1 [2s]
(b) how long it takes to return to the point of projection.[5s]
(c) at what time it will be 19.6m above the point of projection. [1s and 4s]
4. A particle is projected vertically upwards with a velocity of 35ms-1. find
(a) how long it takes to reach the greatest height.[ 3.57s]
(b) distance it ascends during the 3rd second of motion. [10.5m]
5. Two objects are dropped from a cliff of height H. the second is dropped when the first has
travelled a distanc e d. Prove that the instant when the first object reaches the bottom, the
second is a distance 2√𝐷𝐻 − 𝐷 from the top of the cliff.
4
6. A particle is projected vertically upwards from point O with a speed of 𝑣 𝑚𝑠 −1 . After it has
3
2
travelled a distance of 𝑋 𝑚 above O on its upward motion, another particle is projected
5
vertically upwards from the same point with the same initial speed. Given that the particles
2
collide at a height 𝑋 𝑚 above O, prove that
5
(i) the maximum height, H is given by 8𝑣 2 = 9𝑔𝐻
(ii) when the particle collide 9X = 20H.
7. A particle is projected vertically upwards with velocity um/s. After t seconds another particle is
projected vertically upwards from the same point of projection and with the same initial
1
velocity. Prove that the particles collide each other having a velocity of 𝑔𝑡.
2
8. A particle is projected vertically upwards with velocity 28m/s. After 2s another particle is
projected vertically upwards from the same point of projection and with an initial velocity of
21m/s. Find when the two particles are at the same height and the velocity of each body at that
instant. [4.9s after the first particle is projected, 20m/s, 7.4m/s]
9. A particle is projected vertically upwards with velocity 25m/s. After 4s another particle is
projected vertically upwards from the same point of projection and with the same initial
velocity. Find the time and height when the two particles meet.[4.55s after the first particle is
projected, 12.288m]
10. A stone is dropped from the top of a tower. In the last second of its motion, it falls through a
distance which is a fifth of the height of the tower. Find the height of the tower. [439.6m]

Thank You

Dr. Bbosa Science

digitalteachers.co.ug
Equilibrium of three forces L ami’s theorem
For any three forces acting on a particle in equilibrium where none of them is parallel to each other,
Lami’s theorem is applicable

𝑃 𝑄 𝑅
= =
𝑠𝑖𝑛𝛽 𝑠𝑖𝑛𝛼 𝑠𝑖𝑛𝜃

Example 1

A weight of 49N is suspended by two strings of length 21 cm and 72cm attached to 2 points in a
horizontal line a distance of 75cm apart. Find the tension in the strings so that the particle remain in
equilibrium

Similarly, β = 16.260 and α = 73.740


𝑇1 49
= ;
sin(16.26+90) sin 90

∴ 𝑇1 = 47𝑁
𝑇2 49
= ;
sin(73.74+90) sin 90

By cosine rule: ∴ 𝑇2 = 13.72𝑁


752 = 212 + 722 – 2 x 21 x 72cosθ

θ = 900

Example 2

Mass of 30kg hangs vertically at the end of a light string. If the mass is pulled by a horizontal force P
so that the string makes 300 with the vertical. Find the magnitude of the force and the tension in the
string so that the particle remain in equilibrium.

𝑇 30 𝑥 9.8
= ; T = 339.48N
sin 90 sin 120
𝑃 30 𝑥 9.8
= ; P = 16974N
sin(60+90) sin 120
Example 3

One end of a light inextensible string of length 75cm is fixed to a point on a rigid pole. The particle of
weight 12N is attached to the other end of the string. The particle is held 21cm away from the pole
by a horizontal force, P. Find the magnitude of the force, P and the tension of the string so that the
particle remain in equilibrium
21
θ = cos −1 ( ) = 73.740
75

𝑇 12
=
sin 90 sin(180−73.74)

T = 12.5N
𝑃 12
=
sin(90+73.4) sin(180−73.74)

P = 3.5N

Example 4

A light inextensible string AB whose end A is fixed has end B attached to a particle of mass 5kg. A
force P acting perpendicular to the string is applied on the particle keeping it in equilibrium with the
string inclined at 600 to the vertical. Find the value of P and the tension in the string

𝑇 5 𝑥 98
=
sin(90+60) sin 90

T = 24.5N
5 𝑥 98 𝑃
=
sin 90 sin (90+30)

P = 42.44N

Example 5

A non-uniform beam of mass 5kg rests horizontally in equilibrium supported by two strings attached
to the ends of the beam.

The strings makes 300 and 400 with the horizontal beam as shown above. Find the tension in the
strings.
(↑)T1sin30 + T2sin40 = 5g
Solution
0.8846T2sin30 + T2sin40 = 5 x 9.8

T2 = 45.159N

T1 = 0.8846 x 45.159 = 39.94N

(→) T1cos30 = T2cos40; T1 = 0.8846T2


Example 6

A sphere of weight 20N and radius 15cm rests against a smooth vertical wall. A sphere is supported
in its position by a string of length 10cm attached to a point on the sphere and to a point on the wall
as shown.

(i) calculate the reaction on the sphere due to the wall


(ii) Find the tension in the string

Solution Using Lami’s theory

15 𝑇 20
α = cos −1 ( ) = 53.130 = ; T =25N
25 𝑠𝑖𝑛90 sin(180−53.13)

𝑅 20
= ; R = 15N
sin(90+53.13) sin(180−53.13)

Example 7

A particle of weight 20N is held at equilibrium on a smooth plane inclined at 300 to the horizontal by
a horizontal force P.

(i) Find the value of P and the reaction between the particle and the plane.
(ii) If the force P is removed and a string parallel to the plane is used to hold the particle,
find the tension in the string and the new value of the reaction.

Solution
Alternatively: by resolving forces

At equilibrium parallel to plane forces = 0

𝑃 𝑅 20
Pcos30 + 20sin30 = 0; P = 11.55N
= =
𝑠𝑖𝑛150 𝑠𝑖𝑛90 𝑠𝑖𝑛120
R = 20cos30 + Psin30
R = 23.09N and P = 11.55N
R = 20cos30 + 11.55sin30 = 23.09N
(ii) Alternatively by Lami’s theory
𝑇 𝑅 20
= =
𝑠𝑖𝑛150 𝑠𝑖𝑛120 𝑠𝑖𝑛90

Thank you T = 10N


Dr. Bbosa Science
R = 1.3N

Parallel to the plane T = 20sin30 = 10N

Perpendicular to the plane R = 20cos30 = 13N

Example 8

A light inextensible string passes over a smooth fixed pulley at the top of a smooth plane inclined at
300 to the horizontal. A particle of mass 2kg is attached to one end of the string and rests vertically
in equilibrium when the particle of ma resting on the surface of the plane is attached to the other
end of the string. Find

(i) the normal reaction between m and the plane


(ii) tension in the string and the value of m.

Solution

By resolving forces
Alternatively by using Lami’s theorem

For 2kg mass: T – 2 x 9.8 = 0; T = 19.62N

Parallel to the plane For 2kg mass: T – 2 x 9.8 = 0; T = 19.62N


𝑇 𝑚𝑔 𝑅
T – mgsin30 = 0; m = 4kg = =
𝑠𝑖𝑛150 𝑠𝑖𝑛90 𝑠𝑖𝑛120

Perpendicular to the plane 19.62 𝑚𝑔 𝑅


= =
𝑠𝑖𝑛150 𝑠𝑖𝑛90 𝑠𝑖𝑛120
R = mgco30 m = 4kg and R = 33.98N
R = 4 x 98cos30 = 33.98
Revision exercise
1. A particle P of mass 2kg is suspended from a fixed point O by means of a light inextensible
string. The string is taut and makes an angle of 300 with the downward vertical through O and a
particle is held in equilibrium by means of a horizontal force of magnitude F acting on the
particle. Find the value of F and the tension in the string [F = 11.3161, T = 22.6321N]
2. A particle of mass 3kg lies on a smooth plane inclined at angle θ to the horizontal, where
3
tanθ = . The particle is held in equilibrium by horizontal force of magnitude FN. The line of
4
action of this force is the same vertical plane as a line of greatest slope of inclined plane. Find
the value of F. [22.05N]
3. The diagram below shows a body of weight 10N supported in equilibrium by two light
inextensible strings. The tension in the strings are 7N and T and the angle the string makes with
the upward vertical are 600 and θ respectively.

Find T and θ. [T = 8.9N, θ = 430]


4. A particle of weight 8N is attached to a point B by a light inextensible string AB. It hangs in
equilibrium with point A fixed and AB at an angle of 300 to the downward vertical. A force F at B
acting at right angles to AB, keeps the particle in equilibrium. Find the magnitude of force F and
the tension in the string. [4N, 4√3𝑁]
5. The diagram shows a light inextensible string with one end fixed at A and a mass of 5kg
suspended at the other end.

The mass is held in equilibrium at an angle θ to the downward vertical by a horizontal force P.
Find the value of θ, P and the tension in the string [θ = 36.90, P = 36.75N, T = 61.25N)
6. A sphere of mass 5kg and radius 63cm rests against a smooth vertical wall. A sphere is
supported in its position by a string of length 24cm attached to a point on the sphere an to a
point on the wall as shown.

Find the tension in the string. [71.05N]


7. A particle whose weight is 50N is suspended by a light string which is 350 to the vertical under
the action of a horizontal force F. Find the force F and the tension in the string. [35.0N, 61.0N]
8. A particle of weight w rests on a smooth plane which inclined at 400 to horizontal. The particle is
prevented from slipping by a force of 50.0N acting parallel to the plane and up a line of greatest
slope. Calculate w and reaction due to the plane. [77.8N, 59.6N]
9. A mass of 2kg is suspended by two light inextensible strings. One making an angle of 600 with
the upward vertical and the other 300with the upward vertical. Find the tension in each string.
[9.8N, 17.0N]
10. A heavy uniform rod of weight W is hung from a point by two equal strings, one attached to
each end of the rod. A body of weight w is hang half-way between A and the center of the rod.
2𝑊+3𝑤
Prove that the ratio of tension in the string is .
2𝑊+𝑤
11. A non-uniform beam AB of length 8m and its weight 10N acts from a point G between A and B
such that AG = 6m. The beam is supported horizontally by strings attached to A and B. The
string attached to A makes an angle of 300 with AB. Find the angle that the string attached to B
makes with AB and find the tension in the strings. [600, 5N, 8.66N]
12. A light inextensible string of length 40cm has its upper end fixed to a point A and carries a mass
of 2kg at its lower end. A horizontal force applied to the mass keeps it in equilibrium, 20cm
from the vertical through A. Find the magnitude of this horizontal force and the tension in the
string. [11.3N, 22.6N]
13. The diagram shows a body of mass 5kg supported by two light inextensible strings, the other
ends of which are attached to two points A and B on same level as each other end 7m apart.

The body rests in equilibrium at 3m vertically below AB. If angle CBA = 450, find T1 and T2 the
tensions in the strings. [35N, 28√2𝑁]
14. The diagram shows a body of weight 20N supported by two light inextensible strings of length
0.6m and 0.8m from two points 1m apart on a horizontal beam.

The body rests in equilibrium, find T1 and T2 the tensions in the strings. [16N, 12N]
15. A light inextensible string of length 50cm has its upper end fixed at point A and carries a particle
of 8kg at its lower end. A horizontal force P applied to the particle in equilibrium 30cm from the
vertical through A, find the magnitude of P and the tension in the string. [58.8N, 98N}
16. A article is in equilibrium under the action of forces 4N due north, 8N due west, 5√2N south
east and P, find the magnitude and direction of P. [3.16N, N71.60E]
17. A force P holds a particle of mass mkg in equilibrium on a smooth plane which is inclined at 300
to the horizontal.
If P makes an angle β with the plane, find β when R the normal reaction between the particle
and the plane is 15mg [51.70]
18. The diagram below shows masses of 8kg and 6kg lying on smooth planes of inclination θ and β
respectively

Light inextensible strings attached to these masses pass along the line of greatest slopes over
smooth pulleys and are connected to 4kg mass hanging freely. The strings both make an angle
of 600 with the upward vertical as shown above. If the system rest in equilibrium find θ and β.
[θ = 300 and β 41.80]

19. The diagram below shows masses A and B each lying on smooth planes of inclination300.

Light inextensible strings attached to A and B pass along the lines of greatest slopes, over
smooth pulleys and are connected to a third mass C hanging freely. The strings make angles of θ
and β with the upward vertical as shown above. If A, B and C have masses 2m, m, and m
respectively and the system rests in equilibrium show that sinθ = 2sinβ and cosβ +2cosθ =2.
Hence find θ and β. [29.00, 75.50]
Equilibrium of forces
several forces acting on a particle are said to be in equilibrium when the resultant force is equal to
zero
0
i.e. 𝐹𝑅 = ( )
0
Example 1

For the following set of forces in equilibrium find the values of a and b in each case

(i) (6i + 4j)N, (-2i -5j)N, (ai + bj)N


6 −2 𝑎 0
( )+( )+( )=( )
4 −5 𝑏 0
6 – 2 + a = 0 => a = -4
4 – 5 + b = 0 => b = 1
(ii) (5i + aj + ck)N, (bi – 6j – k)N, and (-3i + 2j + ck)
5 𝑏 −3 0 5 + b – 3 = 0 => b = -2
(𝑎) + (−6) + ( 2 ) = (0)
𝑐 −1 𝑐 0 a – 6 +2 = 0 => a = 4

2c – 1 = 0 => c = 0.5

Example 2

In the diagram below, the particle is in equilibrium, find the values of P and Q.

Solution
0 + Pcos30 + 0 -4√3 = 0; => P = 8N
0 𝑃𝑐𝑜𝑠30 0 0
( )+( ) + ( ) + (−4√3) = ( )
𝑄 𝑄𝑐𝑜𝑠30 −6 0 0 Q + Qcos 30 – 6 + 0 = 0; => Q = 2N
Example 3

Diagram below shows three coplanar forces of magnitude 2N, 3N and PN all acting at point O in the
direction shown. Given that the forces are in equilibrium, Find the value of P

-Psinθ + 2sin60 = 0
2𝑠𝑖𝑛60
sin θ = ………………. (ii)
𝑃

Eqn. (i) and (ii)


Solution
2𝑠𝑖𝑛60
θ = tan−1 ( )= 23.413
-Pcos θ + 2cos 60 + 3 = 0 …. (i) 2𝑐𝑜𝑠60+3

2sin 60
cos θ =
2𝑐𝑜𝑠60+3 from (ii) P = =4.3589N
sin 23.413
𝑃

Revision Exercise

1.

(i) The diagram above shows three coplanar forces in equilibrium. Find the value of P and Q.
(ii) If the direction of Q s now reversed, find the magnitude and direction of the resultant
[(i) 24.5N, 22.6N; (ii) 45.2N]

2. Forces F1 = (-3i + 7j)N, F2= (i – j)N and F3 =(pi +qj) act on a particle.

(i) If the particle is in equilibrium, find the vlues of p and q. [p = 2, q= -6]

(ii) Find the magnitude and direction of the resultant of F1 and F2 . [6.3246N, 71.570]

3. Forces of 6N, 5N, 8N, 5N and 9N act on a particles in the direction N300E, N300W, S500E, N600W,
N800E and S400W respectively. Find the additional force that will keep the system of force in
equilibrium. [5.358N at 68.920 above the positive axis]

4. Forces of 7N, 2N, 4N and 5N act on a particle in directions of 0600, 1600 2000 and 3150
respectively. Find the additional force that will keep the system of forces in equilibrium.
[2.3125N at 37.180 below the negative axis]

5. Forces of 2N, 1N, 3N and 4N act on a particle in the direction 00, 900, 2700 and 3300 respectively.
Find the additional force that will keep the system of forces in equilibrium. [6.8N at 360 above the
negative axis]
6. Forces of 6N, 5N, 7N, 4N, 3√2 𝑁 and 7√2N act in direction AB, CB, CD, DA, CA and DB respectively
on a square ABCD. Find the additional force that will keep the system of forces in equilibrium.
[19.2N at 810 above the negative axis]

7. Forces 8N, 7N, 6N, 4N, 7N and 6N act along the sides of a regular hexagon ABCDEF in direction AB,
CB, CD, DE, EF and FA respectively. Find the additional force that will keep the system of forces in
equilibrium. [ 12.49N at 760 above AB]

Thank you
Dr. Bbosa Science
A-level math paper 2: Statistics
This a branch of mathematics dealing with collection, presentation, analysis and interpretation of
data

Types of data
(a) Discrete data
Its information collected by counting and usually takes integral values that do not lie within a
given range
(b) Continuous data
It is information that takes values within a given range

Discrete or ungrouped data


Measures of central tendency

These are values of the distribution that tend to locate the central values. They include
mean, median and mode
(i) Mean or average of a sample
∑𝑥
It is denoted by 𝑥̅ and defined as 𝑥̅ = ; where x is the variable given and n is
𝑛
the number of variable
If assumed mean (working mean) a is given then
∑𝑑
𝑥̅ = 𝐴 + ; where d = x – A
𝑛
∑ 𝑓𝑥 ∑ 𝑓𝑑
If the frequency, f, s given then 𝑥̅ = ∑𝑓
or 𝑥̅ = 𝐴 + ∑𝑓
(ii) Mode
This is the value of the distribution that appears most
(iii) Median
This is the middle value of the distribution obtained after the values have been
arranged either in ascending of descending order.
𝑁 𝑡ℎ
Median = ( ) value.
2

Example 1

Given the following sets of values

2, 1, 3, 4, 5, 6, 7, 8, 9, 10, 3, 4, 6, 8, 9, 6, 3, 2
(a) Form a frequency table of ungrouped data
x f fx cf
1 1 1 1
2 2 4 3
3 3 9 6
4 2 8 8
5 1 5 9
6 4 24 13
7 2 14 15
8 2 16 17
9 2 18 19
10 1 10 20
∑ 𝑓 = 20 ∑ 𝑓𝑥 = 109

(b) Use the table to find the mean and mode


∑ 𝑓𝑥 109
Mean, 𝑥̅ = ∑𝑓
= = 5.45
20
Mode = 6 (the value that appear most
(c) Find the median value
𝑁 𝑡ℎ 20 𝑡ℎ
Median = ( ) value = ( ) = 10th value from cumulative frequency, cf ; median = 6
2 2

Example 2

Given the information below

x 10 11 12 13 14 15 16 17 18
f 4 2 6 3 7 2 1 2 2
Find

(a) Mean
(b) modal value
(c) median

Solution

x f fx Cf
10 4 40 4
11 2 22 6
12 6 72 12
13 3 39 15
14 7 98 22
15 2 30 24
16 1 16 25
17 2 34 27
18 2 36 29
∑ 𝑓 = 29 ∑ 𝑓𝑥 = 387
∑ 𝑓𝑥 387
(a) Mean, 𝑥̅ = ∑𝑓
= = 13.34
29
(b) Modal value = 14 (appear most)
𝑁 𝑡ℎ 29 𝑡ℎ
(c) Median = ( ) value = ( ) = 14.5th value from cumulative frequency, cf ;
2 2
median = 13

Example 3

The following are marks obtained by 30 students in a mathematic test marked out of 10

Mark (x) 0 1 2 3 4 5 6 7 8 9 10
Frequency (f) 2 2 3 2 3 4 5 2 1 3 3

(a) Find the mean using a working mean of 5


(b) Find the modal mark and he median

Solution

x f D= (x-5) fd fc
0 2 -5 -10 2
1 2 -4 -8 4
2 3 -3 -9 7
3 2 -2 -4 9
4 3 -1 -3 12
5 4 0 0 16
6 5 1 5 21
7 2 2 4 23
8 1 3 3 24
9 3 4 12 27
10 3 5 15 30
∑ 𝑓 = 30 ∑ 𝑓𝑥 = 5
∑ 𝑓𝑑 5
(a) 𝑥̅ = 𝐴 + ∑𝑓
=5+ = 5.167
30
(b) Modal value = 6
𝑁 𝑡ℎ 30 𝑡ℎ
(c) Median = ( ) value = ( ) = 15th value from cumulative frequency, cf ; median = 5
2 2

Measures of dispersion
This is a measure of how the observations are spread out from the mean

(a) Range
It is the difference between the largest value and the smallest value.

Example 4

Find the range of the following set of values

2, 1, 3, 4, 5, 6, 7, 8, 9, 10, 3, 4, 6, 8, 9, 6, 3, 2

Range = 10 – 1 = 9

(b) Variance of a sample


The variance of x is denoted by Var(X) is defined as
∑ 𝑥2 ∑𝑥 2
Var(X) = − ( )
𝑛 𝑛
∑ 𝑥2
Var(X) = − 𝑥̅ 2
𝑛
If the frequency is given then
∑ 𝑓𝑥 2 ∑ 𝑓𝑥 2
Var(x) = ∑𝑓
− (∑ )
𝑓
∑ 𝑓𝑥 2
= ∑𝑓
− 𝑥̅ 2

(c) Standard deviation


S.d = √Var(X)
∑ 𝑓𝑥 2 ∑ 𝑓𝑥 2
=√ ∑𝑓
− (∑ )
𝑓
∑ 𝑓𝑥 2
=√ ∑𝑓
− 𝑥̅ 2

Example 5

Find the variance and standard deviation for the following data; 5, 8, 12, 13, 15

Solution

x x2 ∑ 𝑥2 ∑𝑥 2 627 53 2
Var(X) = − ( ) = −( )
𝑛 𝑛 5 5
5 25
8 64 = 125.4 - 112.36 = 13.04
12 144
13 169 S.d = √Var(X) = √13.04 =3.611
15 225
∑ 𝑥 =53 ∑ 𝑥 2 =627
Example 5

The frequency distribution table shows the marks of some student from a certain school

x 45 63 65 66 70 72 75 80 88
f 3 5 6 4 6 2 1 2 1
Calculate standard deviation
Solution
x f fx fx2
∑ 𝑓𝑥 2 ∑ 𝑓𝑥 2
45 3 135 6075 S.d = √ ∑𝑓
− (∑ )
𝑓
63 5 315 19845
134631 1991 2
65 6 390 25350 =√ − ( )
30 30
66 4 264 17424
134631 1991 2
70 6 420 29400 =√ − ( )
30 30
72 2 144 10368
=√4487.7 − 4404.53
75 1 75 5625
= 9.12
80 2 160 12800
88 1 88 7744
∑ 𝑓 =30 1991 ∑ 𝑓𝑥 2 =134631
Using assumed mean to get variance and standard deviation
∑ 𝑓𝑑 2 ∑ 𝑓𝑑 2
Var (X) = ∑𝑓
− (∑ )
𝑓

∑ 𝑓𝑑 2 ∑ 𝑓𝑑 2
s.d = √ ∑𝑓
− (∑ )
𝑓

Example 6

The frequency distribution table shows the heights of some students at a certain shool

Height 154 155 160 164 171 180


Frequency 4 6 8 5 4 3
Determine the variance and standard deviation of the data using a working mean of 160
Solution
x f d = x - A fd fd2 ∑ 𝑓𝑑 2 ∑ 𝑓𝑑 2
154 4 -6 -24 144 Var (X) = ∑𝑓
− (∑ )
𝑓
155 6 -5 -30 150
2058 70 2
160 8 0 0 0 = − ( )
30 30
164 5 4 20 80
171 4 11 44 484 = 63.156
180 3 20 60 1200
2
s.d = √𝑉𝑎𝑟(𝑥) = √63.156 =7.95
∑ 𝑓 =30 ∑ 𝑓𝑑 =70 ∑ 𝑓𝑑 =2058

Quartiles
A quartile is a value that divides given values into four equal parts

q1 is the lower quartile and is defined by

1 𝑡ℎ
q1 = ( 𝑁) value where N is the sum of all the variables
4

q3 is the upper quartile and is defined by

3 𝑡ℎ
q3 = ( 𝑁) value where N is the sum of all the variables
4

Percentiles
A percentile is a value that divides given values into 100 pats.

P10 is the 10th percentile and is defined as

10 𝑡ℎ
P10 = ( 𝑁) value where N is the sum of all the variables
100

P90 is the 10th percentile and is defined as

90 𝑡ℎ
P90 = ( 𝑁) value where N is the sum of all the variables
100
Deciles
A decile is a value that divides given values into 10 pats.

D5 is the 5th decile and is defined as

5 𝑡ℎ
D5 = ( 𝑁) value where N is the sum of all the variables
10

Example 7

The table below shows the marks obtained by 20 students in a mathematics test marked out of 20

Marks 10 11 12 13 14 15 16 17 18 19 20
Number of students 1 2 2 2 2 4 2 1 2 1 1
Find:

(a) Mean mark


(b) Standard deviation
(c) 60th percentile
(d) Interquartile range

Solution

x f cf fx fx2 ∑ 𝑓𝑥 294
(a) Mean,𝑥̅ = ∑𝑓
= = 14.7
10 1 1 10 100 20

11 2 3 22 242 ∑ 𝑓𝑑 2
(b) s.d = √ ∑𝑓
− 𝑥̅ 2
12 2 5 24 288
4470
13 2 7 26 338 =√ − 14.72
20
14 2 9 28 392
= 2.722
15 4 13 60 900 th
(c) 60 percentile
16 2 15 32 512
17 1 16 17 289 60 𝑡ℎ
=( 𝑥 20) value
18 2 18 36 648 100

19 1 19 19 361 = 12th value from cf =15


20 1 20 20 400
∑ 𝑓 =20 2
∑ 𝑓𝑥 =294 ∑ 𝑓𝑥 =4470
1 𝑡ℎ
(d) q1 = ( 𝑥 20) value = 5th value from cf = 12
4

3 𝑡ℎ
Q3 = ( 𝑥 20) value = 15th value from cf = 16
4

Interquartile range = 16 -12 = 4

Example 8

Given the following scores

8, 6, 8, 9, 10, 6, 4, 5, 6, 4, 4, 6, 8, 7, 10, 8, 6, 11, 12,8

(a) Form a frequency distribution table of ungrouped data.


(b) Find the standard deviation
(c) Calculate semi-quartile range
(d) Determine the range of 45th and 90th percentile.

Solution

x f cf fx fx2
∑ 𝑓𝑑 2 ∑ 𝑓𝑑 2
4 3 3 12 48 (b) s.d = √ − (∑ )
∑𝑓 𝑓
5 1 4 5 25
6 5 9 30 180 1168 146 2
=√ − ( ) =2.26
7 1 11 7 49 20 20

8 5 16 40 320 1 𝑡ℎ
(c) q1 = ( 𝑥 20) value
9 1 17 9 81 4
10 2 19 20 200 = 5th value from cf = 6
11 1 20 11 121 3 𝑡ℎ
q3 = ( 𝑥 20) value
12 1 21 12 144 4
∑ 𝑓 =20 ∑ 𝑓𝑥 =146 ∑ 𝑓𝑥 2 =1168 = 15th value, from cf = 8
8−6
Semi-interquartile range = =1
2 (d)
45 𝑡ℎ
(d) 45th percentile = ( 𝑥 20) value = 9th value from cf = 6
10

90 𝑡ℎ
90th percentile = ( 𝑥 20) value = 18th value from cf = 10
100

The range between the 90th percentile and 45th percentile = 10 – 6 = 4

Revision exercise 1
1. The data below represents the length of leaves in cm: 4.5 4.4, 6.2, 9.4, 8.2, 12.6, 10.0, 8.8, 3.8 and
13.6. find the;
(a) Mean
(b) Standard deviation
2. The concentration in m per litre of a trace element in 7 randomly chosen samples of water from
spring wells were: 240.8, 237.3, 236.6, 2333.9 and 232.5. Determine the mean and the variance
of the concentration of the trace elements per litre.
3. The table below shows the length of flowers from a certain plant to the nearest 0.5cm.
Length (cm) 7.5 8.0 8.5 9.0 9.5 10.0 10.5 11.0
Number of flowers 4 9 11 8 10 7 2 3
Find the:
(a) Mean
(b) Mode
(c) The median
(d) Standard deviation
4. The marks scored by 11 students in a test are:52, 61, 78, 49, 47, 79, 54, 58, 62, 73, 72
Find;
(a) Median,
(b) Mean,
(c) Interquartile range
(d) Semi-quartile range
Solutions to revision exercise 1

1. The data below represents the length of leaves in cm: 4.5 4.4, 6.2, 9.4, 8.2, 12.6, 10.0, 8.8, 3.8 and
13.6. find the;
(a) Mean
(b) Standard deviation

Solution

x x2 ∑𝑥 80.7
Mean, 𝑥̅ = =
= 8.07
4.5 20.25 10𝑛

4.4 19.36 ∑ 𝑥2 750.77


6.2 38.44 s.d = √ − 𝑥̅ 2 = √ − (8.07)2 =3.155
𝑛 10
9.2 84.64
8.2 67.24
12 144
10 100
8.8 77.44
3.8 14.44
13.6 184.96
∑ 𝑥 =80.7 ∑ 𝑥 2 =750.77
2. The concentration in m per litre of a trace element in 7 randomly chosen samples of water from
spring wells were: 240.8, 237.3, 236.6, 2333.9 and 232.5. Determine the mean and the variance
of the concentration of the trace elements per litre.
Solution
∑𝑥 1652
x x2 Mean, 𝑥̅ = = = 236
𝑛 7
240.8 57984.64 ∑ 𝑥2
Var(x) = − 𝑥̅ 2
237.3 56311.29 𝑛
236.7 56026.89 389917.5
= − 2362
234.2 54849.64 7

236.6 55979.56 = 55,702.5 – 55,696


233.9 54709.21
= 6.5
232.5 54056.25
2
∑ 𝑥 =1652 ∑ 𝑥 =389917.5

3. The table below shows the length of flowers from a certain plant to the nearest 0.5cm.
Length (cm) 7.5 8.0 8.5 9.0 9.5 10.0 10.5 11.0
Number of flowers 4 9 11 8 10 7 2 3
Find the:
(a) Mean
(b) Mode
(c) The median
(d) Standard deviation
Solution

x f fx fx2 cf ∑ 𝑓𝑥
(a)Mean = ∑𝑓
7.5 4 30 225 4
8.0 9 72 576 13 486.2
= = 9.0
8.5 11 93.5 794.75 24 54
9.0 8 72 648 32 (b) mode = 8.5
9.5 10 95 902.5 42
54 𝑡ℎ
10.0 7 70 700 49 (c) Median = ( ) value
2
10.5 2 21 220.5 51
11.0 3 33 363 54 = 27th value
2
∑ 𝑓 =54 ∑ 𝑓𝑥 =486.5 ∑ 𝑓𝑥 =4429.75 From cf the 27th value = 9

∑ 𝑥2 4429.75
(d) s.d = √ ∑ − 𝑥̅ 2 =√ − 𝑥̅ 2 = 1
𝑓 54
4. The marks scored by 11 students in a test are:52, 61, 78, 49, 47, 79, 54, 58, 62, 73, 72
Find;
(a) Median
Arrange values in ascending order
47, 49, 52, 54, 58, 61, 62, 72,73,78,79
(a) Median =𝑚𝑖𝑑𝑑𝑙𝑒 𝑛𝑢𝑚𝑏𝑒𝑟 = 6th value = 61
(b) Mean,
∑ 𝑥 = 52+ 61+78 + 49 + 47 + 79 + 54 + 58 + 62 + 73 + 72 =685
∑𝑥 685
Mean = = = 62.273
𝑛 11
(b) Interquartile range
1 𝑡ℎ
q1 = ( 𝑥 11) value = 2.75th value = 52
4

3 𝑡ℎ
q3 = ( 𝑥 11) value = 8.25th value = 73
4

Interquartile range = 73 -52 = 21

𝐼𝑛𝑡𝑒𝑟𝑞𝑢𝑎𝑟𝑡𝑖𝑙𝑒 𝑟𝑎𝑛𝑔𝑒 21
(c) Semi-quartile range = = = 10.5
2 2
Continuous or grouped data.
This is data whose scores or values are said to be continuous and take interval values
Example 9
Class 20 – 29 30 – 39 40 – 49 50 – 59 60 – 69 70 – 79 80 - 89
Number of 4 5 7 3 6 4 1
students
Draw a frequency table

Terms used
(a) Class: these are limits of distribution. In the table above, the classes are: (20 – 29),
(30 – 39), (40 – 49), (50 – 59), (60 – 69), (70 – 79), (80 – 89).
(b) Class mark (mark)
This is the mid-point value of the class. It is normally denoted by x. in the table
above, the class marks are 24.5, 34.5, 44.5 …..
(c) Class boundary
These are continuous class limits. In the above table the first class boundary is
(20-0.5) – (29 +0.5). In this case, the lower class boundary is 19.5 and upper class
boundary is 29.5

For class interval 2.0 – 2.9, the class boundary is (2.0 -0.05) – (2.9 + 0.05) = 1.95-2.95.

(d) Class width or class interval


This is the width of each class boundary.
It is given by;
Class width = upper class boundary – lower class boundary
In the table above, class width = 29.5 – 19.5 = 10

Solution

Class Frequency Class width Class mark, x Class boundary


20 – 29 4 10 24.5 19.5 - 29.5
30 – 39 5 10 34.5 29.5 - 39.5
40 – 49 7 10 44.5 39.5 - 49.5
50 – 59 3 10 54.5 49.5 - 59.5
60 – 69 6 10 64.5 59.5 - 69.5
70 – 79 4 10 74.5 69.5 - 79.5
80 – 89 1 10 84.5 79.5 - 89.5

Example 10

The data below shows the heights in centimetres of 70 students.

Height 130 – 135 135 – 140 140 – 145 145 – 150 150 – 160 160 – 170 170 – 180
(cm)
Number 10 12 8 9 11 15 5
of
students
Construct a frequency distribution table for the above data
Class Class boundary Class width Class mark, x Frequency, f
130 – 135 130 – 135 5 132.5 10
135 – 140 135 – 140 5 137.5 12
140 – 145 140 – 145 5 142.5 8
145 – 150 145 – 150 5 147.5 9
150 – 160 150 – 160 10 155 11
160 – 170 160 – 170 10 165 15
170 - 180 170 - 180 10 175 5
Example 11

Use the data below to construct a frequency distribution table

Marks 20-<30 30-<40 40-<50 50-<60 60-<70 70-<80 80-<90


Number 10 14 8 18 11 15 5
of
students
Solution

Class Class boundary Class width Class mark, x Frequency, f


20-<30 20 – 30 10 25 10
30-<40 30 – 40 10 35 14
40-<50 40 – 50 10 45 8
50-<60 50 – 60 10 55 18
60-<70 60 – 70 10 65 11
70-<80 70 – 80 10 75 15
80-<90 80 – 90 10 85 5
Example 12

The table below shows the ages of 35 people

Age 0- 5- 10- 15- 20- 30- 40-


Frequency 4 6 3 5 7 2 8
Draw a frequency table for the data

Solution

Class Class boundary Class width Class mark, x Frequency, f


0- 0–5 5 2.5 4
5- 5 – 10 5 7.5 6
10- 10 – 15 5 12.5 3
15- 15 – 20 5 17.5 5
20- 20 – 30 10 25 7
30- 30 – 40 10 35 2
40- 40 – 45 5 42.5 7
Note : the last class width is 5 since it is the most common

Example 13

The table below shows the marks of 40 students

Marks -20 -30 -40 -50 -60 -65 -70


Frequency 8 4 7 10 2 2 7
Draw a frequency table

Solution

Class Class boundary Class width Class mark, x Frequency, f


-20 10 – 20 10 15 8
-30 20 – 30 10 25 4
-40 30 – 40 10 35 7
-50 40 – 50 10 45 10
-60 50 – 60 10 55 2
-65 60 – 65 5 62.5 2
-70 65- 70 5 67.5 7

Note: 1- the first class width is 10 because it is the most common

2- it is also acceptable for the first class to start from zero. i.e. (0 – 20)

Example 14

The data below shows the length in minutes made of different phone calls made by Airtel clients

Length <20 <30 <35 <40 <50 <60


(minutes)
Frequency 4 20 32 42 48 60
Construct a frequency table for the data.

Solution

Class Class boundary Class width Class mark, x Frequency, f


<20 10 – 20 10 15 4
<30 20 – 30 10 25 16
<35 30 – 35 5 32.5 12
<40 35 – 40 5 37.5 10
<50 40 – 50 10 45 6
<60 50 – 60 10 55 2

Measure of central tendency


(a) Mean or average of sample

The mean of grouped data is given by


∑ 𝑓𝑥
𝑥̅ = ∑𝑓
where f = frequency, x is the mid-mark

(b) Median of grouped data

Median of grouped data is defined by


∑𝑓
− 𝑐.𝑓𝑏
2
Median = Lb + ( )𝐶
𝑓

Where Lb = lower class boundary of the median class

C= class width of the median class


f = frequency of the median class

c.fb= cumulative frequency before that of the median class

Example 15

The ages of people in the town are as follows

Age 0-<5 <15 <30 <50 <70 <90


Number of 44 81 105 147 158 160
people
Calculate the

(i) Mean
(ii) Median

Solution

Age x f fx cf Class
boundary
0-<5 2.5 44 110 44 0-<5
5-<15 10 37 370 81 5-<15
15-<30 22.5 24 540 105 15-<30
30-<50 40 42 1680 146 30-<50
50-<70 60 11 660 158 50-<70
70-<90 80 2 160 160 70-<90
∑ 𝑓 =160 ∑ 𝑓𝑥 =3520
∑ 𝑓𝑥 3520
(a) Mean, 𝑥̅ = ∑𝑓
= = 22years
160
∑𝑓
− 𝑐.𝑓𝑏
(b) Median = Lb + ( 2 )𝐶
𝑓
∑𝑓 160
= = 80
2 2
Median class boundary is 5 -<15, f = 37 and C = 10
80−44
∴ Median = 5 + ( ) 𝑥10 = 14.73 years
37

(c) Mode of grouped data


Mode of grouped data with equal class width is defined as
∆1
Mode = Lb + ( )𝐶
∆1 + ∆2
Where
Lb = lower class boundary of modal class
C = Class width of the modal class
∆1 = Modal frequency (pre-modal frequency)
∆2 = Modal frequency (post modal frequency)
Example 16

The table below shows the weight of 250 students at The Science Foundation College
Weight (kg) 44.0 – 48.0 – 52.0 – 56.0 – 60.0 – 64.0 – 68.0 – 72.0 –
47.9 51.9 55.9 59.9 63.9 67.9 71.9 75.9
Frequency 3 17 50 45 46 57 23 9
Find
(i) Average weight
(ii) Median weight
(iii) Modal weight

Solution

Class Class x f fx cf
boundary
44.0 - 47.9 43.95 - 47.95 45.95 3 137.85 3
48.0 - 51.9 47.95 - 51.95 49.95 17 849.15 20
52.0 - 55.9 51.95 - 55.95 53.95 50 2697.5 70
56.0 -59.9 55.95 - 59.95 57.95 45 2607.75 115
60.0 - 63.9 59.95 - 63.95 61.95 46 2849.7 161
64.0 - 67.9 63.95 - 67.95 65.95 57 3759.15 218
68.0 - 71.9 67.95 - 71.95 69.95 23 1608.85 241
72.0 - 75.9 71.95 - 75.95 73.95 9 665.55 250
250 15175.5
∑ 𝑓𝑥 15175.5
(i) Mean, 𝑥̅ = ∑𝑓
= = 60.702𝑘𝑔
250
∑𝑓
− 𝑐.𝑓𝑏
2
(ii) Median = Lb + ( )𝐶
𝑓
∑𝑓 250
= = 125
2 2
Median class boundary is 59.95 - 63.95, f = 46 and C = 4
125−115
∴ Median = 59.95 + ( ) 𝑥 4 = 60.82kg
46
(iii) Modal class boundary is 63.95 – 67.95, since 57 is the highest frequency and C = 4
∆1 = 57 − 46 = 11 and ∆2 = 57 − 23 = 34
11
𝑀𝑜𝑑𝑒 = 63.95 + ( ) 𝑥 4 = 64.93𝑘𝑔
11+34
(d) Mode of grouped data with unequal class width
Mode of grouped data with unequal class width defined as
∆𝑓 .𝑑1
Mode = Lb + ( )𝐶
∆𝑓 .𝑑1 + ∆𝑓 .𝑑2
Modal class is determined from the highest frequency density
𝑓𝑟𝑒𝑞𝑢𝑒𝑛𝑐𝑦
Frequency density =
𝑐𝑙𝑎𝑠𝑠 𝑤𝑖𝑑𝑡ℎ
Where
Lb = lower class boundary of modal class
C = Class width of the modal class
∆𝑓 . 𝑑1 = Modal frequency density (pre-modal frequency)
∆𝑓 . 𝑑2 = Modal frequency density (post modal frequency)
Example 17

Given the data below

Marks (x) 10-19 20-24 25-34 35-39 40-54 55-64 65-79


Frequency (f) 4 6 7 3 8 6 6
Find the mode

Solution

Class Class f Frequency ∆𝑓 .𝑑1


boundary width density Mode = Lb + ( )𝐶
∆𝑓 .𝑑1 + ∆𝑓 .𝑑2
9.5 - 19.5 10 4 0.4
1.2−0.4
19.5 - 24.5 5 6 1.2 = 19.5 + ((1.2−0.4)+ (1.2−0.7)) 𝑥 5
24.5 - 34.5 10 7 0.7
= 22.58
34.5 - 39.5 5 3 0.6
39.5 - 54.5 15 8 0.53
54.5 - 64.5 10 6 0.6
64.5 - 79.5 15 6 0.4
∑ 𝑓 =40

Example 18

The table shows the weights (kg) of 150 patients who visited a certain health centre.

Weight (kg) 0–9 10 – 19 20 – 29 30 – 39 40 – 49 50 – 59 60 - 69


Frequency (f 30 16 24 32 28 12 8
Calculate

(a) Mean
(b) Mode
(c) Median

Class Class class f fx cf


boundary mark (x)
0-9 0 - 9.5 4.5 30 135 30
10 - 19 9.5 - 19.5 14.5 16 232 46
20 - 29 19.5 - 29.5 24.5 24 588 70
30 - 39 29.5 -39.5 34.5 32 1104 102
40 - 49 39.5 - 49.5 44.5 28 1246 130
50 - 59 49.5 - 59.5 54.5 12 654 142
60 - 69 59.5 - 69.5 64.5 8 516 150
∑ 𝑓 =150 ∑ 𝑓𝑥 =4475
∑ 𝑓𝑥 4475
(a) Mean 𝑥̅ = ∑𝑓
= = 29.83kg
150
∆1
(b) Mode = Lb + ( )𝐶
∆1 + ∆2
Modal class boundary is 29.5 – 39.5, since 32 is the highest frequency and C = 10
∆1 = 32 − 24 = 8 and ∆2 = 32 − 28 = 4
8
𝑀𝑜𝑑𝑒 = 29.5 + ( ) 𝑥 10 = 36.17𝑘𝑔
8+4
∑𝑓
− 𝑐.𝑓𝑏
(c) Median = Lb + ( 2 )𝐶
𝑓
∑𝑓 150
= = 75
2 2
Median class boundary is 29.5 -39.5, f = 32 and C = 10
75 −70
∴ Median = 29.5 + ( ) 𝑥 10 = 30.06kg
32

Measure of dispersion of grouped data


(i) Variance of the sample
The variance of grouped data denoted by Var(x) is defined as
∑ 𝑓𝑥 2 ∑ 𝑓𝑥 2
Var(x) = ∑𝑓
− (∑ )
𝑓
Or
∑ 𝑓𝑥 2
Var(x) = ∑𝑓
− 𝑥̅ 2
Or
∑ 𝑓𝑑 2 ∑ 𝑓𝑑 2
Var(x) = ∑𝑓
− ( ∑ ) where d = x – A and A = suggested mean
𝑓
(ii) s.d = √𝑉𝑎𝑟(𝑥)

Example 19

The table below shows the number of crimes committed by students

Number of 5-<10 10-<20 20-<30 30-<50 50-<100


crimes
Number of 10 15 25 40 26
students
Calculate the variance and standard deviation for the number of crimes committed

Solution

Number of x f fx fx2
crime
5-<10 7.5 10 75 562.5
10-<20 15 15 225 3375
20-<30 25 25 625 15625
30-<50 40 40 1600 64000
50-<100 75 25 1875 140625
∑ 𝑓 =115 2
∑ 𝑓𝑥 =4400 ∑ 𝑓𝑥 =224187.5
∑ 𝑓𝑥 2 ∑ 𝑓𝑥 2 224187.5 4400 2
Var(x) =
∑𝑓
− (∑ ) = − ( ) = 485.56
𝑓 115 115

s.d = √Var(x) = √485.56 = 22.04


Example 20

The table below shows the weight of 250 students at a certain day school

Weight 44.0 – 48.0 – 52.0 – 56.0 – 60.0 – 64.0 – 68.0 – 72.0 –


(kg) 47.9 51.9 55.9 59.9 63.9 67.9 71.9 75.9
Frequency 3 17 50 45 46 57 23 9
Using assumed mean of 57.95, find

(a) average weight


(b) variance
(c) standard deviation

Solution

weight x f d = x - A fd fd2
43.95-47.95 45.95 3 -12 -36 432
47.95-51.95 49.95 17 -8 -136 1088
51.95-55.95 53.95 50 -4 -200 800
55.95-59.95 57.95 45 0 0 0
59.95-63.95 61.95 46 4 184 736
63.95-67.95 65.95 57 8 456 3648
67.95-71.95 69.95 23 12 276 3312
71.95-75.95 73.95 9 16 144 2304
∑ 𝑓 =250 2
∑ 𝑓𝑑 =688 ∑ 𝑓𝑑 =12320
∑ 𝑓𝑑 688
(a) 𝑥̅ = 𝐴 + ∑𝑓
= 57.95 + = 60.702𝑘𝑔
250
∑ 𝑓𝑑 2 ∑ 𝑓𝑑 2
(b) Var(x) = ∑𝑓
− (∑ )
𝑓
12320 688 2
= − ( ) = 41.71𝑘𝑔
250 250
(c) S.d = √𝑉𝑎𝑟(𝑥) = √41.71 = 6.46𝑘𝑔

Percentile and quartile of grouped data.


Percentile

This a value that divides a given distribution into 100 equal parts

The 60th percentile for instance is defined as


60
∑ 𝑓−𝑐.𝑓𝑏
P60 = 𝐿𝑏 + (100 )𝐶
𝑓

Where

Lb = lower class boundary of the 60th class

C= class width

F = frequency of the 60th class

c.fb = cumulative frequency before that one of the 60th class


Quartiles

This a value that divides a given distribution into 4 equal parts

The lower quartile denoted q1 for instance is defined as


1
∑ 𝑓−𝑐.𝑓𝑏
q1 = 𝐿𝑏 + (4 )𝐶
𝑓

Where

Lb = lower class boundary of the q1 class

C= class width

f = frequency of the q1 class

c.fb = cumulative frequency before that one of the q1 class

The upper quartile denoted q1 for instance is defined as


1
∑ 𝑓−𝑐.𝑓𝑏
q3 = 𝐿𝑏 + (4 )𝐶
𝑓

Where

Lb = lower class boundary of the q3 class

C= class width

f = frequency of the q3 class

c.fb = cumulative frequency before that one of the q3 class

Interquartile range = q3 –q1


𝑞3 − 𝑞1
Semi-interquartile range =
2

Example 21

The following table shows the marks obtained by to students in a physics test marked out of 100

Marks (%) 20 -29 30-39 40-49 50-59 60-69 70-79 80-89 90-100
Number of students 4 6 2 5 7 8 5 2
Find

(a) Mean
(b) Standard deviation
(c) Median and mode
(d) Semi-interquartile range
(e) 40th and 85th percentile range
Solution

Class boundary x f fx fx2 cf


19.5-29.5 24.5 4 98 2401 4
29.5-39.5 34.5 6 207 7141.5 10
39.5-49.5 44.5 2 89 3960.5 12
49.5-59.5 54.5 5 272.5 14851.25 17
59.5-69.5 64.5 7 451.5 29121.75 24
69.5-79.5 74.5 8 596 44402 32
79.5-89.5 84.5 5 422.5 35701.25 37
89.5-99.5 94.5 3 283.5 26790.75 40
∑ 𝑓 =40 ∑ 𝑓𝑥 =2420 ∑ 𝑓𝑥 2 =164370
∑ 𝑓𝑥 2420
(i) Mean 𝑥̅ = ∑𝑓
= = 60.5%
40
∑ 𝑓𝑥 2 ∑ 𝑓𝑥 2 164370 2420 2
(ii) S.d = √ ∑𝑓
− (∑ ) =√ −( ) = 21.19%
𝑓 40 40
∑𝑓
− 𝑐.𝑓𝑏
2
(iii) Median = Lb + ( )𝐶
𝑓
∑𝑓 40
= = 20
2 2
Median class boundary is 59.5-69.5, f = 7 and C = 10
20 −17
∴ Median = 59.5 + ( ) 𝑥 10 = 63.786%
7
∆1
Mode = Lb + ( )𝐶
∆1 + ∆2
Modal class boundary is 69.5-79.5, since 8 is the highest frequency and C = 10
∆1 = 8 − 7 = 1 and ∆2 = 8 − 5 = 3
1
𝑀𝑜𝑑𝑒 = 69.5 + ( ) 𝑥 10 = 72%
1+3
1
∑ 𝑓−𝑐.𝑓𝑏
4
(iv) q1 = 𝐿𝑏 + ( )𝐶
𝑓
∑𝑓 40
= = 10 , Lb =29.5, f= 6, C = 10
4 4
10−4
q1 = 29.5 + ( ) 𝑥10 = 39.5
6
3
∑ 𝑓−𝑐.𝑓𝑏
4
q3 = 𝐿𝑏 + ( )𝐶
𝑓
3∑𝑓 3 𝑥 40
= = 30 , Lb =69.5, f= 8, C = 10
4 4
30−24
q1 = 69.5 + ( ) 𝑥10 = 77%
8
𝑞3 − 𝑞1 77−39.5
Semi-quartile range = = = 18.75%
2 2
40
∑ 𝑓−𝑐.𝑓𝑏
100
(v) P40 = 𝐿𝑏 + ( )𝐶
𝑓
40 ∑ 𝑓 40 𝑥 40
= = 16 , Lb =49.5, f= 5, C = 10
100 100
16−12
P40 = 49.5 + ( ) 𝑥10 = 57.5%
5
40
∑ 𝑓−𝑐.𝑓𝑏
100
(vi) P85 = 𝐿𝑏 + ( )𝐶
𝑓
85 ∑ 𝑓 85 𝑥 40
= = 34 , Lb = 79.5, f= 5, C = 10
100 100
34−32
P85 = 79.5 + ( ) 𝑥10 = 83.5%
5
40th and 85th range = 83.5 – 57.5 = 26%

Example 22

Given the information in the table

Class 20-29 30-34 35-44 45-64 65-74 75-84


Frequency 5 5 12 20 10 8
Find

(a) Mean value


(b) Standard deviation
(c) Mode
(d) Median
(e) Interquartile range
(f) 90th percentile

Solution

Class class x f f.d fx fx2 cf


boundary width
19.5-29.5 10 24.5 5 0.5 122.5 3001.25 5
29.5-34.5 5 32 5 1 160 5120 10
34.5-44.5 10 39.5 12 1.2 474 18723 22
44.5-64.5 20 54.5 20 1 1090 59405 42
64.5-74.5 10 69.5 10 1 695 48302.5 52
74.5-84.5 10 79.5 8 0.8 636 50562 60
∑ 𝑓 =60 ∑ 𝑓𝑥 =3177.5 2
∑ 𝑓𝑥 =185113.8
∑ 𝑓𝑥 3177.5
(a) Mean 𝑥̅ = ∑𝑓
= = 52.96
60
∑ 𝑓𝑥 2 ∑ 𝑓𝑥 2 185113.8 3177.5 2
(b) S.d = √ ∑𝑓
− (∑ ) =√ −( ) = 16.75
𝑓 60 60
∆𝑓 .𝑑1
(c) Mode = Lb + ( )𝐶
∆𝑓 .𝑑1 + ∆𝑓 .𝑑2
1.2−1
=34.5 + ((1.2−1)+(1.2−1)) 10 = 39.5
∑𝑓
− 𝑐.𝑓𝑏
(d) Median = Lb + ( 2 )𝐶
𝑓
∑𝑓 60
= = 30
2 2
30− 22
Median = 44.5 + ( ) 𝑥 20 = 552.5
20

1
∑ 𝑓−𝑐.𝑓𝑏
(e) q1 = 𝐿𝑏 + (4 )𝐶
𝑓
1
𝑥 60−10
34.5 + (4 ) 𝑥 10 = 38.67
20
3
∑ 𝑓−𝑐.𝑓𝑏
Q3 = 𝐿𝑏 + (4 )𝐶
𝑓
3
𝑥 60−42
64.5 + (4 ) 𝑥 10 = 67.5
10

Interquartile range = 67.5 – 38.67 = 28.83


90
∑ 𝑓−𝑐.𝑓𝑏
(vii) P90 = 𝐿𝑏 + (100 )𝐶
𝑓
90
(60)−52
= 74.4 + (100 ) 𝑥 10 = 77
8

Graphs
(a) Grouped data with equal class width
(i) Histogram
This is a graph consisting of vertical bars. It is a graph of frequency against class boundary.
The area of the bar is equal to the frequency. Histogram is used to obtain the mode

Example 23

Given the data below

Marks 5-14 15-24 25-34 35-44 45-54 55-64 65-74 75-85 85-94
Frequency 3 7 12 20 30 15 8 3 2
Draw a histogram and use it to determine the mode

(ii) Cumulative frequency curve (Ogive)


This is a curve of cumulative frequency against class boundaries. An Ogive can be
used to determine the median, quartiles, percentiles and deciles

Note: The values of cumulative frequency are plotted against the upper class
boundaries and first value of the lower class boundary is plotted against cumulative
frequency = 0
Example 24

Given the data below

Marks 20-29 30-39 40-49 50-59 60-69 70-79


Frequency 4 6 12 8 7 3
Draw an Orgive and use it to determine

(a) Median
(b) Interquartile range
(c) 10th percentile

Solution

Class 19.5 – 29.5 29.5–39.5 39.5–49.5 49.5 – 59.5 59.5 – 69.5 69.5 – 79.5
boundary
cf 4 10 22 30 37 40

𝑁 𝑡ℎ 40 𝑡ℎ
(a) The median = ( ) =( ) = 20th value from the graph = 48.5
2 2
𝑁 𝑡ℎ 40 𝑡ℎ
(b) q1 = ( ) =( ) = 10th value ; from the graph q1 = 39.5
4 4
3𝑁 𝑡ℎ 3 𝑥 40 𝑡ℎ
q3 = ( ) =( ) = 30th value; from the graph q3 = 59.5
4 4
Interquartile range = 59.5 – 39.5 = 20

10𝑁 𝑡ℎ 10 𝑥 40 𝑡ℎ
(c) P10 = ( ) =( ) = 4th value ; from the graph P10 = 29.5
100 100
Ungrouped data with unequal class width
(i) Histogram

This is a graph of frequency density against class boundary


𝐹𝑟𝑒𝑞𝑢𝑒𝑛𝑐𝑦
Note that frequency density =
𝑐𝑙𝑎𝑠𝑠 𝑤𝑖𝑑𝑡ℎ

Orgive

This is a graph of cumulative frequency against the class boundary

Example 25

The data shows the length in centimetres for different calendars produced by a printing press. A
cumulative frequency distribution was formed

Length (cm) <20 <30 <35 <40 <50 <60


Cumulative 4 20 32 42 48 50
frequency
(a) Construct a frequency table.
(b) Find the mean length of the calendars
(c) Draw a histogram and use it to estimate the modal length
(d) Draw an Orgive and use it to estimate the median length.

Solution

(a) Frequency table

Class boundary x f fx class frequency cf


width density
0 - 20 10 4 40 20 0.2 4
20 - 30 25 16 400 10 1.6 20
30 - 35 32.5 12 390 5 2.4 32
35 - 40 37.5 10 375 5 2 42
40 - 50 45 6 270 10 0.6 48
50-60 55 2 110 10 0.2 50
∑ 𝑓50 ∑ 𝑓𝑥 =1585
∑ 𝑓𝑥 1585
(b) Mean 𝑥̅ = ∑𝑓
= = 31.7
50
𝑁 𝑡ℎ 50 𝑡ℎ
Median length is the ( ) = ( ) =25th value, from the graph = 32
2 2
Revision Exercise 2
UNEB 2002/2/7

The table below shows cumulative distribution of ages (in years of 400 student

Age(years) <12 <13 <14 <15 <16 <17 <18 <19


Cumulative 0 27 85 215 320 370 395 400
frequency
(a) Construct a cumulative frequency curve
(b) Use the curve to estimate
(i) Median age (Answer 14.9)
(ii) 20th and 80th percentile range (Ans. 2.1)

UNEB 2002/2/14

The table below shows the time taken by students to solve a mathematics problem

Time (mins) 5-9 10-14 15-19 20-24 25-29 30-34


Frequency 5 14 30 17 11 3
(a) Draw a histogram and use it to estimate the modal time. (ans. 17.3)
(b) Find the mean and standard deviation of solving the problem (Mean = 18.5mins, s.d =
5.9896 (4D))

UNEB 2004/2/14

The frequency distribution table shows the heights of s.6 students measured to the nearest cm;

Height 149-152 153-156 157-160 161-164 165-168 169-172 173-176


Frequency 5 17 20 25 15 6 2
(a) Calculate
(i) Mean height (Ans. 160.9cm)
(ii) Standard deviation (Ans. 5.5873)
(b) Draw a cumulative frequency curve and use it to estimate the median (Ans. 161cm) and
range of height of the middle 60% of the candidates. (Ans. 10cm)

UNEB 2005/2/15

The table below the weights of some S.5 students from a certain school

Weight 50-53 54-57 58-61 62-65 66-69 70-73 74-77 78-81


Number 3 8 12 18 11 5 2 1
of
student
(a) Calculate
(i) Mean (63.1kg)
(ii) Standard deviation of students’ weight (6kg)
(b) Draw a cumulative frequency curve and use it to estimate
(i) Median weight (63.1kg)
(ii) Number of students with weight between 58.9kg and 66.7kg (29students
UNEB 2006/2/12

The table below is the distribution of weights of a group of animals

Mass (kg) Frequency


21-25 10
26-30 20
31-35 15
36-40 10
41-50 30
51-60 45
66-74 5
(a) Draw a cumulative frequency curve to estimate semi-quartile range (24kg)
(b) Find
(i) Mode (28.8333kg)
(ii) Standard deviation (11.772)

UNEB 2008/2/9

The table below shows the amount of money (in thousands of shillings) that was paid out as
allowances to participants during a certain workshop

Amount 110-114 115-119 120-129 130-134 135-144 145-159


(shs’000s)
Number of 13 20 32 17 16 12
participants
(a) Draw a histogram and use it to estimate the modal allowance (shs. 11800)
(b) Calculate the:
(i) Median allowance (shs. 126,375/=)
(ii) Mean allowance (shs. 128,000/=)

UNEB 2009/2/11

The table below shows the income of 40 factory workers in millions of shillings per annum

1.0 1.1 1.0 1.2 5.4 1.6 2.0 2.5


2.1 2.2 1.3 1.7 1.8 2.4 3.0 2.2
2.7 3.5 4.0 4.4 3.9 5.0 5.4 5.3
4.4 3.7 3.6 3.9 5.2 5.1 5.7 1.5
1.6 1.9 3.4 4.3 2.6 3.8 5.3 4.0
(a) Form a frequency distribution table with class interval of 0.5millin shillings starting with the
lowest limit of 1million shillings
(b) Calculate the
(i) Mean income (shs. 3,175,000)
(ii) Standard deviation (shs. 1,413,992.574)
(c) Draw a histogram to represent the above data. Use it to estimate the modal income
Modal income: (shs. 5, 200,000)
UNEB 2010/2/10

The table below shows the mars obtained by students in a physic test

Marks (%) 25-29 30-34 35-39 40-44 45-49 50-54 55-59 60-64 60-69 70-74
Frequency 9 12 10 17 13 25 18 14 8 8
(a) Draw a histogram and use it to estimate the modal mark. (52.5)
(b) Find the
(i) Mean mark (49.4627)
(ii) Standard deviation (12.424)

UNEB 2012/2/9

The table below shows the marks obtained in an examination by 200 candidates

Marks(%) 10-19 20-29 30-39 40-49 50-59 60-69 70-79 80-89


Number of 18 34 58 42 24 10 6 8
candidates
(a) Calculate the
(i) Mean mark (40.2%)
(ii) Modal mark (35.5%)
(b) Draw a cumulative frequency curve for the data. Hence estimate the lowest mark for a
distinction one if the top 5% of the candidates qualify for the distinction. (75%)

UNEB 2013/2/1

A class performed on an experiment to estimate the diameter of a circular object. A sample of five
students had the following results in centimetre. 3.13, 3.16, 2.94, 3.33 and 3.0.

Determine the sample;


(i) Mean (3.11)
(ii) Standard deviation (0.1356)(05marks)

UNEB 2017/2/12

The times taken for 55 students to have their lunch to the nearest minute are given in the table
below

Time (minutes) 3 -4 5-9 10-19 20 – 29 30 – 44


Number of students 2 7 16 21 9
(a) Calculate the mean time for the student to have lunch. (mean=20.65) (04marks)
(b) (i) Draw a histogram for the given data
(ii) Use your histogram to estimate the modal time for the students to have lunch.
(08marks) (modal time = 22 minutes)
UNEB 2018/2/9

1. The frequency distribution below shows the age of 240 students admitted to a certain
University.
Age (years) Number of student
18 - < 19 24
19 -< 20 70
20 -< 24 76
24 -< 26 48
26 -< 30 16
30 -< 32 6
(a) Calculate the mean age of the students. (mean =22.1458 )(04mark)
(b) (i) Draw a histogram for the given data
(ii) Use the histogram to estimate the modal age (modal age = 19.58) (08mark)

UNEB 2019/2/1

The table shows the masses of bolts bought by a carpenter.


Mass (grams) 98 99 100 101 102 103 104
Number of bolts 8 11 14 20 17 6 4
Calculate the:
(a)median mass (101g)
(b) mean mass of the bolt(100.7625g) (05mark

UNEB 2019/2/10

The table below shows the marks obtained in a mathematic test by a group of student

marks 5 -<15 15-<25 25-<35 35-<45 45-<55 55-<65 65-<75 75-<100


Number of 5 7 19 17 7 4 2 3
students
(a) Construct a cumulative frequency (O give) for the data (05 marks)
(b) Use your Ogive to find the
(i) Range between the 10th and 70th percentiles (26)
(ii) Probability that a student selected at random scored below 50 marks. 0.8125)
(07 marks)

UNEB 2020/2/9

The table below shows the marks obtained by 100 students in a mathematics test

Marks 20-<40 40-<50 50-<55 55-<60 60-<70 70-<90 90-<100


Number of 5 15 10 15 25 25 5
students
(a) Calculate the mean mark (63.125)
(b) Construct a cumulative frequency curve (Ogive) and use it to find the
(i) Median mark (61.5)
(ii) Range of the middle 40% of the mark (15)

Thank You

Dr. Bbosa Science


Trigonometry
The word ‘trigonometry’ suggests ‘tri’-three,
‘gono’-angle, ‘metry’-measurement. As such,
trigonometry is basically about triangles, most
especially right-angled triangles.

Important to note The quadrants are also labelled anti-clockwise


(a) For a right angled triangle below from the positive x – axis.

The signs the trigonometric ratios in the


quadrants are given below

1 𝑐
• sinθ = cosecθ =𝑠𝑖𝑛θ = 𝑎
1 𝑐
• cosθ = secθ =𝑐𝑜𝑠θ = 𝑏
𝑐𝑜𝑠θ 𝑏
• tanθ = • cotθ = = Ratio Quadrant
𝑠𝑖𝑛θ 𝑎
1st 2nd 3rd 4th
(b) All positive angles are measured cosθ
anticlockwise from positive x-axis
sinθ

tanθ

secθ

cosecθ
(c) A circle drawn with the centre O, divides
cotθ
the co-ordinate axis into four equal parts
called quadrants Note

- If θ is the angle in the 1st quadrat


- In the 2nd quadrat the angle is (180 – θ)
- In the 3rd quadrat the angle is (180 + θ)
- In the 4th quadrat the angle is (360 –θ)

digitalteachers.co.ug
4th quadrant θ = 180 + 60 = 2400

,θ: θ= 600, 1200, 2400, 3000}

(iii) cosecθ + 2 = 0

Solution
Solving equations
cosecθ = -2 => sinθ = - ½ (taking reciprocal)
We make use of the quadrants to find the 11
Key angle = = 300
ranges of values within which the angle
follows In the 3rd quadrant θ = 180 + 30 = 2100
Example 1 In the 4th quadrant θ = 360 – 30 = 3300
Solve the following equations for 00 ≤ θ ≤3600 ,θ: θ= 2100, 3300}
(i) 3cosθ + 2 = 0
Solution
cosθ = (iv) 3sec2θ – 4 = 0

Solution
But cos is negative in the 2nd and 3rd
quadrants. √
secθ = => cosθ =

Ignoring the negative sign, the angle obtained
1√
is referred to as the key or principle angle, i.e. Key angle = = 300
1
key angle = = 48.20 (1d.p)

For cosθ = ; θ = 300, 3300
nd 0
In the 2 quadrant, θ = 180 – 48.2 = 131.8

In the 3rd quadrant, θ = 180 + 48.2 = 228.20 For cosθ = ; θ = 1200, 2100

,θ: θ= 131.80, 228.20} ,θ: θ= 300, 1200, 2100, 3300}

Note that: the key angle s not part of the


solution but only a guide. (d) Definitions of angle
2
(ii) 4cos θ – 1 = 0 (i) Acute angle is an angle between 00 and
Solution 900. It lies in the 1st quadrant
1 1 (ii) Right angle is an angle = 900
cosθ = √ = (iii) Obtuse angle is an angle between 900 and
Key angle, θ = 11
= 600 1800. It lies in the 2nd quadrant
(iv) Reflex angle is an angle between 1800 and
When cosθ = ½ (positive is 1st and 4th 3600. It lies in the 3rd and 4th quadrant
quadrants)

1st quadrant θ = 600


Example 2
4th quadrant θ = 360 – 60 = 3000
(a) If sinθ = and 00 ≤ θ ≤ 3600. Find the
nd rd
When cosθ = -½ (positive is 2 and 3 possible values of 3tanθ – cotθ
quadrants)
Solution
3rd quadrant θ = 180 – 60 = 1200

digitalteachers.co.ug
If sinθ = ; θ lies in 1st or 2nd quadrants ,θ: θ=100, 500, 700, 1100, 1300, 1700, 1900,
2300, 2500, 2900, 3100, 3500}

Note

- If θ0 ≤ θ ≤ 3600 then θ0 ≤ 3θ ≤ 10800


[multiply the interval through by 3]
In 1st quadrant
(ii) 2cos2θ + √ = 0
11
3tanθ – cotθ = 3. / . /=
1 Solution
nd
In 2 quadrant √
cos2θ = and θ0 ≤ 2θ ≤ 7200
11
3tanθ – cotθ = 3. / . /= 1 2θ = 1500, 2100, 5100, 5700
11
the possible values are 1
,θ: θ = 750, 1050, 2550, 2850}

(b) If cosθ = and θ is reflex, find the Set square angles: 300, 450, and 600
1
value of 4sec2θ + tanθ (i) From equilateral triangle ABC with side
equal to 1 unit
Solution

If cosθ = and θ is reflex, θ lies in the 3rd


1
quadrant

1
cos 600 =sin300 =

4sec2θ + tanθ = .
1
/
1
=
1 cos 300 = sin 600 =
1
1
tan 300 = cot 600 =
Example 3 √
tan 600 = cot 300 = √
0 0
Solve for θ, where θ ≤ θ ≤ 360 (ii) From the right angled triangle PQR below
(i) 3tan23θ = 1

Solution
1
tan3θ =

1
taking tan3θ =

 3θ = 300, 2100,3900,5700,7500, 9300 1


θ = 100, 700, 1300, 1900, 2500, 3100 cos 450 = sin450 =

1
taking tan3θ = tan450 = 1

 3θ = 1500, 3300,5100,6900,8700, 10500 Example 4


θ = 500, 1100, 1700, 2300, 2900, 3500 Without using tables or calculators find the
value of

digitalteachers.co.ug
(i) cos2400 Identity (i) cos2θ

Solution 1 + tan2θ= sec2θ ……………… (ii)

cos2400 = -cos(240 – 180)0 = - cos 600 =


1 Identiy (i) sin2θ

(ii) tan 39900 1 + cot2θ = cosec2θ ………….(iii)

Solution Example 5

1 Show that
tan 39900 = tan [(360 x 11)+ 30]0 = tan300 =

(i) sin2θ + (1 + cosθ)2 = 2(1 + cosθ)
0
(iii) sin 570
Solution
Solution
sin2θ + (1 + cosθ)2
0 0 1
sin 570 = sin {(360 x 1) + 210] = -sin30 =
= sin2 θ + 1 + 2cosθ + cos2 θ
(iv) sec 2250
= sin2 θ + cos2 θ + 1 + 2cosθ
Solution
= 1 + 1 + 2cosθ (Recall that sin2 θ + cos2 θ = 1)
sec 2250 = sec (225 – 180)0 = sec 450 = -√ = 2 + 2cosθ = 2(1 + cosθ)

sin2 θ + (1 + cosθ)2 = 2(1 + cosθ)


1 1
The Pythagoras theorem (ii) · = tanθ
1 1

Solution

1 1 1 1
1 1
=1
1

1
=1
For any acute angle θ

x = rcosθ and y = rsinθ =

By Pythagoras theorem =
x2 + y2 = r2
= =
Substituting for x and y
1 1
2 2 2 1
· 1
= tanθ
(rcosθ) + (rsinθ) = r

r2(cos2θ + sin2θ) = r2
1
cos2θ + sin2θ = 1 (iii) (tanθ + secθ)2 = 1

Now tanθ = = = Solution


1 1
= (tanθ + secθ)2 = . / =. /

(1 ) (1 )
Identities = =
1
cos2θ + sin2θ = 1 …………………(i)

digitalteachers.co.ug
(1 )(1 ) 1 Squaring both sides
= (1 )(1 )
=
1

1
3tan2θ = sec2θ – 2secθ + 1
2
(tanθ + secθ) = 1
3tan2θ = sec2θ – 2secθ + 1
Example 6
3[sec2θ – 1] = sec2θ – 2secθ + 1
Solve the following equations for
-1800 ≤ x ≤ 1800 2sec2θ + 2secθ – 4 = 0

(i) 2cos2θ + sinθ – 1 = 0 sec2θ + secθ – 2 = 0

Solution (secθ + 2)(secθ – 1) = 0

2(1 – sin2θ) + sinθ – 1 = 0 secθ = -2 or secθ = 1


1
2sin2θ – sinθ – 1 = 0 cosθ = - or cosθ = 1

(sinθ – 1)(2sinθ + 1) = 0 *θ: θ= 00, ± 1200]

Either sinθ = 1 or sinθ =


1 3rd approach

√3sinθ = 1- cosθ
When sinθ = 1; θ = 900
1 Dividing through by sinθ
When sinθ = ; θ= -1500, -300, 2100, 3300
√3 = cosecθ – cotθ
*θ: θ= -1500, - 300, 900 for given range]
Rearranging
(ii) cosθ + √3sinθ = 1
√3 + cotθ = cosecθ
Solution
Squaring both sides
1st approach
3 + 2√3cotθ + cot2θ = cosec2θ
√3sinθ = 1- cosθ
3 + 2√3cotθ + cot2θ = 1+ cot2θ
Squaring both sides
1
2 2 cotθ = -√ ; => tanθ = -√3
3sin θ = 1 – 2cosθ + cos θ

3(1 – cos2θ) = 1 – 2cosθ + cos2θ *θ: θ= -600, 1200]

4cos2θ – 2cosθ – 1 = 0 Example 7

(2cosθ + 1)(cosθ – 1) = 0 (a) Given that 7 tanθ + cotθ = 5secθ, derive a


quadratic equation for sinθ. Hence or
1
Cosθ = cosθ = 1 otherwise, find all values of θ in the interval
00 ≤ θ ≤ 1800 which satisfy the equation, giving
θ = ± 1200 θ = 00 your answer to the nearest 0.10 where
*θ: θ= 00, ± 1200] necessary

2nd approach Solution

√3sinθ = 1- cosθ 7 tanθ + cotθ = 5secθ

Dividing through by cosθ =

√3tanθ = secθ – 1 =

digitalteachers.co.ug
7sin2 + cos2 = 5sin 4 + x2 + 2xy + y2 = x2-2xy + y2

7sin2θ + (1 – sin2θ) = 5sinθ 4xy + 4 = 0

6sin2 – 5sin + 1 =0 xy + 1 = 0 as required

(3sin – 1)(2sin -1) = 0 (b) x = 2 + 3sinθ and y = 3 + 2cosθ show that


1 1
4(x – 2)2 + (y – 3)2 = 36
sin = sin = Solution
θ = 19.50, 160.50 θ = 300, 1500 x = 2 + 3sinθ => sinθ =
y = 3 + 2cosθ => cosθ =
*θ: θ= 19.50, 300, 1500, 160.50]
Using identity sin2θ + cos2θ= 1
Example 8
. / . / =
Find the solution of 3cotθ + cosecθ = 2 for 4(x – 2) + (y – 3)2 = 36 as reqyured
2

00 ≤ θ ≤ 1800. (c) x = 2sinθ and y = tanθ, prove that


Solution =
√(1 )

3cotθ + cosecθ = 2 Solution


x = 2sinθ; => cosecθ =
1
= 1
y = tanθ; => cotθ =
( ) =( ) Using identity: 1 + cot2θ = cosec2θ
1
= 1+. / =. /

= ( ) =
√(1 )

=
√ 1
= Revision exercise 1
1

cosθ = 0.3021 cosθ = 0.7637 1. Solve for θ, where θ0 ≤ θ ≤ 3600


(a) secθcosecθ + 2secθ – 2cosecθ – 4 = 0
θ = 72.40 θ = 40.2 [θ: θ = 600, 2100, 3000, 3300]
*θ: θ = 72.40, 40.20] (b) tan2θ – (√ )tanθ + √ = 0
[θ: θ = 45 , 60 , 2250, 2400]
0 0

Elimination of trigonometric parameter 2. Show that


1 1
This involves the use of identities to eliminate (a) 1
=
the trigonometric values in equation (b) tanθ + cotθ = secθcosecθ
(c) cos4θ-sin4θ+1 = 2cos2θ
Example 9 1
(d) 1
=
(a) If x = tanθ + secθ and y = tanθ – secθ;
1
show that xy + 1 = 0 (e) √1 = secθ + tanθ
Solution 3. Solve the following equations for
x + y = tanθ -1800 ≤ x≤ 1800
x – y = 2secθ (i) 2cos2θ + sinθ – 1 = 0
1
secθ = ( ) *θ: θ = -1500, - 300, 900]
Using identity: 1 + tan2θ = sec2θ (ii) sin2θ + 5cos2θ = 3
1 *θ: θ = ±450, ±1350]
1 + (x + y) 2 = 0 ( )1
(iii) 4cot2θ + 24cosecθ + 39 = 0

digitalteachers.co.ug
*θ: θ = 16.60, 23.60, 156.40, 163.40] (a) Sinxtanx + cosx =secx
(b) Cosecx + tanxsecx = cosecxsec2x
4. Solve each of the following equations in (c) Cosecx – sinx = cotxcosx
the stated range (d) (sinx + cosx)2 - 1 = 2sinxcosx
(a) 4cos2θ + 2sin θ = 4 00 ≤ θ ≤ 3600 12. Eliminate θ from each of the following
*θ: θ = 00, 48.60, 131.40, 1800, 3600] pairs of relationships
(b) 2sec2θ – 4tanθ – 2 = -1800 ≤ θ ≤ 3600 (a) x =3sinθ, y = cosecθ [xy = 3]
*θ: θ = -1350, -161.60, 18.40, 450] (b) 5x = sinθ, y = 2cosθ [100x2 + y2 - 4 = 0]
(c) 5cos23θ = 3(1 + sin3θ), 00 ≤ θ ≤ 3600 (c) x = 3 + sinθ, y = cosθ [(x-3)2 + y2 =1]
*θ: θ = 7.9 , 52.1 , 90 , 127.90, 172.10]
0 0 0
(d) x = 2 + sinθ, cosθ = 1+y
5. Solve for θ; 00 ≤ θ ≤ 3600 [(x-2)2 + (y+1)2 = 1]
(a) tanθ + 3cotθ = 4
*θ: θ = 450, 71.60, 2250,251.60]
(b) 4cosθ – 3sinθ = 2
Measuring angles in radians
*θ: θ = 29.50, 256.70] A radian is defined as an angle subtended at
6. Solve the centre of a circle by an arc that is equal to
(a) cosθ + √3sinθ = 2 0≤θ≤π the radius of the circle. One radian is
0 = 1 represented by π, where =
0 0
(b) 2cosθ – cosecθ = 0 0 ≤ θ≤ 270
*θ: θ = 450, 2250]
(c) 2sin2θ + 3cosθ = 0 00≤ θ≤ 3600

*θ: θ = 2400, 1200]

(d) 3sinθ + 4cosθ = 2 -1800 ≤ θ ≤ 1800


*θ: θ = - 29.550, 103.290]
(e) 3tan2θ + 2sec2 θ = 2(5 – 3tanθ) for
00 < θ <1800
*θ: θ = 38.660, 116.570] How to convert between degrees and radians

1 revolution = circumference of a circle


7. Without using a tables or calculator ,
show that tan 150 = 2 - √3 But circumference of a circle subtends an
8. Solve equation angle 2π at the centre.
8cos4θ – 10cos2θ + 3 for 00 ≤ θ ≤ 1800
 1 revolution = 2π = 3600
*θ: θ = 300, 450, 1350, 1500]
π = 1800
9. Eliminate θ from the following
10 = 1 radians
equation
(a) x = asecθ and y = b + ccosθ x0 = 1 radians
[ac = x(y – b)]
Example 10
(b) x = secθ +tanθ and y = secθ – tanθ
[xy = 1] Convert the following angles to radians
10. Solve the simultaneous equation
Cos x + 4siny = 1 (a) 3300
4secx – 3cosecy = 5 for values of x (b) 900
and y between 00 and 3600 (c) 300
[x = 78.80, 281.50; y = 11.50, 168.50] Solution
11
11. Prove each of the following identities (a) 3300 = 1 = radians

digitalteachers.co.ug
(b) 900 = 1 = radians (a) . /
0
(c) 30 = 1 = radians (b) . /
Converting radians to degrees (c) . /

2π radians = 3600 Solution


1
1 radian = Convert the angles from radian to degrees
1 √
1 (a) . /= . /= =
x radians =
1 1
(b) . /= . /= =
1
(d) . /= . / = tan600 = √

Example 11
Length of an arc
Convert each of the following radians to
degrees Suppose that the angle subtended by the
length L of an arc AB of a circle is θ as shown.
(i) radians
(ii) radians
(iii) π radians

Solution
1
(i) radians = = 600
1
(ii) radians= = 720
1
(iii) π radians = = 1800
=

L = rθ where θ must be in radians


Some equivalent angles in degrees and
radians Example 13

Find the length of an arc of a circle of radius


𝜋
900 = 14 if it subtends an angle
𝜋 𝜋
1350 = 450 = (i)
(ii) 1500

Solution
1800 = 𝜋 00 =
(i) L = rθ = = 11cm
(ii) Convert degrees to radians
0 𝜋
𝜋 315 = 1500 = 1 150 = radians
2250 =
0 𝜋
270 = L= = 36.67cm

Example 12

Find each of the following values

digitalteachers.co.ug
Example 14 = . / =. /
A sector was drawn which had a perimeter of
Where θ must be in radians
80cm, and centre angle of 1300. Calculate the
radius Example 15

Solution Find the area of a sector with radius 14cm and


angle (i) (ii) 1200
The sides of a sector are composed of an arc,
and two more sides which are radii of a circle. Solution

(i) =. / =. / =77cm3
(ii) Converting 1200 to radians

1200 = 1 =

=. / =. / =205.25cm3

Solving trigonometric functions whose range


2r + L = 80 is in radians
L = 80 – 2r When the range of the trigonometric function
Converting 130 to radians 0 is in radians, the answer should be given in
radians
1
1300 = 1 130 = 1
Example 16
But L = rθ
Solve the following equations for the ranges
1 indicated
80 – 2r = 1
1 (i) cosθ + √3sinθ = 1 0≤θ≤π
2r + = 80
1
Solution
( 1 )
=
1 √3sinθ = 1- cosθ
r = 18.74cm Squaring both sides
Area of a sector of a circle 3sin2θ = 1 – 2cosθ + cos2θ
A sector of a circle is a portion of the interior 3(1 – cos2θ) = 1 – 2cosθ + cos2θ
of a circle intercepted by a central angle.
4cos2θ – 2cosθ – 1 = 0

(2cosθ + 1)(cosθ – 1) = 0
1
Cosθ = cosθ = 1

θ = ± 1200 θ = 00

= = Radians
1

The area of a sector of a circle of radius r and 00 = 0 radians


central angle θ is given by
0 = 1

digitalteachers.co.ug
(ii) 2cos2θ + sinθ – 1 = 0 0≤θ≤π 6. Solve the following equations for the
ranges indicated
Solution
(a) 2sec2θ = 3 + tanθ for 0 ≤ θ ≤ 2π
2(1 – sin2θ) + sinθ – 1 = 0 *θ: θ = 0.25π, 0.85π, 1.25π, 1.85π+
(b) 2sin2xcosx + cosx – 1 for 0 ≤ θ ≤ 2π
2sin2θ – sinθ – 1 = 0 *θ: θ = 0.38π, 1.62π, 2π+
(sinθ – 1)(2sinθ + 1) = 0 (c) 2tanθ + 4cotθ = cosecθ for –π ≤ θ ≤ π
1
1 0 = 1
Either sinθ = 1 or sinθ =
Graphs of trigonometric functions
When sinθ = 1; θ = 900
1
The following are the characteristic of the
When sinθ = ; θ= -1500, -300, 2100, 3300 three major trigonometric functions

*θ: θ= = for given range] The sine function


1

Revision exercise 2 - It is continuous (with no breaks)


- The range -1 ≤ sinθ ≤ 1
1. Express each of the following in radians - The shape of the graph from θ = 0 to θ =
(a) 300 0 1 2π is repeated every 2π radians
- This is called a periodic or cyclic function
(b) 450 0 1
and the width of the repeating pattern
(c) 1200 0 1 that is measured on horizontal axis is
called a period. The sine wave has a
(d) 3000 0 1
period of 2π, a maximum value of +1 and
2. Express the following angle in degrees
a minimum value of -1.
(a) rad [600] - The greatest value of sine wave is called
(b) rad [22.50] the amplitude.
(c) 3π rad *5400]
(d) 5.2π rad*9360]
3. A sector of the circle of radius 7 cm
subtends an angle radians at the centre.
Calculate the
(a) Length of the arc 0 1
(b) Perimeter of the sector0 1
(c) Area of the sector 0 1
4. AOB is a sector of a circle, centre O, and is The coosine function
such that OA = OB = 7cm and angle AOB is - It is continuous (with no breaks)
300. Calculate the - The range -1 ≤ sinθ ≤ 1
(a) Perimeter of sector AOB0 1 - Has a period of 2π
- The shape is the same as the sine wave
(b) The area of AOB 0 1
but displaced a distance to the left on
5. Find the value each of the following
the horizontal axis. This is called a phase
(a) Sinπ *0+
shift
(b) cos [-1]
(c) tan [√ ]

digitalteachers.co.ug
Angle ABG = β

Angle [ABG + GBC] = 900

Angle GBC = 90 – β

From triangle GBC,

Angle BCG = 180 – (90 + 90 – β)

Angle BCG = β

The tan function From

- The tan function is found using; (1) Triangle ABC:


= . It follows that tanθ = 0 when cosα = = 1
; => AB = cosα
sin θ = 0; and tanθ is undefined when cosθ (2) Triangle ABE:
=0 cosβ = = ; => AE = cosβcosα
- The graph is continuous, but undefined
sinβ = = ; => BE = cosαsinβ
when θ=
(3) Triangle BCG:
- The range of values for tanθ is unlimited
cosβ = = ; => CG = sinαcosβ
- It has a period π
sinβ = = ; => BG = sinαsinβ
(4) Triangle ACF:
cos(α + β) = = 1
= AE –BG
cos(α + β) = cosαcosβ – sinαsinβ
sin(α + β) = = 1
= CG + GF
sin(α + β) = sinαcosβ + cosαsinβ

It follows that

(i) cos(α + β) = cosαcosβ – sinαsinβ


Compound angles (ii) cos(α - β) = cosαcosβ + sinαsinβ

Consider a cardboard ABCD of unit diagonal [substituting –β for β)


that stands on the edge A, making an angle β
(iii) sin(α + β) = sinαcosβ + cosαsinβ
with the horizontal ground. Let the unit
(iv) sin(α - β) = sinαcosβ - cosαsinβ
diagonal AC be inclined at an angle α to the
side AB (see diagram) [substituting –β for β)

These can also be derived using vector


approach.
Consider two unit vectors and each
inclined at angles α and β, respectively to the
positive x-axis
A (cosα, sinα)
y

B (cosβ, sinβ)
α-β
Angles EAB= ABG(Alternative angles)
α β
O x
digitalteachers.co.ug
Using the definition of a vector product: =
1 √ 1 1
=
√ 1
=0.2588
√ √ √
. = | |.| |cos(α –β)
Example 18
Since and are unit vectors,
1
Prove that tan(450 + A) = 1
| | =| |=1

. = cos(α –β) From tan (α + β) = 1

 (cosα + sinα ). (cosβ + sinβ ) = cos(α –β)


tan(450 + A) = 1
cosαcosβ + sinαsinβ = cos(α –β) 1
=
1
Substituting 90 –α for α
Example 19
cos(90 –α )cosβ + sin(90 –α )sinβ
1
= cos(90 –α –β) Acute angles A and B are such that: cosA = ,
1
sinB . Show without using tables or calculator
sinαcosβ + cosαsinβ = sin(α +β)
√ √
that tan (A + B) =
Other expansions can be similar substitutions
( ) Solution
i.e. tan(α +β) =
( )
Using cos2θ + sin2θ =1
= 1
. / + sin2A =1
Dividing through by cosαcosβ

sin2A = => sinA =
( ) =1
√ 1
tanA = =√
Similarly
Similarly;
( ) =
1
1
cos2B + . / =1
The following is a summary of compound
angles √
cosB =
1. cos (α + β) = cosαcosβ – sinαsinβ
2. cos (α – β) = cosαcosβ + sinαsinβ √ 1 1
tanB = =

3. sin (α + β) = sinαcosβ + sinβcosα
4. sin (α - β) = sinαcosβ – sinβcosα But
5. tan (α + β) = 1
From tan (α + β) = 1
6. tan (α - β) = 1


Example 17 =
1 √

0 0
Calculate the value of sin15 given that sin45 ( √ √ 1)( √ √ )
1 1 √
= ( √ √ )( √
0 0 √ )
= cos45 = , sin30 = and cos30 =

√ √
Sin150 = sin(450 – 300) tan (A + B) =

= sin450cos300 – sin300cos450

digitalteachers.co.ug
Example 20

Solve cos(θ + 350) = sin(θ + 250) tanα + tanβ + tanγ = tanαtanβtanγ


for 00 ≤ θ ≤ 3600
Example 23
0 0 0
Cosθcos35 – sinθsin35 =sinθcos25 +
In a triangle ABC, prove that
cosθsin250 1 1 1

Dividing through by cosθ


1 1 1
0 0 0 0 =
Cos35 – tanθsin35 = tanθcos25 + sin25
Solution
tanθ = = 1
1 1
0 0 0 0
( )= ( )=
θ = 15 , 195 for 0 ≤ θ ≤ 360
1
Example 21 0 ( )1 = =

(a) Prove than 1 = 1 . /


 =
. /
Solution
= . / 1 1 1
1 1= . /
= . /
1
= .
1
/ 1=4 5
1

1 1
= =
(b) Solve sin2θ = cosθ for 00 ≤ θ ≤ 900 1 1 1 1
=
Solution
sin2θ = cosθ 1 1 1 1
= +
2sinθcosθ = cosθ 1 1
1
sinθ =
θ = 300 for 00 ≤ θ ≤ 900 Dividing each side by
1 1 1

Example 22 1 1 1 1 1 1
=
Given that α, β and γ are angles of a triangle,
show that tanα + tanβ + tanγ = tanαtanβtanγ Example 24

Hence find tanγ if tanα = 1 and tanγ = 2. Prove that the angle θ, between the straight
line y = m1x + c1 and the straight line
Solution y = m2x + c2 is given by tanθ = 1
α + β + γ= 1800 (angle sum of a triangle)
Let the lines be inclines at angles α and β with
tan (α + β + γ) = tan1800 = 0 the x-axis respectively
tan* (α + β) + γ+ = 0
( )
=
1 ( )

 ( ) =

1
=
=

digitalteachers.co.ug
From the diagram above (a) tan(θ – 450) = 4, find the value of θ

θ=β–α 0 1
(b) tan (θ + 600) find the value of cotθ
 tanθ = tan(β – α)
[ √ ]
=1
tanθ = 1
Double angles and half angles
Revision exercise 3 (b) From cos (θ + θ) = cosθcosθ – sinθsinθ
 cos2θ = cos2θ – sin2θ…………. (i)
1. (a) show that sin(α +β) - sin(α - β) =
2cosαsinβ Either
(b) If sin(α +β) = 5cos(α - β) show that
cos2θ = cos2θ -1 + cos2θ (cos2θ + sin2θ = 1+
tanα = 1
(c) Without using tables or calculator, cos2θ = 2cos2θ -1 …………………………. (ii)
show that cos150 = sin750 Or
0 1
(d) If α +β = 45 , show that tanα = 1 cos2θ = 1– sin2θ – sin2θ
2. Prove that:
( ) (1 )(1 )  cos2θ = 1 – 2sin2θ …………….(iii)
(i) ( )
=
( ) It follows that
(ii) – =
1
(iii) =
( ) cos2θ = ( ) …………………..(iv)
( ) 1
(iv) = sin2θ = ( ) …………………..(iv)
( )
( ) 1
(v) =
( ) 1 The identities imply
1
(vi) ( )=
Cos6θ = cos23θ – sin23θ
3. (a) Determine solution of tan2x + 2sinx = 0
for 00 ≤ x ≤ 1800 [x: x= 00, 600, 1200, 1800] =2cos23θ – 1 = 1 – 2 sin23θ
(vii) Show that in triangle ABC,
Cos θ = cos2 – sin2
tanA + tanB + tanC = tanAtanBtanC
4. Find the values of tan α for each of the
=2cos2 – 1 = 1 – 2 sin2
following
(a) sin(α - 300) = cos α *√ ] (c) sin(θ + θ) = sinθcosθ + cosθsinθ
(b) sin(α + 450) = cos α *√ ]  sin2θ = 2sinθcosθ
(c) cos(α + 600) = sin α *2 - √3+
(d) sin(α + 600) = cos(α – 600) [1] It follows that
(e) cos(α + 600) = 2cos(α + 300) *4 + 3√3+ sin6θ = 2sin3θcos3θ
0 0 √ √
(f) sin(α + 60 ) = cos(45 – α)0 1
√ 1 sinθ = 2sin cos
5. Given that
(a) tan(α –β) = ½ and tanα = 3 find the (d) tan (θ + θ)=1
value of tanβ *1+
(b) tan(α +β) = 5 and tanβ = 2 find the  =
1
value of tanα0 1 It follows that
11
6. Given that

digitalteachers.co.ug
Example 26
=
1
If tanα = and α is acute, without using tables
=1 or calculator work out the value of

Note that in all cases, the angles on the right (a) tan2α
hand side are half the angles on the left hand
side [half angle formulae] = = = =
Example 25 . /

Show that (b) tan


(a) cosec2θ + cot2θ = cotθ
similarly = =
1
Solution
1  3 =
=
( )( ) =0
1 1
= = =
1 1
=
Since α is acute, tanα cannot be negative
= = 1
=
(b) =
1
Hence deduce that if 3θ + α = 450, then Example 27
1
tanα = 1
(a) Show that cos3α = 4cos2 α -3cos α. Hence

Solution solve the equation 4x3 – 3x - = 0 for
0 0
0 ≤ α ≤ 180
tan3θ=tan(2θ + θ) = 1
Solution
=2. / 3 2 . / 3 Cos3α = cos(2α +α)
1 1

= = = cos2αcosα – sin2αsinα
1 1
=(2cos2α -1)cosα – 2sin2αcosα
= 1
=(2cos2α -1)cosα – 2(1-cos2α)cosα
Hence 3θ + α = 450 => α = 450 - 3θ
= 2cos3α – cosα -2cosα + 2cos3α
0
Tanα = tan(45 - 3θ)
=4cos2 α -3cos α
1
=1 =1 √
Hence 4x3 – 3x =
1 ( )

= i.e. 4cos2 α -3cos α =
1 ( )


1 00 ≤ α ≤ 1800; cos3α =
=
1

1
For the range 00 ≤ α ≤ 1800
tanα = 1
 00 ≤ 3α ≤ 5400

digitalteachers.co.ug
3α = 54.70, 414.70 (b) Prove that cot 2θ =
1
Hence solve
0 0
α = 18.23 , 138.23 (2d.p) the equation cot2θ + 2cotθ = 2 for
00 ≤ θ ≤ 3600
*α: α= 18.230, 138.230]
1
Solution
(b) Given that t = tan22 , show that
t2 + 2t – 1 = 0, cot 2θ = =
1
Hence show that tan22 = -1+ √2 dividing through by sin2θ
Solution
1
cot 2θ =
0
tan45 =
1
Hence, cot2θ + 2cotθ =0
=1
1
= =
2
t + 2t – 1 = 0 (as required)
5cot2θ -4cotθ – 1 = 0
solving
√ ( 1 1) (5cotθ + 1)(cotθ -1) = 0
= 1
1
cotθ = or cotθ = 0

= = √
 tanθ = -5 ot tanθ = 1
1
Since 22 is an acute angle,
When tanθ= -5; θ= 101.30, 281.30
1
tan22 = -1+ √2 is positive
When tanθ = 1, θ = 450, 2250
1
tan22 = -1+ √2
{θ: θ= 450, 101.30, 2250, 281.30}
Example 28
Revison exercise 4
(a) Show that 3sinθ = 3sinθ – 4sin3θ. Hence
1. Prove that
solve the equation sin3θ + sinθ = 0 for
00 ≤ θ ≤ 3600 (a) sinαcosecβ + cosαsecβ = 2sin(α +
β)cosec2β
Solution
Sin3θ = sin(2θ + θ) (b) cos6θ + sin6θ = 1-
= sin2θcosθ + cos2θsinθ (c) = and hence deduce that
1
= 2sinθcos2θ + (1- 2sin2θ)sinθ √ 1
= 2sinθ(1- sin2θ) + (1- 2sin2θ)sinθ = √
= 3sinθ – 4sin3θ 2. (a) Solve the equation for θ, 00≤ θ ≤ 3600
Hence sin3θ + sinθ = 0 sin2θ – 2sinθcosθ – 3cos2θ = 0
3sinθ – 4sin3θ + sinθ= 0 *θ: θ = 71.60, 1350, 251.60, 3150]
4sinθ – 4sin3θ = 0 (b) show that 1 = ( ).
4sinθ(1-sin2θ) = 0 Hence or otherwise solve the equation
4sinθ(1-sinθ)(1+sinθ) = 0 1
= for 00 ≤ θ ≤ 3600 *θ= 36.80]
sinθ = 0; θ = 00, 1800, 3600 1
sinθ = 1; θ = 900 3. (a) solve the equation 4cos2θ – 2cosθ + 3
sino = -1; θ = 2700 = 0 for 00 ≤ θ ≤ 3600
θ: θ= 00, 900, 1800, 2700, 3600 [θ: θ= 600, 104.50, 255.50, 3000]
(c) Solve the equation sinθ + sin = 0 for
00 ≤ θ ≤ 3600

digitalteachers.co.ug
[θ: θ= -3600, -2400, 00, 24050, 3600] (c) y – 3 = cos2θ, x = 2 – sinθ
[2x2 – 8x + y + 4 = 0]
4. (a) Prove that tan . / - tan. /= 10. Solve the following equations for
2tan2θ -1800 ≤ θ ≤ 1800
(b) By expressing 2sinθsin(θ + α) as (a) Sin2θ + sinθ = 0 *±1200, ±1800]
difference of cosines of two angles or (b) Sin2θ – 2cos2θ = 0 *-1350, 450, ±900]
otherwise, where α is constant, find (c) 3cos2θ + 2+ cosθ = 0 *±70.50, ±1200]
its least value 0 1 (d) sin2θ = tanθ *00, ±450, ±1350, ±1800]
11. Solve the following equations for
(c) Solve for θ in the equation
-3600 ≤ θ ≤ 3600, giving your answer
Cosθ – cos(θ + 600) = 0.4 for
correct to 1 decimal place
00 ≤ θ ≤ 3600 *θ = 126.40, 353.60]
5. (a) Show that cos3θ = 4cos3θ – 3cosθ. (a) sinθ = sin. /[00, ±1200, ±3600]

Hence solve the equation 4x3 – 3x - =0 (b) 3cos. / = 2sinθ *±1800, 97.20, 262.80]
[x: x = -0.746, -0.204, 0.959] (c) 2sinθ = tan. /
(b) Find all solutions of the equation [00, ±1200, ±2400, ±3600]
5cosx – 4sinx = 6 in the range
(d) 2cosθ = 15cos. / [±2090]
-1800 ≤ x ≤ 1800 [x: x = -59.10, -18.30]
1
12. Prove the following identities
6. (a) Express √. / in terms of (a) 2cos2θ – cos2θ = 1
1 (b) 2cosec2θ = cosecθsecθ
tanθ[ ]
√( 1) (c) 2cos3θ + sin2θsinθ =2cosθ
(b) Find the solution of the equation (d) tanθ + cotθ = 2cosec2θ
√ = for 0≤θ≤2π (e) cos4θ – sin4θ = cos2θ
1
[ = ] (f) 1
=

(c) Factorize cosθ – cos3θ – cos7θ + cos9θ in (g) Cotθ – tanθ = 2cot2θ
form Acospθsinqθsinrθ where A, p, q and (h) cot2θ + cosecθ = cotθ
r are constants [A= -4, p =5, q = 5, r = 2] (i) =
7. (a) Given that sinα + sinβ = p and (j) =
1
cosα + cosβ = q show that 1
(k) =1
(i) tan. /=
(l) = 1
(ii) cos(α + β)=
(m) . /=1
(b) Solve the simultaneous equation:
1
cosα + 4sinβ = 1 13. Express tan22 in the form a + b√
4secα – 3cosecβ = 5 *θ= 78.50, 281.50] where a and b are integers [a = -1, b= ±1]
8. (a) Express sinθ + sin3θ in form 14. Solve the equation
pcosθsinqθ where p and q are constant
(i) 4cosθ – 2cos2θ = 3 for 0 ≤ θ ≤ π 0 1
[p =2, q = 2]
(b) Find the solution of (ii) Cos2θ + cos3θ + cosθ = 0 for 00 ≤ θ ≤ 3600
cos7θ + cos5θ = 2cosθ for *θ = 450, 1200, 1350]
00 ≤ θ ≤ 3600 [00, 600, 2700, 3600] (iii) cosθ + sin2θ = 0 for 00 ≤ θ ≤ 3600
*θ = 900, 2100, 2700, 3300]
(c) Prove that =
(iv) 2sin2θ = 3cosθ for -1800 ≤ θ ≤ 1800
9. Eliminate θ from each of the following *θ = -900, 48.60, 900, 132.40]
pairs of expression (v) Sinθ – 4sin4θ = sin2θ – sin3θ for
(a) x+1 = cos2θ, y = sinθ *x + 2y2 = 0]
–π ≤ θ ≤ π 0 1
(b) x = cos2θ, y = cosθ -1 [x = 2y2 + 4y + 1]

digitalteachers.co.ug
Harmonic form (a) Express sinθ -√ cosθ in the form
Rsin(θ – α)
These are trigonometric functions expressed
Solution
in the form of Rcos(x ± α) and Rsin(x ±α).
Let sinθ -√ cosθ = Rsin(θ – α)
They are in two ways
=R(sinθcosα – cosθsinα)
(i) solving equations in the form Equating coefficients
acosθ + bsinθ + c = 0 Rcosα = 1 …………………. (i)
(ii) determining the maximum and minimum Rsinα = √ ……………….. (ii)
values of the function Eqn. (ii) eqn. (i)
acosθ + bsinθ + c = 0 tanα = √ ; => α = 600
where a, b and c are constants R2[cos2α + sin2α+ = 4
R2 = 4; R = 2
A: Solving equations
sinθ -√ cosθ = 2sin(θ – 600)
Example 29 (b) Solve the equation
(a) Express 3cosθ – 4sinθ in the form sinθ -√ cosθ + 1 = 0 for 00 ≤ θ ≤ 3600
Rcos(θ + α), where R and α are constants sinθ -√ cosθ = 2sin(θ – 600)
Solution  2sin(θ – 600) + 1 = 0
Let 3cosθ – 4sinθ = Rcos(θ + α) 1
sin(θ – 600) =
= R(cosθcosα – sinθsinα)
= Rcosθcosα – Rsinθsinα θ – 600 = 2100, 3300
Comparing coefficient of cosθ and sinθ
Rcosα = 3 ……….. (i) θ = 2700, 3900
Rsinα = 4 ………… (ii) Hence θ = 2700 for the given range
Eqn (ii) eqn (i)
tanα = ; α = 53.10 Example 31
Eqn. (i)2 + eqn. (ii)2 (a) Express 4cosθ – 5sinθ in the form Acos(θ +
R2cos2α + R2sin2α = 32 + 42 = 25 β), where A is constant and β is an acute
R2[cos2α + sin2α+ = 25 angle
R2 = 25 Let 4cosθ – 5sinθ = Acos(θ + β)
R=5 = A(cosθcosβ – sinθsinβ)
3cosθ – 4sinθ = 5cos(θ + 53.10) = Acosθcosβ – Rsinθsinβ
Comparing coefficient of cosθ and sinθ
(b) Solve the equation 3cosθ – 4sinθ = 5 for Acosβ = 4 ……….. (i)
00 ≤ θ ≤ 3600. Asinβ = 5 ………… (ii)
Eqn (ii) eqn (i)
Solution
tanα = ; α = 51.30
0
3cosθ – 4sinθ = 5cos(θ + 53.1 ) Eqn. (i)2 + eqn. (ii)2
 5cos(θ + 53.10) = 5 A2cos2β + A2sin2β = 42 + 52 = 41
cos(θ + 53.10) = 1 A2[cos2α + sin2α+ = 41
x + 53.10 = 00, 3600 A2 = 41
x = - 53.10, 306.90 A=√
3cosθ – 4sinθ = √ cos(θ + 51.30)
Hence x = 306.90
(b) Solve the equation 3cosθ – 4sinθ = 2.2 for
Example 30 00 ≤ θ ≤ 3600
Solution
3cosθ – 4sinθ = √ cos(θ + 51.30)

digitalteachers.co.ug
 √ cos(θ + 51.30) = 2.2 θ = 2400 =
cos(θ + 51.30) = =0.3436
√ 1
(θ + 51.3 ) =69.9 , 290.10
0 0 The minimum value is . /
θ = 18.60, 238.30
And maximum value occurs when
B: Maximum and minimum values
sin(θ + 300) = 1
The maximum and minimum values of a
 Minimum value = 2(1) + 7 = 9
circular function may be obtained using three
methods Now for sin(θ + 300) = 1

(i) Express the given function either in for θ + 300 = 900


Rcos (θ ± α) or Rsin(θ ± α) if possible,
θ = 600 =
where R and α are constants.
(ii) Differentiating the given function with
The maximum value is . /
respect to the given function say θ
(iii) Sketching the graphs of the function given (b) 5cosθ – 12sinθ – 13
and noting their maximum and minimum Solution
points. Let 5cosθ – 12sinθ = Rcos(θ - α)
= R(cosθcosα + sinθsinα)
In this chapter approach I will be considered. = Rcosθcosα + Rsinθsinα
Example 32 Comparing coefficient of cosθ and sinθ
Rcosα = 5 ……….. (i)
Determine the maximum and minimum values Rsinα = 12 ………… (ii)
of the following, stating the value of θ for Eqn (ii) eqn (i)
which they occur 1
tanα = ; α = 67.40
(a) √ sinθ + cosθ + 7 Eqn. (i)2 + eqn. (ii)2
Let √ sinθ + cosθ = Rsin(θ + α) R2cos2α + R2sin2α = 52 + 122 = 169
=R(sinθcosα +cosθsinα) R2[cos2α + sin2α+ = 169
Equating coefficients R2 = 169
Rsinα = 1 …………………. (i) R = 13
Rcosα = √ ……………….. (ii) 2cosθ – 12sinθ = 13cos(θ – 67.40)
Eqn. (i) eqn. (ii)  5cosθ – 12sinθ – 13 =13cos(θ – 67.40)-13
1
tanα = √ ; => α = 300 The minimum value occurs when
R2[cos2α + sin2α+ = 0 (√ ) 1= 2 cos(θ – 67.40) = -1
R2 = 4; R = 2
 Minimum value = 13(-1) -13 = -26
√ sinθ + cosθ = 2sin(θ + 300)
 √ sinθ + cosθ + 7 = 2sin(θ + 300) + 7 Now for cos(θ – 67.40) = -1

The minimum value occurs when θ – 67.40= 1800

sin(θ + 300) = -1 θ = 247.40

 Minimum value = 2(-1) + 7 = 5 The minimum value is ( )

Now for sin(θ + 300) = -1 And maximum value occurs when

θ + 300 = 2700 cos(θ – 67.40) = 1

 Minimum value = 13(1) - 13 = 0

digitalteachers.co.ug
Now for cos(θ – 67.40) = 1 Using sin2θ = (
1
)and
0 0
θ - 67.4 = 0 1
cos2θ = ( )
θ = 67.40
1
f(x) = (1-cos2θ) - 3sinθcosθ + (1 +cos2θ)
The maximum value is ( )
1 1
= - cos2θ - 3 sinθcosθ + + cos2θ
Example 33 = 3 – 2cos2θ - sin2θ
(a) Given that p = 2cosθ + 3cos2θ and = 3 – *2cosθ + sin2θ+
q = 2sinθ + 3sin2θ, show that Now:
1 ≤ p2 + q2 ≤ 25 3 – *2cosθ + sin2θ+≡ p + qcos(2θ – α)
If p2 + q2 = 19 and θ is acute, find θ and
= 3 + *qcos2θcosα + qsin2θsinα+

show that pq = By comparing: p = 3, qsinα = and
Solution qcosα = 2
 tanα = ; α = 36.90
p2 = 4cos2θ + 12cosθcos2θ + 9cos22θ ….(i)

q2 =4sin2θ + 12sinθsin2θ + 9sin22θ …….(ii) And q = √{. / ( ) }=


Eqn. (i) + eqn. (ii)
 3 – *2cosθ + sin2θ+ = 3 - cos(2θ – 36.90)
2 2
p + q = 4 + 12(cosθcos2θ + sinθsin2θ) + 9 But -1 ≤ cos(2θ – 36.90) ≤ 1
p2 + q2 = 13 + 12cosθ *cos(-θ) = cosθ+ Multiplying through by -

But -1 ≤ cosθ ≤ 1 ≥ - cos(2θ – 36.90)≥ -


Adding 3 throughout
Multiplying through by 12
3- ≤ 3 - cos(2θ – 36.90) ≤ 3+
-12 ≤ 12cosθ ≤ 12 1
( )
1

Adding 13 throughout (c) Find the maximum and minimum points


of the function; f(x) =3cosθ – 4sinθ + 20
1 ≤ 12cosθ +12 ≤ 25
for 00 ≤ θ ≤ 3600
1 ≤ p2 + q2 ≤ 25 as required Solution
Let 3cosθ – 4sinθ = Rcos(θ + α)
If p2 + q2 = 19, =>13 + 12cosθ = 19
1 = Rcosθcosα - Rsinθsinα
cosθ = ; θ =600 *θ is acute+
Comparing coefficient of cosθ and sinθ
1 Rcosα = 3 ……….. (i)
 p = 2cos600 + 3cos1200 = 1 - =
Rsinα = 4 ………… (ii)
√ √ Eqn (ii) eqn (i)
q = 2sin600 + 3sin1200 =√ =
tanα = ; α = 53.10
1 √ √
pq = . /. /= Eqn. (i)2 + eqn. (ii)2
R2cos2α + R2sin2α = 32 + 42 = 25
(b) Express f(x) = 5sin2θ – 3sinθcosθ + cos2θ in R2[cos2α + sin2α+ = 25
the form p + qcos(2θ – α) R2 = 25
1 1
Hence show that ( ) R=5
Solution 3cosθ – 4sinθ = 5cos(θ – 53.10)
 3cosθ – 4sinθ +20 =5cos(θ – 53.10) + 20

digitalteachers.co.ug
The minimum value occurs when denominator is minimum and the vice
versa for the maximum point
cos(θ – 53.10) = -1
The minimum value occurs when
 Minimum value = 5(-1) +20 = 15
sin(θ – 63.40) = 1
Now for cos(θ – 53.1) = -1
1
θ – 53.10= 1800  Minimum value = =

θ = 126.80 Now for sin(θ – 63.4) = 1

The minimum value is ( ) θ – 63.40= 900

And maximum value occurs when θ = 153.40

cos(θ – 53.10) = 1 The minimum value is (153.40. 0.31)

 Minimum value = 5(1) 20 = 25 And maximum value occurs when

Now for cos(θ – 53.10) = 1 sin(θ – 63.40) = -1


1
θ + 53.10 = 00, 3600  Maximum value = =
√ ( 1)
θ = -53.10, 306.80
Now for sin(θ – 63.40) = -1
The maximum value is ( )
θ - 63.40 = 2700
Example 34 θ = 333.40
Find the maximum and minimum points of the
The maximum value is ( )
following
1 1
(a) f(θ) = (b) f(θ) =
Solution Solution
Let sinθ – 2cosθ = Rsin(θ - α) Let 4sinθ – 3cosθ = Rsin(θ - α)

= Rsinθcosα - Rcosθsinα = Rsinθcosα - Rcosθsinα


Comparing coefficient of cosθ and sinθ Comparing coefficient of cosθ and sinθ
Rcosα = 1 ……….. (i) Rcosα = 4 ……….. (i)
Rsinα = 2 ………… (ii) Rsinα = 3 ………… (ii)
Eqn (ii) eqn (i) Eqn (ii) eqn (i)
tanα = ; α = 63.40 tanα = ; α = 36.90
Eqn. (i)2 + eqn. (ii)2 Eqn. (i)2 + eqn. (ii)2
R2cos2α + R2sin2α = 12 + 22 = 5 R2cos2α + R2sin2α = 32 + 42 = 25
R2[cos2α + sin2α+ = 5 R2[cos2α + sin2α+ = 25
R2 = 5 R2 = 25
R=√ R=
sinθ – 3cosθ = sin(θ – 36.90)
sinθ – 2cosθ = √ sin(θ – 63.40)
 sinθ – 3cosθ + 6 = sin(θ – 36.90)+ 6
 =√ sin(θ – 63.40) + 3 1
1  f(θ) =
 f(θ) = ( )
( )

Note: for a fractional function, a The minimum value occurs when
maximum point is obtained when the
sin(θ – 36.90) = 1

digitalteachers.co.ug
1 1 The t- formula is used widely in solving
 Minimum value = (1)
= 11
equations and proving trigonometric
Now for sin(θ – 63.4) = 1 identities. These can be extended as follows

θ – 36.90= 900 (i) For t= tanθ, sin2θ =1 and =1


1

θ = 126.90 (ii) For t= tan. /, sin. / =1 and


1 1
The minimum value is (126.90. 11) . /=1

And maximum value occurs when Example 35

sin(θ – 36.90) = -1 Show that if t = tanθ, then sin2θ = and


1
1 1
 Maximum value = = =1 . Hence solve the equation
( 1)

0
√ cos2θ + sin 2θ = 1 for 00 ≤ θ ≤ 3600.
Now for sin(θ – 36.9 ) = -1
Solution
θ – 36.90 = 2700

θ = 306.90
t √ 𝑡
The maximum value is ( )

The t-formula θ
Although this form has been tackled 1
indirectly, it is formally stated here
From the triangle above
Suppose that t = tan , we have 1
= and =
√1 √1

But =
t √ 𝑡 1
=. / . /
√1 √1
𝜃 1
=1
1
From the triangle above And =
1 1 1
= and = = . /.
1
/
√1 √1
√1 √1
1 1
But = =1
1
=. / . / Hence √ cos2θ + sin 2θ = 1
√1 √1
1
1  √ .1 / .1 /=
= 1

And =
1 1 √ √ =
1
1
(1+ √ ) √ =
= . /. /
√1 √1
√ (1 √ )(1 √ ) √1 1 √
=1 = (1 √ )
= (1 √ )
=1

digitalteachers.co.ug
t =1
1 √
= 1 or Let t = tan θ
√ 3t2- 2(1 + t2) = 2(5 -3t)
1 √ 1 √ 5t2 + 6t – 8 = 0
t=.1 / .1 / = -2 + √
√ √ √ ( )( ) 1
t= ( )
= 1
= -2 or
If tanθ = 1; θ = 450, 2250
Taking t = -2; θ = tan-1(-2) = 116.570
If tanθ = -2 + √ ; θ = 1650, 3450 Taking t = ; θ = tan-1( ) = 38.660
Hence θ = 38.660, 116.570
θ: θ = 450, 1650, 2250, 3450

Example 36 Example 38
(1 )
Find all the solutions of the equation Show that tan4θ = , where t = tanθ.
1
5cosθ – 4sinθ = 6 for -1800 ≤ θ ≤ 1800
Solution
Solution
tan4θ = and tan2 =
1 1
Let t = tan then . /
=
1 1 . /
= 1 (1 )
= 1
=
1
Example 39
1
 5.1 / – 4.1 /=6 Solve the equation cosθ + sinθ + 1 = 0 for
00 ≤ θ ≤ 3600
( ) = ( )
Solution
5 – 5t2 – 8t = 6 + 6t2
cosθ + sinθ + 1
11t2 + 8t + 1 = 0
√ 11 1 1 Let t = tan
= 11
=
1
1 1 1
=
t= = or
1
1 – t2 + 2t = 1(1 + t2)
t= = -0.5669
2t + 2 = 0; t = -1
Taking t = -0.1604
tan = -1
= =
=
Taking t = -0.5669
θ = 2700, 6300
= =
Hence θ = 2700
θ = -59.10, -18.20
Revision exercise 5
Example 37
1. Solve equation 3cosθ + 4sinθ = 2 for
Solve the equation 00 ≤ ≤ 3600 [119.60, 346.70]
1
3tan2θ + 2sec2 θ = 2(5 – 3tanθ) for 2. (a) Show that cos4x = 1
00 < θ <1800 (b) Show that if q = cos2x + sin2x, then
(1+q)tan2x – 2tanx + q – 1 = 0.

digitalteachers.co.ug
Deduce that if the roots of the above cos(A – B) = coaAcosB + sinAsinB ………… (ii)
equation are tanx1 and tanx2, the
sin(A + B) = sinAcosB + sinBcosA ………….(iii)
tan(x1+ x2) = 1
3. Find the values of R and tanα in each of sin(A – B) = sinAcosB – sinBcosA ………….(iv)
the following equations
eqn. (i) + eqn (ii)
(a) 2cosθ + 5sinθ = Rsin(θ + α) 0√ 1
cos(A + B) + cos(A – B) = 2cosAcosB
(b) 2cosθ + 5sinθ = Rcos(θ - α) 0√ 1
(c) √ cosθ + sinθ = Rcos(θ - α) 0
1
1 eqn. (i) – eqn (ii)

1 cos(A + B) - cos(A – B) = -2cosAcosB
(d) 5sinθ – 12cosθ = Rsin(θ - α) 0 1
(e) Cosθ – sinθ = Rcos(θ + α) [√ ] eqn. (iii) + eqn (iv)

sin(A + B) + sin(A – B) = 2sinAcosB


4. Find the greatest and least values and
state the smallest non-negative value of x eqn. (iii) - eqn (iv)
for which each occurs
sin(A + B) - sin(A – B) = -2sinBcosA
(i) 12sinx + 5cosx [13, 67.40; -13, 247.40]
(ii) 2cosx + sinx For simplification, A + B =α and A – B = β
[√ √ -
(iii) 7 + 3sinx – 4cosx Add: 2A = α + β i.e. A = . /
[12, 143.10; 2, 323.10]
(iv) 10 – 2sinx + cosx Subtract 2B = α - β i.e. A = . /
[10 + √ 10 - √ 116.60]
Substituting for A and B in the above equation
1 √ √
(v) 0 1
1 1 1 cosα + cosβ = 2cos. /cos. /
(vi)

0 1
1
(vii) [1, 112.60; 292.60] cosα - cosβ = -2cos. /cos. /
1 1

5. Solve each of the following equations for sinα + sinβ = 2sin. /cos. /
00 ≤ x ≤ 3600
(a) sinx + √ cosx = 1 [900, 3300] sinα - sinβ = 2cos. /sin. /
(b) 4sinx – 3cosx = 2 [60.40, 193.30]
1 Example 40
(c) sinx + cosx = [1050, 3450]

(d) 5sinx + 12cosx = 7 [80.00, 325.20] Show that if X, Y and Z are angles of a triangle,
(e) 7sinx – 4cosx = 3 [51.60, 187.90] then
(f) cosx – 3sinx = 2 [237.70, 339.20]
(a) cosX + cosY + cosZ – 1 = 4sin sin sin
(g) 5cosx + 2sinx = 4[63.80, 339.80]
(h) 9cox2x – 4sin2x = 6 [14.20, 141.80, solution
194.20, 321.80]
(i) 7cosx + 6sinx = 2 [118.10, 323.10] LHS cosX + cosY + cosZ – 1
(j) 9cosx – 8sinx = 12 [313.60, 323.10]
= 2cos cos + 1 – 2sin2 – 1
The factor formulae
(to eliminate -1)
The following identities were developed from
1 –
compound angles = 2cos cos – 2sin2

cos (A + B) = cosAcosB – sinAsinB ……….(i) (since X + Y = 1800 – Z)

digitalteachers.co.ug
= 2sin cos –2 sin2 = 2cos2(1800 – Z)cos2(X – Y) + 2cos22Z

(Since cos(900 – A) = sinA) = 2cos2Z[cos2(X – Y) + cos2{1800 – (X + Y)}]

1 ( ) = 2cos2Z[cos2(X – Y) + cos2 (X + Y)]


= 2sin 0 2 31
= 2cos2Z[2cos2X cos-2Y]
)
= 2sin 0 2 31
Since cos(-A) = cosA
0
(Since sin (90 – A) = cos A) =4cos2Z2cos2X cos2Y

= 2sin 0 1 (d) sin2Y + sin2Z = 1 + cos(Y – Z)cosX

LHS: sin2Y + sin2Z


=2sin 0 1
1 1
= ( ) ( )
(Since sin(-A) = -sinA)
1
4 as required = ( )
1
(b) Sin3X + sin3Y + sin3Z = = ( )
- 4cos
= 1 – cos(1800 –X)cos(Y – Z)
Solution =1 + cos(Y – Z)cosX
LHS: Sin3X + sin3Y + sin3Z Example 41
( ) ( )
= 2sin cos + 2sin cos (a) Factorize cosθcos3θ – cos7θ + cos9θ and
express it in the form Acospθsinqθsinrθ
(1 ) ( )
=2sin cos + 2sin cos wher A, p, q and r are constants

= -2cos cos
( )
+ 2sin cos Solution

f(θ)= cos9θ + cosθ – (cos7θ +cos3θ)


Since sin(2700 – A) = -cosA}
( )
= 2cos5θcos4θ – 2cos5θcos2θ
=-2cos 0 1
=2cos5θ(cos4θ – cos2θ)
( ) *1 ( )+
=-2cos 0 1 = -4cos5θ(-sin3θsinθ)
( ) ( )
=-2cos 0 1 =-4cos5θsin3θsinθ

 A = -4, p = 5, q = 3, r = 1
=-2cos 0 1
(b) Given that
Since cos(-A) = cosA p = sinα + sinβ
q = cosα + cosβ. Show that
=- 4cos
= sin(α + β)
(c) cos4X + cos4Y + cos4Z + 1
= 4cos2Xcos2Ycos2Z Solution

Solution

LHS: cos4X + cos4Y + cos4Z + 1 ( )( )


=
= 2cos2(X + Y)cos2(X – Y) + 2cos22Z -1 + 1

digitalteachers.co.ug
0 10 1 = ( ) ( )
=
(
= 2 + 2cos 2x – 1 + cos2x
0 10 1 2cos2x + 1 = 0
= 1
( ) cos2x = -
1
=
, ( )-,1 ( ) 2x = cos-1( ) = 1200, 2400
,1 ( )-
x = 600, 1200
= sin(α +β)
Alternatively
cos2x = 4cos2x -2sin2x
Example 42 cos2x –sin2x = 4cos2x -2sin2x
3cos2x –sin2x = 0
Solve 5cos23θ =3(1 + sin3θ) for 00 . Sin2x = 3cos2x
Solution tan2x = 3
tanx = √
5cos23θ =3(1 + sin3θ) Either
5(1 – sin23θ) = 3(1 + sin3θ) tanx = √
1 0
x= √ = 60
5 – 5sin23θ = 3 + 3sin3θ
Or
5sin23θ + 3sin3θ – 0 = 0 tanx = √
1
x= √ = 1200
(sin3θ + 1)(5sin3θ – 2) = 0
Hence x = 600, 1200
sin3θ + 1 =0
1 Alternatively
3θ = ( ) = -900, 2700
cos2x = 4cos2x -2sin2x
Example 43 1-2sin2x = 4(1-sin2x)-2sin2x
1 = 4 – 4sin2x
(a) solve the equation cos2x = 4cos2x -2sin2x 4sin2x = 3
for 0
sin2x =
cos2x = 4cos2x -2sin2x
cos2x – sin2x = 4cos2x -2sin2x sinx = √ =

3cos2x - sin2x = 0
4cos2x – 1 = 0
(2cos x + 1)(2cos x – 1) = 0 x = 600, 1200
Either
2cos x + 1 = 0 Alternatively
1 cos2x = 4cos2x -2sin2x
cos x =
1-2sin2x = 4cos2x -2sin2x
1
x = cos-1( ) = 1200 4cos2x = 1
Or 1 1
Cosx = √ =
2cos x - 1 = 0
cos x =
1 x = 600, 1200
1 (b) Show that if sin(x + α) = psin(x – α) then
x = cos-1( ) = 600 1
0 0
tan x = . /tanα.
x(60 , 120 ) 1
Hence solve the equation
Alternatively sin(x + α) = psin(x – α) for p = 2 and α=
cos2x = 4cos2x -2sin2x 200.

digitalteachers.co.ug
1 1
 tanA = and B =

sinxcosα + coxsinα = p(sinxcosα - coxsinα)
cosxsinα (p + 1) = sinxcosα (p - 1)
1 √
cosxsinα. /= sinxcosα 1
1
1
. /=
1 B
1 2
= . 1
/
For sin(x + 200) = 2sin(x – 200)  tan B =
1
1
tanx = 1
= 3tan200
11 1 1
x= 1
( ) = 47.520 LHS = + =A+B

1
Example 44 = , ( )-
1
Prove that in any triangle ABC, = 4 5
1 . /. /
( )
= 1
( ) = 1
1 1
Solution = = =
( ) ( )
= ( )
11 11
( (b) + =
=
Solution
( )( ) 11 11
= Let A = and B =
,1 ( -
1 1
 tanA = and tanB =
. / . / . / . /
= ( ) 1
LHS: ( ) but tan2A = =
( ) ( ) 1 . /
= ( )
1
( ) ( )=
= 1 . /. /
( )

1 1
Inverse trigonometric functions = . /

Note that = 1

1 1 1 =
(a) If θ = . / then cosθ =
1 1
1( 1 ) (c) =
(b) )= tan( =α
(c) 1, ( )- Solution
1 1
= cos[ ( ) = x+y Let θ= ; => x = cosθ = sin. /
1 ) 1( 1 )
(d) ( = sinx =
To avoid errors test the values 1 1
=
Example 45 Example 46
Show that Solve the equations
11 1 1
(a) +

= (a) tan-1(2θ + 1) + tan-1(2θ – 1) = tan-1(2)
Solution Solution
11 1 1
A= and B =

digitalteachers.co.ug
1 1
Let A = ( ) and B = ( ) ( )
=
 tanA = and tanB = 1 . /( )
1 1
A+B= 1
or tan(A +B) =2 1 1
=
1 1
= =
1 ( 1)( 1)
=
Example 50
2
4θ = 2(1 - 4θ – 1)
Without using tables or calculators determine
2
2θ + θ – 1 = 0 11 11 11
the values of .
(2θ – 1)(θ + 1) = 0 Solution
1
θ = or θ = -1 11 11 11
.
(b) tan-1(1+ θ) + tan-1(1 - θ) =32
11
Let A = 1
( ) and B = 1
( ) =
1 . /. /
 tanA = and tanB =
1 11
=
A+B= or tan(A +B) = tan32

Introducing tangents = = 1
. /=
1 . /. /
1 1
=
1 (1 )(1 )
Example 51
2
θ tan32 = 2 Solve equations
θ=√ = (a) 1 1
√ =
Example 47
Solution
If x = tan-1α and y = tan-1β; 1 1
Let A = and B = √
-1
Show that x + y = tan .1 /
A+B=
Solution
Cos (A + B) = cos. /
tanx = α; tany = β

(x + y) = tan[tan-1(x + y)] √ 𝑥 1
1
√ 𝑥
= tan-1. / B
1 A
x 𝑥√
Example 48

Solve the equation


( √ ) .√ / .√ /=
1
tan-1. 1
/ + tan-1(x + 1) = tan(-2)
( √ ) = .√ / .√ /
Solution
8x4 = ( )( )
-1 1 -1
Let A = tan . / and B = tan (x + 1)
1 1- 9x2 = 0
 A + B = tan-1(-2) (1 – 3x)(1 + 3x) = 0

digitalteachers.co.ug
1 1
Either x = or x = Revision exercise 6

We discard the negative value, so the root is 1. If p = sinα + sin β and q = cosα + cosβ show
1 that =
x=
2. (a) Prove that:
1 1
(b) 2 . /+ ( √ )= (i) (sin2x – sinx)(1+ 2cosx) = sin3x
1
Solution (ii) cos4θ =
1
(iii) =
1 1
Let A = . / and B = ( √ )
3. Solve the equation for 00 ≤ x ≤ 1800:
(a) sinx + sn3x + sin5x + sin7x = 0
2A + B =
[x: x = 00, 450, 900, 1350, 1800]
2A = (b) sin7x + sinx + sin5x + sin3x = 0
[x: x = 600, 1800]
Sin(2A) = sin . / (c) sinx + sin4x = 0
[x: x= 00, 600, 720, 1440, 1800]
2SinAcosA = cosB (d) cos (x + 100) -cos(x + 300) = 0

[700]
𝑥√ 1 (e) cos5x – sin2x = cosx
𝑥 1
[x: x = 00, 700, 900, 1100,1800]
B (f) sin2x +sin10x + cos4x = 0
A
√ 𝑥 [x: x = 22.50, 350, 550, 67.50, 950,
𝑥
√ 112.50, 1150, 157.50, 1750]

4. Show that
. / √ =√ 1 1
(a) =
1 1 1 √
(b) 2 . /= . /
√ =√
(c) the positive value that satisfies the
1 1
√ =√ equation = is
1

( )=( ) (d) 1( )= 1

1 1 1 1
X4 – 12x2 + 4= 0 (e) . / . /= . /
5. Prove that
1 √1 ( 1)
= 1 (a) = . / . /
1 √1 (b) Sin3x + sinx = 4sinxcos2x
= = √
(c) =
=√ √ (d) sin(A + B) – sin(A – B) =2cosAsinB
(e) =
After testing for =√ √ and for (f) Cos3x + cosx = 4cos2x – 2cosx
=√ √ , the value that satisfies the
equation is = √ √ = 0.5858
Hence the value of x = 0.5858

digitalteachers.co.ug
Solution to triangles Similarly;

In a triangle ABC b2 = a2 + c2 – 2accosB (2)


C c2 = a2 + b2 – 2abcosC (3)

It follows that
b a =

=
A c B
=
(a) Six elements are considered: three angles
and three sides (b) Given an obtuse angle A
C
Capital letters denote angles and small bold
and italics letters sides a
h b
0
(b) The opposite side of angle A is a, of angle (180 - A) A
D B
B is b and of angle C is c. x A c
(c) The angle sum of a triangle is two right In triangle ABC, A is an obtuse angle and CD is
angles i.e. A + B + C = 1800 the altitude.
(d) The sides are independent except that the
sum of the two sides of the triangle From triangle
should be equal to or greater than the ACD: x2 + h2 = b2 ………………………...(i)
third side BCD: (c – x)2 + h2 = a2
c2 – 2cx + x2 + h2 = a2 …………… (ii)
How to deal with triangles Substituting eqn. (i) into eqn. (ii)
1. The cosine rule c2 – 2cx + b2 = a2
(a) Given an acute angle A But
x = bcos (1800 – A) = -bccosA
C From triangle ACD
a2 = b2 + c2-2bccosA as before

b a The cosine rule can be derived using the


h vector approach.
C
A B
x D c-x
c
b a
From triangle
ACD: x2 + h2 = b2 ………………………...(i)
BCD: (c – x)2 + h2 = a2
c2 – 2cx + x2 + h2 = a2 …………… (ii) A c B
Substituting eqn. (i) into eqn. (ii)
c2 – 2cx + b2 = a2 Given a triangle above with BC = a, AC = c and
But AB = b
x = bcosA
BC = BA + AC = AC – AB
 b2 + c2 – 2bccosA = a2
a2 = b2 + c2-2bccosA (1) a=b–c

digitalteachers.co.ug
 a.a = (b – c)(b – c) a2 = b2 + c2 – 2bc + 4 sin2. /
= b.b – 2b.c + c.c
= b.b + c.c – 2b.c a2 =(b – c)2 + 4bcsin2. /
a = b2 + c2 – 2bccosA
2

since b.c = | |cosA Hence, substituting for sin2. /into tanα


expression we get
Example 52
a2 = (b – c)2 + (b – c)2tan2α
Solve the triangle in which AB = 6cm, BC =
4cm and angle ACB = 48012’ a2 = (b – c)2(1 + tan2α)

Solution a2 = (b – c)2sec2α
C
a = (b – c)secα

4cm 2. The Sine Rule


A
48012’
A 6cm B

Using: b2 = a2 + c2 -2accosB

= 62 + 42 – 2(6)(4)cos48.20 O
r r
10 (degree) = 60’ (minutes) A A

b = 4.47cm B C
½a D
1
Using: = = ( )( ) The figure shows a circle with centre O and
A =41.80 radius r circumscribing triangle ABC

But A + B + C = 1800 Angle BOC = 2A [angle subtended by the same


arc at the centre of the circle is twice the
41.80 + 48.20 + C = 1800 angle formed at any point on the
circumference]
C = 900
Triangle BOC is isosceles
AC = 4.47cm, angles BAC = 41.80 and
ACB = 900 OD bisects angle BOC and side BC
Example 53 BD = ½a
In a triangle ABC, prove that From triangle BOD

(a) a2 =(b – c)2 + 4bcsin2. / hence that sinA = i.e. =


√ . /
a = (b – c)secα where tanα = if instead we consider triangles AOC and AOB,
From cosA = 1- 2 sin2. / we obtain = and =
Substituting for cosA into the cosine
In general: = =
formula a2 = b2 + c2 – 2bccosA
a2 = b2 + c2 – 2bc[1 - 2 sin2. /]

digitalteachers.co.ug
Example 54 ( ) ( ) ( ) ( )
= ( )
Solve the triangle in which AB = 5cm, AC =
4cm and angle ACB = 600 ( ) ( ) ( )
= = ( )
Solution
( )
Hence = ( )
C
Example 56
0
60 Prove that in any triangle ABC,
4cm
1 1
( )=

A B Solution
5cm
From sine rule formula;
a = 2rsinA, b = 2rsinB, c = 2rsinC
Using sine rule
By substitution
1
= => B = . /
= =
1
B= . / = 43.90
But A + B + C = 1800
0
From A + B + C+ = 180
A = 1800 – (B + C)
A = (180 – 60 – 43.9)0 = 76.10
sinA = sin[1800 – (B + C)] = sin(B + C)
1
Similarly a = = = 5.6cm By substitution
̅̅̅̅ = ̂ = ̂ =
= = ( )
Example 55
( ) ( )
Prove that in any triangle =
( ) ( )

( )
= ( )
( ) =
( )
Solution
From A + B + C = 1800
From sine rule formula;
a = 2rsinA, b = 2rsinB, c = 2rsinC B + C = 1800 – A
1 1
By substitution ( )=. /

( ) ( ) 1 1 1
= = ( )= . /=
( )

But A + B + C = 1800 By substitution

C = 1800 – (A + B) ( )
=
sinC = sin[1800 – (A + B)] = sin (A + B)
1 1
( )=
By substitution
( )(
= ( )
== ( )

digitalteachers.co.ug
3. The Tangent Rule ( ) ( )
=
( ) ( )
It states that in a triangle ABC
( ) ( )
1 1 =
( )= . / ( ) ( )

1 1 ( ) ( )
( )= . / =
( )
1 1
( )= . /
( )
=

1 1
Proof = ( )

From = = 1 1
( )= . /
a = 2rsinA, b = 2rsinB, c = 2rsinC Hence
= =
( )=
( ) ( )
= = -0.3621
( ) ( )
1
( ) ( ) ( )= i.e. Q – R = -39.90
=
( ) ( )
But P + Q + R = 180
( ) ( )
= Q + R = 180 - 39.9 = 140.10
( )

Solving Q = 50.150 and R = 89.950


( )
=
1 0 1
Now p = = 1
= 12.79
1 1
= ( )
p = 12.79, Q =50.150, R = 89.950
1 1
( )= . / Example 57
Example 56 Show that =
1 1

Show that in a triangle PQR


Solution
1 1
( )= . / LHS =
=
Hence solve the triangle in which q =15.32, r =
28.6 and P = 39052’ =
( ) ( )
Solution =
( ) ( )
From = = . / ( )
=
. / ( )
p = 2rsinP, q = 2rsinQ, r = 2rsinR ( )
=
( )
= =
( )
=
( )

digitalteachers.co.ug
1
= . /
( ) . ( )/
=
( ) . ( )/
=. /
( ) ( )
= ( )
( ) ( ) =. /
. /
= ( )( )
=. /
1 1
= tan tan
( ) ( )
=. /
Expressions for sinA, sin and cos in
( )( )
=. /
terms of the sides of the triangle
(a) sinA sin
1
= √.
( )( )
/
From the identity
Similarly;
sin2A = 1 – cos2A = (1 – cosA)(1 + cosA)
1 ( )( )
sin = √. /
=. /. /
1 ( )( )
=. /. / sin = √. /

[ ( ) ][( ) ] Also;
=
1 1
2 ( )( )( )( ) Cos2 = ( )
sin A =
1 =. /
Let s = , -
( )
= ,
1
- =. /

( )( )
2s =[a + b + c] =. /

a + b = 2s – c; i.e. a + b – c = 2s – c – c = 2(s – c) =.
( )
/
a + c= 2s – b; i.e. a + c – b = 2s – b – b =2(s – b) ( )
=. /
b + c = 2s – a; i.e. b + c – a = 2s- a – a = 2(s – a)
( ) ( ) ( ) 1 ( )
sin2A = cos = √. /

sinA = √ ( )( )( ) Similarly;

1 ( )
Similarly, sinB = √ ( )( )( ) cos = √. /

sinC = √ ( )( )( ) 1 ( )
cos = √. /
(b) sin and cos
1 1
From sin2 = ( )

digitalteachers.co.ug
1 1
The expression for can be deduced as s= ( )= ( )=
follows
28 + b + c = 84
1 ( )( )
tan = =√ b + c = 56, or c = 56 – b
( )

But Δ2 = ( )( )( )
Similarly;
3362 = 42(42–28)(42– b)(42 – 56 + b)
1 ( )( )
tan =√ ( ) b2 – 56b + 780 = 0

1 ( )( ) √ 1
tan =√ = 1
( )

Area of a triangle b = 30 or 26

C substituting for c = 56 – b

c = 26 or 30
b a the remaining sides are 30m and 26m
h
Applications of trigonometry in finding
A c B distances and bearings
Example 59
1
Area, Δ = ( )( )
A vertical pole BAD stands with its base D on a
1 horizontal plane where BA = a and AD = b. A
=
point P is situated on the horizontal plane at a
1 distance C from D and the angle APB = θ.
=
1
Prove that θ = . /
Substituting for
Solution
sinA = √ ( )( )( )
B
1
Δ= √ ( )( )( ) a

Δ =√ ( )( )( ) A
b θ
This a convenient form given the three sides
of a triangle. The formula is called Hero’s α
c P
D
formula from the first mathematician who
suggested it. Let angle APD = α
Example 58
For triangle APD: tanα =
2
The area of a triangle is 336m . The sum of the
three sides is 84m and one side is 28m. For triangle DPB: tan(θ+α) =
Calculate the length of the remaining two
sides  1
=

Solution Substituting for tanα

Given Δ = 336, a + b + c =84 and a = 28

digitalteachers.co.ug
 =
1 . /
B
c2tanθ + bc = ac + bc –abtanθ – b2tanθ 1
1 13
2 2
(b + ab + c )tanθ = ac 38 C
1
tanθ = 38
α
1 A
θ= . /

Example 60

The angle of the top of a vertical tower from a From the digram
point A is 200 and from another point B is 500. Let B ̂ C = B ̂ A = θ
Given that A and B lie on the same horizontal
1
plane in the same direction where AB = 100m.  2θ + 13 = 1800
Find the height of the tower 1
θ = 83
Solution
1
But 38 + α = θ
Let OT be the height of the tower
1 1
T 38 + α = 83

α = 450

From the sine rule


1
500 200 =
1
O B A
100m
AC = 29km

he has to fly 29km in the direction S450W


A ̂ B = 50 – 30 = 30 0

Example 62
Using sine rule
1 (a) Prove that tan(A – B) =
= 1

( )
1 tan (A – B) = ( )
TB =
=
But OT = TBsin500
1 Diving numerator and denominator on the
OT = = 26.2m
R.H.S by cosAcosB
Example 61

From a point A, a pilot flies in the direction tan (A – B) =


N38020’W to point B 125km from A. He then
flies in the direction S50040’E for 125km. He =1
wishes to return to A from this point. How far
and in what direction must he fly. 1 1 1
Hence show that =
1 1 √
Solution

digitalteachers.co.ug
1 1
=
1 cos (θ + 30)0 – cos (θ + 48)0 = 0.
1 1 1 1
1
= tan (450 – 150) tan300 =  -2sin (θ + 390)sin(-90) = 0.2

sin (θ + 390) = 0.63925
1
(b) Given that cos A = and cos B = 1 where θ + 390 = 39.740
A and B are acute, find the values of
θ = 0.740
(i) tan (A + B)
(ii) cosec (A + B) Example 64

Solution Express 7cos 2θ + 6sin 2θ in form


Rcos (2θ – α), where R is a constant and α is
an acute angle.

7cos 2θ + 6sin 2θ Rcos (2θ – α)


7cos 2θ + 6sin 2θ Rcos 2θ cos α +
Rsin2θsinα
Comparing both sides
Rcosα = 7 ………………… (i)
1 Rsinα = 6 ………………… (ii)
cos A = cos B = 1
(i)2 + (ii)2 gives
sin A = sin B = 1 R=√ =√
From equation (i)
tan A = tanB = √ cosα = 7
1
1
= . / = 40.60
(i) tan (A + B)= √

= = 3.9375 Hence solve 7cos 2θ + 6sin 2θ = 5 for 00


1 1800. (07marks)
(ii) cosec (A + B) =
(
1 7cos 2θ + 6sin 2θ = √ cos(2θ – 40.60) =5
=
1 = 1
. / = 57.160, 302.840
= √

= 1.0317 =

Example 63

Express cos (θ + 30)0 – cos (θ + 48)0 in the Revision exercise 7


form RsinPsinQ, where R is constant. 1. Solve the triangles
Hence solve th3 equation (a) a = 17m, b = 21.42m, B = 51034’
[A =38.440, C = 900, c = 27.34m]
cos (θ + 30)0 – cos (θ + 48)0 = 0.2 (b) b = 107.2m, c= 76.69m, B =102025’
Solution [A =33.260, C = 44.320, a = 60.21m]
(c) a = 7m, b = 3.59m, C= 470
cos (θ + 30)0 – cos (θ + 48)0 [A= 10302’, B = 29052’, c= 5.25m+
(d) A = 600, b = 8m, C = 15
= -2sin. / . / [a = 13, B= 32.20, C = 87.80]
2. Show that for all values of x
= -2sin (θ + 390)sin(-90)
. / . /=

digitalteachers.co.ug
( ) 11 11 11
3. (a) Simplify 0 1 0 1
(b) Express 5sinθ + 12cosθ in the form 8. (a) Prove that = = =
rsin(θ + α) where r and α are constant.
where ABC has all angles acute and R is
Hence determine the minimum value the radius of the circumcircle.
of 5sinθ + 12cosθ + 7.
(b) From the top of a vertical cliff 10m
*r =13, α = 67.40, -6] high, the angle of depression of ship A
(c) Given that tanθ = , where θ is acute, is 100 and ship B is 150. The Bearings
find values of tan2θ and tan of A and B from the cliff are 1620 and
1 202.50 respectively. Find the bearing
*tan2θ = and tan = ]
of B from A [301.50]
1 1 1
4. (a) Show that . / . /= 9. (a) Prove that
(b) Find x given that (sin2x –sinx)(1 + 2cosx) = sin3x
1( ) 1( )= (b) A vertical pole BAO stands with its
[x = ] base O on a horizontal plane, where
(c) Given that sinα+ sinβ = p and BA = c and AO = b, a point P is situated
cosα + cos β = q on horizontal plane at a distance x
Show that sin(α + β) = from O and angle APB = θ
Prove that tanθ =
5. (a) By expressing 2sinθsin(θ + α) as a
difference of cosines of two angles or As P takes different positions on the
otherwise, where αis constant, find the horizontal plane, find the value of x
for which θ is greatest.
least value [minimum value = cosα – 1. It
[18026’, when x = b = c+
occurs when θ = ]
10. (a) Prove that sin3x = 3sinx – 4sin2x.
(b) Solve for x in the equation (b) Find all the solutions to 2sin2x = 1 for
cosx – cos(x + 600) = 0.4 for 00≤ x≤ 3600. [x =100, 500, 1300, 1700,
00 ≤ x ≤ 3600 [x: x = 126.40, 353.60] 2500, 2900]
6. (a) Prove that in any triangle ABC
11. Solve cosx +√ sinx =2 for 00 ≤ x ≤ 3600
( )
= ( ) [x = 600]
(b) Show that for any isosceles triangle 12. From the top of a tower 12.6m high, the
ABC with AB = c the base, is given by angles of depression of ship A and B are
1 120 and 180 respectively. the bearing of
Δ= √ ( ) where s is the
ship A and ship B from the tower are 1480
perimeter of the triangle
and 209.50 respectively
Given that Δ= √ and s = 4, determine Calculate
the sides of the triangle [1, 3.5, 3.5] (i) How far the ships are from each other
1 1
7. Given that = and = , [53.14m]
by expressing α and β as tangents ratio of (ii) The bearing of ship A from ship
x and y and manipulating the ratios show B[108.10]
1
that x + y = . / 1
13. (a) Solve sin3x + = 2cos2x for
1
Hence or otherwise 00 ≤ x ≤ 3600
(i) Solve for x in [x = 300, 600, 1200, 1500, 2400, 3000]
1 1 (b) Given that in any triangle ABC,
. / ( )= ( )
1 tan. /= . / solve the
[x = ]
triangle with two sides 5 and 7 and
(ii) Without using tables of calculators the included angle 450.
determine the value of [A = 450, B = 89.40, C= 45.60]

digitalteachers.co.ug
14. (a) Solve cot2x = 5(coscx + 1) for 23. Without using tables or calculator, show
00 ≤ x ≤ 3600 [9.60, 170.40, 2700] that tan150 = 2-√
(b) Use tan =t to solve 5secθ – 2sinθ = 2 24. (a) Solve the equation cosx + cos2x = 1 for
for 00 ≤ x ≤ 3600 [46.40, 2700] 00 ≤ x ≤ 3600 [x = 38.670, 321.330]
15. Given that sin2x = cos3x, fins the values of (b) (i) Prove that =
sinθ, 0 ≤ x ≤ π *0.309 3dp+ (ii) = where A, B and
16. (a) Show that
C are angles of a triangle
. / . /= 25. Given that sin(θ – 450) = 3cos(θ + 450)
(b) Find in radians the solution of the show that tanθ = 1. Hence find θ if
equation cosθ + sin2θ = cos3θ for 00 ≤ θ ≤ 3600 [450, 2250]
0≤θ≤π0 1 26. (a) Use the factor formula to show
( )
17. (a) Show that cotA + tan2A = cotAsec2A that ( )
= ( )
(b) Show that = , (b) Express y = 8cosx + 6sinx in the form
1
Rcos(x – α) where R is positive and α
where t = tanx. Hence or otherwise
1
is acute
show that .1 / = √ Hence find the maximum and
18. (a) Find all the values θ, 00 ≤ θ ≤ 3600, minimum values of
1
1
which satisfies the equation
[0.2, 0.04]
sin2θ – sin2θ – 3cos2θ = 0 *θ =1350, 3150]
27. Express sinx + cosx in the form Rcos(x – α).
(b) Show that 1 = . /. Hence, find the greatest value of
1
Hence or otherwise solve = ; sinx + cos x – 1. [0.4142]
1
00 ≤ x ≤ 3600 [x = 36.80] 28. (a) Solve cosx + cos3x= cos2x, 0 ≤ x ≤ 3600
19. (a) Given that X, Y and Z are angles of a [x = 450, 600, 1350, 2250, 3000, 3150]
1
triangle XYZ. Prove that (b) Show that . /=
. /= . 11 11 1
29. Show that =
Hence solve the triangle if x = 9cm, y = 30. (a) Solve 3sinx + 4cosx = 2 for
5.7cm and Z = 570. [z = 7.6cm, X= 84.40] -1800 ≤ x ≤ 1800. [-29.550, 103.290]
(b) Use the substitution t = tan. / to (b) Show that in any triangle ABC
( )
solve the equation 3cosθ – 5sinθ = -1 = ( )
for 00 ≤ θ ≤ 3600 [40.840, 201.10] 31. (a) Prove that 1 =
20. Prove that
(b) Solve sin2x = cosx; 00 ≤ x ≤ 900
. / . /= [x = 300, 900]
21. (a) Solve the equation 3cosx + 4sinx = 2 32. (a) Solve the equation
for 00 ≤ x ≤ 3600 [x = 119.50, 346.70] 8cos4x – 10cos2x + 3 = 0; 00 ≤ x ≤ 1800
(b) If A, B, C are angles of a triangle. Show [300, 450, 1350, 1500]
that (b) Prove that cos4A – cos4B – Cos4C =
4sin2Bsin2Ccos2A -1 given that A, B
cos2A + cos2B + cos2C = -1 –4cosAcosB
and C are angles of a triangle
22. (a) Solve 2sin2θ = 3 for -1800 ≤ x ≤ 1800 33. Given that cos 2A - cos 2B = -p and
[-900, 48.60,900, 131.40] sin2A – sin2B = q, prove that
(b) Solve sinx – sin4x = sin2x – sin3x for 1
sec (A + B) = √
–π ≤ x ≤ π
34. Solve
0 1 (a) 4sin2θ – 12sin2θ + 35cos2θ = 0; for
00 [ ]

digitalteachers.co.ug
(b) 3cosθ – 2sinθ = 2, for 00 37. Solve the equation
, = - 3tan2θ + 2sec2 θ = 2(5 – 3tanθ)
35. Solve the equation sin2θ + cos2θcos4θ = for 00 < θ <1800 [θ = 38.660, 116.570]
(1 )
cos4θcos6θ for 0 θ .0 = 1
1 38. (a) Show that tan4θ = , where
1
2
36. (a) solve the equation cos2x = 4cos x - t = tanθ
2sin2x for 0 [θ = 600, 1200] (b) Solve the equation
(b) Show that if sin(x + α) = psin(x – α) sinx + sin5x = sin2x + sin4x
then tan x = .
1
/tanα. Hence solve for 00 < x < 900.[x = 600]
1
39. Solve 2cos2θ – 5cos θ = 4
the equation sin(x + α) = psin(x – α) for
for 00 θ 3600.[ θ = 138.590, 221.410]
p = 2 and α= 200. [x = 47.520]

Thank you
Dr. Bbosa Science

digitalteachers.co.ug
Series
Introductions

Numbers arranged in a definite order a Example 1


sequence. Each number in the sequence is Find the 30th term of a series that has an nth
derived from a particular rule.
term given by
The terms below are examples of sequences
Solution
(a) 1, 3, 5, 7, 9 ….. is a sequence of odd
numbers Un =
(b) 2, 3, 5, 7, 11 … is a sequence of prime
number U30 = =1
(c) 4, 16, 64 …. is a sequence formed by
Example 2
multiplying the proceding number by 4 to
give the next number The first term of an arithmetic progression
(A.P) is 73 and the 9th term is 25. Determine
Series are categorized into two:
the common difference
- Arithmetic progression (A.P)
Solution
- Geometric progression
Un = a + (n – 1)d
Arithmetic progression (A.P)
25 = 73 + (9 – 1)d
This is a series in which each term is obtained
from the preceding one by addition or 25 = 73 + 8d
subtraction of a constant quantity.
d = -6
The series 1 + 3 + 5 + 7 + 9 ….. is an A.P
Example 3
Note the following in an A.P
The 3rd, 5th and 8th terms of A.P are 3n + 8, n +
(i) The first term of an A.P is denoted a. the 34, and n3 + 15 respectively. Find the value of
first letter of the English alphabet n and hence the common difference of the
(ii) There is a common difference d. in the A.P
progression, a = 1 and d = 2.
Solution
(iii) Given the first term, a and the common
difference, d a + 2d = 3n + 8 ……………… (i)
1st term = a
a + 4d = n + 24 ………………(ii)
2nd term = a + d
3rd term = a + 2d a + 7d = n3 + 15 …………….(iii)
nth term (Un) = a + (n – 1)d

digitalteachers.co.ug
Eqn. (ii) – eqn. (i) Solution

2d = -2n + 16 By substituting for n

d = -n + 8…………………… (iv) n = 1:

eqn. (iii) – eqn. (i) 3a + b + c = 4 ……………..(i)

5d = n3 – 3n + 7 ………………… (v) n =2

Substituting eqn. (iv) into eqn. (v) 9a + 2b + c = 13 …………(ii)

5(-n + 8) = n3 – 3n + 7 n=3

n3 + 2n – 33 = 0 27a + 3b + 2 = 46 …………(iii)

By factorizing the equation Eqn. (ii) – eqn. (i)

(n- 3)(n2 + 3n + 11) = 0 6a + b = 9 …………………. (iv)

Either n – 3 = 0 or n2 + 3n + 11 = 0 Eqn.(iii) – eqn (ii)

n = 3 since n2 + 3n + 11 = 0 has no real roots 18a + b = 33 ……………… (v)

Substituting for n in eqn. (iv) Eqn. (v) – eqn. (iv)

d = -n + 8 12a = 24

d = -3 + 8 = 5 a=2

Hence n = 3 and the common difference is 5 Substitute for a in (v)

Example 4 36 + b = 33

An A.P has the first term 3, common b = -3


difference -2 and nth term – 15. Find n and
Substituting for a and b in eqn. (i)
the (n -3)th term
3x2–3+c=4
Solution
c=1
Given, a = 3, d =-2 , Un = -15
Hence a = 2, b = -3 and c = 1
But Un = a + (n – 1)d

-15 = 3 + (n – 1) x ( – 2) The sum of the first n terms of an A.P

n = 10 There are two formulas used for finding the


sum of the first n terms of the A.P depending
substitute n – 3 for n on the terms given
U(n – 3) = 3 + [(10 – 3) – 1](-2) Formula A
= -9 If the first term, a and common difference are
given, then sum (Sn) of the first n terms is
Example 5
given by
The nth term of a series is Un = a3n + bn + c
given that U1 = 4, U2 = 13 and U3 = 46, find the Sn = )
values of a, b and c.
Formula B

digitalteachers.co.ug
If the first term is a and the last term is L, then Substituting d in eqn. (i0
sum of the first n terms (Sn)is given by
a + 9 x 2 = 29
Sn =
a=2
Example 6 Sum of the first 30 terms
The first term of A.P is 73 and the common Sn = )
difference is -6, find the number of terms that
must be added to give a sum of 96 Sn = = 1365
Solution
Example 8
Sn = The 5th term of an arithmetic progression (A.P)
is 12 and the sum of the first 5 terms is 80.
= 93
Determine the first term and common
n(73 – 3(n – 1)) = 96 difference.

73n – 3n2 + 3n = 96 Solution

3n2 – 76n + 96 = 0 Un = a + (n – 1)d [ Un= nth term, a = first term


and d = common difference}

U5 = a + (5 – 1)d

a + 4d = 12 …………………………….(i)

n = 24 The sum of the first n terms,

Hence the number of terms that must be Sn = )


added to give a sum of 96 are 24
S5 = )
Example 7
10a + 20d = 80 x 2 = 160 ………(ii)
The tenth term of an arithmetic progression
(A.P) is 29 and the fifteenth term is 44. Find Eqn. (ii) – 5eqn. (i)
the value of the first term and the common
5a = 100
difference. Hence find the sum of the first 30
terms a = 20
Solution Substituting for a in eqn. (i)
Un = a + (n – 1)d 20 + 4d = 12
U10 = a + (10 – 1)d d = -2
a + 9d = 29 …………….(i) Hence the first term = 20 and the common
difference = -2
U15 = a + (15 – 1)d
Example 9
a + 14d = 44 ………… (ii)

Eqn (ii) – eqn. (i) (a) Prove that ∑

5d = 15

d=3

digitalteachers.co.ug
Solution 1st term a = 2
4th term, ar3 = 16
Sn = 1 + 2 + 3 + ….. + n
2(r3) = 18
+ Sn = n + (n – 1) + (n – 2)+ …. + 1 r=2
the second term, ar = 2 x 2 = 4
2Sn = (n+1) + (n+1) + (n+1) + …. + (n+ 1) the third term, ar2 = 2 x 22 = 8
2Sn = n(n + 1) (b) Insert three geometric means between 1
and 81
Sn = a=1
the 5th term ar4 = 81
(b) Use your answer in (a) to deduce
1(r4) = 81
(i) ∑ r=3
the second term, ar = 1 x 3 = 3
Note to deduce is to use the already
the third term, ar2 = 1 x 32 = 9
existing result to work out other problems
the third term, ar3 = 1 x 33 = 27
[ ]

∑ ( ) Revision exercise 1
1. Find the 5th and 8th terms of a series that
has an nth term given by (-1)n(2n + 1)
[-11, 17]
2. The first term of an arithmetic progression
(ii) ∑ (A.P) is . The sixth term of th A.P is four
times the fourth term. Find the common
∑ ∑
difference of the A.P * +
( ) 3. The sum of p terms of an arithmetic
progression is q and the sum of q terms is
p; find the sum of p + q terms
4. (a) the first four terms of an A.P are 5, 11,
17 and 23. Find the 30th term and the sum
of the first 30 terms [179, 2760]
(b) the second term of an A.P is 7 and the
Inserting geometric means 7th term is -8. Find the first term, common
difference and the sum of the first 14
Like for A.Ps, the terms inserted between terms [ 10, -3, -133]
given two values of a G.P are known as 5. (a) An A.P has the first term of 2 and
geometric means. common difference 5. Given that the sum
If n terms are inserted, then the total number of the first n terms of the progression is
of terms will be n+ 2 with the two extreme 119, calculate n [7]
values representing the first and last terms (b) the sum of the first five terms of an
respectively A.P is . Also , five times the 7th term is
the same as six times the second term.
Example 10
Find the first term and the common
(a) Insert two geometric means between 2 difference * +
and 16
Solution

digitalteachers.co.ug
6. The sum of the first n terms of the a series The common ratio, r =
is n(n + 2). Find the first three terms
[3, 5, 7]  b2 = ac
7. In an A.P, the 1st term is 13 and 15th term b = √ , the geometric mean
is 11. Find the common difference and
Example 10
sum of the first 20 terms [7, 1590]
8. (a) Show that In2r, r = 1, 2,3, is an A GP has 3rd term 7 and 5th term 847. Find the
arithmetic progression possible values of the common ratio and the
(b) find the sum of the first 10 terms of corresponding 4th terms
the progression [38.1231]
(c) Determine the least value of m for Solution
which the sum of the first 2m terms U3 = ar2 = 7
exceeds 883.7 [25]
9. In an arithmetic progression u1 + u2 + u3 +  ar2 = 7 ………………..(i)
u4 = 15 and u16 = -3. Find the greatest U5 = ar4 = 847
integer N such that UN ≥ 0. Determine the
sum of the first N terms of the  ar4 = 847 ……………(ii)
progression. [ N = 14, S14 = 136.5]
Eqn. (ii) eqn. (i)

r2 = 121
Geometric progression (G.P) r = 11
It is a series in which each term is obtained From eqn (i)
from the preceding one by multiplication or
division by a constant quantity. a=
Observations The 4th term, U4 = ar3
- The first term of G.P is also denoted, a
If r = 11, U4 = = 77
- The common ratio is r
- Given a and r If r = -11, U4 = = -77
1st term = a
2nd term = ar Example 11
3rd term = ar2
4th term = ar3 In a G.P the 2nd term is 15 and the 5th term is -
405. Find the sum of the first 8 terms
nth term = arn-1

The sum (Sn) of the first n terms of G.P Solution

U2 = ar = 15
The sum (Sn) of the first n terms of G.P is given
by  ar = 15 ……………………..(i)

Sn = for r > 1 or U5 = ar4 = -405

 ar4 = -405
Sn = for r< 1
Eqn. (ii) eqn. (i)
The geometric mean (G.M)
r3 =-27
Suppose three numbers a, b and c are
consecutive terms of GP, then, the middle r = -3
term is the geometric mean.

digitalteachers.co.ug
From eqn. (i), a = -5 Ar6 – ar3 = 1404
ar3(r3 -1) = 156 ……………. (ii)
Since r < 1
Eqn. (ii) eqn. (i)
Sn = = =
r2 = 9
Example 12 r=
In the geometric series u1 + u2 + u3 + … r = 3 and r = -3

u1+ u3 = 26 and u3 + u5 = 650. Example 14

Find the possible values of u4 (a) The first three terms of a Geometric
progression (G.P) are 4, 8 and 16. Determine
Solution the sum of the first ten terms of the G.P.
u1+ u3 = 26 (04marks)

a + ar2 = 26 solution

a(1 + r2) = 26 ………………… (i) a = 4, ar = 6


4r = 8
u3 + u5 = 650. r=2
ar2 + ar4 = 650 Sn =

ar2(1 + r2) = 650 ……………(ii) S10 = 4( )= 4092

Eqn. (ii) eqn. (i)

r2 = 25 (b) An Arithmetic Progression (A.P) has a


common difference of 3. A Geometric
r= 5
Progression (G.P) has a common ration of 2. A
From eqn. (i) sequence is formed by subtracting the term of
the A.P from the corresponding terms of the
a(1 + 25) = 26 G.P. The third term of the sequence is 4. The
a=1 sixth term of the sequence is 79. Find the first
term of the
u4 = ar3
(i) A.P (08 marks)
If r = 5; u4 = a(5)3 = 125
(ii) G.P (06 mars)
If r = 5; u4 = a(-5)3 = -125
A.P
Example 13 x, x+3, x +6, x + 9, x + 12, x+ 15, ……
In a Geometric Progression (G.P), the G.P
difference between the fifth and the second y, 2y, 4y, 8y, 16y, 32y, ….
term is 156. The difference between the 4y – (x + 6) = 4
seventh and the fourth is 1404 Find the 4y – x = 10 ……………..(i)
possible values of the common ratio. 32y – (x + 15) = 79
32y – x = 94………….. (ii)
U5 – U2 = 156 Eqn. (ii) – Eqn. (i)
ar4 – ar = 156 28y = 84, => y = 3
ar(r3 -1) = 156 ……………. (i) Substituting for y into eqn. (i)
U7 – U4 = 156 12 – x = 10

digitalteachers.co.ug
x =2 when r =
(i) A.P, U1 = 2
(ii) G.P, U1 =3 the terms are
Example 15 Hence the terms in their order are 2, 6, 18 or
The sum of the first n terms of a geometric 18, 6, 2
progression (G.P) is . Find the nth
term as an integral power of 2
Inserting geometric means
Solution
Like for A.Ps, the terms inserted between
Sn = given two values of a G.P are known as
geometric means.
Comparing with Sn =
If n terms are inserted, then the total number
a=4 of terms will be n+ 2 with the two extreme
values representing the first and last terms
r – 1 = 3, r = 4 respectively
The nth term, Un = arn – 1 Example 16
= 4 x 4n-1 = 22 x 22(n-1) = 22+2n-2 = 22n (c) Insert two geometric means between 2
and 16
Example 16
Solution
Find three numbers in geometrical 1st term a = 2
progression such that their sum is 26 and their 4th term, ar3 = 16
product is 216 2(r3) = 18
r=2
Solution
the second term, ar = 2 x 2 = 4
Let the numbers be , a and ar the third term, ar2 = 2 x 22 = 8
(d) Insert three geometric means between 1
Product = ( ) = 216 and 81
a=1
a3 = 216 = 63 the 5th term ar4 = 81
1(r4) = 81
a=6
r=3
the terms are the second term, ar = 1 x 3 = 3
the third term, ar2 = 1 x 32 = 9
Sum of terms = 26 the third term, ar3 = 1 x 33 = 27

 6r2 – 26r + 6 = 0 Mixed terms of A.P and G.P


3r2 – 13r + 3 = 0 These are problems involving both A.Ps and
(r – 3)(3r – 1) = 0 G.Ps. when handling we make use of their
Either r – 3 = 0; r = 3 respective properties.
Or 3r - 1 = 0; r =
Example 17
When r = 3
A geometric progression (G.P) and an
the terms are arithmetic progression (A.P) have the same
first term. The sum of their first, second and

digitalteachers.co.ug
third terms are 6, 10.5 and 18 respectively. 9, 9 + 3d, 9 + 7d
Calculate the sum of their 5th terms.
For a G.P, r =
Solution
 (9 + 3d)2 = 9(9 + 7d)
Terms G.P A.P Sum 81 + 54d + 9d2 = 81 + 63d
1st a a 2a = 6 …………………(i)
9d2 – 9d = 0
2nd a+d ar a + d + ar = 10.5 ….(ii)
9d(d – 1) = 0
3rd a + 2d ar2 a + 2d + ar2 = 18 ..(iii)
Either d – 1 = 0; d = 1
Or d = 0
From eqn. (i): 2a = 6; a = 3
When d = 0 all terms of A.P are equal
From eqn. (ii): 3 + d + 3r = 10.5
Hence the common difference d = 1
d + 3r = 7.5 ……………….. (iv)
Example 19
2
From eqn. (iii) 3 + 2d + 3r = 18
(a) The sum of the first m terms of a
2
2d + 3r = 15 ……………..(v) progression is m(2m + 11)
(i) Show that the progression is an A.P
Eqn. (v) – 2eqn. (iv)
(ii) Determine the nth term of the
3r2 – 6r = 0 progression

3r(r -2) = 0 Solution

r -2 = 0 Given Sm = m(2m + 11)

r=2 First term = S1 = 1(2 x 1 + 11) = 13

Substitute for r into eqn. (iv) S2 = 2(2 x 2 + 11) = 30

d + 6 = 7.5 Second term = 30 – 13 = 17

d = 1.5 S3 = 3(2 x 3 + 11) = 51

Sum of their fifth terms Third term = 51 – 20 = 21

= (a + 4d) + ar4 The progression is 13, 17, 21, ….. Hence A.P
with the first term 13 and common difference,
= (3 + 4 x 1.5) + 3 x 24 = 57 d= 4
Example 18 (ii) Un = Sn – Sn-1
st th
The 1 , 4thand 8 terms of A.P form a G.P. if = n(2n + 11) – (n – 1)(2(n -1) + 11)
the first term is 9, find the
= 9 + 4n
(i) Common difference of the A.P
(ii) Common ratio of the G.P Example 20
(iii) Difference in sums of the first 6 terms of
(a) The first, second and last term of an A.P
the progressions.
are a, b, c respectively. Prove that the sum
Solution of all terms is
Given that a, a + 3d, a + 7d form a G.P Solution
If the 1st term is a and the second term is
Substituting for a = 9, the terms are b; the common difference, d = (b – a)

digitalteachers.co.ug
Last term (nth terms c = a + (n – 1)d

c = a + (n – 1)(b – 1)
(b) Work out the following
i.e. n – 1 = => n = (i) ∑ ( )
Solution
but Sn =
∑ ( )
= ( )
a = 1 and r =
=

(b) The first, second and last terms of a GP (ii) ∑ ( )


are a and b. show that the sum of the first
Solution
n terms is
∑ ( ) =
Solution
A= and r =
Common ratio =
( ( ) )

( ) (iii) ∑
= Solution
a = 1 and r = a
Sum to infinity of a G.P
We have seen that the sum of n terms of a G.P
(iv) ∑
for r < 1 is
Solution
Now for -1 < n < 1 i.e. | | , as n → , rn →0
∑ = 9x2 + 27x3 + 81x4 + …
Therefore
a = 9x2 and r = 3x
Hence the sum of a GP to infinity for| |
converges to and diverges for r > 1
and r < 1 Example 22
Example 21 (a) Express the following as fractions using
approach of sum of a G.P to infinity
(a) Calculate the sum to infinity of the
(i) 0. ̇
following terms
Solution
(i)
0. ̇ =
Solution
= ( )
a = 1 and r =
= ( )

(ii)
Solution (ii) 3.1 ̇ ̇
Solution
a = and r =
3.1 ̇ ̇ = 3 +

digitalteachers.co.ug
=3+ ( ) corresponding values of u1
[ r = 3, a = 156; r = -3, a = ]
= ( ) ( ) 3. The sum of the second and third terms of
a G.P is 9. It the seventh term is eight
times the fourth term, find the
=
(a) The first term and the common ration
Hence 3.1 ̇ ̇ [a = ⁄ and r = 2]
(b) The sum of the fourth and first term
(b) The sum to infinity of a GP is 7 and the [36]
sum of the first two terms is . Find the 4. Find the sum of ten terms of geometrical
common ratio and the first term of the GP series 2, -4, 8 ….. *-682]
with positive common ratio 5. The second and the third terms of a G.P
Solution progression are 24 and 12(b + 1)
respectively. Find b if the sum of the first
three terms of the progression is 76
…………………….(i) * +
6. The sum of the 2nd and 3rd terms of a G.P
But S2 = is 12. The sum of the 3rd and 4th terms is
-36. Find the 1st term and common ratio
a + ar = [a = 2, r = -3]
7. What is the smallest number of terms of
a(1 + r) = ……………………….(ii)
GP 5, 10, 20 …. that can give a sum
substituting eqn. (i) into eqn. (ii) greater than 500, 000 [n= 17]
8. The first, fourth and eighth terms of
= Arithmetic progression (A.P) form a
geometric progression. If the first term is
9, find
49r2 = 1 (i) The common difference of A.P [ 1]
(ii) The common ratio of the G.P * +
r2 = (iii) The difference in the sums of the first
6 terms of the progressions [55.7049]
r=
9. The second, third and ninth terms of an
Considering appositive ratio A.P form a G.P. find the common ratio of
the G.P [6]
From eqn. (i) 10. (a) The sum of the first 10 terms of an AP
is 120. The sum of the next 8 terms is 240.
a = 7(1 - =6
Find the sum of the next 6 terms [264]
Revision exercise 2 (b) the arithmetic mean of the a and b is
three times their geometric mean. Show
1. The common ration of a GP is -5 and the that √
sum of the first seven terms of the 11. The first three terms of a geometric series
progression is 449. Find the first three are 1, p, and q. Given also that 10, q and p
terms. * + are the first three terms of an arithmetic
2. In the geometrical series ∑ , u5 – u2 series. Show that 2p2 – p – 10 = 0
= 156 and u7 – u4 = 1404. Find the possible Hence find the possible values of p and q
values of the common ratio and [p = -2 and q = 4 or p = and q = ]

digitalteachers.co.ug
Application of A.Ps and G.Ps to interest rates = P[( ) ( ) ( ) ]

If a sum of money P is invested at a simple


This is a G.P with:
interest rate of r% per annum, the amount
received after n years is given by A = P + I first term = = (100 + r)%
where I =
And common ratio = (100 + r)%
By substitution we have
(100 + r)%* +
A=P+ = P( )
Example 23
The interest for one year is , for 2 years is
(a) Find the amount at the end of ten years
, for n year = . Therefore the various
when 500000 shillings is invested at 5%
amounts of interest after one, two, three, etc. compound interest
years form an AP (i) the interest being added annually
On the other hand, if the principal P is Solution
invested at compound interest rate of r% per A= ( )
annum, the interest being added annually, the
( )
amount after one year is ( ) , after two
A = 814,447.3134
years is ( ) , after 3 years is (ii) the interest being added twice a year
Solution
( ) and after n years ( ) A= ( )
Hence the amounts after one, two, three, etc. ( )
years for a GP. A = 1,326,648.853
Note: if with compound interest is added half
annually as much as when added yearly, but it (b) Find the amount at the end of ten years
is added twice as much. Hence amount when 500000 shillings is invested at 5%
simple interest
A= ( )
A= =750,000
Now suppose that instead of adding the
interest annually, it is the principal, P which is Proof by induction
added annually, This is a mathematical technique that uses the
reasoning that if a statement is true for a
Amount after 1st year = ( ) particular value say n = 1, then it must be true
for n = 2, 3, 4, …. This involves the proof that
Amount after 2nd year = ( ) the series on the LHS must be equal to the
terms on the RHS
Amount after 3rd year = ( )
Example 24

Amount after nth year = ( ) Prove by induction that

Total amount after n years 1 + 2 + 3 + …+ n =

= ( ) ( ) + …+ ( ) Solution

digitalteachers.co.ug
Here we need to show that the above series
agrees for all values of n = 1, 2, 3 .., q and q +1

Suppose n = 1 =[ ]* +

LHS = 1 [taking only the 1st number]


=[ ]* +
RHs = = 1[substituting for n = 1]
=
LHS = RHS => the series hold for n = 1
LHS = RHS => the series holds for n = k+1
Suppose n = 2 hence for all positive values of n
LHS = 1+ 2 = 3 [taking first 2 numbers] (b) ∑
RHs = = 3[substituting for n = 2] Solution
LHS = RHS => the series hold for n = 2 For n = 1
Suppose n = q LHS = ∑ = 13 = 1

1 + 2 + 3 + …+ q = RHS = =1

For n = q +1(i.e. adding k + 1 on both sides) LHS = RHS => the series holds for n = 1

1 + 2 + 3 + …+ q + (q+1) = + (q + 1) For n = 2

LHS = ∑ = 13 + 2 3 = 1
=
RHS = =9
=
LHS = RHS => the series holds for n = 2
The result is true for n = q + 1, hence true for
all positive values of n For n = k

Example 25 ∑
Prove by induction For n = k+1
(a) 12 + 22 + 32 + …. + n2 = ∑ ∑ +(k + 1)3
Solution
For n = 1 =
LHS = 12 = 1;
= * +
RHS = =1
LHS = RHS => the series holds for n = 1 = [ ]
For n = 2
LHS = 12 + 22 = 5; =
RHS = =5
LHS = RHS => the series holds for n = 2 (c) p + pq + pq2 +…+pqn-1 =p( )
For n = k For n = 1,
2 2 2 2
1 + 2 + 3 + …. + k = LHS = p
For n = k + 1 RHS =p( )=p
12 + 22 + 32 + …. + k2 +(k+ 1)2 the identity is true for n = 1

digitalteachers.co.ug
For n = 1, how much he got at the end of the fifth
LHS = p + pq =p(1+q) year at the compound interest rate of 2%
RHS =p( ) = p( )=p(1+q) per annum. [530,812.1]
4. Prove by induction
the identity is true for n = 2
For n = k (i) ∑
(ii) 13 + 23 + 33 + …. + n3
p + pq + pq2 +…+pqk-1 =p( )
=
For n = k+1
(iii) ∑
p + pq + pq2 +…+pqk-1+pqk = p( )+pqk
(iv)
= p( )
(v) ∑
= p( )
(vi) ∑
the identity is true for n = k+1, hence
true for all positive values of n 5. The sum of the first n terms of a
Geometric Progression (G.P) is .
Revision exercise 3 Find its nth term as an integral power of 2
1. Five millions shillings is invested each year [22n]
at a rate of 15% compound interest by a 6. Prove by mathematical induction the
certain bank. 32n – 1 is a multiple of 8 for all positive
(a) Find how much he will receive at the integers n
end of ten years [116.7464m] 7. Use the method of induction to prove that
(b) How many years will it take to 6n – 1is divisible by 5 for all positive
accumulate to more than 50m [6] integral values of n
2. John opened an account in the bank and 8. Prove by induction that
deposited200,000 shillings every month ∑
for ten months without withdrawing. Find hence evaluate ∑
how much money he accumulated after
10 months if the bank offered 10% Thank you
compound interest per month. Dr. Bbosa Science
[3,506,233.412]
3. Peter deposited sh. 100,000 at the
beginning of every year for 5 years; find

digitalteachers.co.ug
Binomial distribution
A binomial distribution is a special type of a discrete random variable in which an experiment gives
rise to only two outcomes either success or failure.

Conditions for binomial distribution

(i) The experiment has a finite (repeated) number of trials, n


(ii) The trials are independent
(iii) The outcome of each trial is either a success or a failure
(iv) The probability, p of successful outcome is constant for all trials

If a discrete random variable X is the number of successful outcomes in n trials and satisfies the
above conditions, then X follows a binomial distribution written as X~B(n, p) or X~Bin (n, p)

Formula for Binomial distribution


If X~B(n, p), the probability of obtaining, r success in n trials P(X = r) where

P(X = r) =nCrprqn-r for r = 0, 1, 2, 3, …,n where q = 1-p


𝑛!
=(𝑛−𝑟)!𝑟! 𝑝𝑟 𝑞 𝑛−𝑟

Example 1

The random variable X is distributed B(7, 0.2). find

(i) P(X=3) (ii) P(1<X ≤ 4) (iii) P(X>1)

Solution

(i) n = 7, p= 0.2, q= 1- 0.2 = 0.8


P(X=3) = 7C3 x 0.23 x 0.84
7!
= (7−3)!3! 𝑥0.23 𝑥 0.84
7𝑥6𝑥5𝑥4𝑥3𝑥2𝑥1
= (4 𝑥0.23 𝑥 0.84 = 0.115
𝑥 3 𝑥 2 𝑥 1)(3 𝑥 2 𝑥 1)
(ii) P(1<X ≤ 4) = P(x=2) + P(X= 3) + P(X=4)
= 7C2 x 0.22 x 0.85 +7C3 x 0.23 x 0.84 +7C4 x 0.24 x 0.83
= 0.275 + 0.115 + 0.029 = 0.419
(iii) P(X>1) = 1- (P(X≤ 1) = [1 – 𝑃(𝑋 = 0) + 𝑃(𝑋 = 1)]
= 1 – [7C0 x 0.20 x 0.87 +7C1 x 0.21 x 0.86]
=[1 – (0.210 + 0.367)
= 0.423
Example 2

At freedom city super market, 60% of the customers shop on Saturday. Find the probability that in a
randomly selected sample of 10 customers

(i) Exactly 2 shop on Saturday


n= 10, p = 0.6 q = 1- 0.6 = 0.4
P(X = 2) = 10C2 x 0.62x 0.48 = 0.011
(ii) More than 7 shop on Saturday
P(X > 7) = P(X = 8) + P(X = 9) + P(X = 10)
= 10C8 x 0.68x 0.42 + 10C9 x 0.69x 0.41 + 10C10 x 0.610x 0.42
= 0.121 + 0.040 + 0.006 = 0.167

Example 3

The probability that a marble drawn from a box is red is 0.4. if a sample of 6marbles is taken,
find the probability that it will contain;

(i) No red marble


n = 6, p= 0.4, q= 1 – 0.4 = 0.6
P(X = 0) = 6C0 x 0.40 x 0.66 = 0.047
(ii) P(X = 5 or 6) = P(X= 5) + P(X = 6) = 6C5 x 0.45 x 0.61 + 6C6 x 0.46 x 0.60
= 0.037 + 0.004 = 0.041
(iii) Less than half red marbles
P(X<3) = P(X = 0) + P(X = 1) + P(X = 2)
=6C0 x 0.40 x 0.66 + 6C1 x 0.41 x 0.65 + 6C2 x 0.42 x 0.64
= 0.047 + 0.187 + 0.331 = 0.545

Example 4

A biased con is such that the chance of a head appearing upper most when tossed is twice of
the tail appearing uppermost. If the coin is tossed 10 times. Find the probability that

(i) Exactly 6 heads will appear


P(H) + P(T) = 1
2x + x = 1
1
x=
3
2 1
n = 10, p = ; q=
3 3
10 2 6 2 4
P(X = 6) = C6 x ( ) 𝑥 ( ) = 0.228
3 3
2 6 2 4 2 7 2 3
(ii) P(5<X<8) = P(X = 6) + P(X = 7) = 10C6 x ( ) 𝑥 ( ) + 10C7 x ( ) 𝑥 ( )
3 3 3 3
= 0.228 + 0.260 = 0.488

Example 5

A box contains a large number of pens. The probability that a pen is faulty is 0.1. How many pens
would you need to select to be more than 95% certain of picking one faulty pen?

Solution

n? p= 0.1, q = 0.9

P(X≥ 1) = 1- P(X = 0)>0.95


= 1 - nC0 x 0.10 x 0.9n > 0.95

= 0.05 > 0.9n


log10 0.05
n>
log10 0.9

n >29

The least value n = 29

Using Cumulative binomial probability table


The table give values of P(X ≥ x) for values of n and p.

(i) P(X ≤ X) = 1- (P(X ≥ (X+1)


(ii) P(X = x) + P(X ≥ x) – (PX ≥ x+1)

Example 6

The random variable is distributed B(5, 0.3). Find

(i) P(X ≥ 3) (ii) P(X > 1) (iii) P(X ≤ 4) (iv) P (X < 3) (v) P(X = 2)

Solution

n = 5, p = 0.3

(i) P(X ≥ 3) = 0.1631


(ii) P(X > 1) = P(X ≥ 2) = 0.4718
(iii) P(X ≤ 4) = 1 – ( P(X ≥ 5) = 1 – 0.0024 = 0.9976
(iv) P(X < 3) = P(X ≥ 2) = 1 – P(X ≥ 3) = 1 – 0.1631) = 0.8369
(v) P(X = 2) = P(X ≥ 2) – P(X ≥ 3) 0.4718 – 0.1631 = 0.3087

Example 7

An unbiased coin is tossed 15 times. Find th probability that

(i) Exactly eight heads will appear upper most


(ii) Between 6 and 10 heads will appear
(iii) Between 6 and 10 heads inclusive will appear

Solution

n = 15, p = 0.5

(i) P(X = 8) = P(X ≥ 8) – P(X ≥ 9) = 0.5000 – 0.3036 = 0.1964


(ii) P(6< X < 10) = P(X ≤ X ≤ 9) = P(X ≥ 7) – P(X ≥ 10) = 0.6964 – 0.1509 = 0.5455
(iii) P(X ≤ X ≤ 10) = P(X ≥ 6) – P(X ≥ 11) = 0.8491 – 0.0592 = 0.7899

Example 8

A student attempts 20 objective questions by guest work. Each questions has got four possible
alternatives out of which one is correct. Find the probability that he gets

(i) Exactly 9 correct answers


(ii) At least 12 correct answers
(iii) At most 6 correct answers
(iv) Between 6 and 14 correct answers inclusive
(v) Exactly 7 correct answers.

Solution

For correct answers, n = 20, p = 0.25, q = 0.75

(i) P(X = 9) = P(X ≥ 9) – P(X ≥ 10) = 0.0409 – 0.0139 = 0.027


(ii) P(X ≥ 12) = 0.0009
(iii) P(X ≤ 6) = 1 – P(X ≥ 7) = 1 – 0.2142 = 0.7858
(iv) P(X ≤ X ≤ 14) = P(X ≥ 6) – P(X ≥ 15) = 0.3828 – 0.0000 = 0.3828

For incorrect answers, n = 20, P = 0.75, q = 0.25

Revision exercise 1
1. A biased coin is such that a head is three times as likely to occur as a tail. The coin is tossed 5
times. Find the probability that at most two tails occur = 0.8965
2
2. Tom’s chance of passing an examination is . If he sits for four examinations, calculate the
3
probability he passes
(i) Only two examinations = 0.2963
(ii) More than half of the examinations = 0.5926
3. A fair die is rolled 6 times, calculate the probability that
1
(i) A 2 or 4 appears on the first throw =
3
(ii) Four 5’s will appear in the six throws= 0.0080
4. Usain Bolt makes 5 practice ran in the 100m sprint. A run is successful if he runs it in less
than 11 seconds. There are 8 chances out of 10 that he is successful. Find the probability
that;
(i) He records at least no success at all= 0.0003
(ii) Exactly 5 games= 0.3277
2
5. The probability that Alex wins a chess game is . He plays 8 games, what is the probability
3
that he wins
(i) At least 7 games= 0.1951
(ii) Exactly 5 games= 0.2731
6. Usain Bolt makes 5 practice ran in the 100m sprint. A run is successful if he runs it in less
than 11 seconds. There are 8 chances out of 10 that he is successful. Find the probability
that;
(i) He records at least no success at all= 0.0003
(ii) He record at least 2 success= 0.9933
(iii) If he is successful in 5 practice runs, he makes two additional runs. The probability of
success in either of the additional runs is 0.7. Determine the probability that Bolt will
make 7 successful runs = 0.1606
7. In a test there are 10 objective questions each with a choice of five possible alternatives out
of which only one is correct. If a student guesses each of the answers, find the probability
that he gets at least two answers correct. = 0.6242
8. 30% of the students in the school are day scholars. From a sample of 10 students chosen at
random, find the probability that
(i) Only 3 are day scholars= 0.267
(ii) Less than half are day scholars=0.850

9. The probability that a shopper buys a cake is 0.25. Find the probability that in a random
sample of 9 shoppers
(i) Exactly 3 buys a cake=0.2334
(ii) More than 7 buy a cake= 0.0001
10. A bag contains counter books of which 40% are blue and the rest are black. A counter book
is taken from the bag, its colour is noted then replaced. This is performed 8 times in all.
Calculate the probability that
(i) Exactly 3 will be blue= 0.279
(ii) At least one will be blue= 0.983
(iii) More than half will be black= 0.594

11. At a certain school, the records taken from admission’s office the ratio of male to female s. 5
applicants is 4:6. Basing on this experience, what is the probability that will be more female
applicants in a random collection of a dozen applicants? = 0.665
12. The random variable X is B(6, 0.42). find
(i) P(X = 6) = 0.00549 (ii) P(X = 4) = 0.157 (iii) P(X ≤ 2) = 0.503
13. An unbiased die is thrown seven times. find the probability of throwing at least 5 sixes.=
0.002
14. In a family a couple is likely to produce a girl or a boy. Find the probability that in a sample of
5 children there will be more boys than girls = 0.5
15. The probability that it will rain on any given day during examination period is 0.3. calculate
the probability that in a given week during examination period, it will rain on;
(i) Exactly 2 days = 0.318
(ii) At most two days = 0.647
(iii) Exactly three days that are consecutive = 0.0324
16. A fair coin is tossed 6 times. find the probability of throwing at least four heads = 0.344
17. The random variable x is B(4, p) and P(X = 4) = 0.0256. find P(X = 2) = 0.3456
18. In agriculture lab, Silvia plants bean seeds and the probability that they germinate
1
successfully is .
3
(a) She takes 9 seeds. Find the probability that
(i) More than five seeds germinate = 0.0424
(ii) At least three seeds germinate = = 0.623
(b) Find the number of seeds that she needs to take in order to 99% certain at least one
germinate = 12
19. In a shooting competition, the probability of hitting the target with a single shot is 0.6, if 7
shots are taken; find the probability that the target is hit more than twice. = 0.9037
20. In mass production of shirts, it is found that 5% are defective. Shirts are selected at random
of put into packets of 10.
(a) A packet is selected at random. Find the probability that it contains
(i) Three defective shirt = 0.0105
(ii) Less than three defective shirts = 0.9885
(b) Two packets are selected at random. Find the probability that there are no defective
shirts in either packet = (10C0(0.05)0(0.95)10 x (10C0(0.05)0(0.95)10 = 0.358
21. A biased coin is such that it is twice as likely to show a head as a tail. If the coin is tossed 5
times. find the probability that
(i) Exactly three heads are obtained = 0.329
(ii) More than three heads are obtained = = 0.3333
22. The probability that a target is hit 0.3. Find the probability the least number of times shots
should be fired if the probability that the target is hit is at least once is greater than 0.95. =9
23. 1% of the light bulbs in a box are faulty. Find the largest sample size which can be taken if it
required that the probability that there is no faulty bulb in the sample is greater than 0.5
(0.99)n > 0.5
n = 68
24. In a test there are 10 multiple choice questions. Each question has got four possible
alternatives out of which only one is correct. If a student guesses each of the answers, find
the probability that he gets
(i) More than 7 correct answers = 0.0004
(ii) More than half correct answers = 0.0197
25. X~𝐵(𝑛, 𝑜. 3). find the value of n such that P(X ≥ 1) = 0.8 :
26. The random variable X is B(n, 0.6). find the value of n such that P(X<1) = 0.0256

Solutions to revision exercise 1

1. A biased coin is such that a head is three times as likely to occur as a tail. The coin is tossed 5
times. Find the probability that at most two tails occur
Solution
n=5
p+q=1
x + 3x = 1, x = 0.25
p = 0.25, q = 0.73
P(X ≤ 2) = P(X = 0) + P(X =1) + P(X = 2)
= 5C0 x 0.250 x 0.755 +5C1 x 0.251 x 0.754+ 5C2 x 0.252 x 0.753
=0.2373 + 0.3955 + 0.2637 = 0.8965

2
2. Tom’s chance of passing an examination is . If he sits for four examinations, calculate the
3
probability he passes
(i) Only two examinations
2 1
n = 4, p = , q =
3 3
4 2 2 1 2
P(X = 2) = C2 x ( ) ( ) = 0.2963
3 3
(ii) More than half of the examinations
2 3 1 1 2 4 1 0
P(X < 2) = P(X= 3) + P(X =4) = 4C3 x ( ) ( ) + 4C4 x ( ) ( ) = 0.5926
3 3 3 3
3. A fair die is rolled 6 times, calculate the probability that
(i) A 2 or 4 appears on the first throw
1 1 2 1
P( X = 2 or 4) = P(X =2 ) + (X = 4) = + = =
6 6 6 3
(ii) Four 5’s will appear in the six throws
1 5
n = 6, P= , q =
6 6
1 4 5 2
P(X = 4) = 6C4 x ( ) ( ) = 0.0080
6 6
4. Usain Bolt makes 5 practice ran in the 100m sprint. A run is successful if he runs it in less
than 11 seconds. There are 8 chances out of 10 that he is successful. Find the probability
that;
(i) He records at least no success at all
n = 5, p = 0.8, q = 0.2
P(X = 0) = 5C0 x (0.8)0 (0.2)5 = 0.0003
(ii) Exactly 5 games

P(X = 5) = 5C5 x (0.8)5 (0.2)0 = 0.3277


2
5. The probability that Alex wins a chess game is . He plays 8 games, what is the probability
3
that he wins
(i) At least 7 games
2 1
n = 8, p = , q =
3 3
2 7 1 1 2 8 1 0
P(X ≤ 7) = P(X = 7) + P(X= 8) = 8C7 x ( ) ( ) + 8C8 x ( ) ( )
3 3 3 3
=0.1561+ 0.390 = 0.1951
(ii) Exactly 5 games
2 5 1 3
P(X =5) = 8C5 x ( ) ( ) = 0.2731
3 3
6. Usain Bolt makes 5 practice ran in the 100m sprint. A run is successful if he runs it in less
than 11 seconds. There are 8 chances out of 10 that he is successful. Find the probability
that;
(i) He records at least no success at all
n = 5, p = 0.8, q = 0.2
P(X = 0) = 5C0 x (0.8)0 (0.2)5 = 0.0003
(ii) He record at least 2 success
P(X ≥ 2) = 1 –P(X <2) = 1- [P(X=0) + P(X = 1)
= 1 – [5C0 x (0.8)0 (0.2)5 + 5C1 x (0.8)1 (0.2)4 ) = 1- (0.0003 + 0.0064) = 0.9933
(iii) If he is successful in 5 practice runs, he makes two additional runs. The probability of
success in either of the additional runs is 0.7. Determine the probability that Bolt will
make 7 successful runs
Probability = 5C5x 0.85 x 0.20 and 2C2 x 0.72(0.3)0
= 0.32768 x 0.49= 0.1606
7. In a test there are 10 objective questions each with a choice of five possible alternatives out
of which only one is correct. If a student guesses each of the answers, find the probability
that he gets at least two answers correct.
Solution
n = 10, p = 0.2, q = 0.8
P(X ≥ 2) = 1- P(X<2)
= 1 – [P(X = 0) + P(X = 1)
= 1 – [ 10C0 x 0.20 x 0.810 x 10C1 x 0.21 x 0.89)
= 1- (0.1074 + 0.2684) = (1 – 0. 3758) = 0.6242
8. 30% of the students in the school are day scholars. From a sample of 10 students chosen at
random, find the probability that
(i) Only 3 are day scholars
n = 10, p = 0.3, q = 0.7
P(X = 3) = 10C3(0.3)3(0.7)7 = 0.267
(ii) Less than half are day scholars
P(X < 5) = P(X = 0) + P(X = 1) + P(X = 2) + P(X = 3) + P(X = 4)
=10C0(0.3)0(0.7)10 + 10C1(0.3)1(0.7)9 + 10C2(0.3)2(0.7)8 + 10C3(0.3)3(0.7)7
+10C4(0.3)4(0.7)6
=0.850
9. The probability that a shopper buys a cake is 0.25. Find the probability that in a random
sample of 9 shoppers
(i) Exactly 3 buys a cake
n = 9, p = 0.25, q = 0.75
P(X = 3) = 9C3(0.25)3(0.75)6 =0.2334
(ii) More than 7 buy a cake
P(X > 7) = P(X = 8) + P(X = 9)
= 9C8(0.25)8(0.75)1 + 9C9(0.25)9(0.75)0 = 0.0001
10. A bag contains counter books of which 40% are blue and the rest are black. A counter book
is taken from the bag, its colour is noted then replaced. This is performed 8 times in all.
Calculate the probability that
(i) Exactly 3 will be blue
n = 8, p = 0.4, q = 0.6
P(X = 3) = 8C3(0.4)3(0.6)5 = 0.279
(ii) At least one will be blue
P(X ≥ 1) = 1 – P(X<1) = 1- P(X=0) = 1 – 8C0(0.4)0 (0.6)8 = 0.983
(iii) More than half will be black
P(X≤3) = P(X = 0) + P(X= 1) + P(X = 2) + P(X = 3)
=8C0(0.4)0 (0.6)8 + 8C1(0.4)1 (0.6)7 + 8C2(0.4)2 (0.6)6 + 8C3(0.4)3 (0.6)5
= 0.594
11. At a certain school, the records taken from admission’s office the ratio of male to female s. 5
applicants is 4:6. Basing on this experience, what is the probability that will be more female
applicants in a random collection of a dozen applicants?
Solution
n = 12, p = 0.4, q = 0.6
P(X≤ 5) = P(X = 0) + P(X =1) + P(X =2) + P(X= 3) + P(X = 4) + P(X = 5)

= 12C0(0.4)0 (0.6)12 + 12C1(0.4)1 (0.6)11 + 12C2(0.4)2 (0.6)10 + 12C3(0.4)3 (0.6)9 +


12
C4(0.4)4 (0.6)8 + 12C5(0.4)5 (0.6)7

= 0.6652

12. The random variable X is B(6, 0.42). find


(i) P(X = 6)
n = 6, p = 0.42, q= 1- 0.42= 0.58
P(X = 6) = 6C6(0.42)6(0.58)0 = 0.00549
(ii) P(X = 4)
6
C4(0.42)4(0.58)2= 0.157
(iii) P(X ≤ 2)
P(X ≤ 2) = P(X = 0) + P(X = 1) + P(X= 2)
=6C0(0.42)0(0.58)6 + 6C1(0.42)1(0.58)6+ 6C2(0.42)2(0.58)4= 0.503
13. An unbiased die is thrown seven times. find the probability of throwing at least 5 sixesn = 7,
1 5
p= ,q=
6 6
P(X ≤ 5) = P(X = 5) + P(X = 6) + P(X = 7)
1 5 1 1 5
14. =7C5( )5( )2 + 7C6( )6(0.58)1 +7C7( )7( )0.= 0.002
6 6 6 6 6
15. In a family a couple is likely to produce a girl or a boy. Find the probability that in a sample of
5 children there will be more boys than girls
n = 5, p = 0.5, q = 0.5
P(X ≥ 3) = P(X = 3) + P(X = 4) + P(X= 5)
= 5C3(0.5)3(0.5)2 +5C4(0.5)4(0.5)1 + 5C5(0.5)5(0.5)0 = 0.5
16. The probability that it will rain on any given day during examination period is 0.3. calculate
the probability that in a given week during examination period, it will rain on;
(i) Exactly 2 days
n = 7, p = 0.3, q = 0.7
P(X = 2) = 7C2(0.3)2(0.7)5= 0.318
(ii) At most two days = 0.671
P(X ≤ 2) = P(X = 0) + P(X = 1) + P(X = 2)
=7C0(0.3)0(0.7)7 + 7C1(0.3)1(0.7)6 + 7C2(0.3)2(0.7)5 = 0.647
17. A fair coin is tossed 6 times. find the probability of throwing at least four heads
n = 6, p = 0.5, q = 0.5
P(X ≤ 4) = P(X = 4) + P(X =5) + P(X =6)
= 6C4(0.5)4(0.5)2 + 6C5(0.5)5(0.5)1 + 6C6(0.5)6(0.5)0= 0.344

18. The random variable x is B(4, p) and P(X = 4) = 0.0256. find P(X = 2)
n = 4, p = p, q = (1-p)
P(X = 4) = 4C4(p)4(q)0 = 0.0256
(p)4= 0.0256; p = 0.4
P(X = 2) =4C2(0.4)2(0.6)2= 0.3456
19. In agriculture lab, Silvia plants bean seeds and the probability that they germinate
1
successfully is .
3
(a) She takes 9 seeds. Find the probability that
(i) More than five seeds germinate
P(X<5) = P(X =6) + P(X = 7) + P(X = 8) + P(X = 9)
1 2 1 2 1 2 1 2
= 9C6( )6( )3 + 9C7( )7( )2 +9C8( )8( )1 +9C9( )9( )0 = 0.0424
3 3 3 3 3 3 3 3
(ii) At least three seeds germinate
P(X ≥ ) = 1 –[P(X=0) + P(X = 1) + P(X = 2)]
1 2 1 2 1 2
= 1 –[9C0( )0( )9 + 9C1( )1( )8 +9C2( )2( )7] = 0.623
3 3 3 3 3 3
(b) Find the number of seeds that she needs to take in order to 99% certain at least one
germinate
1 2
P(X≤ 1) = 1 – P(X = 0) = 1 - nC0( )0( )n = 0.99
3 3
2
( )n = 0.01
3
n = 11.36
therefore the number = 12
20. In a shooting competition, the probability of hitting the target with a single shot is 0.6, if 7
shots are taken; find the probability that the target is hit more than twice.
n = 7, p = 0.6, q = 0.4
P(X > 2) = 1 – P(X ≤2) = 1 – [P(X = 0) + P(X = 1) + P(X =2)]
= 1- [7C0(0.6)0(0.4)7 + 7C1(0.6)1(0.4)6 +7C2(0.6)2(0.4)5]
= 0.9037
21. In mass production of shirts, it is found that 5% are defective. Shirts are selected at random
of put into packets of 10.
(a) A packet is selected at random. Find the probability that it contains
(i) Three defective shirt
P(X= 3) = 10C3(0.05)3(0.95)7= 0.0105
(ii) Less than three defective shirts
P(X < 3) = P(X = 0) + P(X = 1) + P(X=2)
= 10C0(0.05)0(0.95)10 x 10C1(0.05)1(0.95)9 + 10C2(0.05)2(0.95)8= 0.9885
(b) Two packets are selected at random. Find the probability that there are no defective
shirts in either packet = (10C0(0.05)0(0.95)10 x (10C0(0.05)0(0.95)10 = 0.358
22. A biased coin is such that it is twice as likely to show a head as a tail. If the coin is tossed 5
times. find the probability that
(i) Exactly three heads are obtained
2 1
n = 5, p = , q =
3 3
5 2 3 1 2
P(X = 3) = C3 ( ) ( ) = 0.3292
3 3
(ii) More than three heads are obtained = 0.3333
P(X > 3) = P(X = 4) + P(X = 5)
2 1 2 1
=5C4( )4( )1 + 5C5( )0( )5 =
3 3 3 3
23. The probability that a target is hit 0.3. Find the probability the least number of times shots
should be fired if the probability that the target is hit is at least once is greater than 0.95. =9
n = ? p = 0.3, q = 0.7
P(X ≥ 1) = 1 – P(X = 0) = 1 - nC0(0.3)0(0.7)n > 0.95
(0.7)n > 0.05
log 0.05
n= = 8.399
log 0.7
therefore n =9
24. 1% of the light bulbs in a box are faulty. Find the largest sample size which can be taken if it
required that the probability that there is no faulty bulb in the sample is greater than 0.5
(0.99)n > 0.5
n = 68
25. In a test there are 10 multiple choice questions. Each question has got four possible
alternatives out of which only one is correct. If a student guesses each of the answers, find
the probability that he gets
(i) More than 7 correct answers = 0.0004
(ii) More than half correct answers = 0.0197
26. X~𝐵(𝑛, 𝑜. 3). find the value of n such that P(X ≥ 1) = 0.8 : 0.7n = 0.2. = 5
27. The random variable X is B(n, 0.6). find the value of n such that P(X<1) = 0.0256
0.4n = 0.0256, n = 4

Expectation, variance and standard deviation


If X~B(n, p)

E(X) = np

Var(X) = npq where q = 1- p

s.d = √𝑛𝑝𝑞
Example 9

The random variable x is B(4, 0.8). the mean, variance and standard deviation

Mean = np = 4 x 0.8 = 3.2

Variance = npq = 4 x0.8 x 0.2 = 0.64

s.d = √𝑛𝑝𝑞 = √0.64 = 0.8

Example 10

The probability that, it will be a sunny day is 0.4. Find the expected number if sunny days in a week
and also find the standard deviation

Solution

E(X) = np = 7 x 0.4 = 2.8

s.d = √𝑛𝑝𝑞 = √(7 𝑥 0.4 𝑥 0.6) = 1.296

Example 11

X is B(n, p) with mean 5 and standard deviation2. Find the value of n and p.

E(X) = np

np = 5 ………………….. (i)

s.d = √𝑛𝑝𝑞

√𝑛𝑝𝑞 =2

npq = 4 …………………(ii)

Eqn. (ii) ÷ eqn. (i)


𝑛𝑝𝑞 4
= = 0.8
𝑛𝑝 5

q = 0.8

p = 0.2
5
n= = 25
0.2

Mode of the binomial distribution


The mode is the value of X that is most likely to occur. The value of X with the highest probability
and its close to the mean gives the mode.

Example 12

The probability that a student is awarded a distinction in mathematics examination is 0.15. In a


randomly selected group of 15 students, what is the most likely number of students awarded a
distinction
Solution

E(X) = np = 15 x 0.15 = 2.25

P(X = 2) = 15C2(0.15)2(0.85)13 = 0.286

P(X = 3) = 15C3(0.15)3(0.85)12 = 0.216

The most likely number of students awarded a distinction = 2

Example 13

In a school 80% of the students find difficulties in Physics. If a sample of 12 students is chosen

(i) What is the most likely number of students who find it difficult in physics.
E(X) = np = 0.8 x 12 = 9.6
P(X = 9) = 12C9(0.8)9(0.2) 3 = 0.236
P(X = 10) = 12C10(0.8)10(0.2) 2 =0.294
The most likely number is 10
(ii) Find the probability that fewer than half find difficulty in Physics.
P(X<6) = 0.0004

Revision exercise 2
1. 10% of drugs at a certain Pharmacy are expired. A sample of 25 drugs is taken. Find the
expected number and standard deviation of expired drugs (25,1.5)
2. The probability that a student scores above 60% in mathematics test is0.5. In a random sample
of 15 students, what is the most likely number of students who score above 60%. (7 and 8)
3. The probability that an apple picked at random from a sack is bad is 0.15.
(a) Find the standard deviation of the number of bad apples in a sample of 15 apples.=1.38
(b) What is the most likely number of bad apples in a sample of 30 apples = 4
4. The random variable X is B(n, 0.3) and E(X) = 2.4. Find n and standard deviation (8, 1.30)
5. In a group of people, the expected number who wear glasses is 2 and the variance is 1.6, find
the probability that;
(i) A person chosen at random from the group wear glasses = 0.2
(ii) 6 people in the group wear glasses. =0.00551
6. New vision publishes a governance article in its newspaper each day of the week Sunday. A man
is able to read 8 out of ten articles
(i) Find the expected value and the standard deviation of the number of read articles in a
given week= 4.8
(ii) What is the probability that he will read at least 5 articles in a given week? = 0.98
1
7. A die is biased and probability, p of throwing a six is known to be less than . An experiment
6
consists of recording the number of sixes n 25 throws of the die. The standard deviation of the
number of sixes I 1.5. calculate the
(i) value of p. = 0.1
(ii) the probability that exactly three sixes are recorded during a particular experiment =
0.23
8. The random variable X is B(10, p) where p < 0.5. The variance of X is 1.875. find
(i) Value of p. = 0.25
(ii) E(X) = 2.5
(iii) P(X = 2) = 0.282
9. In a bag there are 6 red pens, 8 blue pens and 6 black pens. an experiment consists of taking a
pen at random from the bag, noting its colour and then replacing it in the bag. This procedure is
repeated 10 times in all. find
(i) Expected number of red pens drawn=3
(ii) Most likely number of black pens drawn = 3
(iii) Probability that not more than four blue pens are drawn = 0.633
10. The random variable X is distributed binomially with mean 2 and variance 1.6, Find
(i) the probability that x is less than 6 = 0.994
(ii) the most likely value of X = 2
11. Each day a bakery delivers the same number of loaves to a certain shop which sells on
average98%of them. Assuming that the number of loaves sold per day has a binomial
distribution with standard deviation 7. Find the expected number of loaves the shop would
expect to sell per day = 2500
12. On average 20% of the bolts produced by a machine are faulty. Samples of 10 bolts are to be
selected at random each day. Each bolt will be selected and replaced in the set of bolts which
have been produced on that day.
(i) Find the probability that in any one sample, two bolts or less will be faulty = 0.68
(ii) Calculate the expected value and variance of the number of bolts in a sample which will
not be faulty. = 8, 1.6
13. An experiment consists of taking 12 shots at a target and counting the number of hits. The
expected number of hit was found to be 3. Calculate
(i) The probability of hitting the target with a single shot. = 0.25
(ii) Standard deviation of the number of hits in an experiment = 1.5
14. In a certain family, the probability that they will have a baby boy is 0.6. If there are 5 children in
a family determine
(a) Expected number of girls = 2
(b) The probability that there are at least three girls = 0.317
(c) The probability they are all boys. = 0.0778
15. The probability of winning a game is 0.8. Ten games are played. What is the;
(a) Mean number of success and variance. = 8, 1.6
(b) The probability of at least 8 success in the ten = 0.6778
16. In a test there are 10 multiple choices questions. Each question has four possible alternatives
out of which one is correct. If a student guesses each of the answers, find the
(i) The probability that at least fur answers are correct = 0.2241
(ii) Most likely number of correct answers = 2
17. A biased coin is such that a head is twice as likely to occur as a tail. the coin is tossed 15 times.
Find the
(i) The expected number of heads = 10
(ii) Probability that at most two tails occur = 0.0793

Solutions to revision exercise 2


14. In a certain family, the probability that they will have a baby boy is 0.6. If there are 5 children in a
family determine
(a) Expected number of girls
n = 5, p = 0.4, q = 0.6
E(X) = np = 5 x 0.4 = 2
(b) The probability that there are at least three girls
P(X≤3) = P(X= 3) + P(X = 4) + P(X = 5)
= 5C3(0.4)3(0.6)2 + 5C4(0.4)4(0.6)1 +5C5(0.4)5(0.6)0 = 0.317
(c) The probability they are all boys.
P(X = 0) = 5C0(0.4)0(0.6)5= 0.0778
15. The probability of winning a game is 0.8. Ten games are played. What is the;
(a) Mean number of success and variance.
Mean = np = 10 x 0.8 = 8
Variance = npq = 10 x 0.8 x 0.2 =1.6
(b) The probability of at least 8 success in the ten
P(X ≤ 8) = P(X = 8) + P(X = 9) + P(X = 10)
= 10C8(0.8)8(0.2)2 + 10C9(0.8)9(0.2)1 + 10C10(0.8)10(0.2)0 = 0.6778
16. In a test there are 10 multiple choices questions. Each question has four possible alternatives
out of which one is correct. If a student guesses each of the answers, find the
(i) The probability that at least four answers are correct
n = 10, p = 0.25, q = 0.75
P(X ≥ 4) = 1 –P(X ≤ 3) = 1 – [P(X= 0) + P(X= 1) + P(X = 2) + P(X = 3)
= 1 – [10C0(0.25)0(0.75)10+10C1(0.25)1(0.75)9+10C2(0.25)2(0.75)8+10C3(0.25)3(0.75)7]
= 0.2241
(ii) Most likely number of correct answers
E(X) = np = 0.25 x 10 = 2.5
P(X = 2) =10C2(0.25)2(0.75)8 = 0.2816
P(X = 3) =10C3(0.25)3(0.75)7 = 0.2503
∴ the most likely number of correct answers = 2
17. A biased coin is such that a head is twice as likely to occur as a tail. The coin is tossed 15 times.
Find the
(i) The expected number of heads
2 1
n =15, p = , q =
3 3
2
E(X) = np = 𝑥 15 = 10
3
(ii) Probability that at most two tails occur
1 2
n = 15, p = , q =
3 3
P(X≤ 2) = P(X = 0) + P(X = 1) + P(X =2)
1 0 2 15 1 1 2 14 1 2 2 13
= 15C0( ) ( ) + 15C1( ) ( ) + 15C2( ) ( ) = 0.0793
3 3 3 3 3 3

Thank you
Dr. Bbosa Science
Normal distribution
A continuous random variable X follows a normal distribution with mean, μ and variance, σ2 if

X~N(μ, σ2) root


(𝑥−𝜇)2
1
Its p.d.f is given by fx = 𝑒 2𝛿2 , −∞ < 𝑥 < ∞
𝜎√2𝜋

A sketch of f(x) gives a normal curve Properties of the curve

f(x) - It is bell shaped


1 (𝑥−𝜇)2
𝑒 2𝛿2 - It is symmetrical about μ
𝜎√2𝜋 - It extends from−∞ < 𝑥 < ∞
(𝑥−𝜇)2
1
The maximum value of f(x) = 𝑒 2𝛿2
𝜎√2𝜋

-∞ 𝜇 +∞ - The total area under the curve = 1

How to read the cumulative normal distribution table

(i) Between 0 and any z value


(a) P(0 ≤ Z ≤ Z1) = φ(Z1) = region P (b) P(-Z1≤ Z ≤ 0)= P(0 ≤ Z ≤ Z1) = φ(Z1) = region P
By symmetrical

P
-Z1 0 Z 0 Z1 Z
(ii) z Less than any positive z value z
(a) P(Z < Z1) = 0.5 + P(0 ≤ Z ≤ Z1) = φ(Z1) = region P
(b) P(Z > -Z1) = P(Z < Z1) = 0.5 + P(0 ≤ Z ≤ Z1) = 0.5 + φ(Z1) = region P
By symmetrical

P
-Z1 0 Z 0 Z1 Z
z
(iii) Greater than any positive z value
P(Z > Z1) = 0.5 - P(0 ≤ Z ≤ Z1) = 0.5 - φ(Z1) = region P
P(Z < -Z1) = P(Z > Z1) = 0.5 - P(0 ≤ Z ≤ Z1) = 0.5 - φ(Z1) = region P
By symmetrical
t
P
-Z1 0 Z 0 Z1 Z
z
Example 1

Find (i) P(Z< 2) (ii) P(Z > 0.85) (iii) P(X< 0.345)

Solution
(i) P(X < 2) = 0.5 + φ(2) = 0.5 + 0.4772 = 0.9772
(ii) P(Z > 0.85) = 0.5 - φ(0.85) = 0.5- 0.3023 = 0.1977
(iii) P(X< 0.345) = 0.5 + - φ(0.345) =0.5 + 0.1331 + 0.0019 = 0.6350

Example 2

Find (i)P(Z < -0.25) (ii) P(Z > -1.377) (iii) P(Z < -1.377)

Solution

(i) P(Z < -0.25) = P(Z > 0.25) = 0.5 - φ(0.25)= 0.5 – 0.0987 = 0.4013
(ii) P(Z > -1.377) = P(Z < 1.377) = 0.5 + φ(1.377)= 0.5 + 0.4147 + 0.0011 = 0.9158
(iii) P(Z < -1.377) = P(Z > 1.377 = 0.5 - φ(1.377) = 0.5 – (0.4147 + 0.0011) = 0.0842

Other important results

-Z4 -Z3 0 Z1 Z2 Z

(i) Between two Z values on the same side of the mean


(a) P(Z1 < Z < Z2) = P(0 < Z < Z2) - P(0 < Z < Z1) = φ(Z2) - φ(Z1)
(b) P(-Z4 < Z < -Z3) = P(0 < Z < Z4) - P(0 < Z < Z3) = φ(Z4) - φ(Z3)
(ii) Between two Z values on the opposite side of the mean
(a) P(-Z3 < Z < Z1) = P(0 < Z < Z3) + P(0 < Z < Z1) = φ(Z3) + φ(Z1)
(b) P(|Z|< Z1) = P(-Z1 < Z < Z1) = 2 x P(0 < Z < Z1) = 2 x φ(Z1)
(c) P(|Z|> Z1) = 1 - P(-Z1 < Z < Z1) = 1 -2 x φ(Z1)

Example 3

Find (i) P(1.5 < Z < 1.88) (iii) P(-2.696 < Z < 1.865) (v) P(|Z|< 1.75)
(ii) P(-2.5 < Z < 1) (iv) P (-1.4 < Z < -0.6) (vi) P(|Z|> 1.433)
Solution
(i) P(1.5 < Z < 1.88) = φ(1.88) - φ(1.5) = 0.4699 – 0.4332 = 0.0367
(ii) P(-2.5 < Z < 1) = φ(1) + φ(2.5) = 0.3413 + 0.4938 = 0.8351
(iii) P(-2.696 < Z < 1.865) = φ(1.865) + φ(2.696) = 0.469 + 0.4964 = 0.9654
(iv) P (-1.4 < Z < -0.6) = φ(1.4) + φ(0.6) = 0.4192 – o.2257 =0.1935
(v) P(|Z|< 1.75) = P(-1.75 < Z < 1.75) = 2 x φ(1.75) = 2 x 0.4625= 0.925
(vi) P(|Z|> 1.433) = 1 - P(|Z|< 1.433) = 1 – 2 x φ(1.433) = 1 – 2 x 0.424 = 0.152
Standardizing a random variable X
If a random variable X follows a normal distribution with mean, μ and variance, σ2, then X ~N(μ, σ2)
and can be standardized using the equation below and read from a cumulative normal table
𝑿− 𝝁
Z=
𝝈

Example 4

Given that the random variable X is X ~N(300, 25). Find

(i) P(X > 305) (ii) P(X < 291) (iii) P(X < 312) (iv) P(X > 286)

Solution
305−300
(i) P(X> 305) = 𝑃 (𝑍 < ) = P(X > 1) = 0.5 – φ(1) = 0.5 – 0.3413 = 0.1587
5
291−300
(ii) P(X < 291) = 𝑃 (𝑍 < ) = P(Z < -1.8)
5
= P(Z >1.8) = 0.5 - φ(1.8) = 0.5 – 0.4641 = 0.0359
312−300
(iii) P(X < 312) = 𝑃 (𝑍 < ) = P(X < 2.4) = 0.5 + φ(2.4)
5
= 0.5 – 0.4918 = 0.9918
286−300
(iv) P(X > 286) = 𝑃 (𝑍 < ) = P(Z < -2.8)
5
= P(Z < 2.8) = 0.5 + φ(2.8) = 0.5 + 0.4974= 0.9974

Example 5

Given that the random variable X is X ~N(10, 4). Find

Find (i) P(X < 7) (ii) P(X > 12) (iii) P(7 < X < 12) (iv) P(9 < X < 11)

Solution
7−10
(i) P(X < 7) = 𝑃 (𝑍 < ) = P(Z < - 0.15) = P(Z > 1.5)
2
= 0.5 – φ(1.5) = 0.5 – 0.4332 0.0668
12−10
(ii) P(X > 12) = 𝑃 (𝑍 > ) = P(Z > 1) = 0.5 – φ(1) = 0.5 – 0.3413 = 0.1587
2
7−10 12−10
(iii) P(7 < X < 12) = 𝑃 ( <𝑍> )
2 2
= P(-1.5 < Z < 1) = φ(1.5) + φ(1) = 0.4332 + 0.3413 = 0.7745
9−10 11−10
(iv) P(9 < X < 11) = 𝑃 ( <𝑍> )
2 2
= P(-0.5 < Z < 0.5) = φ(0.5) + φ(0.5) = 2 x 0.1915 = 0.3830

Example 6

Given that the random variable X is X ~N(50, 8). Find

(i) P(48 < X < 54) (ii) P(52 < X < 55) (iii) P(46 < X < 49) (iv) P(|X- 50|< √8)

Solution
48−50 54−50
(i) P(48 < X < 54) = 𝑃 ( <𝑍> ) = P(-0.707 < Z < 1.414)
√8 √8

= φ(1.414) + φ(0.707) = 0.4213 + 0.2601 = 0.6814


52 −50 55−50
(ii) P(52 < X < 55) = 𝑃 ( <𝑍> ) = P(0.707 < Z < 1.768)
√8 √8

= φ(1.768) - φ(0.707) = 0.4615 -0.2601 = 0.2014


46 −50 49−50
(iii) P(46 < X < 49) = 𝑃 ( <𝑍> ) = P(-1.414 < Z < -0.354)
√8 √8

=φ(1.414) – φ(0.354) = 0.4213 – 0.1383 -0.283


−√8+50−50 √8+50−50
(iv) P(|X- 50|< √8) = 𝑃 ( <𝑍> ) P(-1 < X < 1) = 2 x φ(1) = 2 x 0.3413 = 0.6826
√8 √8

Example 6

A random variable X is normally distributed with mean 65 and variance 100, find the probability that
X assumes a value between 50 and 90.
50−65 90−65
P(50 < X < 90) = 𝑃 ( <𝑍< )= P(-1.5 < Z < 2.5) = φ(1.5) + φ(2.5) = 0.4332 + 0.4938 = 0.927
10 10

Example 7

Lengths of metal strips produced by a machine are normally distributed with mean length of 150cm
and standard deviation of 10cm. find the probability that the length of a randomly selected strip is

(i)shorter than 165 (ii) within 5cm of the mean

Solution
165−150
P(X < 165) = 𝑃 (𝑍 < ) =P(Z< 1.5) = 0.5 + φ(1.5) = 0.5 + 0.4332 = 0.9332
10

−5 5
P(150 – 5 < X < 150 + 5) = 𝑃 ( <𝑍< )= P(-0.5 < Z < 0.5) = 2 x φ(0.5) = 2 x 0.1915 = 0.383
10 10

Example 8

In end of year exams, the marks are normally distributed with a mean mark of 50 and standard
deviation 5. If a mark 45 is required to pass the exam, what percentage of the students failed the
exam.
45−50
P(X < 45) = 𝑃 (𝑍 < ) = P(Z < -1) = P(Z > 1) = 0.5 – φ(1) = 0.5 – 0.3413 = 0.1587
5

Example 9

A bakery supplies bread to a shop every day. The time to deliver bread to the shop is normally
distributed with mean 12 minutes and standard deviation of 2 minutes. Estimate the number of days
the year when he takes

(i) longer than 17 minutes (ii) less than 10 minutes (iii) between 9 and 13 minutes

Solution
17−12
(i) P(X > 17) = 𝑃 (𝑍 > )= P(Z > 2.5) = 0.5 – φ(2.5) = 0.5 – 0.4938 = 0.0062
2

The number of days = 0.0062 x 365 = 2 days


10−12
(ii) P(X < 10) = 𝑃 (𝑍 > )= P(Z < -1) = P(Z> 1) = 0.5 – φ(1) + 0.5 – 0.3413 0.1587
2
The number of days = 0.1587 x 365 = 58 days.
9−12 13−12
(iii) P(9 < X <13) = 𝑃 ( <𝑍< )= P(-1.5 < Z < 0.5) = φ(1.5) + φ(0.5) 0.4332 + 0.1915 = 0.6247
2 2

Number of days = 0.6247 x 365 = 228 days.

Example 10

(a) In a certain athletics competition, points are awarded according to level of performance. The
average grade was 82 points with standard deviation of 5 points. All competitors whose
grades ranged between 88 to 94 points received certificates. If the grades are normally
distributed and 8 competitors received certificates. How many participants took part in the
competition?
88−82 94−82 8
P(88 < X < 94) = 𝑃 ( <𝑍< )= P(1.2 < Z < 2.4) =
5 5 𝑛
8
φ(2.4) – φ(1.2) = 0.4918 – 0.3849 = 0.1069 = ; n = 74.84
𝑛
hence 75 participants took part.
(b) If certificates were to be awarded to only those having between 90 and 94 points. What
proportion of the participants would acquire certificates.
90−82 94−82
P(90 < X < 94 = 𝑃 ( <𝑍< )=P(1.6 < Z < 2.4 = φ(2.4) – φ(1.6)
5 5
= 0.4918 – 0.4452 = 0.0466
= 0.0466 x 100% = 4.66%

Revision exercise 1

1. The amount of meat sold by a butcher is normally distributed with mean 43kg and standard
deviation 4kg. Determine the probability that the amount of meat sold is between 40kg and
50kg. (0.7333)
2. Given that a random variable X is X~N(2, 2.89). Find P(X<0)(0.1198)
3. In a school of 800 students their average weight is 54.5kg and standard deviation 6.8kg. given
that the weight of students are normally distributed, find
(i) Probability that the weight of any student randomly selected is 52.8 kg or less = 0.4014
(ii) Number of students who weigh over 75kg = 1
(iii) Weight of the middle 56% of the students (49.251< X < 59.750)
4. A sugar factory sells sugar in bags of mean weight 50kg and standard deviation 2.5kg. given that
the weight of the bags is normally distributed, find the
(i) Probability that the weight of any bag of sugar randomly selected lies between 51.5kg
and 53kg = 0.1592
(ii) Percentage of bags whose weight exceeds 54kg = 5.48%
(iii) Number of bags that will be rejected out of 1000 bags purchased for weighing below
45.0kg = 23
5. A certain maize firm sells maize in bags of mean weight 40kg and standard deviation 2kg,. given
that the weight of the bags are normally distributed, find
(i) Probability that the weight of any bag of maize randomly selected lies between 41.0
and 42.5kg = 0.2029
(ii) Percentage of bags whose weight exceeds 43kg = 6.68%
(iii) Number of bags that will be rejected out of 500 bags purchased for weighing below
38.5kg = 113
6. Given that the random variable X is X~N(300, 25) Find
(i) P(X>308) = 0.0548 (ii) P(X> 311.5) = 0.0107 (iii) P(X < 294) = 0.8849
(iv) P(X < 290.5) = 0.9713 (v) P(X > 302) = 0.6554 (vi) P(X > 312) = 0.9918
7. If X~N(50, 20). Find
(i) P(X > 60.3) = 0.0106 (ii) P(X < 47.3) = 0.273 (iii) P(X > 48.9) = 0.5972
(iv) P(X > 53.5) = 0.2831 (v) P(X < 59.8) = 0.9857 (vi) P(X < 62.3) = 0.9970
8. If X~N(-8, 12). Find
(i) P(X < -9.8) = 0.1587 (ii) P(X > 0) = 0.8413 (iii) P(X < -3.4) =0.9079
(iv) P(X > -5.7) =0.2533 (v) P(X < 10.8) = 0.2097 (vi) P(X > -1.6) = 0.0323
9. If X~N(α, α2) . Find
(i) P(X < 0) = 0.1587 (ii) P(X > 0) = 0.8413 (iii) P(X < 0.5α) = 0.6915 P(X > 0.5α) = 0.3085
10. If X~N(100, 80) . Find
(i) P(85 < X < 112) = 0.8634 (ii) P(105 < X < 115) = 0.2413
(iii) P(85 < X < 92) = 0.1388 (iv) P(|X|< √80 = 0.6826
11. If X~N(84, 12) . Find
(i) P(80 < X < 89)= 0.8014 (ii) P(X <79 or X > 92) = 0.085 (iii) P(76 < X < 82) = 0.2714
(iv) P(|X- 84|> 2.9) = 0.4028 (v) P(87 < X < 93) = 0.1886
12. The masses of packages from a particular machine are normally distributed with a mean of 200g
and standard deviation of 2g, find the probability that a randomly selected package from the
machine weighs
(i) less than 197g = 0.0668
(ii) more than 200.5g = 0.4013
(iii) between 198.5g and 199.5g = 0.1747
13. The heights of boys at a certain school follow a normal distribution with mean = 150.3cm and
variance 25cm, find the probability that a boy picked at random from the group has a height;
(i) less than 153cm = 0.7054
(ii) more than 158cm = 0.018
(iii) between 150 cm and 158 cm= 0.4621
(iv) more than 10cm difference from the mean height = 0.0046
14. The masses of a certain type of cabbages are normally distributed with mean of 1000g and
standard deviation of 0.15kg. In a batch of 800 cabbages, estimate how many have a mass
between 750g and 1290g = 740
15. Cartons of milk from quality super market are advertised as containing 1 litre, but in fact the
volume of the content is normally distributed with a mean of 1012ml and standard deviation of
15ml.
(i) Find the probability that a randomly chosen carton contains more than 1010ml =0.6554
(ii) In a batch of 1000 cartons, estimate the number of cartons containing less than the
advertised volume of milk =8
16. A random variable X is such that X~N(-5, 9). Find the probability that;
(i) A randomly chosen item from the population will have positive value = 0.0478
(ii) Out of 10 items chosen randomly, exactly 4 will have a positive value = 0.00082
17. The life of a laptop is normally distributed with a mean of 2000 hours and standard deviation of
120 hours. Estimate the probability that the life of such a laptop will be
(i) greater than 2150 hours = 0.1056
(ii) greater than 1910hours = 0.7734
(iii) within a range 1850 hours to 2090 hours = 0.6678
Solutions to revision questions 1

1. The amount of meat sold by a butcher is normally distributed with mean 43kg and standard
deviation 4kg. Determine the probability that the amount of meat sold is between 40kg and
50kg.
X ~N(43,4)
40−43 50−43
P(40<x<50) = 𝑃 ( <𝑍< )
4 4
= P(- 0.75 < Z < 1.75)

P(40<x<50) = P(-0.75 < Z < 0) + P(0 < Z < 1.75)


By property of symmetry
P(40<x<50) = P(-0.75 < Z < 0) + P(0 < Z < 1.75)
= 0.2735 + 0.4599
= 0.733

2. Given that a random variable X is X~N(2, 2.89). Find P(X<0)


μ = 2, σ = √2.89 = 1.7
0−2
P(X < 0) = 𝑃(𝑍 < ) = P(Z < - 1.176) = P(X> 1.176)
1.7
= 0.5 – P(0 < Z < 1.176)
= 0.5 – 0.3802 = 0.1198
3. In a school of 800 students their average weight is 54.5kg and standard deviation 6.8kg. given
that the weight of students are normally distributed, find
(i) Probability that the weight of any student randomly selected is 52.8 kg or less
Let x be the weight of the student
52.8−54.5
P(x ≤ 52.8) = 𝑃 (𝑍 < )= P(Z <-0.25)
6.8

-0.25 0 0.25
Z
= P(Z> 0.25) = 0.5- P(0 < Z < 0.25) = 0.5 -0.0987 = 0.4013
(ii) Number of students who weigh over 75kg = 1
75−54.5
P(Z > 75) = 𝑃 (𝑍 > ) = P(Z > 3.01)
6.8

0 3.01
P(ZZ > 3.01) = 0.5 – P(0 < Z < 3.01) = 0.5 – 0.4990= 0.001
Number of students who weigh more than 75g = 800 x 0.001 =1
(iii) Weight of the middle 56% of the students
𝑥1 −54.5
𝑍1 =
6.8
𝑥1 −54.5
-0.772 = ; x1 = 49.251
Z1 0 Z2 Z 6.8
𝑥1 −54.5
𝑍2 =
6.8
P(X1 < X < X2) = P(Z1< Z < Z2) = 0.56 𝑥1 −54.5
0.772 = ; x2 = 59.750
But P(0< Z < Z2) = 2.8; Z2 = 0.772 and Z1 = -0.772 6.8
Hence the weight range of the middle 56% of students of the school is 49.251< X < 59.750

4. A sugar factory sells sugar in bags of mean weight 50kg and standard deviation 2.5kg. given that
the weight of the bags is normally distributed, find the
(i) Probability that the weight of any bag of sugar randomly selected lies between 51.5kg
and 53kg
51.5− 50 53− 50
P(51.5 < X < 53) = <𝑍< = P(0.6 < Z 1.2)
2.5 2.5
= φ(1.2) – φ(0.6) = 0.3849 – 0.2257== 0.1592
(ii) Percentage of bags whose weight exceeds 54kg
54− 50
P(X > 54) =P(𝑍 > ) = 𝑃(𝑍 > 1.6)= 0.5 –φ(1.6) = 0.5 – 0.4452 = 0.0548
2.5
= 0.0548 x 100 = 5.48%
(iii) Number of bags that will be rejected out of 1000 bags purchased for weighing below
45.0kg
45− 50
P(X < 45) = P(𝑍 < ) = 𝑃(𝑍 < −2) = 𝑃(𝑍 < 2) = 0.5 –φ(2)= 0.5- 0.4772 = 0.0228
2.5
Number of bags rejected = 0.0228 x 1000 = 22.8 ≈23
5. A certain maize firm sells maize in bags of mean weight 40kg and standard deviation 2kg, given
that the weight of the bags are normally distributed, find
(i) Probability that the weight of any bag of maize randomly selected lies between 41.0
and 42.5kg
41− 40 42.5− 40
P(41.0 < X < 42.5) = <𝑍< = P(0.5 < Z 1.25)
2 2
= φ(1.25) – φ(0.5) = 0.3944 - 0.1915 = 0.2029

(ii) Percentage of bags whose weight exceeds 43kg


43−40
P(X > 43) =P(𝑍 > ) = 𝑃(𝑍 > 1.5)= 0.5 –φ(1.5) = 0.5 – 0.4332 = 0.0668
2
= 0.0668 x 100 = 6.68%

(iii) Number of bags that will be rejected out of 500 bags purchased for weighing below
38.5kg
38.5−40
P(X < 38.5) = P(𝑍 < ) = 𝑃(𝑍 < −0.77) = 𝑃(𝑍 < 0.75) = 0.5 –φ(0.75)
2
= 0.5- 0.2734 = 0.2266
Number of bags rejected = 0.2266 x 500 = 113
7. If X~N(50, 20). Find
(i) P(X > 60.3)
60.3−50
P(X > 60.3) = P(Z > ) = P(Z > 2.303) = 05 –φ(2.303)
√20
= 0.5 –(0.4893 + 0.0001) = 0.0106
(ii) P(X < 47.3)
47.3−50
P(X < 47.3) P(Z > ) = P(Z <-0.6037) =P(Z> 0.6037) = 05 –φ(0.6037)
√20
= 0.5 –(0.2257 + 0.0013) = 0.273
(iii) P(X > 48.9)
48.9−50
P(X < 48.9) = P(Z > ) = P(Z <-0.246) = P(Z < 0.246) = 0.5 +φ(0.246)
√20
= 0.5 + 0.0948 + 0.0022 = 0.597
(iv) P(X > 53.5)
53.5−50
P(X > 53.5) = P(Z > ) = P(Z > 0.783) =0.2823 + 0.0008 = 0.2831
√20
(v) P(X < 59.8)
59.8−50
P(X < 59.8) = P(Z < ) = P(Z < 2.191) = 0.5 + φ(2.191)
√20
= 0.5 + 0.4826 +0.0001 =0.9857
(vi) P(X < 62.3)

62.3−50
P(X < 62.3) = P(Z < ) = P(Z < 2.750) = 0.5 + φ(2.730
√20
= 0.5 + 0.4970 = 0.9970

How to obtain Z-values from a given probabilities


If you are interested in finding the Z-values whose probabilities are given, it is important to note that
the Z-value may be positive or negative.

Sign Probability Z-value


< < 0.5 -
> >0.5 -
< >0.5 +
> <0.5 +
Note: for the above table the probability given in the question always correspond to Q in the critical
table

Example 11
P(Z<Z1) = 0.5, find Z1
P(Z<Z1) = 0.5, find Z1
Z1 = -0.674 (negative since 0.25 < 0.5 read directly
P(Z<Z1) = 0.5, find Z1
from a critical table)
Solution

P(Z<Z1) = 0.5(Q)

Example 12

P(Z < Z1) = 0.0968, find Z1 Z1 = -1.3 (negative since 0.0968 < 0.5 read directly
from a critical table)
Since 0.0968(Q) is not on critical table

P(Z < Z1) = 0.5 – 0.0968 = 0.403(P)

Example 13

P(Z < Z1) = 0.5, find Z1 Z1 = -1.645 (negative since 0.05 < 0.5 read directly
from a critical table)
Solution

P(Z < Z1) = 0.05(Q)

Example 14
and 0.2881. Since the extra information to the
P(Z< a) = 0.787, find a
right hand side is add, we consider the smallest
Since 0.787 is not on critical table value i.e. 0.2852 but 2852 corresponds to 0.79

P(Z < a) = 0.787 – 0.5 = 0.287 to get the next

From the table0.287 lies between 0.2852 0.2870 – 0.285 = 0.0018


So we look for 0.0018 on the add column which gives 6

∴ a = 0.79 + 0.006 = 0.796

Example 15

P(Z > b) = 0.01, find b

P(Z > b) = 0.01(Q)

b = 2.326 read directly from critical table

Inverse process (De-standardizing Z)

It involves converting the Z-value to raw data (X) form

Example 16

If X~(100, 36) and P(X > α) = 0.8907, find the value of α


𝛼−100
Since 0.8907 is not critical on a critical table 1.23 =
6
𝛼−100
P(Z < ) = 0.8907 – 0.5 = 0.390(P) α = 100 + 1.23 x 6 =107.38
6

From the table Z =1.23

Example 17

If X~(24, 9) and P(X > b) = 0.974, find the value of b


𝑏−24 𝑏−24
P(Z < ) = 0.974 – 0.5 = 0.474(P) 1.943 =
3 3

From the table Z =-1.943 α = 24 -1.943 x 3 =18.171

Example 18

The height of flowers in a farm is normally distributed with the mean 169 cm and standard deviation
9cm. if X stands for the height of flowers in cm, find X values for

(a) P(X < a) = 0.8

Solution From the table Z = 0.842

P(X < a) = 0.8(Q) a = 0.842 x 9 + 169 = 176.38

𝑎−169
𝑃(𝑍 < ) = 0.8 – 0.5 = 0.3(P)
9

(b) P(X > b) = 0.6

Solution From the table Z = -0.253


P(X > b) = 0.6 (Q) b = -0.253 x 9 + 169 = 166.72
𝑎−169
𝑃(𝑍 < ) = 0.6 – 0.5 = 0.1(P)
9
Example 19

The period of a certain machine approximately follows a normal distribution with mean of five years
and standard deviation of 1 year. Given that the manufacturer of this machine replaces the machine
that fails under guarantee, determine the

(i) Length of the guarantee required so that not more than 2% of the machine that fail are
replaced.
P(X < X0) = 0.02(Q)
𝑋0 −5
𝑃 (𝑍 < ) = 0.02(Q)
1
From the table Z = -2.054
X0= -2.054 x 1 + 5 = 2.946
∴ the guarantee period is 2.946 years
(ii) The proportion of the machines that would be replaced if the guarantee period was four
years
4−5
P(X < 4) = 𝑃 (𝑍 < ) = P(Z < -1) = P(Z > 1) = 5 – φ(1) = 5 – 0.3413 = 0.1587
1
P(Z < 4) = 0.1587 x 100 = 15 .87%

Example 20

The marks of 500 students in a mock examination for which the ass mar was 50%. Their marks are
normally distributed with mean 45 marks and standard deviation 20 marks.

(a) Given that the pass mar is 41, estimate the number of candidates who passed the
examination.
41−45
P(X ≥ 41) = 𝑃 (𝑍 < ) = P(Z ≥ -0.2) = P(Z ≤ 0.2) = 0.5 + φ(0.2) = 0.5 + 0.0793 = 0.5793
20
Number of candidates who assed =0.5793 x 500 = 290
(b) If 5% of the candidates obtain a distinction by scoring X marks or more, estimate the value of
X.
P(X > X0) = 0.05(Q)
𝑋0 −45
𝑃 (𝑍 < )= 0.5 – 0.05 = 0.45(P); from the table Z = 1.645
20
X0 = 1.645 x 20 + 45 = 78
∴the distinction starts at 78%
(c) Estimate the interquartile range of the distribution
Interquartile range = q3 – q1
𝑞3 −55
𝑃 (0 < 𝑍 < ) = 0.25(P); from the table Z = 0.674
20
𝑞3 = 0.674 𝑥 20 + 45 = 58.48
𝑞1 −45
𝑃( < 𝑍 < 0) = 0.25(P); from the table Z = -0.674
20
𝑞1 = −0.674 𝑥 20 + 45 = 31.52
∴ interquartile range = 58.48 – 31.52 = 26.96

Example 21

If X~N(70, 25) and P(|X - 70|< a) = 0.8, find the value of a and hence the limits within which the
central 80% of the distribution lies.

P(|X - 70|< a) = P(-a <X – 70 < a) = P(-a + 70 < X < a + 70) = 0.8
−𝑎+70−70 𝑎+70−70 −𝑎 𝑎
𝑃( <𝑋< ) = 𝑃( < 𝑋 < ) = 0.8
5 5 5 5
𝑎 𝑎
2 𝑥 𝑃 (0 < 𝑋 < )= 0.8; 𝑃 (0 < 𝑋 < )= 0.4(P)
5 5

From table Z = 1.282


𝑎
1.282 = ; a = 6.41
5

But P(-a + 70 < X < a + 70) = 0.8

P(63.59 < X < 76.41) = 0.8

∴ Central 80% of the distribution lies between 63.59 and 76.41

Revision Exercise 2

1. Find the value of the following


(i) P(Z < a) = 0.506 [a = 0.015] (ii) P(Z < a) = 0.787 [a= 0.796] (iii)P(Z< a) =0.0296[a= -0.1887]
(iv) P(Z > a) = 0.713 [ a = -0.562] (iii)P(Z< a) = 0.325 [a=-0.454] (vi) P(|Z| > a) = 0.5 [a =0.674]
(vii) P(|Z|> a) = 0.6 [a = 0.842] (viii) P(Z< a) = 0.9738 [a =1.94] (ix) P(Z< a)= 0.2435[a = -0.695]
(x) P(Z > a) = 0.82 [a = - 0.915] (xi) P(Z >a ) = 0.2351 [a= 0.628] (xii) P(|Z|<a)= 0.6372[a=0.91]
(xiii) P(Z>a) = 0.097 a=1.66] (xv) P(|Z|> a) = 0.0404[a= 2.05]

2. Find the value of a if


(i) P(Z< a) =0.9693 [a =1.87] (ii) P(Z > a) = 0.3802 [a = 0.305] (iii) P(Z > a)= 0.7367 [a = -0.633]
(iv) P(Z< a) = 0.0793[a = -1.41] (v) P(|Z|< a) = 0.9 [a = 1.645]
3. If X~N(60, 25) find a if
(i) P(X > a) = 0.2324 [a = 63.66] (ii) P(X > a) = 0.0702 [a =67.37]
(iii) P(X > a) = 0.837 [a = 55.09] (iv) P(X > a) =0.7461 [a = 56.69]
4. If X~N(45, 16) find a if
(i) P(X < a) = 0.0317 [a = 37.57] (ii) P(X < a) = 0.895 [a = 50.01]
(iii) P(X < a) = 0.0456 [a = 38.24] (iv) P(X < a) = 0.996 [a = 55.6]
5. If X~N(400, 64) find a if
(i) P(|X-400|< a) = 0.75 [9.2] (ii) P(|X-400|< a) = 0.98 [18.61]
(iii) P(|X-400|< a) = 0.95 [15.68] (iv) P(|X-400|< a) = 0.975 [17.92]
(v) The limits within which the central 95% of distribution lies. [384.32 < X < 415.68]
(vi) Interquartile range of distribution [394.61, 405.39]
6. Bags of flour paced by a particular machine have masses which are normally distributed with
mean 500g and standard deviation 20g. 2% of the bags are rejected for being overweight.
Between what ranges of values should the mass of a bag of flour lie if it is to be accepted.
[0458.92, 546.52]
7. The masses of mangoes sold at a market are normally distributed with mean mass 600g and
standard deviation 20g.
(i) If a mango is chosen at random, find the probability that its mass lies between 570g and
610g [0.6247]
(ii) Find the mass exceeded by 7% of mangoes [629.52]
(iii) In one day 1000 mangoes are sold. Estimate how many weigh less than 545g [3]
8. The length of metal strips are normally distributed with mean of 120cm and standard deviation
of 10cm.
(a) Find the probability that a strip selected at random has a length
(i) greater than 105cm [0.9332] (ii) within 5cm of the mean =[0.383]
(b) Strips that are shorter than L cm are rejected. Estimate the value of L, if 9% or all the strips
are rejected. [106.6cm]
(c) In a sample of 500 strips, estimate the number having a length over 126cm. [137]
9. The number of shirts sold in a week by a boutique are normally distributed with a mean 2080
and standard deviation of 50. Estimate
(i) The probability that in a given week fewer than 2000 shirts are sold [0.0548]
(ii) The number of weeks in a year that between 2060 and 2130 shirts are sold [26]
(iii) The least number n of shirts such that the probability that more than n are sold in a
given week is less than 0.02 [2183]
10. Batteries for a transistor radio have a mean life under normal usage of 160 hours, with standard
deviation of 30 hours. Assuming the battery life follow normal distribution
(i) Find the percentage of batteries which have a life between 150 hours and 180 hours.
[37.8%]
(ii) Calculate the range, symmetrical about the mean, within which 75% of the batteries lives
lie. [125.5, 194.5]
(iii) If the radio takes four of these batteries and require all of them to be working, find the
probability that the radio will run for at least 135 hours. [0.405]
11. The length of type A rod is normally distributed with mean of 15cm and a standard deviation of
0.1cm. the length of another type B is also normally distributed with mean of 20cm and
standard deviation 0.16cm. For type A rod to be acceptable, its length must be between 14.8cm
and 15.2 cm and type B rod, the length must be between 19.8cm and 20.2cm.
(i) What is the proportion of type a rod is of acceptable length? [95.44%]
(ii) What is the probability that one of them is of acceptable length [0.7528, 0.2375]
12. The marks of 1000 students in an examination were normally distributed with mean 55 marks
and standard deviation 8 marks.
(i) If a mark of 71 or more is required for A-pass, estimate the number of a-passes awarded.
[23]
(ii) If 15% of the candidates failed, estimate the minimum mark required to pass. [47]
(iii) Calculate the probability that two candidates chosen at random both passes examination
[0.7225]
13. The burning life of a bulb approximately follows a normal distribution with mean of 1300hours
and standard deviation of 125 hours
(i) What is the probability that the bulb selected at random will burn for more than 1500
hours. [0.0548]
(ii) Given that the manufacturer guarantees to replace any bulb that burns for less than
1050hours, what percentage of the bulbs will have to be replaced. [2.28%]
(iii) If two bulbs are installed at the same time, what is the probability that bot will burn less
than 1400 hours but more than 1200 hours [0.3320]
14. The marks in an examination were found to be normally distributed with mean 53.9 and
standard deviation 16.5. 20% od the candidates who sat this examination failed. Find the pass
mark [40.007]
Finding the value of mean, μ or standard deviation, σ or both
Hint: X = Zσ + μ

Example 22

If X~N(100, σ2) and P(X < 106) = 0.8849, find the value of standard deviation, σ.

Solution
106−100
P(X < 106) = 0.8849 (Q –value) = 1.2
𝜎
106−100
𝑃 (𝑍 < ) = 0.8849 – 0.5 = 0.3849 (P – value) σ=5
𝜎

From table Z = 1.2

Example 23

The length of a certain item follows a normal distribution with mean, μ cm and standard deviation of
6cm. it is known that 4.78% of the items have length greater than 82cm, find the mean, μ.

Solution

P(X > 82) = 0.0478 (Q – value) 82−𝜇


= 1.667
6
82−𝜇
𝑃 (𝑍 > ) = 0.5 – 0.0478 = 0.4522(P – value) μ = 72 cm
6

From table Z = 1.667

Example 24

The masses of boxes of oranges are normally distributed such that 30% of them are greater than
4.00kg and 20% are greater than 4.53kg. Estimate the mean and standard deviation of the masses

Solution

P(X > 4) = 0.3 (Q – value) 4.53−𝜇


= 0.842
𝜎
4−𝜇
𝑃 (𝑍 > ) = 0.3 (Q – value)
𝜎 4.53 = 𝜇 + 0.842𝜎 ……………….(ii)
4−𝜇
= 0.524 Eqn. (ii) – eqn. (i): 0.53 = 0.318σ; σ = 1.67kg
𝜎

4 = 𝜇 + 0.524𝜎 ……………….(i) From eqn. (i)


4.53−𝜇 μ = 4 – 0.524 x 1.67 = 3.13kg
𝑃 (𝑍 > ) = 0.2 (Q – value)
𝜎

Example 25

The speed of cars passing certain Entebbe high way can be taken to be normally distributed. 95% of
the cars are travelling at less than 85m/s and 10% are travelling at less than 55m/s.

(i) Find the average speed of the cars passing through the high way
(ii) Find the proportion of the cars that travel at more than 70m/s
Solution

P(X < 85) = 0.95 (Q – value) 55 = μ – 1.282σ ……………………………. (ii)


85−𝜇 Eqn. (i) – eqn. (ii): 30 = 2.927σ; σ = 10.25m/s
𝑃 (𝑍 < ) = 0.45 (Q – value)
𝜎

85−𝜇
From eqn. (i)
= 1.645
𝜎
μ = 85 – 1.645x 10.25 = 68.14m/s
85 = 𝜇 + 1.645𝜎 ……………….(i)
70− 68.14
(ii) P(X > 70) = 𝑃 (𝑍 < )= P(Z > 0.182)
10.25
P(X < 55) = 0.1 (Q – value)
55−𝜇
= 0.5 – 0.0722
𝑃 (𝑍 < ) = 0.1 (Q – value)
𝜎
= 0.4278
55−𝜇
= −1.282
𝜎

Example 26

The masses of articles produced in a particular shop are normally distributed with mean μ and
standard deviation σ. 5% of the articles have greater than 85g and 10% have masses less than 25g.

(i) Find the values of μ and σ


(ii) Find the symmetrical limits, about the mean, within which75% of the masses lie.

Solution

P(X > 85) = 0.05 (Q – value) (ii) P(|X-51.3| < a) = 0.75

𝑃 (𝑍 >
85−𝜇
) = 0.05 (Q – value) P(-a + 51.3 < X < a + 51.3) = 0.75
𝜎
−𝑎 + 51.3 − 51.3 𝑎 + 51.3 − 51.3
85−𝜇 𝑃( <𝑍< )
= 1.645 20.5 20.5
𝜎
= 0.75
85 = 𝜇 + 1.645𝜎 ……………….(i)
−𝑎 𝑎
𝑃( <𝑍< ) = 0.75
P(X < 25) = 0.1 (Q – value) 20.5 20.5

𝑎
𝑃 (𝑍 <
25−𝜇
) = 0.1 (Q – value) 2 x 𝑃 (0 < 𝑍 < )= 0.375 (P – value)
20.5
𝜎
𝑎
25−𝜇
= −1.282 = 1.15 ; a = 23.575
20.5
𝜎

55 = μ – 1.282σ ……………………………. (ii) Lower limit = -23.575 + 51.3 = 27.73

Eqn. (i) – eqn. (ii): 60 = 2.927σ; σ = 20.5m/s Upper limit = 23.575 + 51.3 = 74.88

From eqn. (i)

μ = 85 – 1.645x 20.5 = 513g

Example 27

A total population of 700 students sat a mock examination for which the pass mark was 50%. Their
marks were normally distributed. 28 students scored below 40% while 35 students scored above
60%.

(a) Find the mean mark and standard deviation of the students’ marks.
(b) What is the probability that a student chosen at random passed the exam?
(c) Suppose the pass mark is lowered by 2%, how many more students will pass.

Solution
50−50.312
P(X < 40) =
28
=0.04 (Q – value) (ii) P(X ≥ 50) = 𝑃 (𝑍 ≥ )= P(Z ≥ - 0.053)
5.889
700

40−𝜇
𝑃 (𝑍 > ) = 0.04 (Q – value)
𝜎

40−𝜇
= −1.751
𝜎
-0.053 0 0.053 Z
40 = 𝜇 − 1.751𝜎 ……………….(i)
P(Z ≥ -0.053)
z = 0.5 + P(0 < Z < 0.053)
35
P(X > 60) = =0.05 (Q – value) = 0.5 + 0.0211 = 0.5211
700

60−𝜇 48−50.312
𝑃 (𝑍 > ) = 0.05 (Q – value) (iii) P(X ≥ 48) = 𝑃 (𝑍 ≥ )= P(Z ≥ - 0.053)
𝜎 5.889
60−𝜇
= 1.645
𝜎

60 = μ + 1.645σ ……………………………. (ii)

Eqn. (ii) – eqn. (ii): 20 = 3.396σ; σ = 5.889 -0.393 0 0.393 Z


z
P(Z ≥ -0.392) = 0.5 + P(0 < Z < 0.392)
From eqn. (i)

μ = 40 + 1.751 x 5.889 = 50.312 = 0.5 + 0.1528 = 0.6528

More proportion = 0.6528 – 0.5211 = 0.1317

More students = 0.1317 x 700 = 92

Example 28

A random variable X has a normal distribution with P(X > 55) = 0.2 and P(35 < X < 55) = 0.5. Find

(a) The value of the mean, μ and standard deviation, σ.


(b) The percentage of those with P(X > 45)

Solution
35 −𝜇
28 𝑃( < 𝑍 < 0) = 0.2(P – value)
P(X > 55) = =0.2 (Q – value) 𝜎
700
35−𝜇
55 −𝜇 = −0.842
𝑃 (𝑍 > ) = 0.2 (Q – value) 𝜎
𝜎

40−𝜇
35 = 𝜇 − 0.842𝜎 ……………….(ii)
= 0.842
𝜎
μ = 70.8797, σ = 42.5872
55 = 𝜇 + 0.842𝜎 ……………….(i) 45 −70.8797
(b) P(X > 45) = 𝑃 (𝑍 > )= P(Z > -0.608)
42.5872
P(35 < X < 55) = 0.5
0.3 = 0.5 + P(0 < Z < 0.608)
0.2
P 0.2
Q = 0.5 + 0.2283 = 0.7283
35−𝜇 55−𝜇
−( ) 0 ( ) Z Percentage = 0.7283 x 100 = 72.83
𝜎 𝜎
z
Revision exercise 3
1. X~N(45, σ2) and P(X > 51) = 0.288. find σ. [σ = 10.7]
2. X~N(21, σ2) and P(X < 27) = 0.9332. find σ. [σ = 4]
3. X~N(μ, 25) and P(X < 27.5) = 0.3085. find μ. [μ = 30]
4. X~N(μ, 12) and P(X > 32) = 0.8438. find μ. [μ = 35.5]
5. X~N(μ, σ2) and P(X > 80) = 0.0113 and P(X > 30) = 0.9713. find σ and μ. [μ =52.73 and σ= 11.96]
6. X~N(μ, σ2) and P(X > 102) = 0.42 and P(X < 97) = 0.25. find σ and μ. [μ =100.8 and σ= 5.71]
7. X~N(μ, σ2) and P(X <57.84) = 0.90 and P(X < 50) = 0.5. find σ and μ. [μ =50 and σ= 6.12]
8. X~N(μ, σ2) and P(X < 35) = 0.20 and P(35 < X < 45) = 0.65. find σ and μ. [μ =39.5 and σ= 5.32]
9. The length of rods produced in a workshop follow a normal distribution with mean μ and variance
4. 10%of the rods are less than 17,4cm long. Find the probability that a rod chosen at random will
be between 18cm and 23 cm. [0.7725]
10. The length of a stick follow a normal distribution. 10% are of length 250cm or more while 55% have
a length over 240cm. Find the probability that a stick chosen at random is less than 235cm long.
[0.203]
11. A certain make of car tyres can be safely used for 25000km on average before replaced. The
makers guarantee to pay compensation to anyone whose tyre does not last for 22000km. they
expect 7.5% of all the tyres sold to qualify for compensation. If the distance X travelled before a
tyre is replaced has normal distribution.
(i) Find the standard deviation [2080]
(ii) Estimate the number of tyres per 1000 which will not have been replaced when they have
covered 26500km. [236]
12. The continuous random variable X is normally distributed with mean μ and standard deviation σ. If
P(X < 53) =0.04 and P(X < 65) = 0.97, find the interquartile range [4.46]
13. Tea sold in packages marked 750g. The masses are normally distributed with mean 760g and
standard deviation σ. What is the maximum value of σ, if less than 1% of the packages are
underweight? [4.299]
14. In an examination 30% of the candidates fail and 10% achieve distinction. Last year the pass mark
(out of 200) was 84 and the minimum mark required for a distinction was 154. Assuming that the
marks of candidates are normally distributed, estimate the mean mark and standard deviation.
[μ = 104.31, σ = 38.76]
15. AT St Noa junior, the heights of students are normally distributed. 10% are over 1.8m and 20% are
below 1.6m.
(i) Find the mean height μ and standard deviation σ. [μ =1.68, μ = 0.09]
(ii) Find the interquartile range [0.13]
16. Observation of a very rage number of cars are certain point on a motor way established that the
speeds are normally distributed. 90% of the cars have speed less than 77.7km/h and only 5% of
cars have speed less than 63.1km/h. find the mean speed μ and standard deviation σ.
[μ = 71.305, σ = 4.988]
17. A sample of 100 apples is taken from a load. The apple have the following distribution of size.
Diameter (cm) 6 7 8 9 10
Frequency 11 21 38 17 13
Assuming that the distribution is approximately normal with mean μ and standard deviation σ.
(i) Determine μ and σ [μ = 8, σ = 1.16]
(ii) Find the range of sizes of apples for packing, if 5% are to be rejected as too small and 5%
are to be rejected as too large [ 6.10, 9.90]
18. The volumes of soda in bottles are normally distributed with mean of 333ml. Given that 20% of the
bottles contain more than 340ml, find
(i) Standard deviation of the volume of bottle. [8.31]
(ii) Percentage of bottles that contain less than 330ml. [35.9%]
19. The heights of 500 students are normally distributed with a standard deviation of 0.080cm. If the
heights of 129 of the students are greater than the mean height but less than 1.806m find the
mean height. [1.75]
20. The masses of boxes of apples are normally distributed such that 20% of them are greater than
5.08kg and 15% are greater than 5.62kg; find the mean and standard deviation. [μ= 2.74, σ = 2.78]
21. The masses of sugar are normally distributed. If 5% of the packets have mass greater than 510g
and 2% have masses greater than 515g. Find the mean and standard deviation. [μ = 490, σ = 12.2g]
22. Sugar packed in 500g packets is observed to be approximately normally distributed with standard
deviation of 4. If only 2% of the packets contained less than 500g of sugar. Find the mean weight of
sugar in the packets. [508.216g]
23. Sixty students sat for a mathematics contest whose pass mark was 40marks. Their scores in the
contest were approximately normally distributed. 9 students scored less than 20 marks while 3
scored more than 70 marks. Find the
(i) Mean scored and the standard deviation of the contest. [μ = 39.32, σ = 18.65]
(ii) Find the probability that a student chosen at random passed the contest. [ 0.4856]
24. The number of cows owned by residents n a village is assumed to be normally distributed. 15% of
the residents have less than 60 cows while 5% of residents have over 90 cows.
(a) Determine the values of the mean and standard deviations of cows [μ =71.5926, σ= 11.1899]
(b) If there are 200 residents, find how many have more than 80 cows. [45]
25. A random variable X has a normal distribution when P(X > 9) = 0.9192 and P (X < 11) = 0.7580. find
(a) the value of the mean and standard deviation. [μ= 10.3333, σ = 0.9524]
(b) P(X > 10) [0.6386

26. The marks in an examination were normally distributed with mean μ and standard deviation σ.
20% of the candidates scored less than 40marks and 10% more than 75 marks. Find the
(a) values of μ and σ. (08marks)
(b) percentage of the candidate who scored more than 50 marks.(04marks)

Solutions to revision exercise 3


20. The masses of boxes of apples are normally distributed such that 20% of them are greater than
5.08kg and 15% are greater than 5.62kg; find the mean and standard deviation.

Solution
5.62−𝜇
= 1.036
P(X > 5.08) = 0.2 (Q – value) 𝜎

5.08 −𝜇 5.62 = 𝜇 + 1.036𝜎 ……………….(ii)


𝑃 (𝑍 > ) = 0.2 (Q – value)
𝜎
Eqn. (ii) – eqn. (i)
5.08−𝜇
= 0.842
𝜎
0.54 = 0.194σ; σ = 2.7835kg
5.08 = 𝜇 + 0.842𝜎 ……………….(i)
From eqn. (i)
P(X > 5.62) = 0.15 (Q – value)
μ = 5.08 – 0.842 x 2.7835 = 2.7363kg
5.62 −𝜇
𝑃 (𝑍 > ) = 0.15 (Q – value)
𝜎

21. The masses of sugar are normally distributed. If 5% of the packets have mass greater than 510g
and 2% have masses greater than 515g. Find the mean and standard deviation.

Solution
515−𝜇
= 2.054
P(X > 510) = 0.05 (Q – value) 𝜎

510 −𝜇 515 = 𝜇 + 2.054𝜎 ……………….(ii)


𝑃 (𝑍 > ) = 0.05 (Q – value)
𝜎
Eqn. (ii) – eqn. (i)
510−𝜇
= 1.645
𝜎
5 = 0.409σ; σ = 12.225kg
510 = 𝜇 + 1.645𝜎 ……………….(i)
From eqn. (i)
P(X > 515) = 0.02 (Q – value)
μ = 510 – 1.645 x 12.225 = 489.89kg
515 −𝜇
𝑃 (𝑍 > ) = 0.02 (Q – value)
𝜎

22. Sugar packed in 500g packets is observed to be approximately normally distributed with standard
deviation of 4. If only 2% of the packets contained less than 500g of sugar. Find the mean weight of
sugar in the packets.

P(X < 500) = 0.02 (Q – value)


500 −𝜇
𝑃 (𝑍 < ) = 0.02 (Q – value)
4

500−𝜇
= −2.054
𝜎

Mean weight, μ = 500 + 2.054 x 4 = 508.216g

23. Sixty students sat for a mathematics contest whose pass mark was 40marks. Their scores in the
contest were approximately normally distributed. 9 students scored less than 20 marks while 3
scored more than 70 marks. Find the
(i) Mean scored and the standard deviation of the contest.

Solution
70−𝜇
9 = 1.645
P(X < 20) = =0.15 (Q – value) 𝜎
60
70 = 𝜇 + 1.645𝜎 ……………….(ii)
20 −𝜇
𝑃 (𝑍 > ) = 0.15 (Q – value)
𝜎
Eqn. (ii) – eqn. (i)
20−𝜇
= −1.036 50 = 2.681; σ = 18.65
𝜎

20 = 𝜇 − 1.036𝜎 ……………….(i) From eqn. (i)


3
P(X > 70) = = 0.05 (Q – value) μ = 20 + 1.036 x 18.65 = 39.3214
60

70 −𝜇
𝑃 (𝑍 > ) = 0.05 (Q – value)
𝜎
(ii) Find the probability that a student chosen at random passed the contest.
40−39.3214
P(X > 40) =𝑃 (𝑍 > )= P(Z > 0.0364) = 0.5 – φ(0.036) = 0.5 – 0.0144 = 0.4856
18.65

24. The number of cows owned by residents n a village is assumed to be normally distributed. 15% of
the residents have less than 60 cows while 5% of residents have over 90 cows.
(a) Determine the values of the mean and standard deviations of cows
(b) If there are 200 residents, find how many have more than 80 cows. [45]

Solution
70−𝜇
= 1.645
(i) P(X < 60) =0.15 (Q – value) 𝜎

60 −𝜇 90 = 𝜇 + 1.645𝜎 ……………….(ii)
𝑃 (𝑍 < ) = 0.15 (Q – value)
𝜎
Eqn. (ii) – eqn. (i)
20−𝜇
= −1.036
𝜎
30 = 2.681; σ = 11.1899
60 = 𝜇 − 1.036𝜎 ……………….(i)
From eqn. (i)
P(X > 90) = 0.05 (Q – value)
μ = 60 + 1.036 x 11.1899= 71.5927
70 −𝜇
𝑃 (𝑍 > ) = 0.05 (Q – value)
𝜎

80 −71.5927
(ii) P(X > 80) = 𝑃 (𝑍 > )= P(Z > 0.751)
11.1899

P(Z > 0.751) = 0.5 – (0 < Z < 0.751)

= 0.5 – 0.2737

=0.2263

Number of residents =200 x 0.2263 = 45

25. A random variable X has a normal distribution when P(X > 9) = 0.9192 and P (X < 11) = 0.7580. find
(a) The values of the mean and standard deviation (08marks)
9− 𝜇
P(X, >9) = P(𝑧1 > ) =0.9192
𝛿
11− 𝜇
P(X, < 11) = P(𝑧2 > ) =0.0.7580
𝛿
𝑧1 = −(0.4192) = −1.4
𝑧2 = (0.258) = 0.7
9− 𝜇
 = −1.4
𝛿
9 − μ = -1.4δ ……………………. (i)
11− 𝜇
 = 0.7
𝛿
11 − μ = 0.7δ …………………… (ii)
𝐸𝑞𝑛 (𝑖) − 𝐸𝑞𝑛 (𝑖𝑖)
−2 = -2.1δ
−2
δ= = 0.9524
− 2.1
From (i)
9 − μ = -1.4 x 0.9524
𝜇 = 10.333

(b) P(X>10) (04marks)


10−10.333
P(X>10) = P(𝑧 > )
0.9524
= P(z > -0.35)

P(X > 10) = P(0.5 + P(0 < z < 0.35)


= 0.5 + 0.1368
= 0.6368
26. The marks in an examination were normally distributed with mean μ and standard deviation σ.
20% of the candidates scored less than 40marks and 10% more than 75 marks. Find the
(c) values of μ and σ. (08marks)
Let x = marks scored
P(x < z< z0) = 20% = 0.2
P(x < 40) = P(z < z0) = 0.2

P(0 < x < z0) = 0.3


z0 = -0.842
40− 𝜇
but z =
𝜎
40− 𝜇
-0.842 =
𝜎
-0.842𝜎 = 40 − 𝜇 …………………….. (i)
P(x > 75) = 10% = 0.1

P(0 < z < z1) = 0.4


z1= 1.282
75− 𝜇
1.282 =
𝜎
1.282𝜎 = 75 − 𝜇 …………………….. (ii)
Eqn. (ii) – eqn. (i)
2.124𝜎 = 35
𝜎 = 16.478 (3𝐷)
𝑠𝑢𝑏𝑠𝑡𝑖𝑡𝑢𝑡𝑖𝑛𝑔 𝜎 𝑖𝑛𝑡𝑜 𝑒𝑞𝑛. (𝑖𝑖)
1.282 x 16.478 = 75 – 𝜇
𝜇 = 53.875
Hence μ = 53.875 and σ = 16.478
(d) percentage of the candidate who scored more than 50 marks.(04marks)
50−53.875
P(x > 50) = P(𝑧 − )
16.478

= P(z > -0.235)

= 0.5 + (0< z < 0.235)

= 0.5 + 0.0929

= 0.5929

= 59.29%

Binomial approximation to a normal distribution


Under the following conditions, the normal distribution is used to approximate binomial distribution

Conditions

(i) the number of trials of the binomial experiment should be large, n > 20.
(ii) The probability of success not to small or too large i.e. p constant and very close to 0.5
X~N(np, npq)
𝑋±0.5−𝑛𝑝
The z-value is obtained from Z =
√𝑛𝑝𝑞
Where ±0.5 is used to make the binomial distribution continuous.
Note; 0.5 must be subtracted from the minimum value and added to the maximum value
(𝑋−0.5)−𝑛𝑝
(i) P(X≥ 𝑥1 ) = 𝑃 (𝑍 ≥ )
√𝑛𝑝𝑞
(𝑋+0.5)−𝑛𝑝
(ii) P(X≤ 𝑥1 ) = 𝑃 (𝑍 ≤ )
√𝑛𝑝𝑞
(𝑋−0.5)−𝑛𝑝 (𝑋=0.5)−𝑛𝑝
(iii) P(𝑥1 ≤X≤ 𝑥2 ) = 𝑃 ( ≤𝑍 ≤ )
√𝑛𝑝𝑞 √𝑛𝑝𝑞

Example 29

In a box containing different pens, the probability that a pen is red is 0.35. Find the probability that
in a random sample of 400 pens from the box

(i) Less than 120 are red pens


(ii) More than 160 are red pens
(iii) Between 120 and 150 inclusive are red pens.

Solution
116.5 −140
N = 400, p = 0.35, q = 0.65 = 𝑃 (𝑍 ≤ ) = 𝑃(𝑍 ≥ 2.149)
√91

Mean, μ = np = 400 x 0.35 = 140 =0.5 – φ(2.149) = 0.5 – 0.4821 = 0.0158

σ = √𝑛𝑝𝑞 = √400 𝑥 0.35 𝑥 0.65 = √91 (iii) P(120 ≤ X ≤ 150)


119.5 −140 119.5 −140 150.5 −140
(i) P(X < 120) = 𝑃 (𝑍 ≤ ) = 𝑃(𝑍 ≤ 2.149) =𝑃 ( ≤𝑍 )
√91 √91 √91
=𝑃(𝑍 ≥ 2.149) = 0.5 – φ(2.149)
=P(-2.149 ≤ Z ≤ 1.101)
=0.5 – 0.4842 = 0.0158
(ii) P(X > 160) = P(X ≥ 161) =0.4842 + 0.3645 = 0.8487

Example 30

In unbiased coin is tossed 100 times, what is the probability that

(i) There will be more than 60 heads (ii) there will be less than 43 head

(iii) there will be between 45 heads and 55 head

Solution
42.5 −50
N = 100, p = 0.5, q = 0.5 = 𝑃 (𝑍 ≤ ) = 𝑃(𝑍 ≤ −1.5
5

Mean, μ = np = 100 x 0.5 = 50 = 𝑃(𝑍 ≥ 1.5) =0.5 – φ(1.5)

σ = √𝑛𝑝𝑞 = √100 𝑥 0.5 𝑥 0.5 = 5 = 0.5 – 0.4332 = 0.0668

(i) P(X > 60) = P(X ≥ 61) (iii) P(45 ≤ X ≤ 55)


60.5 −50
= 𝑃 (𝑍 ≥ ) = 𝑃(𝑍 ≥ 2.1) 44.5 −50 55.5 −50
5 =𝑃 ( ≤𝑍 )
5 5
=0.5 – φ(2.149) =0.5 – 0.4821 = 0.0179
=P(-1.1 ≤ Z ≤ 1.1) = 2 x φ(1.1)
(ii) P(X < 43) = P(X ≤ 42)
=2 x 0.3643 = 0.7286
Example 31

It is known that 72% f NTV viewer watch news at 9 pm. What is the probability that a sample of 500
viewer chosen at random

(i) More than 350 watch news (ii) fewer than 340 watch news (ii) exactly 350 watch news

Solution

(i) P(X > 350) = P(X ≥ 351) (iii) P(X = 350) = P(349.5 ≤ X ≤ 350.5)
350.5 −500 𝑥 0.72
= 𝑃 (𝑍 ≥ ) =𝑃 (
349.5 −500 𝑥 0.72
≤𝑍 ≤
350.5 −500 𝑥 0.72
)
√500 𝑥 0.72 𝑥 0.28
√500 𝑥 0.72 𝑥 0.28 √500 𝑥 0.72 𝑥 0.28
= 𝑃(𝑍 ≥ −0.946) = 𝑃(𝑍 ≤ 0.946
=0.5 + φ(0.946) =0.5 + 0.328 = 0.8280 = φ(-1.046 – φ0.946)
(ii) P(X < 340) = P(X ≤ 339)
338.5 −500 𝑥 0.72
= 0.3522 – 0.3280 = 0.0242
= 𝑃 (𝑍 ≤ )
√500 𝑥 0.72 𝑥 0.28
∴ P(X = 350) = 0.0242
= 𝑃(𝑍 ≤ −2.042) = 𝑃(𝑍 ≥ 2.042
=0.5 - φ(2.042) =0.5 - 0.4794= 0.0206

Example 32

A pair of balanced dice, each numbered 1 to 6 is tossed 150 times. Determine the probability that a
sum of seven appear at least 26 times
6 1
P(sum of 7) = p= = = 𝑃 (𝑍 ≥
25.5 −25
) = P(Z ≥ 0.11)
36 6
4.56

1 = 0.5 – φ(0.11) = 0.5 – 0.0438


(26−0.5) −150 𝑥
6
P(X ≥ 26) = 𝑃 (𝑍 ≥ 1 5
)
√150 𝑥 𝑥 = 0.4562
6 6

Example 33

Two players play a game in which each of them tosses a balanced coin. The game ends in a draw if
both get the same result. Determine the probability that in 100 trials, the game ends in a draw.

(i) At least 53 times (ii) at most 53 times


2 1
P(sum of 7) = p= = 1
4 2 (53+0.5) −100 𝑥
5
P(X ≤ 53) = 𝑃 (𝑍 ≤ 1 1
)
1 √100 𝑥 𝑥
(53−0.5) −100 𝑥 2 2
2
P(X ≥ 53) = 𝑃 (𝑍 ≥ 1 1
)
√100 𝑥 𝑥 53.5 −50
2 2 = 𝑃 (𝑍 ≤ ) = P(Z ≤ 0.7)
5
52.5 −50
= 𝑃 (𝑍 ≥ ) = P(Z ≥ 0.5) = 0.5 + φ(0.5) = 0.5 + 0.2580 = 0.7580
5

= 0.5 – φ(0.5) = 0.5 – 0.1915 = 0.3085 ∴ P(X ≤ 53) = 0.7580

Example 34

In a certain book of words per page follow normal distribution with mean 800 words and standard
deviation 40 words. Three pages are chosen at random, what is the probability that

(i) None of them has between 830 and 845 words.


(ii) At least tw pages have between 830 and 845 words
Solution
830−800 845−800
(i) P(830 ≤ X ≤ 845) = 𝑃 ( ≤𝑍≤ )= P(0.75 ≤ Z ≤ 1.25)
40 40

= φ(1.25) –φ(0.75) = 0.3522 – 0.3280 = 0.0962

P(X = 0) = 3C0(0.0962)0(0.9038)3 = 0.7383

(ii) P(X ≥ 2) = P(X = 2) + P(X = 3)


=3C2(0.0962)2(0.9038)1 + 3C3(0.0962)3(0.9038)0 = 0.02509 + 0.00089 = 0.02598

Revision exercise 4
1. A random variable X~B(200, 0.7). Find
(i) P(X ≥ 130) [0.9474] (ii) P(136 ≤ X < 148) [0.6325] (iii) P(X < 142) [0.5914] (iv) P(X= 152) [0.0111]
2. An ordinary unbiased die is thrown 120 times. Find the probability of obtaining at least 24 sixes.
[0.1958]
3. A pair f dice is tossed 144 times and the sum of the outcomes recorded. Find the probability
that a sum of 7 occurs at least 26 times. [0.3688]
4. In a school 45% f the boys are circumcised. Find the probability that in a group of 200 boys 97
are circumcised. [0.1432]
5. 10% of phones imported to Uganda are I-phones, a random sample of 1000 phones is taken.
Find the probability that
(i) Less than 80 are I-phones [0.0154]
(ii) Between 90 and 115 inclusive are I-phones [0.8145]
(iii) 120 or more are I-phones [0.02]
6. During Christmas, the probability that message is sent on phone successfully is 0.85.
(i) When 8 messages are sent, find the probability that at least 7 are successfully sent [0.657]
(ii) When 50 messages are sent, find the probability that at least 45 are successfully sent
[0.2142]
7. One –fifth f tourist have COVID 19. Find the probability that the number of tourist with COVID
19 is
(i) More than 20 in a sample f 100 people [0.4502]
(ii) Exactly 20 in a random sample of 100 people [0.0996]
(iii) More than 200 in a random sample of 1000 people [ 0.484]
8. If a fair die is thrown 300 times, what is the probability that
(i) There will be more than 60 sixes [0.0519]
(ii) There will be fewer than 45 sixes [0.1971]
9. A coin is biased such that head is twice as likely to occur as a tail. The coin is tossed 120 times.
find the probability that there will be
(i) Between 42 and 51 tails inclusive [0.3729]
(ii) 48 tails or less [0.9501]
(iii) Less than 34 tails [0.1039]
(iv) At least 72 and at most 90 heads [0.9290]
10. A lorry of potatoes has average one rotten potato in six. A green grocer tests a random sample
of 100 potatoes and decides to turn away the lorry if he finds more than 18 rotten potatoes in
the sample. Find the probability that he accepts the consignment. [0.6886]
11. On a certain farm, 20% of all the cows are infected by a tick disease. Find the probability that in
a sample of 50 cows selected at random not more than 10% of the cows are infected. [0.0558]
12. A pair of balanced dice, each numbered from 1 to 6 is tossed 180 times. determine the
probability that a sum of seven appears;
(i) Exactly 40 times [0.0108]
(ii) Between 25 and 35 inclusive times [0.7286]
13. On average 20% of all the eggs supplied by a farm have cracks. Find the probability that in a
sample of 900 eggs supplied by a far will have more than 200 cracked eggs. [0.0439]
14. On average 15% of all boiled eggs sold in a restaurant have cracks. Find the probability that in a
sample of 300 boiled eggs will have more than 50 cracked eggs [0.215]
15. Among spectators watching a football watch, 80% were the home supporters while 20% were
the visiting team supporters. If 2500 of the spectators are selected randomly, what is the
probability that there are at least 541 visitors in the sample? [0.0215]
16. A die is tossed 40 times and the probability of getting at any one toss is 0.122, estimate the
probability of getting between 6 to 10 sixes. [0.2048]
17. In an examination which consists of 100 questions, a student has a probability of 0.6 of getting
each answer correct. A student fails the examination if he obtains a mark less than 55, and
obtains a distinction for a mark of 68 or more. Calculate
(i) The probability that he fails the examination [0.1308]
(ii) The probability that he obtains a distinction [0.0629]
18. A research station supplies three varieties of seeds s1, S2 and S3 in the ratio 4:2:1. The
probabilities of germination of S1, S2 and S3 are 50%, 60% and 80% respectively
(i) Find the probability that a selected seed will germinate [
(ii) Given that 150 seeds are selected at random, find the probability that less than 90 seed
will germinate.
19. A biased die with faces labelled 1, 2, 3, 4, 5, and 6 is tossed 45 times. calculate the probability
that 2 appears
(i) More than 18 times [0.1342]
(ii) Exactly 11 times[0.0568]

Solutions to revision exercise 4


11. On a certain farm, 20% of all the cows are infected by a tick disease. Find the probability that in
a sample of 50 cows selected at random not more than 10% of the cows are infected. [0.0558]

Since n is large, we use the normal distribution to approximate binomial distribution

Given: n = 50, p = 0.2, q = 1 – 0.2 = 0.8, μ = np = 50 x 0.2 = 10,


=√𝑛𝑝𝑞 = √50 𝑥 0.2 𝑥 0.8 = √8
10% of 50 cows = 0.1 x 50 = 5
5.5
P(x < 5) = 𝑃 (𝑍 < ) = 𝑃(𝑍 < −1.591)
√8

-1.591 0 1.591 Z
𝑃(𝑍 < −1.591) = 𝑃(𝑍 > 1.591)
= 0.5 – P(0 < Z < 1.591)

= 0.5 – 0.4442 = 0.0558


12. A pair of balanced dice, each numbered from 1 to 6 is tossed 180 times. determine the
probability that a sum of seven appears;
(i) Exactly 40 times [0.0108]
(ii) Between 25 and 35 inclusive times [0.7286]

Solution

Dice 1 2 3 4 5 6
1 2 3 4 5 6 7
2 3 4 5 6 7 8
3 4 5 6 7 8 9
4 5 6 7 8 9 10
5 6 7 8 9 10 11
6 7 8 9 10 11 12
Let E = event that the sum 7 is picked when a pair of dice is tossed
6 1
n(E) = 7 and P(E) = =
36 6

Since n is large, we use the normal distribution to approximate binomial distribution


1 1 5 1
Given: n = 180, p = , q = 1 – = , μ = np = 180 x = 30,
6 6 6 6
1 5
=√𝑛𝑝𝑞 = √180 𝑥 𝑥 =5
6 6
Let x = number of times a sum of 7 appears
39.5−30 39.5−30
(i) P(x = 40) = P(39.5 < X < 40.5) = 𝑃 ( <𝑍< ) = 𝑃(1.9 < 𝑍 < 2.1)
5 5
=P(0 < Z < 2.1) - (0 < Z < 1.9) = 0.4821 – 0.4713 = 0.0108
24.5−30 35.5−30
(ii) P(25 ≤ x ≤ 35) = 𝑃 ( <𝑍< )= P(-.1.1 < Z < 1.1)
5 5

-1.1 0 1.1 Z

P(-.1.1 < Z < 1.1) = 2 x P() < Z < 1.1) = 2 x 0.3643 = 0.7286

Hence P(25 ≤ x ≤ 35) = 0.7286

13. On average 20% of all the eggs supplied by a farm have cracks. Find the probability that in a
sample of 900 eggs supplied by a far will have more than 200 cracked eggs.

Since n is large, we use the normal distribution to approximate binomial distribution


Given: n = 900, p = 0.2, q = 1 – 0.2 = 0.8, μ = np = 900 x 0.2 = 180,
=√𝑛𝑝𝑞 = √900 𝑥 0.2 𝑥 0.8 = 12
let x = number of eggs with cracks
P(x > 200) = P(Z > Z1)
200.5−180
Where 𝑍1 = = 1.708
12

0 1.708 Z
P(Z > 1.708) = 0.5 – P(0 < Z < 1.708)

= 0.5 – 0.4561 = 0.0439

14. On average 15% of all boiled eggs sold in a restaurant have cracks. Find the probability that in a
sample of 300 boiled eggs will have more than 50 cracked eggs
Since n is large, we use the normal distribution to approximate binomial distribution
Given: n = 300, p = 0.15, q = 1 – 0.15 = 0.85, μ = np = 300 x 0.15 = 45,
=√𝑛𝑝𝑞 = √300 𝑥 0.15 𝑥 0.85 = 6.1847

let x = number of eggs with cracks


P(x > 50) = P(Z > Z1)
50.5−45
Where 𝑍1 = = 0.889
6.1847

0 0.889 Z

P(Z > 0.889) = 0.5 – P(0 < Z < 0.889)

= 0.5 – 0.2850 = 0.215

15. Among spectators watching a football watch, 80% were the home supporters while 20% were
the visiting team supporters. If 2500 of the spectators are selected randomly, what is the
probability that there are at least 541 visitors in the sample?
Solution
Since n is large, we use the normal distribution to approximate binomial distribution
Given: n = 2500, p = 0.2, q = 1 – 0.2 = 0.8, μ = np = 2500 x 0.2 = 500,
σ =√𝑛𝑝𝑞 = √2500 𝑥 0.2 𝑥 0.8 = 20
let x = number of visitors to support their team
P(x > 540) = P(Z > Z1)
540.5−500
Where 𝑍1 = = 2.025
20

0 2.025 Z
P(Z > 2.025) = 0.5 – P(0 < Z < 2.025)

= 0.5 – 0.4785 = 0.0215

16. A die is tossed 40 times and the probability of getting at any one toss is 0.122, estimate the
probability of getting between 6 to 10 sixes.
Solution
Given: n = 40, p = 0.122, q = 1 – 0.122 = 0.878, μ = np = 40 x 0.122 = 4.88,
σ =√𝑛𝑝𝑞 = √40 𝑥 0.122 𝑥 0.878 = 2.07
Let x be the number of sixes
P(6 < x < 10) = P(7≤ x ≤ 9)
Using normal approximation to binomial
𝑋±0.5− 𝜇
Z=
𝜎
6.5−4.88 9.5−4.88
P(7≤ x ≤ 9) = 𝑃 ( ≤𝑍 ≤ )
2.07 2.07
=P(0.78 < 𝑍 < 2.23
= P(0 < Z < 2.23) – P(0 < Z < 0.78)
= 0.4871 – 0.2823
= 0.2048
17. In an examination which consists of 100 questions, a student has a probability of 0.6 of getting
each answer correct. A student fails the examination if he obtains a mark less than 55, and
obtains a distinction for a mark of 68 or more. Calculate
(i) The probability that he fails the examination [0.1308]
(ii) The probability that he obtains a distinction [0.0629]

Solution

Given: n = 100, p = 0.6, q = 0.4, μ = np = 100 x 0.6 = 60 and σ = √𝑛𝑝𝑞 = √100 𝑥 0.6 𝑥 0.4 = 4.899

Let x = mark scored


54.5−60 67.5−60
(i) P(x < 55) = P(x ≤ 54) = 𝑃 (𝑍 ≤ ) (ii) P(x ≥ 68) = 𝑃 (𝑍 ≤ )
4.899 4.899

= (Z ≤ -1.123) =P(Z ≥ 1.531)

0 1.531 Z
-1.123 0 1.123 Z
P(Z < -1.123) = P(Z > 1.123) P(Z ≥ 1.531) = 0.5 – P(0 ≤ Z ≤ 1.531)
= 0.5 – P(0 ≤ Z ≤1.123) = 0.5 -0.4371
=0.5 -0.3692 = 0.0629
= 0.1308

18. A research station supplies three varieties of seeds s1, S2 and S3 in the ratio 4:2:1. The
probabilities of germination of S1, S2 and S3 are 50%, 60% and 80% respectively
(i) Find the probability that a selected seed will germinate [
(ii) Given that 150 seeds are selected at random, find the probability that less than 90 seed
will germinate.

Solution

Given

4:2:1
4 + 2+ 1 = 7
4 2 1
P(S1) = ; P(S2) = ; P(S3) =
7 7 7
Let G = germination of seeds
P(𝐺⁄𝑆 ) = 50% = 0.5
1
𝐺
P( ⁄𝑆 ) = 60% = 0.6
2
𝐺
P( ⁄𝑆 ) = 80% = 0.8
3
(a) Find the probability that a seed selected at random will germinate.

P(G) = 𝑃(𝑠1 ∩ 𝐺) + 𝑃(𝑠2 ∩ 𝐺) + 𝑃(𝑠3 ∩ 𝐺)


= P(S1).P(𝐺⁄𝑆 ) + P(S2).P(𝐺⁄𝑆 ) + P(S3 ). P (𝐺⁄𝑆 )
1 2 3
4 2 1
= 𝑥 0.5 + 𝑥 0.6 + 𝑥 0.8
7 7 7
2 1.2 0.8
= + +
7 7 7
4
=
7
Or Using factor tree diagram

0.5 G/S1
S1
4 0.5 G’/S1
7 2
0.6 G/S2
7
S2 G’/S2
0.4
1 0.8 G/S3
7 S3
0.2 G’/S3

4 2 1
P(G) = 𝑥 0.5 + 𝑥 0.6 + 𝑥 0.8
7 7 7
2 1.2 0.8
= + +
7 7 7
4
=
7

(b) Given that 150 seeds are selected at random, find the probability that less than 90 of the
seeds will germinate. Give your answer to two decimal places.
4 3
n = 150; P = ; q =
7 7
since n = large (= 150), we use the normal approximate this binomial
4 600
μ = np = 𝑥150 =
7 7
600 3 30√2
σ = √𝑛𝑝𝑞 = √ 𝑥 =
7 7 7
Let X = number of seeds that will germinate
P(x < 90) = P(x ≤ 89)
600
89.5−
7
= P(𝑧 ≤ 30√2
)
7
600
7(89.5− )
7
= P(𝑧 ≤ )
30√2
628.5−600
= P(𝑧 ≤ )
30√2
= P(𝑧 ≤ 0.6250)

= 0.5 + (0≤ 𝑧 ≤ 0.625)


= 0.5 + 0.2340
= 0.7340
= 0.73 (2D)

19. A biased die with faces labelled 1, 2, 3, 4, 5, and 6 is tossed 45 times. calculate the probability
that 2 appears
(i) More than 18 times (07marks)
2 1 2
n =45, p = = , q =
6 3 3
1
μ = np = 45 x = 15
3
1 2
σ =√𝑛𝑝𝑞 = √45 𝑥 𝑥 = √10
3 3
Changing binomial to normal distribution.
P(X> x) = P(X> 18 + 0.5) = P(X> 18.5)
̅̅̅̅̅̅̅
𝑥− 𝜇
Standardizing using z=
𝜎
18.5−15
P(X> 18.5) = P(z > = P(z > 1.107)
√10

P(z > 1.107) =0.5 – P(0 < z < 1.107)


= 0.5 – 0.3658
= 0.1342
∴ P(X > 18) = 0.1342
(ii) Exactly 11 times (05marks)
P(X = 11) = P(11 – 0.5 < X < 11 + 0.5)
= P(10.5 < X < 11.5)
10.5−15 11.5−15
= P( <𝑧< )
√10 √10
= P(-1.423 < z < 1.107)
By symmetry
P(-1.423 < z < 1.107) = P(1.107 < z < 1423)
P(1.107 < z < 1423) = P(0 < z < 1.423) – P(0 < z < 1.107)
= 0.4226 – 0.3658
= 0.0568

Distribution of sample mean of a normal distribution population


If a random variable X of a sample of size n from a normal distribution with mean μ and variance σ2,
then distribution of the sample mean 𝑥 is also said to be normally distributed with mean μ and
𝜎2 𝜎2
variance , such that 𝑥 ≈ (𝜇, )
𝑛 𝑛

𝑥−𝜇
Then 𝑍 = 𝜎2
𝑛

Example 35

At a certain school, the masses of students are normally distributed with mean 70kg and standard
deviation 5kg. If 4 students are randomly selected, find the probability that their mean is less than
65.

65−70
P(𝑋 < 65) = 𝑃 (𝑍 < 5 ) = 𝑃(𝑍 < −2)
√4

𝑃(𝑍 < −2) = 𝑃(𝑍 > 2) = 0.5 − 𝑃(0 < 𝑍 < 2) =0.5 – 0.4772 = 0.0228

Example 36

A random sample of size 15 is taken from a normal population with mean 60 and standard deviation
4. Find the probability that the mean of the sample is less than 58

58−60
P(𝑋 < 58) = 𝑃 (𝑍 < 4 ) = 𝑃(𝑍 < −1.936)
√15

𝑃(𝑍 < −1.936) = 𝑃(𝑍 > 1.936) = 0.5 − 𝑃(0 < 𝑍 < 1.936) =0.5 – 0.4736 = 0.0264

Example 37

The height of students are normally distributed with mean 164cm and standard deviation 7.2cm.
Calculate the probability that the mean height of a sample of 36 students will be between 162cm
and 166cm.

162−164 166−164
P(162 < 𝑋 < 166) = 𝑃 ( 7.2 <𝑍< 7.2 ) = 𝑃(−1.667 < 𝑍 < 1.667)
√36 √36

𝑃(−1.667 < 𝑍 < 1.667) = 2 𝑥 𝑃(0 < 𝑍 < 1.667 = 2 x 0.4522 = 0.9044
Example 38

The height of a certain plant follows a normal distribution with mean 21cm and standard deviation
√90𝑐𝑚. A random sample of 10 plants is taken and the mean height calculated. Find the probability
that this sample mean lies between 18cm and 27 cm

18−21 27−21
P(18 < 𝑋 < 27) = 𝑃 ( √90
<𝑍< √90
) = 𝑃(−1 < 𝑍 < 2)
√10 √10

𝑃(−1 < 𝑍 < 2) = 𝑃(0 < 𝑍 < 1) + 𝑃(0 < 𝑍 < 2) = 0.3413 + 0.4772 = 0.8185

Example 39

A large number of random sample of size n is taken from a distribution X where X ~N(74, 36) and the
sample mean 𝑥 for each sample is noted. If P(𝑥 > 72) = 0.854, 𝑓𝑖𝑛𝑑 𝑡ℎ𝑒 𝑣𝑎𝑙𝑢𝑒 𝑜𝑓 𝑛.

72−74
P(𝑋 > 72) = 𝑃 (𝑍 > 6 ) = 0.854
√𝑛

−√𝑛
𝑃 (𝑍 > ) = 0.854
3

From table Z = - 1.054


−√𝑛
= −1.054
3

n = 10

Example 40

The distribution of a random variable x is X ~N(25, 340) and the sample mean 𝑥 for each sample is
calculated. If P(𝑥 > 28) = 0.005, 𝑓𝑖𝑛𝑑 𝑡ℎ𝑒 𝑣𝑎𝑙𝑢𝑒 𝑜𝑓 𝑛.

28−25
P(𝑋 > 28) = 𝑃 (𝑍 > √340
) = 0.005
√𝑛

3√𝑛
𝑃 (𝑍 > ) = 0.005
√340

From table Z = 2.576


3√𝑛
= 2.576
√340

n = 250

Revision exercise 5
1. If X~𝑁(200, 80) and a random sample of size 5 is taken from the distribution, find the
probability that the sample mean
(i) is greater than 207 [0.0401]
(ii) lies between 201 and 209 [0.3891]
2. If X~𝑁(200, 10) and a random sample of size 10 is taken from the distribution, find the
probability that the sample mean lies outside the range 198 and 205 [0.3206]
3. If X~𝑁(50, 12) and a random sample of size 12 is taken from the distribution, find the
probability that the sample mean
(i) Is less than48.5 [0.0668]
(ii) Is less than 52.3[0.9893]
(iii) Lie between 50.7 and 51.7 [0.1974]
4. Biscuits are produced with weight (W g) where W is N(10, 4) and are packed at random into
boxes consisting of 25 biscuits. Find the probability that
(i) a biscuit chosen at random weigh between 9.25g and 10.7g [0.2924]
(ii) the content of a box weighs between 245g and 255g [0.0796]
(iii) the average weight of the biscuit in the box lies between 9.7g and 10.3g [0.5468]
5. A normal distribution has a mean of 40 and standard deviation of 4. If 25 items are drawn at
random, find the probability that their mean
(i) 41.4 or more [0.0401]
(ii) Between 38.7 and 40 .7 [0.7571]
(iii) Less than 39.5 [0.2660]
6. A random sample of size 25 is taken from a normal population with mean 60 and standard
deviation 4. Find the probability that the mean of the sample
(i) Less than 58 [0.0062]
(ii) Greater than 58 [0.9918]
(iii) Between 58 and 62[0.9876]
7. At St. Noa Junior, the marks of the pupils can be modelled by a normal distribution with mean
70% and standard deviation 5%. If four pupils are chosen at random, find the probability that
the mean mark is
(i) Less than 65% [0.9772]
(ii) Greater than 65% [0.0228]
(iii) Greater than 75%[0.0228]
(iv) Between 72% and 75% [0.1891]
8. The volume of soda in bottle are normally distributed with mean 758ml and standard deviation
of 12ml. a random sample of 10 bottles is taken and mean volume is found. Find the probability
that the sample mean is less than 750ml. [0.0176]
9. The height of cassava plants are normally distributed with mean of 2m and standard deviation
of 40cm. a random sample of 50 cassava plants is taken and the mean height found. Find the
probability that the sample mean lies between 195cm and 205cm. [0.6234]
10. In an examination, marks are normally distributed with mean 64.5 and variance 64. The mean
ark in a random sample of 100 scripts is denoted by X. find
(i) P(X> 65.5) [0.1056]
(ii) P(63.8 < 𝑋 < 64.5[0.3092]
11. The marks of an examination were normally distributed. 20% of the students scored below 40
marks while 10%of the students scored above 75 marks
(i) Find the mean mark and standard deviation of the students [μ = 53.87, σ = 16.473]
(ii) If 25 students were chosen at random from those who sat for the examination, what is
the probability that their average mark exceeds 60. [0.0313]
(iii) If a sample of 8 students were chosen, find the probability that not more than 3 scored
between 45 and 65 marks. [0.5419]
12. The life time of batteries produced by a certain factory is normally distributed. Out of 10,000
batteries selected at random, 668have life tie less than 130 hours and 228 have life time more
than 200 hours.
(i) Find the mean mark and standard deviation of the battery life time[μ =160, σ = 20]
(ii) Find the percentage of the batteries with life time between 150 and 180 hours.
(iii) If the sample of 25 batteries is selected at random, find the probability that the mean of
the life time exceeds 165 hours [0.1056]
13. A normal distribution has a mean of 30 and a variance of 5. Find the probability that
(i) The average of 10 observation exceeds 30.5 [0.2399]
(ii) The average of 40 observation exceeds 30.5[0.0787]
(iii) The average of 100 observation exceeds 30.5 [0.0127]
(iv) Find n such that the probability that the average of observations exceed 30.5 is less
than 1%, [n > 108]
14. The random variable is such that X~𝑁(μ, 4). A random sample size n is taken from the
population. Find the least n such that P(|𝑋 − 𝜇| < 0.5) = 0.95 [62]
15. Boxes made in a factory have weight which are normally distributed with a mean of 4.5kg and a
standard deviation of 2.0kg. if a sample of 16 boxes is drawn at random, find the probability
that their mean is
(i) between 4.6 and 4.7 kg [0.0761]
(ii) between 4.3 and 4.7g [0.3108]
16. the masses of soap powder in a certain packet are normally distributed with mean 842g and
variance 225g. find the probability that a random sample of 120 packets has sample mean mass
(i) between 844g and 846g [0.0702]
(ii) less than 843g[0.7673]

Estimation of population parameters


Statistical estimation is used to describe the unknown characteristics of the population (population
parameters) by using sample characteristics.

A sample is a representation of the population parameter such as population mean, μ and


population variance, σ2.

Types of parameter estimation

- point estimation
- interval estimation
(a) point estimates
(i) the unbiased estimate of the population mean, μ is
∑𝑥 ∑ 𝑓𝑥
𝑥= 𝑜𝑟 𝑥 = ∑𝑓
where 𝑥 is sample mean
𝑛
𝑛
(ii) the unbiased estimate of the population variance, σ2 is 𝜎̂ 2 where 𝜎̂ 2 = 𝑠 2 where
𝑛−1
𝑠 2 is sample variance
𝑛 ∑ 𝑥2 ∑𝑥 2 𝑛 ∑ 𝑓𝑥 2 ∑ 𝑓𝑥 2
OR 𝜎̂ 2 = [ −( ) ] or 𝜎̂ 2 = [ ∑𝑓
−(∑ ) ]
𝑛−1 𝑛 𝑛 𝑛−1 𝑓

Example 41

Find the best unbiased estimate of mean μ and variance σ2 of the population from each of the
following sample is drawn

(i) 46, 48, 50, 45, 53, 50, 48, 51

Solution
x f fx fx2
45 1 45 2025
46 1 46 2116
48 2 96 4608
50 2 100 5000
51 1 51 2601
53 1 53 2809
∑ 𝑓= 8 ∑ 𝑓𝑥= 391 ∑ 𝑓𝑥 2 = 19159
∑ 𝑓𝑥 391
Unbiased estimate for the mean 𝑥 = ∑𝑓
= = 48. 875
8

𝑛 ∑ 𝑓𝑥 2 ∑ 𝑓𝑥 2
The unbiased estimate of the population variance, 𝜎̂ 2 = [ ∑𝑓
−(∑ ) ]
𝑛−1 𝑓

8 19159 391 2
= [ −( ) ] = 6.982
8−1 8 8

(ii) ∑ 𝑥 = 100, ∑ 𝑥 2 = 1028, n = 10


∑ 𝑓𝑥 100
Unbiased estimate for the mean 𝑥 = ∑𝑓
= = 10
10

𝑛 ∑ 𝑓𝑥 2 ∑ 𝑓𝑥 2
The unbiased estimate of the population variance, 𝜎̂ 2 = [ ∑𝑓
−(∑ ) ]
𝑛−1 𝑓

10 1028 100 2
= [ −( ) ] = 3.11
10−1 10 10

(iii) ∑ 𝑥 = 120, ∑ 𝑥 2 = 2102, n = 8

∑ 𝑓𝑥 120
Unbiased estimate for the mean 𝑥 = ∑𝑓
= = 15
8

𝑛 ∑ 𝑓𝑥 2 ∑ 𝑓𝑥 2
The unbiased estimate of the population variance, 𝜎̂ 2 = [ ∑𝑓
−(∑ ) ]
𝑛−1 𝑓

8 2102 120 2
= [ −( ) ] = 43.14
8−1 8 8

(iv) ∑ 𝑥 = 330, ∑ 𝑥 2 = 23700, n = 34

∑ 𝑓𝑥 330
Unbiased estimate for the mean 𝑥 = ∑𝑓
= = 9.71
34

𝑛 ∑ 𝑓𝑥 2 ∑ 𝑓𝑥 2
The unbiased estimate of the population variance, 𝜎̂ 2 = [ ∑𝑓
−(∑ ) ]
𝑛−1 𝑓

34 23700 330 2
= [ −( ) ] = 621.12
34−1 34 34

(v) ∑ 𝑥 = 738, ∑ 𝑥 2 = 16526, n = 50

∑ 𝑓𝑥 738
Unbiased estimate for the mean 𝑥 = ∑𝑓
= = 14.76
50
𝑛 ∑ 𝑓𝑥 2 ∑ 𝑓𝑥 2
The unbiased estimate of the population variance, 𝜎̂ 2 = [ ∑𝑓
−(∑ ) ]
𝑛−1 𝑓

50 16526 738 2
= [ −( ) ] = 114.96
50−1 50 50

Example 42

The fuel consumption of a new car model was being tested. In one trials 8 cars chosen at random
were driven under identical conditions and distance x km covered on one litre of petro was
recorded. The following results were obtained. ∑ 𝑥 = 152.98, ∑ 𝑥 2 = 2927.1. Calculate the
unbiased estimate of the mean and variance of the distance covered by the car.

Solution
∑ 𝑓𝑥 152.98
Unbiased estimate for the mean 𝑥 = ∑𝑓
= = 19.1225
8

𝑛 ∑ 𝑓𝑥 2 ∑ 𝑓𝑥 2
The unbiased estimate of the population variance, 𝜎̂ 2 = [ ∑𝑓
−(∑ ) ]
𝑛−1 𝑓

8 2927.1 152.98 2
= [ −( ) ] = 0.25
8−1 8 8

(b) Interval estimate

Here we are interested in obtaining the interval over which the true population mean lies
(confidence interval)
𝛼
The unbiased estimate of the population 𝑃 (−𝑍𝛼⁄ < 𝑍 < 𝑍𝛼⁄ )=
2 2 2

mean, μ is 𝑥 𝑥− 𝜇 𝛼
𝑃 (−𝑍𝛼⁄ < 𝜎 < 𝑍𝛼⁄ )=
2 2 2
𝑥− 𝜇 √𝑛
Z= 𝜎 where n is the sample size
√𝑛 𝜎 𝜎 𝛼
𝑃 (𝑥 − 𝑍𝛼⁄ < 𝜇 < 𝑥 + 𝑍𝛼⁄ )=
2 √𝑛 2 √𝑛 2
Z is the area under the normal curve leaving
𝜎 𝜎
𝛼 Confidence interval [𝑥 − 𝑍𝛼⁄ < 𝜇 < 𝑥 + 𝑍𝛼⁄ ]
an area of on either side of the curve 2 √𝑛 2 √𝑛
2
𝜎 𝜎
Confidence Limits [𝑥 − 𝑍𝛼⁄ < 𝜇 < 𝑥 + 𝑍𝛼⁄ ]
2 √𝑛 2 √𝑛
P
𝜎
Q Or 𝜇 < 𝑥 ± 𝑍𝛼⁄
2 √𝑛
-𝑍𝛼⁄ 0 𝑍𝛼⁄ Z
2 2

(i) Confidence interval for population mean μ


- of a normal or non-normal population
- with known population variance σ2 or standard deviation σ
- using any sample size
𝜎
The confidence interval is obtained from 𝜇 < 𝑥 ± 𝑍𝛼⁄ where 𝑥 is sample mean
2 √𝑛

Example 43

The length of a bar of a metal is normally distributed with mean of 115cm and standard deviation of
3cm. find the 95% confidence limits for the length of the bar
𝛼 0.95
= = 0.475
2 2
𝛼 0.95 𝜎 Lower limit =109.12
= = 0.475 𝜇 < 𝑥 ± 𝑍𝛼⁄
2 2 2 √𝑛

3 Upper limit = 120.88


𝑍𝛼𝛼⁄ =
𝑍 = 1.96
1.96 𝜇 < 115 ± 1.96
⁄2 2 √1

Example 44

The mass of vitamin in a capsule is normally distributed with standard deviation 0.042mg. a random
sample of 5 capsules was taken and the mean mass of vitamin e found to be 5.12. calculate a
symmetric confidence interval for the population mean mass.

𝛼 0.95 𝜎
= = 0.475 𝜇 < 𝑥 ± 𝑍𝛼⁄ Lower limit =5.08
2 2 2 √𝑛

0.042 Upper limit = 5.16


𝑍𝛼⁄ = 1.96 𝜇 < 5.12 ± 1.96
2 √5

Example 44

It is known that an examination paper is marked in such a way that the standard deviation of the
marks is 15.1. in a certain school, 80 candidates took the examination and they had an average mark
of 57.4. find

(i) 95% confidence limits for the mean mark in the exam.

𝛼 0.95 𝜎
= = 0.475 𝜇 < 𝑥 ± 𝑍𝛼⁄ Lower limit =54.091
2 2 2 √𝑛

15.1 Upper limit = 60.709


𝑍𝛼⁄ = 1.96 𝜇 < 5.12 ± 1.96
2 √80

(ii) 99% confidence limits for the mean mark in the exam.

𝛼 0.99 𝜎
= = 0.495 𝜇 < 𝑥 ± 𝑍𝛼⁄ Lower limit =53.053
2 2 2 √𝑛

15.1 Upper limit = 61.746


𝑍𝛼⁄ = 2.575 𝜇 < 5.12 ± 2.595
2 √80

Example 45

After a particular rainy night, 12 worms were picked and their length in cm measured;
9.5, 9.5, 11.2, 10.6, 9.9, 11.1, 10.9, 9.8, 10.1, 10.2, 10.9, 11.0

Assuming that this sample came from a normal population with standard deviation 2, find the 95
confidence interval for the mean length of all the worms
∑ 𝑥 9.5 + 9.5 + 11.2 + 10.6 + 9.9 + 11.1 + 10.9 + 9.8 + 10.1 + 10.2 + 10.9 + 11.0
𝑥= =
𝑛 12
= 10.39

𝛼 0.95 𝜎
= = 0.475 𝜇 < 𝑥 ± 𝑍𝛼⁄ Lower limit =9.81
2 2 2 √𝑛

2 Upper limit = 10.97


𝑍𝛼⁄ = 1.96 𝜇 < 10.39 ± 1.96
2 √12
Example 45
The height of students are normally distributed with mean μ and standard deviation σ. On the bsis of
results obtained from a random sample of 100 students from school, the 95% confidence interval of
the mean was calculated and found to be (177.22cm, 179.18cm). Calculate
(i) the value of the sample mean
𝛼 0.95
= = 0.475 𝜎
2 2 179.18< 𝑥 + 1.96 …. (ii)
𝑍𝛼⁄ = 1.96 √100
2
𝜎 Eqn. (i) + eqn. (ii)
𝜇 < 𝑥 ± 𝑍𝛼⁄
2 √𝑛
2𝑥 = 356.4; 𝑥 = 178.2
𝜎
177.22< 𝑥 − 1.96 …. (i)
√100

(ii) the value of standard deviation


𝜎
177.22< 178.2 − 1.96 ;
√100
σ=5
(iii) 90% confidence interval of the mean, μ
𝛼 0.90
= = 0.45; 𝑍𝛼⁄ = 1.645
2 2 2

Example 46

A plant produces steel sheets whose weight are normally distributed with standard deviation of
2.4kg. A random sample of 36 sheets had a mean weight of 31.4kg.

(i) Find the 99% confidence limit for the population


𝛼 0.99 𝜎
= = 0.495 𝜇 < 𝑥 ± 𝑍𝛼⁄ Lower limit =30.37kg
2 (ii) 2 2 √𝑛

2.4 Upper limit = 32.43kg


𝑍𝛼⁄ = 2.575 𝜇 < 31.4 ± 2.595
2 √36
Thank you
(ii) Find the width of the 99% confidence limit

=32.43kg - 30.37kg = 2.06kg

Example 47

The marks scored by students are normally distributed with mean μ and standard deviation1.3. it is
required to have 95% confidence interval for μ with width less than 2. Find the least number of
students that be sampled to achieve this.

𝛼 0.95 𝜎 𝜎
= = 0.475 width = 2 𝑥𝑍𝛼⁄ <2 2 𝑥 1.96 < √𝑛; n < 6.49
2 2 2 √𝑛 2
𝜎 n > 6.49
𝑍𝛼⁄ = 1.96 2 𝑥 1.96 <2
2 √𝑛
the least number = 7

(ii) Confidence interval for population mean μ


- of a normal or non-normal population
- with unknown population variance σ2 or standard deviation σ
- using a large sample size (n ≥ 30)
If the population variance σ2 s not given or unknown, the confidence interval is obtained
̂
𝜎 𝑛
from 𝜇 < 𝑥 ± 𝑍𝛼⁄ where 𝑥 is sample mean, 𝜎̂ = 𝑠 2 and s = sample variance
2 √𝑛 𝑛− 1

𝑛 ∑ 𝑥2 ∑𝑥 2
𝜎̂ 2 = [ −( ) ]
𝑛−1 𝑛 𝑛
Or
𝑛 ∑ 𝑓𝑥 2 ∑ 𝑓𝑥 2
𝜎̂ 2 = [ ∑𝑓
−(∑ ) ]
𝑛−1 𝑓

Example 48

the fuel consumption of a new car was being tested. In one trials 50 cars chosen at random were
driven under identical conditions and the distance x km covered on one litre of petrol was recorded.
the following results were obtained. ∑ 𝑥 = 525 , ∑ 𝑥 2 = 5625. Calculate the 95%confidence
interval for the mean petrol consumption, in km per litre of cars of this type..
∑ 𝑓𝑥 525
Unbiased estimate for the mean 𝑥 = ∑𝑓
= = 10.5
50

𝑛 ∑ 𝑓𝑥 2 ∑ 𝑓𝑥 2
The unbiased estimate of the population variance, 𝜎̂ 2 = [ ∑𝑓
−(∑ ) ]
𝑛−1 𝑓

50 5625 525 2
= [ −( ) ] = 2.2952
50−1 50 50

𝛼 0.95 𝜎
= = 0.475 𝜇 < 𝑥 ± 𝑍𝛼⁄ Lower limit =10.08km/litre
2 2 2 √𝑛
Upper limit = 10.92km/litre
𝑍𝛼⁄ = 1.96 𝜇 < 10.5 ± 1.96
√2.2952
2
√50

Example 49

The height x cm of each man in a random sample of 200 men living in Nairobi was measured. The
following results were obtained∑ 𝑥 = 35050 , ∑ 𝑥 2 = 6163109.

(a) calculate the unbiased estimate of the mean and variance of the heights of men living Nairobi

∑ 𝑓𝑥 35050
Unbiased estimate for the mean 𝑥 = ∑𝑓
= = 175.25
200

𝑛 ∑ 𝑓𝑥 2 ∑ 𝑓𝑥 2
The unbiased estimate of the population variance, 𝜎̂ 2 = [ ∑𝑓
−(∑ ) ]
𝑛−1 𝑓

200 6163109 35050 2


= [ −( ) ] = 103.5
200−1 200 200

(b) Determine the 90% confidence interval for the mean height of mean living in Nairobi.

𝛼 0.90 𝜎
= = 0.45 𝜇 < 𝑥 ± 𝑍𝛼⁄ Lower limit =174.07cm
2 2 2 √𝑛
Upper limit = 176.43cm
𝑍𝛼⁄ = 1.645 𝜇 < 175.25 ± 1.645
√103.5
2
√200
Example 50

A random sample of 100 observations from a normal population with mean μ gave the following
results ∑ 𝑥 = 8200 , ∑ 𝑥 2 = 686000.

(a) calculate the unbiased estimate of the mean and variance of the heights of men living Nairobi

∑ 𝑓𝑥 8200
Unbiased estimate for the mean 𝑥 = ∑𝑓
= = 82
100

𝑛 ∑ 𝑓𝑥 2 ∑ 𝑓𝑥 2
The unbiased estimate of the population variance, 𝜎̂ 2 = [ ∑𝑓
−(∑ ) ]
𝑛−1 𝑓

200 68600 8200 2


= [ −( ) ] = 11.72
200−1 100 100

(b) Determine the 98% confidence interval for the mean

𝛼 0.98 𝜎
= = 0.49 𝜇 < 𝑥 ± 𝑍𝛼⁄ Lower limit =79.274
2 2 2 √𝑛

11.72 Upper limit = 84.726


𝑍𝛼⁄ = 2.326 𝜇 < 82 ± 2.326
2 √100

(c) determine the 99% confidence interval for the mean

𝛼 0.99 𝜎
= = 0.495 𝜇 < 𝑥 ± 𝑍𝛼⁄ Lower limit =78.981
2 2 2 √𝑛

11.72 Upper limit = 85.726


𝑍𝛼⁄ = 2.575 𝜇 < 82 ± 2.595
2 √100

Example 50

The mean and standard deviation of a random sample of size 100 is 900 and 60 respectively. Given
that the population is normally distributed, find 96% confidence interval of the population mean.

𝑛 100
𝜎̂ = √ 𝑠2 = √ 𝑥 602 = 60.302
𝑛−1 100−1

𝛼 0.96 𝜎
= = 0.48 𝜇 < 𝑥 ± 𝑍𝛼⁄ Lower limit =887.614
2 2 2 √𝑛

60.302 Upper limit = 912.386


𝑍𝛼⁄ = 2.054 𝜇 < 900 ± 2.054
2 √100

Revision exercise 6
1. the concentration in mg per litre of a trace element in 7 randomly chosen samples of water
from a spring were 240.8, 237.3, 236.7, 236.6, 234.2, 233.9, 232.5.
Determine the unbiased mean and variance of the concentration of the trace element per litre
from spring [236, 7.58]
2. Cartons of oranges are filled by a machine. A sample of 10 cartons selected at random from the
population contained the following quantities in ml) 201.2, 205.0, 209.1, 202.3,204.6, 206.4,
210.1,201.9, 203.7, 207.3. Determine the unbiased mean and variance of the population from
which the sample was taken. [203.16, 9.223]
3. A factory produces cans of meat whose masses are normally distributed with standard
deviation 18g. A random sample of 25 cans is found to have the mean of 458g. find the 99%
confidence interval for the population mean of a can of meat produced at the factory. [448.7,
467.3g]
4. The height of bounce of a tennis ball is normally distributed with standard deviation 2cm. A
sample of 60 tennis balls is tested and the mean height of bounce is 140cm. Find
(i) 95% [139.5, 140.51] (ii) 98% [139.4, 140.6] confidence interval for the mean height of
bounce of the tennis ball
5. A random sample of 100 is taken from a population. The sample is found to have a mean of 76.0
and standard deviation of 120. Find
(i) 90% [747.51, 748.49] (ii) 95% [747.42, 748.58] (iii) 98% [747.31, 748.69] confidence
interval for the mean of the population
6. 150 bags of flour of a particular brand are weighed and the mean mass is found to be 748g with
standard deviation 3.6g. Find
(i) 90% [74.02, 77.98] (ii) 97% [73.38, 78.62] (iii) 99% [72.89, 79.11] confidence interval for
the mean mass of bags of flour of this brand.
7. A random sample of 100 readings taken from a normal population gave the following data:
𝑥 = 82, ∑ 𝑥 2 = 686800. Find
(i) 98% [79.19, 84.81] (ii) 99% [78.89, 85.11] confidence interval of the population mean
8. 80 people were asked to measure their pulse rates when they woke up in the morning. The
mean was 69 beats and standard deviation 4 beats. find
(i) 95% [68.12, 69.88] (ii) 99% [67.84, 70.16] (iii) 97% [68.0, 70.0] confidence interval of the
population mean
9. The 95% confidence interval for the mean length of a particular brand of light bulbs is [1023.3h,
1101.7h]. This is interval is based on results from a sample of 36 light bulbs. Find the 99%
confidence interval for the mean length of life of this brand of light bulbs assuming that the
length of life is normally distributed. [1011, 1114]
10. A random sample of 6 items taken from a normal population with variance 4.5cm2 gave the
following data: 12.9cm, 13.2cm, 14.6cm, 12.6cm, 11.3cm, and 10.1 cm.
(i) Find the 95% confidence interval for the population mean. [10.75, 14.15].
(ii) What is the width of this confidence interval [3.4]
11. A random sample of 60 loaves is taken from a population whose mean masses are normally
distributed with mean μ and standard deviation 10g.
(i) calculate the width of 95% confidence interval for μ bases on the sample [5.06]
(ii) Find the confidence level having the same width as in (i) but based on a random sample
of 40 loaves. [89%]
12. The distribution of measurements of masses of a random sample of bags packed in a factory
is shown below
Mass (kg) 72.5 77.5 82.5 87.5 92.5 97.5 102.5 107.5
frequency 6 18 32 57 102 51 25 9
(i) Find the mean and standard deviation of the masses [μ =91.317, σ=7.41]
(ii) find the 95% confidence limits[90.5, 92.2]

Thank you
Dr. Bbosa Science
Moment of a force
This is the product of a force and perpendicular distance from the pivot to the line of action of the
force. The unit of moments is Nm.

The moment of force about point O is F x d

Matrix approach of finding sum of moments about the origin


If forces (𝑎1 i + 𝑏1 j)N, (𝑎2 i + 𝑏2 j)N, ……….(𝑎𝑛 i + 𝑏𝑛 j) act on the body at point (𝑥1 + 𝑦1 ), (𝑥2 + 𝑦2 ), …..
((𝑥𝑛 + 𝑦𝑛 ). The sum of the moments about the origin is
𝑥1 𝑎1 𝑥2 𝑎2 𝑥𝑛 𝑎𝑛
𝐺 = |𝑦 𝑏1 | + |𝑦2 𝑏2 | + ⋯ + |𝑦𝑛 𝑏𝑛 |
1

𝐺 = (𝑏1 𝑥1 − 𝑎1 𝑦1 ) + (𝑏2 𝑥2 − 𝑎2 𝑦2 ) + ⋯ + (𝑏𝑛 𝑥𝑛 − 𝑎𝑛 𝑦𝑛 )


Note

If G is positive, the sum of moments will be anticlockwise and if G is negative the sum of moments
will be clockwise.

Example 1

Find the moment about the origin of a force of 4jN acting at a point which has position vector -5iN

Solution
−5 0
𝐺=| |= -5x4 – 0x0 = -20Nm clockwise
0 4
Example 2

Find the moment about the origin of a force of 4jN acting at a point which has position vector 5iN
5 0
𝐺=| |= 5x4 – 0x0 = 20Nm anticlockwise
0 4
Example 3

Forces of (2i-3j)N, (4i + j)N and (5i -3j)N act on a body at points with Cartesian co-ordinates (1,1),
(2, 4), and (-1, 3) respectively. Find the sum of moments of the forces about the origin.
Solution
1 2 2 4 −1 5
𝐺=| |+| |+| |= (1 x -3 – 2 x 1)+(2 x 1 – 4x 4)+(-1 x -3 – 3 x 5) = -31Nm
1 −3 4 1 3 −3
= 31Nm clockwise

Example 4

Forces (2i -3j)N, (5i + j)N and (-4i+4j) act on a body at points with position vector (i + j), (-2i +2j) and
(3i -4j) respectively. Find the sum of moments of forces about the

(i) origin
1 2 −2 5 3 −4
𝐺=| |+| |+| |= (1 x -3 -2 x 1) + (-2 x 1 – 2 x 5) + (3 x 4 - –4x -4)
1 −3 2 1 −4 4
= -21Nm = 21Nm clockwise
(ii) point with position vector (i –j)

G = (5 x 3) + (1 x 3) +(2 x 0) + (2 x 2) + (4 x 3) – (4 x 2) = 26Nm clockwise

Revision exercise
1. Find the moment about the origin of a force of 3i acting at a point which has position vector
(2i + 3j)m. [9Nm clockwise]
2. Find the moment about the origin of force (4i + 2j)N acting at a point which has position
vector (3i + 2j)m. [2Nm clockwise]
3. A force of (3i -2j)N act at a point which has position vector (5i + j)m. Find the moment about
the point which has a position vector (i +2j)m. [5Nm clockwise]
4. A force of (2i + j)N act at a point which has position vector (2i + 2j)m and a force of 5iN at a
point which has position vector (-2i + j)m. Find the sum of moments of these forces about
the origin. [7Nm clockwise]
5. A force of (3i + 2j)N act at a point which has position vector (5i + j)m and a force of (I + j)N
act at a point which has position vector (2i + j)m. Find the sum of moments of these forces
about the point which has position vector (i + 3j)m. [17Nm anticlockwise]
Couple of forces
These are equal forces acting in opposite direction

Conditions for forces to form a couple

Forces reduce to a couple if;

- resultant force is zero


- the sum of moments about a point is not zero

Example 1

Forces of (-5i – j)N, -3j and (5i + 4j) act on a body a point with position vectors (i – j)m, (2i + j)m and
(4i – 5j)m respectively. Show that these forces reduce to a couple

Solution
−5 0 5 0
𝑅= ( )+( )+( )= ( )
−1 −1 4 0
1 −5 2 0 4 5
𝐺= | |+| |+| |
−1 −1 1 −3 −5 4
= [(1 x -1) – (-5 x -1)]+[(3 x -3) – (1 x 0)] + [(4 x 4) – (5 x -5)] = 29Nm

Since the resultant force is zero an the sum of moment G is not zero, the forces reduce to a couple.

Example 2

ABCD is a square of side 3m. Forces of magnitude 1N, 2N, 3N, sN and tN act along the line AB, BC,
CD, DA and AC respectively, in each case the direction of the force being given by the order of
letters. Taking AB as horizontal and BC as vertical, find the values of s and t so that the resultant of
the forces is a couple.
1 0 −3 0 𝑡𝑐𝑜𝑠45 0
𝑅 = ( )+( )+( )+( )+( )=( )
0 2 0 −𝑠 𝑡𝑠𝑖𝑛45 0
2
(→) tcos45 = 2; t = =2 2
𝑐𝑜𝑠45

(↑) 2 – s + tsin45 = 0; s = 2 + 2 2 sin45 = 4N

It must also be shown the G≠0

A G = 2 x 3 + 3 x 3 = 15Nm
Example 3

ABCDEF is a regular hexagon of side 4m. Forces of magnitude 5N, 1N, 3N, 4N, 2N and 2N act along
the lines AB, BC, CD, DE, EF and FA respectively. In each case the direction of the force being given
by the order of the letters. Given that AB is horizontal, show that these forces reduce to a couple.

Solution

OM = 42 − 22 = 2 3𝑚

(→) 5 – 4 + (1 – 3 – 2 +2)cos60 = 0

(↑) (1 + 3 – 2 – 2) sin60 = 0

O G = 2 3 (5 + 1 + 3 + 4 + 2)

= 34 3N anticlockwise

Since R = 0 and G≠ 0 then is a couple.

Revision exercise
1. Forces of 6N, 8N, 6N and 8N act along sides of a rectangle ABCD where AB =8m and BC = 6m in
the direction AB, BC, CD and DA respectively.
(a) show that the forces reduce to couple
(b) find the moment of the couple about A. (100Nm)
2. Forces of 5N, 3N, 5N and 3N act along the side of a square ABCD of side 4m in the directions AB,
BC, CD and DA respectively.
(a) show that the forces reduce to couple
(b) find the moment of the couple about A. (100Nm
3. ABCD is a rectangle with AB = 6m and BC = 2m. A force of 3N acts along each of the four sides
AB, BC, CD and DA in the directions indicated by the order of the letters. Show that the forces
form a couple and find its moment. [21Nm]
4. ABCD is a rectangle with AB = 6m and BC = 2m. A forces of 5N, 5N, xN, and xN acts along each
of the four sides CB, AD, AB and CD in the directions indicated by the order of the letters. If the
system is in equilibrium, find the value of x. [15N]
5. ABCD is a square of side 40cm. Forces of magnitude 20N, 15N, 20N and Y act along the line AB,
BC, CD and DA respectively in each case the direction of the force being given by the order of
letters. If the system is equivalent to a couple, find the magnitude of Y and the moment of the
couple. [15N, 14Nm]
6. A force of (3i – 5j)N acts at a point which has position vector (6i + j)m and a force of (-3i + 5j)N
acts at the point which has position vector (4i + j)m. Show that the forces reduce to a couple
and find the moment of the couple [10Nm]
7. A force of (4i + 3j)N acts at a point which has position vector (6i + 3j)m and a force of (-4i -3j)N
acts at the point which has position vector (3i – j)m. Show that the forces reduce to a couple
and find the moment of the couple [7Nm]
8. Force of (i + j)N, (-4i + j)N and (3i – 2j)N act at the points having position vectors (2i + 2j)m, (-
1 + 4j)m and (4i – 2j)m respectively. Show that the forces reduce to a couple and find the
moment of the couple [13Nm]
9. Forces of (ai + bj)N and (6i – 4j)N act at points having position vectors (-2i – 2j)m and (3i – j)m
respectively. If these forces reduce to a couple, find a and b and the moment of the couple.
[ -6, 4 and 26Nm]

Thank You

Dr. Bbosa Science


Line of action of the resultant force
The equation of line of action is given by
𝑥 𝐹𝑥
𝐺 = |𝑦 𝐹𝑦 | = 𝑥𝐹𝑦 − 𝑦𝐹𝑥

𝐺 − 𝑥𝐹𝑦 + 𝑦𝐹𝑥 =0

Note

The line of action cuts the horizontal when y = o and the cuts the vertical axis when x = 0

Example1

Forces of 3N, 2N, 4N and 1N act along the sides of a square ABCD of side 4m in the direction AB, BC,
CD and DA respectively, the direction of force in each case being the order of the letters. Find the;

(i) Magnitude and direction of the resultant force


(ii) equation of the line of action
(iii) point where the line of action of the resultant of forces cuts AB

Solution

1
Direction, θ =tan−1 ( ) = 450 to AB
1

A: G = 4 x 4 + 2 x 4 = 24Nm anticlockwise

Equation is 24 – x + y = 0
3 0 −4 0 −1 (ii) Line cuts AB when y = 0; 24 – x + 0 = 0
𝑅 =( )+( )+( )=( )=( )
0 2 0 −1 1
x = 24m from A
|𝑅| = √(−1)2 + 12 = √2𝑁

digitalteachers.co.ug
Example 2

Five forces of magnitude 3N, 4N, 4N, 3N and 5N act along AB, BC, CD, DA and AC respectively of
squares of side 1m. The directions of the forces being given in the order of the letters Taking AB and
AD as horizontal and vertical respectively. Find the

(a) magnitude and direction of the resultant force


(b) equation of the line of action

Solution

4.5355
Direction, θ =tan−1 ( ) = 60.80 to AB
2.5355

A: G = 4 x 1 + 4 x 1 = 8Nm anticlockwise

Equation is 8 – 4.5355x + 2.5355y = 0


3 4 −4 0 5𝑐𝑜𝑠45
𝑅 =( )+( )+( )=( )+( )
0 0 0 −3 5𝑠𝑖𝑛45 (ii) Line cuts AB when y = 0; 8 – 4.5355x + 0 = 0
2.5355 8
=( ) x= = 1.769m from A
4.5355 4.5355

|𝑅| = √(2.5355)2 + (4.5355)2 = 5.1961𝑁

Example 3

ABCD is a rectangle with AB = DC = 5m and AD = BC = 6m. Forces of magnitude 8N, 5N, 3N, 6N and
10N act along AB, BC, CD, AD and DB of the rectangle respectively. The directions of the forces being
given by the order of the letters. Find

(i) the magnitude and the direction of a single force that could replace this system of force
(ii) equation of the line of action
(iii) where the line of action cuts AB

Solution

5
(i) 𝜃 = tan−1 ( ) = 3980
6
8 0 −3 0 𝑐𝑜𝑠39.8
𝑅 =( )+( )+( )+( )+( )
0 5 0 6 −𝑠𝑖𝑛39.8
11.4011
=( )
3.3172
|𝑅| = √11.40112 + 3.31722 =11.8739N
3.3172
Direction, α =tan−1 ( ) = 16.20 to AB
11.4011

digitalteachers.co.ug
(ii) A G= (5 x 0.05) + (3 x 0.06) + ( 10 sin39.8 x 0.06) = 0.046Nm anticlockwise

0.046 - 0.3172x + 11.4011y = 0

(iii) line cuts AB where y = 0

0.046 - 0.3172x =0

x = 0.0139m from A

Example 4

ABCDEF is a regular hexagon 2m. Forces of magnitude 7N, 6N, 8N, 4N, 3N and 2N act along AB, BC,
CD, DE, EF and FA respectively in each case the direction of force being given by the order of letters.
Given that AB is horizontal, find the

(a) the magnitude and direction of resultant force


(b) where its line of action of the resultant cuts AB

Solution

2m 2m

7 6𝑐𝑜𝑠60 −8𝑐𝑜𝑠60 −4 −3𝑐𝑜𝑠60 2𝑐𝑜𝑠20 1.5


𝑅 = ( )+( )+( )+( )+( )+( ) =( )
0 6𝑠𝑖𝑛60 8𝑠𝑖𝑛60 0 −3𝑠𝑖𝑛60 −2𝑠𝑖𝑛60 7.794
|𝑅| = √1.55 + 7.7942 = 7.94𝑁
7.794
Direction, α = tan−1 ( )= 79.110 to AB
1.5

A: G =(6sin600 x 2) + (8sin600 x 4) + (4 x 4sin600) + (3sin600 x 2) = 33√3 = 57.158Nm

𝐺 = 𝑥𝐹𝑦 − 𝑦𝐹𝑥

57.158 = 7.794x - 1.5y

Line cuts AB when y = 0


57.158
x= = 7.334m from A
7.794

Example 5

ABCDEF is a regular hexagon 1m. Forces of magnitude 6N, 7N, 5N, 3N, 2N, and 52N act along AB, BC,
CD, DE, EF and FA respectively in each case the direction of force being given by the order of letters.
Given that AB is horizontal, find the

(a) the magnitude and direction of resultant force


(b) where its line of action of the resultant cuts AB

digitalteachers.co.ug
Solution

1m 1m

6 7𝑐𝑜𝑠60 −5𝑐𝑜𝑠60 −3 −2𝑐𝑜𝑠60 5𝑐𝑜𝑠20 5.5


𝑅 = ( )+( )+( )+( )+( )+( ) =( )
0 7𝑠𝑖𝑛60 5𝑠𝑖𝑛60 0 −2𝑠𝑖𝑛60 −5𝑠𝑖𝑛60 4.33
|𝑅| = √5.55 + 4332 = 7.0𝑁
4.33
Direction, α = tan−1 ( )= 38.210 to AB
5.5

A: G = (7sin600 x 1) + (5sin600 x 2) + (4 x 4sin600) + (3 x 2sin600)+ (2sin60 x 1= 12.5√3

= 21.65Nm anticlockwise

𝐺 = 𝑥𝐹𝑦 − 𝑦𝐹𝑥

21.65 = 4.33x - 5.5y

Line cuts AB when y = 0


21.65
x= = 5m from A
4.33

Example 6

The center of a regular hexagon ABCDEF of side 2a is O. Forces of magnitude 4N, sN, tN, 1N, 7N and
3N act along the sides AB, BC, CD, DE, EF and FA respectively, in each case the direction of the force
being given by the order of the letters.

(i) Given that the resultant of these six forces is of magnitude 2√3𝑁 acting in the direction
perpendicular to BC, determine the value of s and t.
(ii) show that the sum of moments of forces about O is 27a√3𝑁𝑚
(iii) If the midpoint of BC is M, find the equation f the line of action of the resultant, refer to OM as
x-axis and OD as y-axis

Solution

digitalteachers.co.ug
−2√3𝑐𝑜𝑠30 4 𝑠 𝑐𝑜𝑠60 −𝑡𝑐𝑜𝑠60 −1 −7𝑐𝑜𝑠60 3𝑐𝑜𝑠60
( )= ( )+( )+( )+( )+( )+( )
2√3𝑠𝑛30 0 𝑠 𝑠𝑖𝑛60 𝑡𝑠𝑖𝑛60 0 −7𝑠𝑖𝑛60 −3𝑠𝑖𝑛60

𝑠 = 2 𝑎𝑛𝑑 𝑡 = 10

(ii)
OM = √(2𝑎)2 − 𝑎2 =a√3

O: G =(4 + 2 + 10 + 1 + 7 + 3) a√3 = 27 a√3Nm

Since the resultant is in direction OM, => x = 2√3, y =0

G = xY + yX; resultant cuts the x-axis when Y = 0

27 a√3 = -2√3y; y = -13.5a

Example 7

The diagram below shows a trapezium rule ABCD, AD = DC = CB = 1 and AB = 2meters. Forces of
magnitude 1N, 3N, 5N, 6N and 2√3 N respectively.

(a) Calculate the magnitude of the resultant force and the angle it makes with side AB
(09marks)
𝑥 −6 3 2√3 cos 300 cos 600 ) + ( 5cos 600 ) = ( 3 )
(𝑦 ) = ( ) + ( ) + ( ) + (
0 0 2√3 sin 300 sin 600 −5sin 600 −√3
2
𝑅𝑒𝑠𝑢𝑙𝑡𝑎𝑛𝑡 𝑓𝑜𝑟𝑐𝑒, 𝑅 = √(3)2 + (−√3) = 3.464N
√3
𝐷𝑖𝑟𝑒𝑐𝑡𝑖𝑜𝑛, 𝛼 = 𝑡𝑎𝑛−1 ( ) = 300
3

(b) Given that the line of action of the resultant force meets AB at X, find AX. (03marks)

digitalteachers.co.ug
Equation of the line action of the resultant is given by G-xY + yX = 0
Taking moments about A
G= -3 x 1cos 300 – 5 x 2 cos 300
√3 √3 −13√3
= -3 x -10 x =
2 2 2
By substitution
−13√3
+ √3𝑥 + 3𝑦 = 0
2
The line of action of the resultant cuts AB when y = 0
−13√3
+ √3𝑥 + 3 𝑥 0 = 0
2
𝑥 = 6.5𝑚
̅̅̅̅= 6.5m
Hence 𝐴𝑋

Example 8

Six forces, 9N, 5N, 7N, 3N, 1N and 4N act along the sides PQ, QR, RS, ST, TU and UP of a regular
hexagon of side 2m, their direction being indicated by the order of letters. Taking PQ as the
reference axis, express each of the forces in vector form. Hence find the

(i) magnitude and direction of the resultant of forces


(ii) distance from P, where the line of action of the resultant cuts PQ.

Solution
9 −3
PQ =( ); ST = ( )
0 0
5 −1
0 0
QR = (5𝑐𝑜𝑠600 ) = (5√3) TU = (5𝑐𝑜𝑠600 ) = (−√3)
2 2
5𝑠𝑖𝑛60 5𝑠𝑖𝑛60
2 2

7 0 2
0 UP= ( 4𝑐𝑜𝑠60 0 ) = ( )
RS= (−7𝑐𝑜𝑠600 ) = (7√3)
2
−4𝑠𝑖𝑛60 −2√3
7𝑠𝑖𝑛60
2

5 7 −1
9 2 2 −3 2 2 6.5
R = ( ) + (5√3) + (7√3) + ( ) + (−√3) + ( )=( )
0 0 −2√3 3.5√3
2 2 2

2
|𝑅| = √(6.5)2 + (3.5√3) = 8.9N

digitalteachers.co.ug
3.5√3
θ = tan−1 ( )= 430
6.5

(ii) Equation of the line of the resultant is given by: G - xY + yX

P: G = 5sin600 + 7 x 2 √3 +3 x 2 √3 + 1 x 2sin600

= 5√3 + 14√3 + 6√3 +√3= 26√3Nm

By substitution

26√3 − 3.5√3x + 6.5y = 0

The line cuts PQ when y = 0

 26√3 − 3.5√3x = 0; x = 7.43m

∴The lines cuts PQ at a distance 7.43m from P

Revision exercise
1. ABCD is a square of side 5m. Forces of 4N, 6N, 8N and 10N act along BD, DC, CA and CB
respectively. When a force P acts along AD and force Q acts along AB, the whole system is
equivalent a couple. Find the magnitude of P and Q and the moment of the couple. [12.8N,
2.49N, 65,9Nm]
2. ABCD is a square of side am. Forces of 1N, 4N, 3N and 6N act along AB, CB, DC and AD
respectively. Calculate the magnitude and direction of a single force that could replace this
system of forces and find where its line of action cuts AB. [4.47N, 26.60 to AB, 3.5a from A]
3. ABCD is a rectangle with AB = 3m and angle CAB = 300. Forces of 10N, 20N and 20N act along AC,
AD and DB respectively. Calculate the magnitude and direction of a single force that could
replace this system of forces and where its line of action cuts AB. [30N, 300 to AB, 2m from A]
4. ABCD is a rectangle with AB = 5m and BC = 3m. Forces of magnitude 2N, 4N, 3N and 1N act along
the lines AB, BC, DC and DA respectively, in each case the direction of force being given by the
order of the letters. Given AB is horizontal, find the
(i) magnitude and direction of the resultant force [6N]
(ii) equation of the line of action of the resultant
(iii) distance from A where the line of action of the resultant force cuts AB. [1.57m]
5. ABCD is a rectangle with AB = 4m and BC = 3m. Forces of magnitude 3N, 5N, 6N, 4N and 7N act
along the lines AB, BC, CD, DA and AC respectively, in each case the direction of force being given
by the order of the letters. Given AB is horizontal, find the
(i) magnitude and direction of the resultant force. [5.81N]
(ii) distance from A where the line of action of the resultant force cuts AB[7.31m]
6. ABCD is a square of side 2m. Forces of magnitude 10N, 9N, 8N, 7N and 5N act along the lines AB,
BC, CD, DA and AC respectively, in each case the direction of force being given by the order of
the letters. Given AB is horizontal, find the
(i) magnitude and direction of the resultant force [20.305N]

digitalteachers.co.ug
(ii) sum of moments about A [34Nm]
(iii) distance from A where the line of action of the resultant force cuts AB. [1.74m]
7. ABCDEF is a regular hexagon of side 4m. Forces of magnitude 6N, 4N, 7N, 8N, 4N and 2N act
along the lines AB, BC, CD, DE, EF and FA respectively, in each case the direction of force being
given by the order of the letters. Given AB is horizontal, find the
(i) magnitude and direction of the resultant force [7.55N]
(ii) distance from A where the line of action of the resultant force cuts AB. [21.71m]
8. ABCDEF is a regular hexagon of side 34m. Forces of magnitude 5N, 6N, 2N, 3N, 6N and 1N act
along the lines AB, BC, CD, DE, EF and FA respectively, in each case the direction of force being
given by the order of the letters. Given AB is horizontal, find the
(i) magnitude and direction of the resultant force [4.583N]
(ii) distance from A where the line of action of the resultant force cuts AB. [13.2m]
9. ABCDEF is a regular hexagon of side 2m. Forces of magnitude 2N, 3N, 4N and 5N act along the
lines AC, AE, AF and ED respectively, in each case the direction of force being given by the order
of the letters. Given AB is horizontal, find the
(i) magnitude and direction of the resultant force [8.84N, 57.60 to AB]
(ii) distance from A where the line of action of the resultant force cuts AB. [2.32m from A]
10. ABCDEF is a regular hexagon of side 2m. Forces of magnitude 3N, 4N, 2N, 1N, 2N and 6N act
along the lines AB, BC, DC, ED, EF and AF respectively, in each case the direction of force being
given by the order of the letters. Given AB is horizontal, find the
(i) magnitude and direction of the resultant force [6N]
(ii) Equation of the line of action of the resultant force [ y = √3𝑥]
11. ABCDE is a regular pentagon of side 2m. Forces of 5N act along AB, BC and AD. find the
(i) magnitude and direction of the resultant force [12.5N,at 49.60 to AB]
(ii) distance from A where the line of action of the resultant force cuts AB. [1m from A]

Thank You

Dr. Bbosa Science

digitalteachers.co.ug
Parallel forces in equilibrium
Conditions for a body to be in equilibrium

When a system of parallel forces act on a body then it will be in equilibrium when;

(i) the sum of forces acting in one direction are equal to the sum of forces acting in
opposite direction.
(ii) sum of clockwise moments about a point are equal to the sum of anticlockwise moment
about the same point

Example 1

Given the diagram below. Find the value of y

500 x y = (300 x 3) + (50 x 1)


950
y= = 1.9𝑚
500

Example 2

A uniform beam of weight 50N and length 2m rests horizontally on two supports pivoted at each
end. A load of weight 500N is placed 0.5m from one end. Find the reaction on each support.

B: Ra x 2 =50 x 2 + 500 x 1.5

2Ra = 50 + 750

Ra = 400N

Also Ra + Rb = 500N + 50 N

Rb = 550N – 400N = 150N

digitalteachers.co.ug
Example 3

A uniform beam of mass 50kg and length 4m rests horizontally on two supports pivoted at each end.
A load of 20kg is placed 1m from one end. Find the reaction on each support

B: Ra x 4 =50g x 2 + 20g x 3

4Ra = 100g + 60g = 160 x 9.8

Ra = 392N

Also Ra + Rb = 500N + 50 N

Rb = 20gN + 50gN – 392N = 294N

Example 4

A non- uniform beam AB of length 4m has its weight 5N acting at a point 1.8m from end A. The beam
rests horizontally on two supports pivoted at each end. Find the reaction on each support.

B: Ra x 4 =5 x 2.2 =11

Ra = 2.75N

Also Ra + Rb = 5N

Rb = 5N – 2.75N = 2.25N

Example 5

A non- uniform beam AB of length 4m rests in horizontal position on vertical support at A and B. The
centre of gravity is at 1.5m from end A. The reaction at B is 37.5N fine the

(a) mass of the beam (b) reaction at B

digitalteachers.co.ug
A: 37.5 x 4 = mg x 2.5

m = 10.2kg

Also Ra + 37.5 = 10.2 x 9.8

Ra = 62.5N

Revision exercise
1. A uniform beam AB of length 10m rests horizontally on two supports A and B. If the beam
has a mass of 20g, find the reaction on each support. [98N, 9N]
2. A uniform beam of length 14m and mass 20kg rests horizontally on two supports, one at A
and another at C which is 4m from B. find the reactions at each support [58.8N, 137.2N]
3. A uniform beam AB of length 10m and mass 20kg rests horizontally on two supports, one at
A and another at C which is 2m from B. If a weight of mass 20kg is attached to the beam at a
point 6m from A. Find the reaction on the supports. [392N, 196.2N]
4. A uniform beam AB of length 4m and mass 10kg rests horizontally on two supports at A and
the other at C which is 1m from B. Where must a body of mass 50kg stand on the beam so
that the reaction on each support is equal? [1.4m]
5. A uniform beam AB of length 12m and mass 12kg rests on two supports A and B. At what
distance must a particle of mass 4g be tied so that the reaction of each support is equal.
[9m from A]
6. A playground sea saw consists of a uniform beam of length 4m supported at its mid-point. If
a girl of mass 25kg sits at one end of the sea saw, find where her brother of mass 40kg must
sit if the sea saw is to balance horizontally. [75cm from other end]
7. A broom consists of a uniform broom stick of length 120cm and mass 4kg and a broom head
of mass 6kg attached at the other end. Find where a support should be placed so that the
broom balances horizontally. [24cm from the head]
8. A non-uniform beam AB of length 4m rests horizontally on two supports, one at A and the
other at B. The reaction at the supports are 5gN and 3gN respectively. If instead the rod the
rod were to rest horizontally on one support, find how far from A this support would have to
be placed. [1.5m from A]
9. A uniform beam AB of mass 80g and of length 100cm is pivoted at 30cm from A, a force of
10N is placed on the beam at the 80cm from end A and a string is tied at the 40cm from end
B so that the beam rests horizontally. Find the tension in the string. [17.2N]
10. A uniform beam AB of length 100cm is pivoted at 60cm from end B. The beam rests
horizontally when a mass at A is 35g. Calculate the mass 9m) of the beam. [0.14kg]
11. A uniform meter rule pivoted at 10cm mark balances when a mass of 400N is suspended at
the 0cm mark. If the system is in equilibrium. Find the mass of the ruler [10kg]
12. Two boys are carrying a uniform ladder of weight 800N, if the boys hold the ladder at 2m
and 3m respectively from the centre of gravity, calculate the weight that each boy support.
[480N, 320N]

Thank You

digitalteachers.co.ug
Dr. Bbosa Science

digitalteachers.co.ug
Linear interpolation and extrapolation
Linear interpolation
Deals with computations of values that lie within a given range

Example 1

The table below shows values of a function f(x)

x 1.8 2.0 2.2 2.4


f(x) 0.532 0.484 0.436 0.384
Find values of (i) f(1.88) (ii) x corresponding to f(x) = 0.4

Solution

1.8 1.88 2.0 (ii) 2.2 x0 2.4


0.532 y 0.484 0.436 0.4 0.384
𝑥0 −2.2 2.4−2.2
𝑦−0.532
=
0.884−0.532 =
0.4−0.436 0.384−0.436
1.88−1.8 2.0−1.8

y = 0.513 x0 =2.34

Example 2

Given the table below

x 9 10 11 12
f(x) 2.66 2.42 2.18 1.92
Using linear interpolation find

(i) f(x) when x = 10.15 (ii) f-1 (2.02)

Solution
11 x0 12
10 10.15 11 2.18 2.02 1.92
𝑥0 −11 12−11
2.42 y 2.18 =
𝑦−2.42 2.18−2.42 2.02−2.18 1.92−2.18
=
10.15−10 11−10
x0 = 11.62
y =2.384

Example 3

Given table below


x0 40.00 40.40 40.80 50.40
sin x0 0.6428 0.6481 0.6534 0.7705
Find (i) sin 40.50 (ii) sin-1 0.6445

Solution
40.00 x0 40.40
40.40 40.50 40.80
0.6428 0.6445 0.6481
0.6481 y 0.6534 𝑥0 −40.0 40.4−40.0
𝑦−0.6481 0.6534−0.6481 =
= 0.6445−0.6428 0.6481−0.6428
40.5−40.4 40.8−40.4
x0 = 40.13
y = 0.6494

Linear extrapolation

This deals with computation of values that lie outside given values

Example 4

Given the table below

x 2.2 2.6 3.1


x3 10.648 17.576 29.791
Find 3.43

2.6 3.1 3.4


17.576 29.791 y
𝑦−29.791 29.791−17.576
=
3.4−3.1 3.1−2.6
y = 37.12

Example 5

The table below is an extract from table of sec x

x = 600 0’ 12’ 24’ 36’ 48’


𝑠𝑒𝑐 𝑥 2.0000 2.0122 2.0245 2.0371 2.0498
Use linear interpolation to determine
(i) sec 60015’
(ii) angle whose secant is 2.0436 [60042’]

Solution

(i) (ii)

12’ 15’ 24’ 36’ x 48’


2.0122 y 2.0245 2.0371 2.0436 2.0498
𝑦−2.0122 2.0245−2.0122 𝑥−36 48−36
= =
15−12 24−12 2.0436−2.0371 2.0498−2.0371

y = 2.03065 x = 42’ hence angle = 60042’


Example 6
The table below shows the values of a function f(x)
x 1.8 2.0 2.2 2.4
f(x) 0.532 0.484 0.436 0.384
Use linear interpolation to find the value of
(i) F(2.08)
1.8 2.08 2.0
0.532 f(x) 0.484
𝑓(𝑥)−0.436 0.436−0.484
=
2.08−2.0 2.2−2.0
𝑓(𝑥)−0.436 −0048
=
0.08 0.2
f(x) = 0.4648 or 0.465 (3D)

(ii) x corresponding to f(x) = 0.5 (05marks)


1.8 x 2.0
0.532 0.5 0.484
0.5−0.532 0.484−0.532
=
𝑥−1.8 2.0−18
−0.032 −0.048
=
𝑥−1.8 0.2
𝑥 = 1.9333 or 1.9 (1D)
Example 7
Given the table below,
x 0 10 20 30
y 6.6 2.9 -0.1 -2.9
Use linear interpolation to find
(a) y when x = 16
Extract
x 10 16 20
y 2.9 y0 -0.1
𝑦0 −2.9 −0.1−2.9
=
16−10 20−10
𝑦0 −2.9 −3.0

6 10
y0 = 1.1
hence when x = 16, y =1.1
(b) x when y = -1
Extract
x 20 x0 30
y -0.1 -1 -2.9
𝑥0 −20 30−20
=
−1−(−0.1) −2.9−(−0.1)
𝑥0 = 23.2
Hence when y = -1; x = 23.2
Example 8

The table below shows the values of a function f(x) for given values of x.

x 9 10 11 12
f(x) 2.66 2.42 2.18 1.92
Use linear interpolation or extrapolation to find
(a) f(10.4)
Extract
10 10.4 11
2.42 f(x) 2.1
Using gradient approach
2.18−𝑓(𝑥) 2.18−2.42
=
11−10.4 11−10
2.18−𝑓(𝑥) −0.24
=
0.6 1
f(x) = 2.18 + 0.24 x 0.6 = 2.324

(b) the value of x, corresponding to f(x) = 1.46 (05marks)


Extract
x 12 11
1.46 1.92 2.18
Using gradient approach
2.18−1.46 2.18−1.92
=
11−𝑥 11−12
0.72 0.26
=
11−𝑥 −1
−1 𝑥 0.72
X = 11 - = 13.769
0.26

Revision exercise
1. Table below is an extract from the table of cos x
x 00 100 200 300 400 500
Cos x 0.1736 0.1708 0.1679 0.1650 0.1622 0.1593
0 -1 0
Use linear interpolation to determine: (i) cos 80 36’ [0.1633] cos (0.1685) [80 18’]
2. The table below shows variation of temperature with time in a certain experiment.
Time (s) 0 120 240 360 480 600
Temperature (0C) 100 80 76 65 50 48
Use linear interpolation to determine
(i) value of 0C corresponding to 400s [620C]
(ii) time at which the temperature is 770C [192s]
3. The table below shows the value of a function In(x) for given values of x
x 1.4 1.5 1.6 1.7
In(x) 0.3365 0.4055 0.4700 0.5306
Using linear interpolation or extrapolation, find
(i) In(1.66) [0.5064] (ii) find value of x corresponding to In(x) = 0.400 [1.492]
4. The table below shows variation of temperature with time in certain experiment.
Time (s) 0 10 15 20 30
0
Temperature ( C) 80 70.2 65.8 61.9 54.2
Use linear interpolation to determine
(i) value of θ0 corresponding to T= 18s [63.50C]
(ii) Time T at which the temperature θ0 = 600C [22.5s]
5. Given the table below
x -1.0 -0.5 -1.4
y -1.0 -2.2 -3.7
Using linear interpolation or linear extrapolation to find
(i) y when x = 0.5 [-4.6] (ii) x when y = -4.5 [ 0.458]
6. In an examination, scaling is done such that candidate A who originally scored 35% gets 50%
and candidate B with 40% gets 65%, determine the original mark for candidate C whose new
mark is 80% [45%]
7. The table below is an extract of log10 𝑥
x 80.00 80.20 80.50 80.80
log10 𝑥 1.9031 1.9042 1.9058 1.9074
Using linear interpolation find
(i) log10 80.759[1.9072]
(ii) the number whose logarithm is 1.90388 [80.14]
8. The table below shows the values of a function f(x) for given values of x
x 2 3 4 5
𝑓(𝑥) 3.88 5.11 8.14 11.94
Use linear interpolation to determine
(i) f(2.15) [4.06]
(ii) the value of x corresponding to f(10.6) [4.68]
9. The table below shows distance in km a truck moves with a given amount of fuel in litres (l)
Distance (km) 20 28 33 42
𝐹𝑢𝑒𝑙 (𝑙) 10 13 21 24
Use linear interpolation or extrapolation to find
(i) How far the truck can move on 27.5l of fuel [52.5km]
(ii) the amount of fuel required to cover29.8km [15.88l]
10. The table below shows the values of a continuous f(t) with respect to t
t 0 0.3 0.6 1.2 1.6
𝑓(𝑡) 2.72 3.00 3.32 4.06 4.95
Use linear interpolation or extrapolation, find
(i) f(t) when t = 0.9 [3.69]
(ii) the value of t corresponding to f(t) = 4.48 [1.48]
11. The table below shows the delivery charges by courier company
Mass (g) 200 400 600
𝑐ℎ𝑎𝑟𝑔𝑒𝑠 (𝑠ℎ𝑠. ) 700 1200 300
Use linear interpolation or extrapolation, find
(i) the delivery charge of a parcel weighing 352g [1080]
(ii) mass of a parcel whose delivery charge is shs. 3,300 [ 633.33kg]
12. The table below shows the cost y shillings for hiring a motor cycle for a distance x kilometres.

Distance x (km) 10 20 30 40
Cost (shs.) 2800 3600 4400 5200
Use linear interpolation or extrapolation, find
(i) the cost of hiring the motor cycle for distance of 45km [shs. 5600]
(ii) distance travelled if he paid shs. 4000 [ 25km]
13. The table below shows the values of a function f(x) for given values of x
x 0.4 0.6 0.8
f(x) -0.9613 -0.5108 -0.2231
Use linear interpolation to determine f-1(-0.4308} correct to 2 decimal places [0.66]
14. The table below shows how T caries with S
T -2.9 -0.1 2.9 3.1
S 30 20 12 9
Use linear interpolation/extrapolation to estimate values of
(a) T when S = 26 [-1.78] (ii) S when T = 3.4 [4.5]
15. The table below shows the commuter bus fare from stages A to B, C, D and E
Stage A B C D E
Distance (km) 0 12 16 19 23
Fare (shs) 0 1300 1700 2200 2500
(a) Jane boarded from A and stopped at a place 2km after E. How much did she pay?
(03marks) [shs. 2650]
(b) Okello paid shs 2000. How far from A did the bus leave him? (02marks) [17.km]
16. The table below shows the value of x and corresponding values of a function f(x)
The table below shows how T caries with S
x 0.3 0.6 0.9 1.2
f(x) 3.00 3.22 3.69 4.06
Use linear interpolation/extrapolation to estimate values of
(i) f(x) when x = 0.4 [3.0733] (ii) x when f(x) = 3.82 [ 1.0054]

Thank you
Dr. Bbosa Science
Probability theory
Probability is the measure of chance of an event occurring or not occurring

Sample space

A sample space(s) is the set of all possible outcomes of an experiment

Example 1

When a dice is rolled once s = (1, 2, 3, 4, 5, 6)

When a coin is tossed once s = (H, T)

Events

An event (E) is a subset of a sample space.

Example2

When a coin is tossed twice, s = (HH, TT, HT, TH)

When interested in getting one head, E = (TH, HT)

Probability of an event

Given an event, E over a sample space, S


𝑛(𝐸)
P(E) =
𝑛(𝑆)

Example 3

Find the probability of getting two heed when an ordinary coin is tossed thrice

S = (HHH, HHT, HTH, HTT, THH, THT, TTH, TTT)

E= (HTH, HHT, THH)


𝑛(𝐸) 3
P(E) = =
𝑛(𝑆) 8

Example 4

Find the probability of getting a number greater than 3 when ordinary die is tossed once

S = (1, 2, 3, 4, 5, 6)

E = (4, 5, 6)
1
P(E) =
3

Intersection of events
For any two events A and B, the probability that A and B occur together is (A∩B)

Union of event
For any two events A and B, the probability that event A or B or both occur is P(A∪ 𝐵)

P(A∪ 𝐵) = 𝑃(𝐴) + 𝑃(𝐵) − 𝑃(𝐴 ∩ 𝐵)

P(A or B) = P(A) + P(B) – P(A and B)

Example 5
2 4
The probability that a student passes economics is , the probability that he passes mathematics is .
3 9
4
If the probability that he passes at least one of them is . Find the probability that he passes both
5
subjects.

P(E∪ 𝑀) = P(E) + P(M) – P(E∩ 𝑀)


2 4 4 14
P(E∩ 𝑀) = + − =
3 9 5 45

Complement of events
A’ denotes event A does not occur

For events A and B

(i) P(A) + P(A’) = 1


(ii) P(B) + P(B’) = 1
(iii) P(A∪ 𝐵) + 𝑃(𝐴 ∪ 𝐵)′ = 1
(iv) P(A∩ 𝐵) + 𝑃(𝐴 ∩ 𝐵)′ = 1

Venn diagram

(i) P(A) = P(A∩ 𝐵′) + 𝑃(𝐴 ∩ 𝐵)


(ii) P(B) = P(B∩ 𝐴′) + 𝑃(𝐴 ∩ 𝐵)

Contingency table
B B’ (i) P(A) = P(A∩ 𝐵) + P(A∩ 𝐵′)
A P(A∩ 𝐵) P(A∩ 𝐵′) P(A) (ii) 𝑃(𝐴’) = 𝑃(𝐴′ ∩ 𝐵) + 𝑃(𝐴’ ∩ 𝐵′)
A’ P(A’ ∩ 𝐵) P(A’∩ 𝐵′) P(A’) (iii) P(B) = P(A∩ 𝐵) + P(A′ ∩ 𝐵)
P(B) P(B’) 1 (iv) P(B’) = P(A∩ 𝐵′) + P(A′ ∩ 𝐵′)
Demorgan’s rule
(i) P(A’ ∩ 𝐵′) = P(A∪ 𝐵)′ = 1 − P(A ∪ 𝐵)
P(neither A nor B = 1 – P(A or B)
(ii) P(A’∪ 𝐵′) = 𝑃(𝐴 ∩ 𝐵)′ = 1 − 𝑃(𝐴 ∩ 𝐵)

Types of events
- Undefined events
- Mutual events
- Independent events
- Exhaustive events

Unidentified events

For unidentified events, there is no restriction on 𝑃(𝐴 ∩ 𝐵)

Example 6
19 2 4
Event A and B are such that P(A) = , P(B) = and P(A∪ 𝐵) = find
30 5 5

(i) P(A∩ 𝐵)
Using, P(A∪ 𝐵) = 𝑃(𝐴) + 𝑃(𝐵) − 𝑃(𝐴 ∩ 𝐵)
19 2 4 19+12−24 7
P(A∩ 𝐵) = + − = =
30 5 5 30 30
(ii) P(A’ ∩ 𝐵′)
Using, P(A’ ∩ 𝐵′) = P(A∪ 𝐵)′ = 1 − P(A ∪ 𝐵)
4 1
P(A’ ∩ 𝐵′)= 1 - =
5 5
(iii) P(B∩ 𝐴′)
Using, P(B) = P(B∩ 𝐴′) + 𝑃(𝐴 ∩ 𝐵)
2 7 5 1
P(B∩ 𝐴′) = − = =
5 30 30 6
(iv) P(A’∪ 𝐵)
P(A’∪ 𝐵) = 𝑃(𝐴’) + P(B) – P(A’∩ 𝐵)
19 2 1 3
= (1 − )+ − =
30 5 6 5
(v) P(A∩ 𝐵′)
P(A∩ 𝐵′) = P(A) – P(A∩ 𝐵)
19 7 12 2
= − = =
30 30 30 5

Example 7

Event X and Y are such that P(C) = 0.3 P(Y) = 0.4 and P(X∩ 𝑌) = 0.1, find

(i) P(Y’)
P(Y’) = 1- P(Y) = 1 – 0.4 = 0.6
(ii) P(X∩ 𝑌′)
P(X∩ 𝑌′) = P(X) – P(X∩ 𝑌) = 0.3 – 0.1 = 0.2
(iii) P(X’∩ 𝑌′)
P(X’∩ 𝑌′) = P(X’) – P(X’∩ 𝑌) = (1-0.3)- 0.3 = 0.4
(iv) P(X’∪ 𝑌′)
P(X’∪ 𝑌′)= P(X∩ 𝑌)′ = 1 − 0.1 = 0.9
Example 8

Events A and B are such that P(A) = 0.4, P(A∩ 𝐵) = 0.1 𝑎𝑛𝑑 𝑃(𝐴 ∪ 𝐵) = 0.9, find

(i) P(B)
P(A∪ 𝐵) = P(A) + P(B) – P(A∩ 𝐵)
0.9 = 0.4 + (P(B) – 0.1
P(B) = 0.6
(ii) P(A’∩ 𝐵)
P(A’∩ 𝐵) = 𝑃(𝐵) − 𝑃(𝐴 ∩ 𝐵)
= 0.6 – 0.1
= 0.5
(iii) P(A∩ 𝐵′)
P(A∩ 𝐵′) = 𝑃(𝐴) − 𝑃(𝐴 ∩ 𝐵)
= 0.4 – 0.1
= 0.3
(iv) P(A∪ 𝐵′)
P(A∪ 𝐵′) = P(A) + P(B’) – P(A∩ 𝐵′)
= 0.4 +(1-0.6) – 0.3
= 0.5

Example 9

Events A and B are such that P(A) = 0.7, P(A∩ 𝐵) = 0.45 and P(A’∩ 𝐵′) = 0.18, find

(i) P(B’)
P(A’) =P(A’∩ 𝐵) + 𝑃(𝐴′ ∩ 𝐵′)

1 - 0.7 = P(A’∩ 𝐵) + 0.18

P(A’∩ 𝐵) = 0.3 – 0.18 = 0.12


P(B) = P(A’∩ 𝐵) + P(A∩ 𝐵)

1-P(B’) =0.12 + 0.45

P(B’) = 0.43

Alternatively

P(A’∩B’) = P(AUB)’ =1- P(AUB)

1 – P(AUB) = 0.18

P(AUB)= 0.82

P(AUB) = P(A) + P(B) – P(A∩B)

0.82 = 0.7 + P(B) – 0.43

P(B) = 0.57

P(B’) = 1- 0.57 = 0.43

(ii) P(A or B, but not both A and B)

= P(A’∩B) + P(A∩B’) = 0.12 + P(A) – P(A∩B) = 0.12 + 0.7 – 0.45 = 0.37


Example 10

The probability that Anne reads the NewVision is 0.75 and the probability that she reads
NewVision and bot Daily-monitor is 0.65. The probability that she reads neither of the
papers is 0.15. Find the probability that she reads Daily-monitor.

P(N) = 0.75, P(N∩D’) = 0.65, P(N’∩D’) = 0.15

P(D’) = P(N∩D’) + P(N’∩D’)

1-P(D’) =0.65 + 0.15

P(D’) = 0.2

Example 11
4
Event A and B are such that P(A’∩B) = 3x, P(A∩B’) = x and P(B) = . Use a Venn diagram
7
to find the value of

(i) x
(ii) P(A∩B)
Solution

(i) P(AUB) = P(A∩B’) + P(B)


4
1 – P(AUB)’ = 2x +
7
4
1 – x =2x +
7
1
x=
7
(ii) P(A∩B) = P(B) – P(A’∩B)
4 1 1
= − 3( ) =
7 7 7

Exercise A
1. Event C and D are such that P(C) = 0.5, P(D) = 0.7 and PCUD) = 0.8, find
(i) P(C∩D)
(ii) (ii) P(C∩D’)
2. Events A and B are such that P(A) = 0.36, P(B) = 0.25 and P(A’∩B) =0.24. Find
(i) P(A’) (ii) P(A∩B) (iii) P(AUB) (iv) P(A∩B’) (v) P(A’UB’)
3. Events C and D are such that P(C) = 0.7, P(C∩D)= 0.3 and P(CUD) = 0.9. Find
(i) P(D), (ii) P(C∩D’) (iii) P(C’∩D) (iv) P(C’∩D’)
2 1 1
4. Events A and B are such that P(A’) = , P(B) = and P(A∩B) = . Find
3 2 12
(i) P(AUB) (ii) P(A∩B’)
2 5 3
5. Events A and B are such that P(A) = , P(A∩B) = and P(AUB) = . Find
3 12 4
(i) P(B) (ii) P(A’∩B)
6. Events A and B are such that P(A) =P(B), P(A∩B) = 0.1 and P(AUB) = 0.7. Find
(i) P(A) (ii) P(A’UB)
7 3 2
7. Event A and B are such that P(A) = , P(B) = and PA∩B) = . Find
12 4 5
(i) P(AUB) (ii) P(A∩B’)

Solutions to exercise A
1. Event C and D are such that P(C) = 0.5, P(D) = 0.7 and PCUD) = 0.8, find
(i) P(C∩D)
P(C∪ 𝐷) = 𝑃(𝐶) + 𝑃(𝐷) − 𝑃(𝐶 ∩ 𝐷)
0.8 = 0.5 + 0.7 −𝑃(𝐶 ∩ 𝐷)
𝑃(𝐶 ∩ 𝐷) = 0.4
(ii) (ii) P(C∩D’)
P(C) = P(C∩ 𝐷) + P(C∩ 𝐷′)
P(C∩ 𝐷′) = 0.5 – 0.4 = 0.1

2. Events A and B are such that P(A) = 0.36, P(B) = 0.25 and P(A’∩B) =0.24. Find
(i) P(A’) = 1 – P(A) = 1 – 0.36 = 0.64
(ii) P(A∩B)
P(B) = P(A∩ 𝐵) + P(A′ ∩ 𝐵)
P(A∩ 𝐵) = 0.25 -0.24 – 0.01
(iii) P(AUB)
P(AUB) = P(A)+ P(B) – P(A∩B)
= 0.36 + 0.25 – 0.01 = 0.6
(iv) P(A∩B’)
P(A) = P(A∩B) + P(A∩B’)
P(A∩B’) = 0.36 -0.01 = 0.35
(v) P(A’UB’)
P(A’∪ 𝐵′) = 𝑃(𝐴 ∩ 𝐵)′ = 1 − 𝑃(𝐴 ∩ 𝐵)
= 1 – 0.01 = 0.99
3. Events C and D are such that P(C) = 0.7, P(C∩D)= 0.3 and P(CUD) = 0.9. Find
(i) P(D)
P(CUD)= P(C) + P(D)- P(C∩D)

0.9 = 0.7 + P(D) – 0.3

P(D) = 0.5

(ii) P(C∩D’)
P(C) = P(C∩D) + P(C∩D’)
P(C∩D’)= 0.7 – 0.3 = 0.4
(iii) P(C’∩D)
P(D) = P(C∩D) + P(C′ ∩D)
P(C′ ∩D) = 0.5 – 0.3 = 0.2
(iv) P(C’∩D’)
P(C’∩D’)= 1 - P(CUD) = 1 – 0.9 = 0.1

2 1 1
4. Events A and B are such that P(A’) = , P(B) = and P(A∩B) = . Find
3 2 12
(i) P(AUB)
2 1
P(A) = 1- P(A’) = 1 - =
3 3
1 1 1 9 3
P(AUB) = P(A) + P(B) - P(A∩B) = + − = = = 0.75
3 2 12 12 4
(ii) P(A∩B’)
P(A) = P(A∩B) + P(A∩B’)
1 1 3 1
P(A∩B’) = − = = = 0.25
3 12 12 4

2 5 3
5. Events A and B are such that P(A) = , P(A∩B) = and P(AUB) = . Find
3 12 4
(i) P(B)
P(AUB) = P(A) + P(B) - P(A∩B)
3 5 2 6 1
P(B) = + − = = = 0.5
4 12 3 12 2
(ii) (ii) P(A’∩B)
P(B) = P(A∩B) + P(A′ ∩B)
1 5 1
P(A′ ∩B) = − =
2 12 12
6. Events A and B are such that P(A) =P(B), P(A∩B) = 0.1 and P(AUB) = 0.7. Find
(i) P(A)
P(AUB) = P(A) + P(B) - P(A∩B)
2P(A) = 0.7 + 0.1 =0.8
P(A) = 0.4
(ii) P(A’UB)
P(A) = P(A∩B) + P(A∩B’)
P(A∩B’) = 0.4 -0.1 = 0.3
P(A∪ 𝐵′) = P(B) + P(A’) – P(A∩ 𝐵′)
= 0.4 +(1-0.4) – 0.3
= 0.57
7 3 2
7. Event A and B are such that P(A) = , P(B) = and PA∩B) = . Find
12 4 5
(i) P(AUB)
P(AUB) = P(A) + P(B) - PA∩B)
7 3 2 56 14
= + - = =
12 4 5 60 15

(ii) (ii)P(A∩B’)
P(A) = PA∩B)+ P(A∩B’)
7 2 11
P(A∩B’) = − = ]
12 5 60

Mutually exclusive events


Two events are mutually exclusive if they do not occur together i.e. P(A∩B)=0
P(AUB) = P(A) + P(B)

Example12
1 2
Events A and B are mutually exclusive such that P(B) = , P(A) = , find
2 5

(i) P(AUB)
1 2 9
P(AUB) = P(A) + P(B) = + = = 0.9
2 5 10
(ii) P(A’∩B)
P(A’∩B) = P(B) - P(A∩B) = 0.5 – 0 = 0.5
(iii) P(A’∩B’)
P(A’∩B’) = P(AUB)’ = 1 – P(AUB) = 1- 0.9 = 0.1

Example 13
7 3
Event A and B are mutually exclusive such that P(AUB) = , P(A) = ; find
10 5

(i) P(B)
P(B) = P(AUB) + P(A∩B) – P(A)= 0.7 + 0 – 0.6 = 0.1
(ii) P(A’∩B)
P(B) = P(A∩B) + P(A′ ∩B)
P(A′ ∩B) = 0.1 – 0 = 0.1
(iii) P(A∩B’)
P(a) = P(A∩B) + P(A∩B’)
P(A∩B’) = 0.6 – 0 = 0.6
(iv) P(A’∩B’)
P(A’∩B’) = P(AUB)’ = 1-P(AUB) = 1- 0.7 = 0.3
(v) P(A’UB’)
P(A’UB’)= P(A∩B)’ = 1 - P(A∩B) = 1- 0 = 1
(vi) P(AUB’)
P(AUB’) = P(A) + P(B’) –P(A∩B’)
= 0.6 + 1- 0.1 -0.6 = 0.9
(vii) P(A’UB)
P(A’UB) = P(B) + P(A’) –P(A′ ∩B)
= 0.1 + 1- 0.6 -0.1 = 0.4

Example 14

In an athletics competition in which there no dead heats, the probability that Kiplimo wins is 0.5, the
probability that Bekele wins is 0.2, the probability that Cheptegei wins is 0.1.

Find the probability that

(i) Bakele or Kiplmo wins


P(BUK) = P(B) + P(K) = 0.2 + 0.5 = 0.7
(ii) Neither Kiplimo nor Cheptegei wins
P(K’∩B’) = P(KUB)’ = 1-P(KUB) = 1- (0.5 + 0.1) = 0.4

Revision exercise B
1 2
1. Events A and B are mutually exclusive such that P(A) = and P(B) = ; find
2 5

(i) P(AUB) (ii) P(A∩B’) (ii) P(A’∩B’)


3 3
2. Events A and B are mutually exclusive such that P(B) = , P(A) = , find
10 5

(i) P(AUB) (ii) P(A’) (iii) P(A’∩B)

3. Events A and B are mutually exclusive such that P(B) = 0.4, P(A) = 0.5, find

(i) P(A’UB) (ii) P(B’) (ii) P(A’∩B’)


8 2
4. Events A and B are mutually exclusive such that P(AUB) = , P(A) = , find
10 5

(i) P(B) (ii) P(A’∩B) (iii) P(A∩B’) (iv) P(A’∩B’) (v) P(AUB’) (vi) P(A’UB)

5. Events A and B are mutually exclusive such that P(A’∩B) = 0.3, P(A’UB) = 0.45, find

(i) P(B) (ii) P(A) (iii) P(A’∩B) (iv) P(A’∩B’) (v) P(AUB’) (vi) P(A’UB)

6. Event A and B are mutually exclusive such P(AUB) = 0.9, P(AUB’) = 0.6, find

(i) P(B) (ii) P(A) (iii) P(A’UB) (iv) P(A’∩B’) (v) P(A’UB’)

Solutions to exercise B
1 2
1. Events A and B are mutually exclusive such that P(A) = and P(B) = ; find
2 5
1 2 9
(i) P(AUB) = P(A) + P(B) = + = = 0.9
2 5 10

(ii) P(A∩B’) = P(A) – P(A∩B) = 0.4 – 0 = 0.4

(iii) P(A’∩B’) = P(AUB)’ = 1-P(AUB) = 1- 0.9 = 0.1


3 3
2. Events A and B are mutually exclusive such that P(B) = , P(A) = , find
10 5
3 3 9
(i) P(AUB) = P(A) + P(B) = + = = 0.9
10 5 10

(ii) P(A’) = 1- P(A) = 1 – 0.6 = 0.4

(iii) P(A’∩B) = P(B) – P(A∩B) = 0.3- 0 = 0.3

3. Events A and B are mutually exclusive such that P(B) = 0.4, P(A) = 0.5, find

(i) P(A’UB)

P(A’UB) = P(B) + P(A’) –P(A′ ∩B)


= 0.4 + 1- 0.5 -0.4 = 0.5

(ii) P(B’) = 1- P(B) = 1 – 0.4 =0.6

(iii) P(A’∩B’) = P(AUB)’ = 1 – P(AUB) = 1- (P(A) + P(B) = 1-(0.4 + 0.5) =0.1


8 2
4. Events A and B are mutually exclusive such that P(AUB) = , P(A) = , find
10 5

(i) P(B) =P(AUB) – P(A) = 0.8- 0.4 = 0.4

(ii) P(A’∩B) = P(B) – P(A∩B) =0.4 – 0 =0.4

(iii) P(A∩B’) = P(A) - P(A∩B) = 0.4 – 0 = 0.4

(iv) P(A’∩B’) = P(AUB)’ = 1- P(AUB) = 1- 0.8 = 0.2

(v) P(AUB’) =P(A) + P(B’) – P(A∩B’) = 0.6 + (1-0.4) – 0.4 = 0.6

(vi) P(A’UB) = P(A’) + P(B) – P(A∩B’) = (1-0.4) + 0.4 – 0.4 = 0.6

5. Events A and B are mutually exclusive such that P(A’∩B) = 0.3, P(A’UB) = 0.45, find

(i) P(B) = P(A∩B) + P(A’∩B) = 0+ 0.3 = 0.3


(ii) P(A)

P(A’UB) = P(A’) + P(B) – P(A’∩B)

0.45 = P(A’) + 0.3 – 0.3

P(A’) = 0.45

P(A) = 1- P(A’) = 1- 0.45 = 0.55

(iii) P(A’∩B) = 0.55

P(B)= P(A∩B) + P(A’∩B)

P(A’∩B) = P(B) - P(A∩B) = 0.3 – 0 = 0.3

(iv) P(A’∩B’) = P(AUB)’ = 1 – P(AUB) = 1 – (P(A) + P(B) = 1- (0.55 + 0.3) = 0.15

(v) P(A’UB’) = P(A∩B)’ = 1- P(A∩B) = 1-0 = 1

(vi) P(A’UB) = P(A’) + P(B) – P(A’∩B)

=(1 – 0.55) + 0.3 - 0.3 =0.45

6. Event A and B are mutually exclusive such P(AUB) = 0.9, P(AUB’) = 0.6, find

(i) P(B) = 0.4

P(A) + P(B’) – P(A∩B’) =0.6

P(A) + [1 – P(B)] – P(A) = 0.6

1-P(B) = 0.6

P(B) = 0.4

(ii) P(A) = 0.5

P(A) + P(B) = 0.9

P(A) = 0.9 – 0.4 = 0.9

(iii) P(A’UB) = P(A’) + P(B)– P(A’∩B)

= (1 - 0.5) + 0.4 – 0.4 = 0.5

(iv) P(A’∩B’) = P(AUB)’ = 1- P(AUB) = 1 – 09 = 0.1

(v) P(A’UB’) = P(A∩B)’ = 1 - P(A∩B) = 1 - 0 = 1

Independent event
Two events A and B are independent if the occurrence of one does not affect the other

(i) P (A∩B) = P(A) x P(B)


(ii) P (A′ ∩B) = P(A’) x P(B)
(iii) P (A∩B’) = P(A) x P(B’)
(iv) P (A′ ∩B’) = P(A’) x P(B’)
Example 15
8 1
Event A and B are independent such that P(AUB) = , P(A) = , 𝑓𝑖𝑛𝑑
10 2

(i) P(B)
P(AUB)= P(A) + P(B) – P(A) x P(B)
0.8 = 0.5 + x – 0.5x
0.5x = 0.3; x = 0.6.
P(B) = 0.6
(ii) P(A’∩B)
P(A’∩B) = P(A’) x P(B) = (1-P(A)) x P(B) = (1-0.5)(0.6) = 0.3
(iii) P(A∩B’)
P(A∩B’) = P(A) x P(B’) = 0.5(1-0.6) = 0.2
(iv) P(A’∩B’)
P(A’∩B’) = P(A’) x P(B’) = (1-0.5)(1-0.6) = 0.2
(v) P(A’UB)
P(A’UB) = P(A’) + P(B) – P(A∩B) = 0.5 + 0.6 – (0.5 x 0.6) = 0.8

Example 16
1 1
Events A and B are independent such that P(A∩B) = , P(A) = , find
12 3

(i) P(B)
P(A∩B) = P(A) x P(B)
1 1 1
= 𝑥 𝑃(𝐵); P(B) =
12 3 4
(ii) P(AUB)
1 1 1
P(AUB) = P(A) + P(B) – P(A∩B) = + − = 0.5
3 4 12
(iii) Show that a’ and B are independent
P(A’∩B) = P(B) – P(A∩B)
P(B) – P(A) x P(B)
= P(B)(1-P(A)
= P(A’) x P(B)

Example 17

Events A and B are independent

(i) Show that the events A and B’ are also independent.


P(A∩B’) =P(A) – P(A∩B)
= P(A) – P(A) x P(B)
= P(A)(1-P(B)
= P(A) x P(B’)
(ii) Find P(B) given that given that P(A) = 0.4 and P(AUB) = 0.8
P(AUB) = P(A) + P(B) – P(A∩B)
0.8 = 0.4 + y – 0.4y
0.6y = 0.4
0.4 2
y= =
0.6 3
Example 17
9 2
Events A and B are independent such that P(AUB’) = , P(A) = , find
10 5

(i) P(B)
P(AUB’) = P(A) + P(B’) – P(A∩B’)
0.9 = 0.4 + (1-y) – 0.4(1- y) = 0.4 + 1 –y – 0.4 + 0.4y = 1 – 0.6y
0.6y = 0.1
0.1 1
y= =
0.6 6
(ii) P(AUB)
2 1 2 1 12+5−2 15
P(AUB) = P(A) + P(B)- P(A∩B)= + − 𝑥 = = = 0.5
5 6 5 6 30 30

(iii) Show that A’ and B’ are independent


P(A’ ∩B’) = 1 – P(AUB)
= 1- [P(A) + P(B)- P(A)x P(B)]
= 1- [P(A) + P(B) (1-P(A)]
= 1 – P(A) - P(B)x P(A’)
= P(A’) - P(B)x P(A’)
= P(A’)[1-P(B)]
= P(A’) x P(B’)

Example 18
1
The probability of two independent events A and B occurring together is . The probability that
8
5
either or both events occur is . Find
8

(i) P(A)
(ii) P(B)

Solution
5 1
Given P(AUB) = and P(A∩B) =
8 8

Let P(A) = x and P(B) = y


5
x + y – xy = ….. (i)
8
1
xy =
8
1
x= ………… (ii)
8𝑦

(i) and (ii)


1 1 5
+𝑦− =
8𝑦 8 8

1 6
+𝑦 =
8𝑦 8

Multiplying through by 8y

1 + 8y2 = 6y
8y2 – 6y + 1 = 0

Using quadratic equation


6±√(−6)2 −4(8)(1) 6±2
𝑦= =
2(8) 16

8 4 1
Either y = = 0.5 𝑂𝑟 𝑦 = = = 0.25
16 16 4
1
When y = 0.5; x = = 0.25
8 𝑥 0.5
1
When y = 0.25; x = = 0.5
8 𝑥 0.25

Hence P(A) = 0.25 and P(B) = 0.5 or P(A) = 0.5 and P(B) = 0.25

Example 19

Abel, Bob and Charles applied for the same job in a certain company. The probability that Abel will
3 1
take the job is , the probability that Bob takes it is while the probability that Charles will take the
4 2
2
job is ; what is the probability that
3

(i) None of them will take the job


1 1 1 1
P(none takes it) = P(A’∩B’∩C’) = P(A’) x P(B’) x P(C’) = 𝑥 𝑥 =
4 2 3 24
(ii) (one takes)
P(one takes it) = P(A∩B’∩C’) + P(A’∩B∩C’) + P(A’∩B’∩C)
3 1 1 1 1 1 1 1 2 1
( 𝑥 𝑥 ) +( 𝑥 𝑥 )+( 𝑥 𝑥 )=
4 2 3 4 2 3 4 2 3 4

Revision exercise C
1. Events A and B are independent such that P(A) = 0.4, P(B) = 0.25. Find
(i) P(A∩B) (i) P(A∩B’) (iii) P(A’∩B’)
2. Events A and B are independent such that P(A) = 0.3, P(B) = 0.5. Find
(i) P(A∩B) (ii) P(AUB) (iii) P(A’∩B’)
3. Events A and B are independent such that P(A) = 0.4, PAUB) = 0.7 Find
(i) P(B) (ii) P(A∩B) (iii) P(A’∩B)
1 3
4. Events A and B are independent such that P(A) = , P(B) = . Find the probability that
3 4
(i) Both occur
(ii) Only one occurs
2
5. The probability that two independent events A and B occur together = and the probability
15
3
that either A or B or both occur is . Find
5
(i) P(A) (ii) P(B)
6. A mother and her daughter both enter a competition. The probability that a mother wins a prize
1 2
= and the probability that her daughter wins the prize is . Assuming that the two events are
6 7
independent, find the probability that
(i) Either the mother of the daughter but not both wins the prize
(ii) At least one of them wins the prize
7. Two athletes, Kiprotich and Chebet attempt to qualify for Olympics games. The probabilityof
Kiprotich qualifying is 0.8 and the probability that both Kiprotich and Chebet qualifying is 0.6.
given that the probability of the athletes qualifying are independent events, find the probability
that only one qualifies
1
8. The probability of two independent events A and B occurring together = . The probability that
10
8
either or both events occur is . Find
10
(i) P(A)
(ii) P(B)

9. The probability that a certain types of computer will break down on the first month of use = 0.1.
If the school has two such computer bought at the same time, find the probability that at the
end of the first month just one has broken. Assume that the performance of the two computers
are independent
10. Three athletes enter a marathon race. The respective probabilities of them completing the race
are 0.9, 0.7 and 0.6. assuming that their performance are independent, find the probability that
(i) they all complete the race
(ii) at least two complete the race
1 2
11. The probability that two twins pass an interview are 𝑎𝑛𝑑 . Assuming that their performance
3 5
are independent, find the probability
(i) They all pass the interview
(ii) Only one passes the interview
12. The probability that Angela can solve a certain number is 0.4 and the probability that Jane can
solve the same number is 0.5, find the probability the number will be solved if both students try
the number independently
13. Three target men take part in a shooting competition, their chances of hitting the target are
1 1 1
, 𝑎𝑛𝑑 . Assuming that their performance are independent, find the probability that
4 3 2
(i) Only one will hit the target
(ii) Target will be hit
14. Three football teams Noa, Kitende and Budo are playing in Nationals. The probability that Noa,
2 3 1
Kitende and Budo qualify for the finals is , 𝑎𝑛𝑑 . Find the probability that only two teams
3 5 4
will qualify for the finals
15. Three Athletes Kiprop, Chebet and Aloysius are competing for a place in Olympics games. The
2 2 5
probability that Kiprop, Chebet and Aloysius will qualify for the Olympics games is , 𝑎𝑛𝑑 .
3 5 6
Find the probability that only one athlete will qualify for the Olympics games .
2 2 3
16. The probability that three girls Faith, Jane and Angela will pass exams is , 𝑎𝑛𝑑 respectively.
3 5 4
Find the probability that
(i) All the three will fail
(ii) All the three will pass
(iii) Only two will pass
17. The interview involves written, oral and practical tests. The probability than an interviewee
passes written = 0.8, oral is 0.6 and practical is 0.7. what is the probability that the interviewee
will pass
(i) The entire interview
(ii) Exactly two of the interview test
18. The probabilities that the players A, B and C score in a netball game are
1 1 1
, 𝑎𝑛𝑑 𝑟𝑒𝑠𝑝𝑒𝑐𝑡𝑖𝑣𝑒𝑙𝑦. If the player play together in a game, find the probability that
5 4 3
(i) Only one score
(ii) At least one player scores
(iii) Two and only two player score
1 1
19. Events a and B are such that P(A) = , P(B) = . Find P(AUB) when A and B are
5 2
(i) Independent events
(ii) Mutually exclusive events

Solutions to revision exercise C


1. Events A and B are independent such that P(A) = 0.4, P(B) = 0.25. Find
(i) P(A∩B) P(A) x P(B) = 0.4 x 0.25 = 0.1
(ii) P(A∩B’) =P(A) x P(B’) = 0.4 x 0.75 = 0.3
(iii) P(A’∩B’) = P(A’) x P(B’) = 0.6 x 0.75 =0.45
2. Events A and B are independent such that P(A) = 0.3, P(B) = 0.5. Find
(i) P(A∩B) = P(A) x P(B) = 0.3 x 0.5 = 0.15
(ii) P(AUB) =P(A0 + P(B) - P(A∩B) = 0.3 + 0.5 – 0.16 = 0.65
(iii) P(A’∩B’) = 1 – P(AUB) = 1 – 0.65 = 0.35
3. Events A and B are independent such that P(A) = 0.4, PAUB) = 0.7 Find
(i) P(B)
P(AUB) = P(A) + P(B) – P(A) x P(B)
0.7 = 0.4 + x – 0.4x
0.6x = 0.3
x = 0.5
hence P(B) = 0.5
(ii) P(A∩B) = P(A) x P(B) = 0.4 x 0.5 =0.2
(iii) P(A’∩B) = 1 – P(AUB) = 1 – 0.7 = 0.3
1 3
4. Events A and B are independent such that P(A) = , P(B) = . Find the probability that
3 4
(i) Both occur
1 3 1
P(A∩B) = P(A) x P(B)= 𝑥 =
3 4 4
(ii) Only one occurs
2 3 1 1 7
P(A∩B’) + P(A’∩B) = P(A) x P(B’) + P(A’) x P(B) = 𝑥 + 𝑥 =
3 4 3 4 12
2
5. The probability that two independent events A and B occur together = and the probability
15
3
that either A or B or both occur is . Find
5
Let the P(A) = x and P(B) = y
2
xy = ……. (i)
15
3
x + y - xy= ……. (ii)
5
Eqn (i) and eqn. (ii)
2 2 3
+𝑦− =
15𝑦 15 5
2
2 + 15y – 2y= 9y
15y2 – 11y + 2 = 0
Using quadratic equation
11±√(−11)2 −4(15)(2) 11±1
y= =
2(15) 30
10 1 12
y= = 𝑂𝑟 𝑦 = = 0.4
30 3 30
1 1
Either P(A) = 0.4 and P(B) = or P(A) = and P(B) =0.4
3 3

6. A mother and her daughter both enter a competition. The probability that a mother wins a prize
1 2
= and the probability that her daughter wins the prize is . Assuming that the two events are
6 7
independent, find the probability that
5
(i) Either the mother of the daughter but not both wins the prize =
14
1 5 5 2 15 5
P(M∩D’) + P(M’∩D) = P(M)x P(D’) + P(M’) x P(D) = 𝑥 + 𝑥 = =
6 7 6 7 42 14
(ii) At least one of them wins the prize
1 2 1 2 17
P(AUB) = P(A) + P(B) – P(A∩B) = + − x =
6 7 6 7 42
7. Two athletes, Kiprotich and Chebet attempt to qualify for Olympics games. The probabilityof
Kiprotich qualifying is 0.8 and the probability that both Kiprotich and Chebet qualifying is 0.6.
given that the probability of the athletes qualifying are independent events, find the probability
that only one qualifies = 0.35
P(K∩B) = P(K) x P(C) = 0.8 x P(C) = 0.6
0.6 3
P(C) = =
0.8 4
P(K∩C’) + P(K’∩C) = 0.8 x 0.25 + 0.2 x 0.75 = 0.35

1
8. The probability of two independent events A and B occurring together = . The probability that
10
8
either or both events occur is . Find
10
(iii) P(A)
(iv) P(B)

Solution

Let the P(A) = x and P(B) = y


1
xy = ……. (i)
10
8
x + y - xy= ……. (ii)
10
Eqn (i) and eqn. (ii)
1 1 8
+𝑦− =
10𝑦 10 10
1 + 10y2 – y= 8y
10y2 – 9y + 1 = 0
Using quadratic equation
9±√(−9)2 −4(10)(1) 9±6.4
y= =
2(10) 20
2.6 12
y= = 0.13 𝑂𝑟 𝑦 = = 0.77
20 30

Either P(A) = 0.13 and P(B) = 0.77 or P(A) = 0.77 and P(B) = 0.13

9. The probability that a certain types of computer will break down on the first month of use = 0.1.
If the school has two such computer bought at the same time, find the probability that at the
end of the first month just one has broken. Assume that the performance of the two computers
are independent= 0.18
Let the computers be A and B
P(one has broken down) = P(A∩B’) + P(A’∩B)
=0.1 x 0.9 + 0.9 x 0.1
= 0.18
10. Three athletes enter a marathon race. The respective probabilities of them completing the race
are 0.9, 0.7 and 0.6. Assuming that their performance are independent, find the probability that
(i) they all complete the race
Let the athletes be A, B, C
P(all complete the race) = P(A∩B∩C) = 0.9 x 0.7 x 0.6 = 0.378
(ii) at least two complete the race
P(At least two complete the race) = P(A’∩B∩C) + P(A∩B’∩C) + P(A∩B∩C’) + P(A∩B∩C)
= 0.1 x 0.7 x 0.6 + 0.9 x 0.3 x 0.6 + 0.9 x 0.7 x 0.4 + 0.378
= 0.834
1 2
11. The probability that two twins pass an interview are 𝑎𝑛𝑑 . Assuming that their performance
3 5
are independent, find the probability
(i) They all pass the interview
Let the twins be A and B
1 2 2
P(all pass) =𝑃(𝐴 ∩ 𝐵) = 𝑥 =
3 5 15
(ii) Only one passes the interview
2 2 1 3 7
P(A∩B’) + P(A’∩B) = 𝑥 + 𝑥 =
3 5 3 5 15
12. The probability that Angela can solve a certain number is 0.4 and the probability that Jane can
solve the same number is 0.5, find the probability the number will be solved if both students try
the number independently = 0.7

P(the number solved) = P(A ∩J’) + P(A’∩J) + P(A∩J)

= 0.4 x 0.5 + 0.6 x 0.5 + 0.4 x 0.5

= 0.7

13. Three target men take part in a shooting competition, their chances of hitting the target are
1 1 1
, 𝑎𝑛𝑑 . Assuming that their performance are independent, find the probability that
4 3 2
Let the men be A, B and V
(i) Only one will hit the target =
P(one hit the target) = P(A’∩B’∩C) + P(A’∩B∩C’) + P(A∩B’∩C’)
3 2 1 3 1 1 1 2 1
= 𝑥 𝑥 + 𝑥 𝑥 + 𝑥 𝑥
4 3 2 4 3 2 4 3 2
11
=
24
(ii) Target will be hit
P(target hit)= P(A’∩B’∩C) + P(A’∩B∩C’) + P(A∩B’∩C’) + P(A’∩B∩C) + P(A∩B∩C’) +
P(A∩B’∩C) + P(A∩B∩C)
11 3 1 1 1 1 1 1 2 1 1 1 1
= + 𝑥 𝑥 + 𝑥 𝑥 + 𝑥 𝑥 + 𝑥 𝑥
24 4 3 2 4 3 2 4 3 2 4 3 2
18
= = 0.75
24
14. Three football teams Noa, Kitende and Budo are playing in Nationals. The probability that Noa,
2 3 1
Kitende and Budo qualify for the finals is , 𝑎𝑛𝑑 . Find the probability that only two teams
3 5 4
will qualify for the finals
P(only two teams qualify) = P(N’∩K∩B) + P(N∩K’∩B) + P(N∩K∩B’)
1 3 1 2 2 1 2 3 3
= 𝑥 𝑥 + 𝑥 𝑥 + 𝑥 𝑥
3 5 4 3 5 4 3 5 4
25 5
= =
60 12
15. Three Athletes Kiprop, Chebet and Aloysius are competing for a place in Olympics games. The
2 2 5
probability that Kiprop, Chebet and Aloysius will qualify for the Olympics games is , 𝑎𝑛𝑑 .
3 5 6
Find the probability that only one athlete will qualify for the Olympics games .
P(only one qualifies) = P(K∩C’∩A’)+ P(K’∩C∩A’) + P(K’∩C’∩A)
2 3 1 1 2 1 1 3 5
= 𝑥 𝑥 + 𝑥 𝑥 + 𝑥 𝑥
3 5 6 3 5 6 3 5 6
23
=
90
2 2 3
16. The probability that three girls Faith, Jane and Angela will pass exams is , 𝑎𝑛𝑑 respectively.
3 5 4
Find the probability that
(i) All the three will fail
1 3 1 1
P(all fail) = P(F’∩J’∩A’) = 𝑥 𝑥 =
3 5 4 20
(ii) All the three will pass
2 2 3 1
P(all pass) = P(F∩J∩A) = 𝑥 𝑥 =
3 5 4 5
(iii) Only two will pass
P(only two pass) = P(F’∩J∩A) + P(F∩J∩A’) + P(F∩J’∩A)
1 2 3 2 2 1 2 3 3 28 7
= 𝑥 𝑥 + 𝑥 𝑥 + 𝑥 𝑥 = =
3 5 4 3 5 4 3 5 4 60 15
17. The interview involves written, oral and practical tests. The probability than an interviewee
passes written = 0.8, oral is 0.6 and practical is 0.7. what is the probability that the interviewee
will pass
(i) The entire interview
P(pass entire interview) = P(W∩O∩P) = 0.8 x 0.6 x 0.7 =0.336
(ii) Exactly two of the interview test
P(pass exactly two) = P(W’∩O∩P) + P(W∩O’∩P) + P(W∩O∩P’)
= 0.2 x 0.6 x 0.7 + 0.8 x 0.4 x 0.7 + 0.8 x 0.6 x 0.3
= 0.452
18. The probabilities that the players A, B and C score in a netball game are
1 1 1
, 𝑎𝑛𝑑 𝑟𝑒𝑠𝑝𝑒𝑐𝑡𝑖𝑣𝑒𝑙𝑦. If the player play together in a game, find the probability that
5 4 3
(i) Only one score
P(only one scores) = P(A∩B’∩C’) + P(A’∩B∩C’) + P(A’∩B’∩C)
1 3 2 4 1 2 4 3 1 26 13
= 𝑥 𝑥 + 𝑥 𝑥 + 𝑥 𝑥 = =
5 4 3 5 4 3 5 4 3 60 30
(ii) At least one player scores
P(at least one scores) = P(A∩B’∩C’) + P(A’∩B∩C’) + P(A’∩B’∩C) + P(A∩B∩C’) +
P(A’∩B∩C) + P(A∩B’∩C) + P(A∩B∩C)
13 1 1 2 4 1 1 1 3 1 1 1 1
= + 𝑥 𝑥 + 𝑥 𝑥 + 𝑥 𝑥 + 𝑥 𝑥
30 5 4 3 5 4 3 5 4 3 5 4 3
36 3
= =
60 5
(iii) Two and only two player score
= P(A∩B∩C’) + P(A’∩B∩C) + P(A∩B’∩C)
1 1 2 4 1 1 1 3 1
= 𝑥 𝑥 + 𝑥 𝑥 + 𝑥 𝑥
5 4 3 5 4 3 5 4 3
9 3
= =
60 20
1 1
19. Events a and B are such that P(A) = , P(B) = . Find P(AUB) when A and B are
5 2
(i) Independent events
1 1 1 1 6 3
P(AUB) = P(A) + P(B) – P(A∩B) = + − 𝑥 = =
5 2 5 2 10 5
7
(ii) Mutually exclusive events =
10
1 1 7
P(AUB) = P(A) + P(B) – P(A∩B) = + − 0 =
5 2 10

Conditional probability
If A and B are two events, then the conditional probability that A occurs given that B has already
occurred is P(A/B)
𝑃(𝐴∩𝐵)
P(A/B) =
𝑃(𝐵)

Example 20
1 1 2
Events A and B are such that P(A) = , P(B) = and P(A/B) = . Find
5 4 5

(i) P(A∩B)
𝑃(𝐴∩𝐵)
P(A/B) =
𝑃(𝐵)
2 1 2
P(A∩B) = 𝑥 = = 0.1
5 4 20

(ii) P(AUB)
1 1
P(AUB) = P(A) + P(B) - P(A∩B) = + − 0.1 = 0.35
5 4
(iii) P(B/A)
𝑃(𝐴∩𝐵) 0.1
P(B/A) = = = 0.5
𝑃(𝐴) 0.2

Example 21
4 1 5
Events A and B are such that P(A) = , P(A∩B’) = and P(A/B)= . Find
7 3 14

(i) P(A∩B)
4 1 12−7 5
P(A∩B) = P(A) - P(A∩B’) = − = =
7 3 21 21
(ii) P(B)
𝑃(𝐴∩𝐵)
P(A/B) =
𝑃(𝐵)
5 5 5 14 2
P(B) = ÷ = 𝑥 =
21 14 21 5 3
(iii) P(AUB)
4 2 5
P(AUB) = P(A) + P(B) - P(A∩B) = + − =1
7 3 21

Example 22
1 1
Events A and B are independent. Given that P(A∩B’) = and P(A’/B) = . Find
4 6
(i) P(A)
𝑃(𝐴′∩𝐵) 𝑃(𝐴′ ) 𝑥 𝑃(𝐵)
P(A’/B) = =
𝑃(𝐵) 𝑃(𝐵)
1 1 5
P(A’) = and P(A) = 1- =
6 6 6
(ii) P(B)
P(A∩B’) = P(A) x P(B’) =
1 5 1 6 3
P(B’) = P(A∩B’)÷ 𝑃(𝐴) = ÷ = 𝑥 =
4 6 4 5 10
P(B) = 1 – P(B’) = 1- 0.3 = 0.7
5 7 7
(iii) P(A∩B) = P(A) x P(B) = 𝑥 =
6 10 12
(iv) P(AUB)’
5 7 7 19
P(AUB) = P(A) + P(B) - P(A∩B) = + − =
6 10 12 20
19 1
P(AUB)’ =1- P(AUB) = 1 - =
20 20

Revision exercise D
3 1 1
1. Events X and Y are such that P(X’) = , P(Y/X’) = and P(Y’/X) = . Find
5 3 4
1
(i) P(Y) = (ii) P(X’/Y)
2
2 1 2
2. Events A and B are such that P(A)= , P(A/B) = , P(B/A) = . Find
5 2 3
4 8 2
(i) P(A∩B) = (ii) P(B) = (iii) P(AUB) =
15 15 3
1 1 4
3. Events A an B are such that P(A) = , P(B/A) = and P(B’/A’) = . Find
3 4 5
3 1 13 7
(i) P(B’/A) = (ii) P(A∩B) = (iii) P(B) = (iv) P(AUB) =
4 12 60 15
1 1 1
4. Events A and B are such that P(A) = , P(B) = and P(A∩B’) = . Find
2 3 3
5 1
(i) P(A’UB’) = (ii) P(A’/B’) =
6 2
1 1 1
5. Events A and B are such that P(A∩B) = , P(B/A) and P(B) = . Find
12 3 6
1 1 1
(i) P(A) = (ii) P(A/B) = (iii) P(A/B’) =
4 2 5
6. Events A and B are such that P(AUB) = 0.8, P(A/B) = 0.2 and P(A’∩B’) = 0.4. Find
(i) P(A∩B) = 0.1 (ii) P(B) = 0.5 (ii) P(A) = 0.4 (iv) P(A/B’) = 0.6 (v) P(A’/B’) =0.4
7. Events A and B are independent. Given that P(A) = 0.2 and P(B) = 0.15. find
(i) P(A∩B) = 0.03 (ii) P(A/B) = 0.2 and P(AUB) = 0.32
8. Events A and B are such that P(A) = 0.2, P(A/B) = 0.4 and P(B) = 0.25. Find
(i) P(A∩B) = 0.1 (ii) P(B/A) = 0.5 (ii) P(AUB) = 0.35
1 1 2
9. Events A and B are such that P(A) = , P(B) = and P(A/B) = . Find
3 4 5
(i) P(A∩B) = 0.1 (ii) P(B/A) = 0.3
2 1 2
10. Events A and B are such that P(A) = , P(B) = and P(A/B) = . Find
3 4 3
1 1
(i) P(A∩B) = (ii) P(B/A) =
6 4
1 1 1
11. Events A and B are such that P(B) = , P(A) = and P(A∩B’) = . Find
3 2 3
5 2
(i) P(A’UB’) = (ii)P(B’/A’) =
6 3
1 3 7
12. Events A and B are such that P(A) = , P(B) = and P(A/B) = . Find
2 8 12
7
(i) P(A∩B) =
32
3
(ii) P(B/A’) =
8
13. A and B are intersecting sets as shown in the Venn diagram below
A B
x y z
0.15
5
Given that P(A) = 0.6, P(A’/B) = and PAUB) = 0.85. Find
7

(i) the values x, y and z (x = 0.5, y = 0.1 and z = 0.25)


2 10
(ii) P(A/B) = (iii) P(A/B’) =
7 13
14. Events A and B are independent with A twice likely to occur as B. If P(A) = 0.5, find
5 𝐴∩𝐵 1
(i) P(AUB) = (ii) P( )=
8 𝐴 8
2 1 1
15. Two events A and B are such that P(A/B) = , P(B) = and P(A) = . Find
5 4 5
(i) P(A∩B) = 0.1 (ii) P(AUB) = 0.35
1 1 1
16. Two events a and B are such that P(B/A) = , P(B) = and P(A∩B) = . Find
3 8 10
(i) P(A) = 0.3 (ii) P(AUB) = 0.325 (iii) P(A/B) = 0.8
17. Two events A and B are such that P(A/B)= 0.1, P(A) = 0.7 and P(B) = 0.2. Find
(i) P(AUB) = 0.88 (ii) P(A∩B’) = 0.68

Solutions to revision exercise D


3 1 1
1. Events X and Y are such that P(X’) = , P(Y/X’) = and P(Y’/X) = . Find
5 3 4
1
P(Y) =
2
𝑃(𝑌∩𝑋 ′ )
P(Y/X’) =
𝑃(𝑋 ′ )
1 3 1
P(Y∩X’) = 𝑥 =
3 5 5
2 1 2
2. Events A and B are such that P(A)= , P(A/B) = , P(B/A) = . Find
5 2 3
(i) P(A∩B)
𝑃(𝐴∩𝐵)
P(B/A) =
𝑃(𝐴)
2 2 4
P(A∩B)) = 𝑥 =
3 5 15
(ii) P(B)
𝑃(𝐴∩𝐵)
P(A/B) =
𝑃(𝐵)
4 1 4 8
P(B) = ÷ = 𝑥2=
15 2 15 15

(iii) P(AUB)
2 8 4 10 2
P(AUB) = P(A) + P(B) – P(A∩B) = + − = =
5 15 15 15 3

1 1 4
3. Events A an B are such that P(A) = , P(B/A) = and P(B’/A’) = . Find
3 4 5
3
(i) P(B’/A) =
4
P(B/A) =
1 13 7
(ii) (ii) P(A∩B) = (iii) P(B) = (iv) P(AUB) =
12 60 15
1 1 1
4. Events A and B are such that P(A) = , P(B) = and P(A∩B’) = . Find
2 3 3
5 1
(i) P(A’UB’) = (ii) P(A’/B’) =
6 2
1 1 1
5. Events A and B are such that P(A∩B) = , P(B/A) and P(B) = . Find
12 3 6
1 1 1
(i) P(A) = (ii) P(A/B) = (iii) P(A/B’) =
4 2 5
6. Events A and B are such that P(AUB) = 0.8, P(A/B) = 0.2 and P(A’∩B’) = 0.4. Find
(i) P(A∩B)
(ii) P(B) = 0.5
(iii) P(A) = 0.4
(iv) P(A/B’) = 0.6
(v) P(A’/B’) =0.4
7. Events A and B are independent. Given that P(A) = 0.2 and P(B) = 0.15. find
(i) P(A∩B) = P(A) x P(B) = 0.2 x 0.15 = 0.03
𝑃(𝐴∩𝐵) 0.03
(ii) P(A/B) = = = 0.2
𝑃(𝐵) 0.15
(iii) P(AUB) = P(A) + P(B) – P(A∩B) = 0.2 + 0.15 – 0.03 = 0.32
8. Events A and B are such that P(A) = 0.2, P(A/B) = 0.4 and P(B) = 0.25. Find
(i) P(A∩B)
𝑃(𝐴∩𝐵)
P(A/B) =
𝑃(𝐵)
P(A∩B) = 0.4 x 0.25= 0.1
(ii) P(B/A)
𝑃(𝐴∩𝐵) 0.1
P(B/A) = = = 0.5
𝑃(𝐴) 0.2
(iii) P(AUB)
P(AUB) = P(A) + P(B) - P(A∩B) = 0.2 + 0.25 – 0.1= 0.35
1 1 2
9. Events A and B are such that P(A) = , P(B) = and P(A/B) = . Find
3 4 5
(i) P(A∩B)
𝑃(𝐴∩𝐵)
P(A/B) =
𝑃(𝐵)
2 1 1
P(A∩B) = 𝑥 =
5 4 10
(ii) P(B/A) = 0.3
𝑃(𝐴∩𝐵) 1 1 3
P(B/A) = = ÷ =
𝑃(𝐴) 10 3 10
2 1 2
10. Events A and B are such that P(A) = , P(B) = and P(A/B) = . Find
3 4 3
(i) P(A∩B)
𝑃(𝐴∩𝐵)
P(A/B) =
𝑃(𝐵)
2 1 1
P(A∩B) = 𝑥 =
3 4 6
(ii) P(B/A)
𝑃(𝐴∩𝐵) 1 2 1 3 1
P(B/A) = = ÷ = 𝑥 =
𝑃(𝐴) 6 3 6 2 4
1 1 1
11. Events A and B are such that P(B) = , P(A) = and P(A∩B’) = . Find
3 2 3
(i) P(A’UB’)
1 1 5
P(A’UB’) = 1 – P(A∩B) = 1 - 𝑥 =
2 3 6
(ii) P(B’/A’)
2 1
𝑃(𝐴′ ∩𝐵′ ) 𝑥 2
3 2
P(B’/A’) = = 1 =
𝑃(𝐴′ ) 3
2
1 3 7
12. Events A and B are such that P(A) = , P(B) = and P(A/B) = . Find
2 8 12
(i) P(A∩B)
𝑃(𝐴∩𝐵)
P(A/B) =
𝑃(𝐵)
7 3 7
P(A∩B) = 𝑥 =
12 8 32
3 1
𝑃(𝐵∩𝐴′) 𝑥 3
8 2
(ii) P(B/A’) = = 1 =
𝑃(𝐴′) 8
2
13. A and B are intersecting sets as shown in the Venn diagram below
A B
x y z
0.15
5
Given that P(A) = 0.6, P(A’/B) = and PAUB) = 0.85. Find
7

(i) the values x, y and z


from the Venn diagram
x + y + z = 0.85 …….. (i)
x + y = 0.6 ………….. (ii)
subtracting eqn. (ii) from eqn. (i)
z = 0.85 – 0.6 = 0.25
𝑃(𝐴′ ∩𝐵) 5
P(A’∩B) = =
𝑃(𝐵) 7
P(A’∩B) = P(B) only = 0.25
5
 0.25 = 𝑃(𝐵)
7
0.25 𝑥 7
P(B) = = 0.35
5

But P(B) = y + z = y + 0.25 = 0.35

y = 0.1

Substituting y in eqn.(i)

x + 0.1 = 0.6; x = 0.5


𝑃(𝐴∩𝐵) 0.1 10 2
(ii) P(A/B) = = = =
𝑃(𝐵) 0.35 35 7
𝑃(𝐴∩𝐵′) 0.5 50 10
(iii) P(A/B’) = = = =
𝑃(𝐵′) 0.65 65 13
14. Events A and B are independent with A twice likely to occur as B. If P(A) = 0.5, find
(i) P(AUB)
P(B) = 0.25
P(AUB) = P(A) + P(B) – P(A∩B) = 0.5 + 0.25 – 0.5 x 0.25 = 0.625
0.125 𝑥 0.5
(ii) P(𝐴 ∩ 𝐵/𝐴) = =0125
0.5
2 1 1
15. Two events A and B are such that P(A/B) = , P(B) = and P(A) = . Find
5 4 5
(i) P(A∩B)
𝑃(𝐴∩𝐵)
P(A/B) =
𝑃(𝐵)
P(B) = 0.4 x 0.25 = 0.1
(ii) P(AUB)
P(AUB) = P(A) + P(B) – P(A∩B) = 0.2 + 0.25 – 0.1 = 0.35
1 1 1
16. Two events a and B are such that P(B/A) = , P(B) = and P(A∩B) = . Find
3 8 10
(i) P(A) = 0.3
𝑃(𝐴∩𝐵)
P(B/A) =
𝑃(𝐴)
1 1 3
P(A) = ÷ = = 0.3
10 3 10
(ii) P(AUB)
P(AUB) = P(A) + P(B) – P(A∩B) = 0.3 + 0.125 – 0.1 = 0.325
(iii) P(A/B)
𝑃(𝐴∩𝐵) 0.1
P(A/B) = = = 0.8
𝑃(𝐵) 0.125
17. Two events A and B are such that P(A/B)= 0.1, P(A) = 0.7 and P(B) = 0.2. Find
(i) P(AUB) =
𝑃(𝐴∩𝐵)
P(A/B) =
𝑃(𝐵)
P(A∩B) = 0.1 x 0.2 = 0.02
P(AUB) = P(A) + P(B) - P(A∩B)
= 0.7 + 0.2 – 0.02 =0.88
(ii) P(A∩B’)
P(A∩B’) = P(A) – P(A∩B) = 0.7 – 0.02 = 0.68

Combinations
The number of combinations of r objects from n unlike objects is 𝑛𝑐𝑟 where
𝑛!
𝑛𝑐𝑟 = (𝑛−𝑟)!𝑟!

Example 23
5! 5𝑥4𝑥3𝑥2𝑥1
Find the value of 5𝑐2 = (5−3)!.2! = 10
(3 𝑥 2 𝑥 1)(2 𝑥 1)

Example 24

Find the number of ways of selecting a football team from 15 players

15𝑐11 = 1365 𝑤𝑎𝑦𝑠


Example 25

A committee of 4 men and 3 women is to be formed from 10 men and 8 women. In how many ways
can the committee be formed?

10𝑐4 𝑥 8𝐶3 = 210 𝑥 56 = 11760 𝑤𝑎𝑦𝑠


Example 26

A group of 9 has to be selected from 8 girls it can consist of either 5 boys or 4 girls of 4 boys and 5
girls. Find how many different groups can be chosen

10𝑐5 𝑥 8𝑐4 + 10𝑐4 𝑥 8𝑐5 = 252 𝑥 70 + 210 𝑥 56 = 29400 𝑤𝑎𝑦𝑠


Exercise E
1. A bag contains 5 Pepsi and 4 Mirinda bottle tops. Three bottle tops are picked at random from
the bag one after the other without replacement. Find the probability that the bottle tops
picked are of the same type.
2. In a group of 12 international referees, there are 3 from Africa, 4 from Asia and 5 from Europe.
To officiate at a tournament 3 referees are chosen at random from the group, find the
probability that:
(i) A referee is chosen from each continent
(ii) Exactly 2 referees are chosen from Asia
(iii) 3 referees are chosen from the same continent
3. Box P contains 4 red and 3 green sweets and box Q contains 7 red and 4 green sweets. A box is
selected and 2 sweets are randomly picked from it, one a time without replacement. If P is
twice likely to be picked as Q, find the probability that both sweets are
(i) same colour
(ii) of different colour
(iii) from P given that they are different colours.
4. A bag contains 20 good and 4 bad oranges. If 5 oranges are selected at random without
replacement. Find the probability that 4 are good and the other is bad = 0.456

Solutions to exercise E
1. A bag contains 5 Pepsi and 4 Mirinda bottle tops. Three bottle tops are picked at random from
the bag one after the other without replacement. Find the probability that the bottle tops
picked are of the same type.
Solution
n(S) = 3 tops from the 9 = 9𝑐3 = 84 𝑤𝑎𝑦𝑠
n(E) = 3 Pepsi from 5 + 3 Mirinda from 4 = 5𝑐3 𝑥 4𝑐0 + 5𝑐0 𝑥 4𝑐3 = 10 + 4 = 14 𝑤𝑎𝑦𝑠
14 1
P(same type) = =
84 6
2. In a group of 12 international referees, there are 3 from Africa, 4 from Asia and 5 from Europe.
To officiate at a tournament 3 referees are chosen at random from the group, find the
probability that:
(iv) A referee is chosen from each continent
(v) Exactly 2 referees are chosen from Asia
(vi) 3 referees are chosen from the same continent

Solution

(i) n(S) = 3 referees from 12 = 12𝑐3 = 220𝑤𝑎𝑦𝑠

n(E) =3𝑐1 𝑥 4𝑐1 𝑥 5𝑐1 = 60 𝑤𝑎𝑦𝑠


60 3
P(1 from each) = =
220 11

(ii) 𝑛(𝐸) = 4𝑐2 𝑥 3𝑐1 𝑥 5𝑐0 + 4𝑐2 𝑥 3𝑐0 𝑥 5𝑐1 = 18 + 30 = 48 𝑤𝑎𝑦𝑠


48 12
P(2 from Asia) = =
220 55
(iii) 𝑛(𝐸) = 4𝑐3 𝑥 3𝑐0 𝑥 5𝑐0 + 4𝑐0 𝑥 3𝑐3 𝑥 5𝑐0 + 4𝑐0 𝑥 3𝑐0 𝑥 5𝑐3 = 4 + 1 + 10 = 15ways
15 3
P(3 from the same) = =
220 44
3. Box P contains 4 red and 3 green sweets and box Q contains 7 red and 4 green sweets. A box is
selected and 2 sweets are randomly picked from it, one a time without replacement. If P is
twice likely to be picked as Q, find the probability that both sweets are
(iv) same colour
(v) of different colour
(vi) from P given that they are different colours.

Solution
2 4𝑐2 𝑥 3𝑐0 2 4𝑐0 𝑥 3𝑐2 1 7𝑐2 𝑥 4𝑐0 1 7𝑐0 𝑥 4𝑐2
(i) P(both the same colour) = [ ]+ [ ]+ [ ]+ [ ]
3 7𝑐2 3 7𝑐2 3 11𝑐2 3 11𝑐2
= 0.4494
2 4𝑐1 𝑥 3𝑐1 1 7𝑐1 𝑥 4𝑐1
(ii) P(both different colour) = [ ]+ [ ] = 0.5506
3 7𝑐2 3 11𝑐2
2 4𝑐1 𝑥 3𝑐1
[ ]
3 7𝑐2
(iii) P(from P/different colour) = = 0.6919
0.5506
4. A bag contains 20 good and 4 bad oranges. If 5 oranges are selected at random without
replacement. Find the probability that 4 are good and the other is bad
20𝑐4 .4𝑐1
P(X= 4) = = 0.456
24𝑐5

Probability tree diagram


Example 27

A factory makes cakes. When an inspector tests a random sample of cakes, the probability of any
cake being contaminated is 0.025. Jane bought two of the cakes made from the factory. Find the
probability

(i) both are contaminated


(ii) only one is contaminate

Solution

(i) P(both contaminated) = P(C∩ 𝐶) =0.025 x 0.025 = 0.000625

(ii) P(one contaminated) =P(C∩ 𝐶′) + P(C′ ∩ 𝐶) =0.025 x 0.975 + 0.975 x 0.025 = 0.04875

Example 28

A box contains 3 red balls and 4 blue balls. Two balls are randomly drawn one after the other
without replacement. Find the probability that

(i) 1st ball is blue


(ii) 2nd ball is red
(iii) 2nd ball is red given that the 1st was blue
(iv) Both balls are of the same colour
(v) Different colour

2R, 4B
3R, 4B
3R, 3B

4 3 4 3
(i) P(1st is blue) = 𝑥 + 𝑥 = 0.5714
7 6 7 6
3 2 4 3
(ii) P(2nd is red ) = 𝑥 + 𝑥 = 0.4286
7 6 7 6
3
(iii) P(2nd is red given 1st is blue) = = 0.5
6
3 2 4 3
(iv) P(both same colour) = 𝑥 + 𝑥 = 0.4286
7 6 7 6
3 4 4 3
(v) P(different colour) = 𝑥 + 𝑥 = 0.5714
7 6 7 6
Or = 1- 0.4286 = 0.5714

Example 29

A bag contains 8red pens and 3 green pens. Two pens are randomly picked one after the other, find
the probability of drawing two pens of different colours, if the

(i) first pen is not replaced


(ii) first pen is replaced

Solution

(i)
2R, 4B
8R, 3G
3R, 3B

8 3 3 8 24
P(different colour) = 𝑥 + 𝑥 =
11 10 11 10 55
(ii)

2R, 4B
8R, 3G
3R, 3B

8 3 3 8 48
P(different colour) = 𝑥 + 𝑥 =
11 11 11 11 121
Example 30

Bag A contains 8 red pens and 5 green pens. Bag B contains 6 red and 10 green pens. a pen is
randomly picked from bag A and placed in bag B. A pen is then randomly picked from bag B, fins the
probability that it will be red

Solution
B
A 7R, 10G
8R, 5G
B
6R, 11G

8 7 5 6 86
P(Red pen) = 𝑥 + 𝑥 = =0.3891
13 17 13 17 221

Example 31

Box A contains 6 red and 8 green sweets and box B contains 9 red and 7 green sweets. A box is
randomly picked and 2 sweets are randomly picked from it, one at a time without replacement. If A
is likely to be picked as B, find the probability that both sweets are

(i) same colour


(ii) from A given that are of same colour
(iii) of different colours

5R, 8G
A
6R, 8G
6R, 7G
Bags
A

B 8R, 7G
9R, 7G
9R, 6G

1 6 5 1 8 7 1 9 8 1 7 6
(i) P(same colour) = 𝑥 𝑥 + 𝑥 𝑥 + 𝑥 𝑥 + 𝑥 𝑥 = 0.4738
2 14 13 2 14 13 2 16 15 2 16 15
(ii) P(both different colour) = 1 - 0.4738 = 0.5262
1 6 5 1 8 7
𝑥 𝑥 + 𝑥 𝑥
(iv) P(from A given that are of same colour) = 2 14 13 2 14 13
= 0.4987
0.4738
Exercise F
1. A box contains 15 red and 5 black balls. Two balls are picked at random one after the other
without replacement. find the probability that:
21
(i) both are red =
38
15
(ii) are different colour =
38
(iii) both are black, given that the second ball is black
2. A box contains 4 red balls and 6 black balls two balls are randomly drawn one after the other
without replacement. Find the probability that
1
(i) second ball is red given that the first ball is red =
3
2
(ii) both balls are red =
15
8
(iii) both balls of different colour =
15
3. A box contains 3 black balls and 5 white balls. Two balls are randomly drawn one after the
other without replacement. find the probability
5
(i) second ball is white =
8
4
(ii) first ball is white given that the second ball is white =
7
4. A box contains 3 red sweets, 8 blue sweets and 7 green sweets. Three sweets are randomly
drawn one after the other without replacement. Find that
7
(i) all sweets are blue =
102
1
(ii) all sweets are red =
816
7
(iii) one of each colour =
34
5. A box contains 7 black sweets and 3 white sweets. Three sweets are randomly drawn one
after the other with replacement. Find the probability that:
(i) all three sweets are black = 0.34
(ii) a white, black and a white sweet in that order are chosen = 0.063
(iii) two white and one black sweets are drawn = 0.189
(iv) at least one black sweet drawn = 0.97
5
6. A coin is tossed four times, find the probability of obtaining less than two heads. =
16
7. The probability that I am late for work is 0.05. Find the probability that on two consecutive
mornings:
(i) I am late for work twice = 0.0025
(ii) I am late for work once = 0.095
8. A box A contains 3 red balls and 4 brown balls while box B contains 3 red balls and 2 brown
balls. A box is drawn at random and one ball is randomly drawn from it. Find the probability
that;
18
(i) the ball is red =
35
5
(ii) the ball came from box A given that it is red =
12
9. a bag contains 10 white and 6 red balls two balls are randomly drawn one after the other
without replacement. Find the probability that the second ball drawn is
(i) red given that the first one was white = 0.4
(ii) white = 0.675
10. Box P contains 2 red balls and 2 blue balls while Box Q contains 2 red balls and 3 blue balls.
A box is drawn at random and two balls are randomly drawn from it, one after the other
19
without replacement. Find the probability that the balls are of different colour. = .
30
11. A box contains 4 white balls and 1 black ball. A second box contains 1 white and 4 black
balls. A ball is drawn at random from the first bag and put into the second bag, then a ball is
taken from the second bag and put into the first bag. Find the probability that a white ball
7
will be picked when a ball is selected from the first bag =
10
12. (a) a box contains 7 red balls and 6 blue balls . Three balls are selected at random without
replacement. find the probability that:
(i) they are the same colour = 0.1923
(ii) at most two are blue = 0.9301

(b) Two boxes P and Q contain white and brown cards. P contains 6 white and 4 brown. Q
contains 2 white and 3 brown. A box is selected at random and a card is selected at random.
Find the probability that

(i) a brown card is selected = 0.5

(ii) Box Q is selected given that the card is white = 0.4

13. A box contains two types of balls, red and black. When a ball is picked from the box, the
7
probability that it is red is . Two balls are picked at random from the box without
12
replacement. find the probability that
5
(i) The second ball is black =
11
7
(ii) The first ball is red, given that the second one is black =
11
14. A bag contains 30 white, 20 blue and 20 red balls. Three balls are selected at random
without replacement. Find the Probability that the first ball is white and the ball is also white
= 0.18
15. A box A contains 4 white and 2 re ball. Box B contains 3 white and 2 red balls. A box is
selected at random and two balls are picked one after the other without replacement.
(i) Find the probability that the two balls picked are red. = 0.1333
(ii) Given that two white balls are picked, what is the probability that they are from box
B = 0.3333

Solutions to exercise F
1. A box contains 15 red and 5 black balls. Two balls are picked at random one after the other
without replacement. find the probability that:

14R, 5B

15R, 5B
15R, 4G

(i) both are red


15 14 21
P(both red) = P(R∩) = 𝑥 =
20 19 38
(ii) are different colour
15 5 5 15 15
P(different colour) = P(R∩B) + P(B∩R) = 𝑥 + 𝑥 =
20 19 20 19 38
20
(iii) Both are black, given that the second ball is black =
83
15 5 5 4 95
P(2nd black) = P(R∩B) + P(B∩B) = 𝑥 + 𝑥 =
20 19 20 19 380
5 4 20
P(both black) = P(B∩B) = 𝑥 =
20 19 380
20 95 20 4
P(B/2nd B) = ÷ = =
380 380 95 19

2. A box contains 4 red balls and 6 black balls two balls are randomly drawn one after the other
without replacement. Find the probability that

3R, 6B

4R, 6B
4R, 5G

(i) second ball is red given that the first ball is red
3 1
P(2nd red/1st red) = =
9 3
(ii) both balls are red
4 3 12 2
P(both red) =P(R∩R) = 𝑥 = =
10 9 90 15
(iii) both balls of different colour
4 6 6 4 8
P(both different colour) = P(R∩B) + P(B∩R) = 𝑥 + 𝑥 =
10 9 10 9 15
3. A box contains 3 black balls and 5 white balls. Two balls are randomly drawn one after the
other without replacement. find the probability

2B, 5W

3B, 5W
3B, 4W

(i) second ball is white


3 5 5 4 5
P(2nd W) = P(B∩W) + P(W∩ 𝑊) = 𝑥 + 𝑥 =
8 7 8 7 8
4
(ii) first ball is white given that the second ball is white =
7

4. A box contains 3 red sweets, 8 blue sweets and 7 green sweets. Three sweets are randomly
drawn one after the other without replacement. Find that
(i) all sweets are blue
8 7 6 7
P(all blue) = 𝑥 𝑥 =
18 17 16 102
(ii) all sweets are red
3 2 1 1
P(all red) = 𝑥 𝑥 =
18 17 16 816
(iii) one of each colour

P(each colour) = P(R∩B∩G) + P(R∩G∩B) + P(B∩R∩G) + P(B∩G∩R)+ P(G∩B∩R) + P(G∩R∩B)


3 8 7 7
=6( 𝑥 𝑥 )=
18 17 16 34

5. A box contains 7 black sweets and 3 white sweets. Three sweets are randomly drawn one
after the other with replacement. Find the probability that:
(i) all three sweets are black
P(all black) = P(B∩B∩B) = 0.7 x 0.7 x 0.7 = 0.343
(ii) a white, black and a white sweet in that order are chosen
=0.3 x 0.7 x 0.3= 0.063
(iii) two white and one black sweets are drawn
P(2W, 1B) = P(W∩W∩B) + P(W∩B∩W) + P(B∩W∩W)
= 3(0.3 x 0.3 x 0.7) = 0.189
(iv) at least one black sweet drawn
P(at least 1B) = [P(W∩W∩B) + P(W∩B∩W) + P(B∩W∩W)]+ P(W∩B∩B) +
P(B∩B∩W) + P(B∩W∩B) + P(B∩B∩B)
= 0.189 +3(0.3 x 0.7 x 0.7) + 0.7 x 0.7 x 0.7
= 0.973
6. A coin is tossed four times, find the probability of obtaining less than two heads.
P(less than 2H) = P(H∩T∩T∩T) + P(T∩H∩T∩T) + P(T∩T∩H∩T)+ P(T∩T∩T∩H) + P(T∩T∩T∩T)
1 1 1 1 5
= 5( 𝑥 𝑥 𝑥 ) =
2 2 2 2 16
7. The probability that I am late for work is 0.05. Find the probability that on two consecutive
mornings:
(i) I am late for work twice
= P(L∩L) = 0.05 x 0.05= 0.0025
(ii) I am late for work once
= P(L∩L’) + P(L’∩L) = 2(0.05 x 0.95) = 0.095
8. A box A contains 3 red balls and 4 brown balls while box B contains 3 red balls and 2 brown
balls. A box is drawn at random and one ball is randomly drawn from it. Find the probability
that;
(i) the ball is red
1 3 1 3 3 3 18
P(R) = P(A∩R) + P(B ∩ R) = 𝑥 + 𝑥 = + =
2 7 2 5 14 10 35
(ii) the ball came from box A given that it is red
P(A∩R) 3 35 5
P(A/R) = = 𝑥 =
𝑃(𝑅) 14 18 12
9. A bag contains 10 white and 6 red balls two balls are randomly drawn one after the other
without replacement. Find the probability that the second ball drawn is
(i) red given that the first one was white
6
= = 0.4
15
(ii) white
10 9 6 10
P(2nd W) = P(W∩W) + P(R∩W) = 𝑥 + 𝑥 = 0.675
16 15 16 15
10. Box P contains 2 red balls and 2 blue balls while Box Q contains 2 red balls and 3 blue balls.
A box is drawn at random and two balls are randomly drawn from it, one after the other
19
without replacement. Find the probability that the balls are of different colour. = .
30
P(different colour) = P(P∩R∩B) + P(P∩B∩R) + P(Q∩R∩B)+ P(Q∩R∩B)
1 2 2 1 2 2 1 2 3 1 3 2
= 𝑥 𝑥 + 𝑥 𝑥 + 𝑥 𝑥 + 𝑥 𝑥
2 4 3 2 3 4 2 5 4 2 5 4
1 1 3 3 10+10 +9+9 19
= + + + = =
6 6 20 20 60 30
11. A box contains 4 white balls and 1 black ball. A second box contains 1 white and 4 black
balls. A ball is drawn at random from the first bag and put into the second bag, then a ball is
taken from the second bag and put into the first bag. Find the probability that a white ball
7
will be picked when a ball is selected from the first bag =
10
Let A = 1st bag and B = 2nd bag
W1 = a white ball drawn from A for the first time
B1 = a black ball draw from A for the first time
W2 = a white ball drawn from A for the second time
B2 = a black ball draw from A for the second time
W3 = a white ball drawn from A for the second time
B3 = a black ball draw from A for the second time

A
4W,1B
B
2W,4B A
3W,2B
A
4W,1B
A
5W,0B
B
1W,5B
A
4W,1B

Let W = event that a white ball is then selected from Bag A


P(W) = P(W1∩W2∩W3) + P(W1∩B2∩W3)+ P(B1∩W2∩W3)+ P(B1∩B2∩W3)
4 2 4 4 4 3 1 1 5 1 5 4 32 48 5 20 105 7
= 𝑥 𝑥 + 𝑥 𝑥 + 𝑥 𝑥 + 𝑥 𝑥 = + + + = =
5 6 5 5 6 5 5 6 5 5 6 5 150 150 150 150 150 10

12. (a) A box contains 7 red balls and 6 blue balls. Three balls are selected at random without
replacement. find the probability that:
(i) they are the same colour
7 6 5 6 5 4
P(same colour) = P(R∩R∩R) + P(B∩B∩B) = 𝑥 𝑥 + 𝑥 𝑥 = 0.1923
13 12 11 13 12 11
(ii) at most two are blue
P(at most 2B) = P(R∩R∩R) + P(R∩R∩B)+ P(B∩R∩R) + P(R∩B∩R)+ P(R∩B∩B) +
P(B∩B∩R) + P(B∩R∩B)
6 5 4
= 1- P(B∩B∩B) = 1 - 𝑥 𝑥 = 0.9301
13 12 11

(b) Two boxes P and Q contain white and brown cards. P contains 6 white and 4 brown. Q
contains 2 white and 3 brown. A box is selected at random and a card is selected at random.
Find the probability that

(i) a brown card is selected


1 4 1 3
P(B) = P(P∩B) + P(Q∩B) = 𝑥 + 𝑥 = 0.5
2 10 2 5

(ii) Box Q is selected given that the card is white


1 6 1 2
P(W) = P(P∩W) + P(Q∩W) = 𝑥 + 𝑥 = 0.5
2 10 2 5

P(Q∩W) 0.2
P(Q/W) = = = 0.4
𝑃(𝑊) 0.5

13. A box contains two types of balls, red and black. When a ball is picked from the box, the
7
probability that it is red is . Two balls are picked at random from the box without
12
replacement. find the probability that
(i) The second ball is black
7 5
P(R) = ; P(B) =
12 12
7 5 5 4 5
P(B) = P(R∩B) + P(B∩B) = 𝑥 + 𝑥 =
12 11 12 11 11
(ii) The first ball is red given that the second one is black
P(R∩B) 35 5 35 11 7
P(1st B/B) = = ÷ = 𝑥 =
𝑃(𝐵) 121 11 121 5 11
14. A bag contains 30 white, 20 blue and 20 red balls. Three balls are selected at random
without replacement. Find the Probability that the first ball is white and the third ball is also
white
30 20 29 30 20 29 30 29 28
P = P(W∩B∩W) + P(W∩R∩W) + P(W∩W∩W) = 𝑥 𝑥 + 𝑥 𝑥 + 𝑥 𝑥
70 69 68 70 69 68 70 69 68
= 0.18
15. A box A contains 4 white and 2 re ball. Box B contains 3 white and 3 red balls. A box is
selected at random and two balls are picked one after the other without replacement.
(i) Find the probability that the two balls picked are red.
1 2 1 1 3 2 8
P(R) = P(A∩R1∩R2) + P(B∩R1∩R2) = 𝑥 𝑥 + 𝑥 𝑥 = = 0.1333
2 6 5 2 6 5 60
(ii) Given that two white balls are picked, what is the probability that they are from box
B
1 4 3 1 3 2 18
P(W) = P(A∩W1∩W2) + P(B∩W1∩W2) = 𝑥 𝑥 + 𝑥 𝑥 =
2 6 5 2 6 5 60
P(B∩W1∩W2) 6 18 6 1
P(B/W) = = ÷ = =
𝑃(𝑊) 60 60 18 3
= 0.3333

Conditional probability using a tree diagram/ Baye’s Rule


Example 32
2 1
During planting season a farmer treats of his seeds and of the seed are left untreated. The seeds
3 3
which are treated have a probability of germinating of 0.8 while the untreated seeds have a
probability of germinating of 0.5, find the probability that a seed selected at random

(i) will germinate


(ii) had been treated, given that it had germinated.

Solution

T
2 1
(i) P(G) = P(T∩ G) + P(T’∩G) = 𝑥 0.8 + 𝑥 0.5 = 0.7
3 3
2
𝑃(𝑇∩𝐺) 𝑥 0.8
3
(ii) P(T/G) = = = 0.762
𝑃(𝐺) 0.7

Example 33

The probability that a golfer hits the ball on the green if it is windy as he strikes the ball is 0.4 and
the corresponding probability if it is not windy as he strikes the ball is 0.7. The probability that the
3
wind blow as he strikes the ball is . Find the probability that
10

(i) He hits the ball on the green


(ii) It was not windy, given that he does not hit the ball on the green

T’

(i) P(S) = P(W∩S) + P(W’∩S) = 0.3 x 0.4 + 0.7 x 0.7 = 0.61


7
𝑃(𝑊 ′ ∩𝑆 ′ ) 𝑥 0.3
10
(ii) P(W’/S’) = = = 0.5385
𝑃(𝑆 ′ ) (1−0.61)

Example 33

When students were to go for a geography tour, the school hired three different types of vehicles,
buses, coaster and taxis. Of the hiring’s 40% were buses, 50% were coasters and 10% were taxis. For
bus hired 9% arrive late, the corresponding percentages for a coaster and a taxi being 6% and 20%
respectively. Find the probability that the next vehicle hired

(i) will be a bus and will not arrive late


(ii) will arrive late
(iii) will be a coaster given that it will arrive late

(i) P(B∩ 𝐿′) = 0.4 x 0.91 = 0.364


(ii) P(L) = P(B∩L) + P(C∩L) + P(T∩L)
= 0.4 x 0.09 + 0.5 x 0.06 + 0.1 x 0.2
= 0.086
𝑃(𝐶∩𝐿) (0.5 𝑥 0.06)
(iii) P(C/L) = = = 0.3488
𝑃(𝐿) 0.086
Revision exercise G
1. 55% of the teachers at a certain school are male. 30% of the male teachers are science teachers
and 5% of female teachers teach sciences. If a teacher is selected at random, what is the
probability that
(i) Teaches sciences = 0.1875
(ii) She is a female given that she does not teach sciences. = 0.5262
2. The probability that a golfer hits the ball on the green if it is windy as he strikes the ball is 0.4
and the corresponding probability if it is not windy as he strikes the ball is 0.7. The probability
3
that the wind blows as he strikes the ball is . Find the probability that:
10
(i) he hits the ball on the green= 0.61
(ii) it was not windy, given that he does not hit the ball on the green = 0.5385
3. In a restaurant, 40% of the customer’s order for chicken. If a customer orders, the probability
that he will take juice is 0.6. If he does not order chicken, the probability that he will take juice
is 0.3. Find the probability a customer picked at random will order
(i) Chicken and juice = 0.24
(ii) Juice = 0.42
4. In a factory, there are two different machines A and B. Items are produced from A and B with
respective probabilities of 0.2 and 0.8. it was established that 5% and 8%produced by A and B
respectively are defective. If one item is selected randomly, find the probability that
(i) it is defective = 0.074
(ii) it is produced by A given that t is defective = 0.1351
5. Data from electoral commission showed that in the previous electron, of all the Kampala
parliamentary contestants, 70% were N.R.M, 20% were F.D.C and 10% were independents. 5%
N.R.M contestants won elections, 95% of F.D.C contestants won elections and 25% of the
independent contestants won elections. If a contestant is chosen at random, find the
probability that the person
(i) won election = 0.25
(ii) an F. D.C won elections = 0.19
6. A student travels to school by route A or route B. The probability that she uses route A is 0.25.
2
The probability that she is late to school if she uses route A is and the corresponding
3
1
probability if she uses route B is .
3
5
(i) Find the probability that she will be late to school = .
12
3
(ii) Given that she is late, what is the probability that she used route B = .
5
7. A student is to travel to school for an interview. The probability that he will be in time for the
interview when he travels taxi and boda respectively are 0.1 and 0.2. The probability that he will
travel by taxi and boda are 0.6 and 0.4 respectively.
(i) find the probability that he will be on time = 0.14
(ii) given that he is not on time, what is the probability that he travelled by boda = 0.372

8. Of the group of students studying A-level in a school, 56% are boys and 44% are girls. The
1
probability that a boy of this group is studying chemistry is and the probability that a girl of
5
1
this group is studying chemistry is :
11
(i) Find the probability that a student selected at random from this group is a girl studying
1
chemistry. =
25
(ii) Find the probability that a student selected at random from this group is not studying
106
chemistry. =
125
(iii) Find the probability that a chemistry student selected at random from this group is
14
male =
19
9. When a school wants to buy chalk, the school phones three suppliers A, B or C of the phone
calls to them. 30% are to A, 10% to B and 60% to C. the percentage of occasions when the
supplies deliver chalk after a phone call to them are 20% for A, 6% for B and 9% for C.
(i) Find the probability that the suppler phoned will not deliver chalk on the day if
phoning.=0.88
(ii) Given that the school phones a supplier and the supplier can deliver chalk that, find the
probability that the school phoned supplier B. = 0.05
10. In Kampala city, 30% of the people are F.D.C, 50% are N.R.M and 20% are independent.
Records show that in previous elections, 65% of the F.D.C voted, 85% of the N.R.M voted and
50% of the independent voted. A person is randomly selected from the city.
(i) Find the probability that the person voted= 0.72
(ii) Given that the person didn’t vote, determine the probability that is an F.D.C = 0.375
11. A shop stocks two brands of toothpaste, Colgate toothpaste and Fresh up toothpaste, and two
sizes, large and small. Of the stock 70% is Colgate and 30% is Fresh up. Of the Colgate, 30% are
small size and of Fresh up 40% are small size. Find the probability that;
(i) A toothpaste chosen at random from the stalk will be of small size = 0.33
7
(ii) Small toothpaste chosen at random from the stalk will be of Colgate. =
11
12. At a bus park, 60% of the buses are of Teso coaches, 25% are Kakise buses and the rest are Y.Y
buses. Of the Teso coaches 50% have TVs, while for the Kakise and Y.Y buses only 55 and 15
have TVs respectively. If a bus is selected at random from the park, determine the probability
that
(i) Has a TV = 0.314
(ii) Kakise bus is selected that it has TV= 0.0398
13. On a certain day, fresh fish from lakes, Kyoga, Victoria, Albert and George were supplied to a
market in ratio 30%, 40%, 20% and 10% respectively. Each lake had an estimated ratios of
poisoned fish of 2%, 3%, 3% and 1% respectively. If a health inspector picked a fish at random
(i) What is the probability that the fish was poisoned = 0.025
(ii) Given that the fish was poisoned, what is the probability that it was from L. albert. =
0.24
14. The chance that a person picked from a Kampala street is employed is 30 in every 48. The
probability that a person is a university graduate is employed is 0.6. Find
(i) the probability that the person picked at random from the street is a university
graduate and is employed = 0.375
(ii) number of people that are not university graduates and are employed from a group of
120 people. = 30
15. A mobile phone dealer imports Nokia and Motorola phones. In a given consignment, 55% were
Nokia and 45% were Motorola phones. The probability that a Nokia phone is defective is 4%.
The probability that a Motorola phone is defective is 6%. A phone is picked at random from the
consignment. Determine that it is
(i) defective = 0.049
(ii) a Motorola given that it is defective = 0.551
Solutions to revision exercise G
1. 55% of the teachers at a certain school are male. 30% of the male teachers are science teachers
and 5% of female teachers teach sciences. If a teacher is selected at random, what is the
probability that
P(M) = 0.55, P(F) =0.45, P(S/M) = 0.3, P(S/F) =0.05, P(S’/F) =0.95
(i) Teaches sciences
P(S) = P(M∩S) + P(F∩S) = 0.55 x 0.3 + 0.45 x 0.05 = 0.1875
(ii) She is a female given that she does not teach sciences.
P(S’) = 1 – P(S) = 1- 0.1875 = 0.8125
𝑃(𝐹∩𝑆 ′ ) 0.45 𝑥 0.95
P(F/S’) = = = 0.5262
𝑃(𝑆 ′ ) 0.8125

2. The probability that a golfer hits the ball on the green if it is windy as he strikes the ball is 0.4
and the corresponding probability if it is not windy as he strikes the ball is 0.7. The probability
3
that the wind blows as he strikes the ball is . Find the probability that:
10
Summary
P(S/W) = 0.4, P(S/W’) = 0.7, P(W) = 0.3, P(W’) = 0.7
(i) he hits the ball on the green
P(S) = P(S∩W) + P(S∩W) = 0.4 x 0.3 + 0.7 x 0.7= 0.61
(ii) it was not windy, given that he does not hit the ball on the green
P(S’) = 1- 0.61 = 0.39
𝑃(𝑊 ′ ∩𝑆 ′ ) 0.7 𝑥 0.3
P(W’/S’) = = = 0.5385
𝑃(𝑆 ′ ) 0.39
3. In a restaurant, 40% of the customer’s order for chicken. If a customer orders, the probability
that he will take juice is 0.6. If he does not order chicken, the probability that he will take juice
is 0.3. Find the probability a customer picked at random will order
P(C) = 0.4, P(C’) = 0.6, P(J/C) = 0.6, P(J/C’) = 0.3
(i) Chicken and juice
P(C∩J) = 0.4 x 0.6 = 0.24
(ii) Juice
P(J) = P(C∩J) + P(C’∩J) = 0.24 + 0.6 x 0.3= 0.42
4. In a factory, there are two different machines A and B. Items are produced from A and B with
respective probabilities of 0.2 and 0.8. it was established that 5% and 8%produced by A and B
respectively are defective. If one item is selected randomly, find the probability that
Summary
P(A) = 0.2, P(B) = 0.8, (D/A) = 0.05, P(D/B) 0.08
(i) it is defective
P(D) = P(A∩D) + P(B∩B) = 0.2 x 0.05 + 0.8 x 0.08 =0.074
(ii) it is produced by A given that t is defective
𝑃(𝐴∩𝐷) 0.2 𝑥 0.05
P(A/D) = = = 0.1351
𝑃(𝐷) 0.074
5. Data from electoral commission showed that in the previous electron, of all the Kampala
parliamentary contestants, 70% were N.R.M, 20% were F.D.C and 10% were independents. 5%
N.R.M contestants won elections, 95% of F.D.C contestants won elections and 25% of the
independent contestants won elections. If a contestant is chosen at random, find the
probability that the person
Summary
P(N) = 0.7, P(F) = 0.2, P(I) = 0.1, P(W/N) = 0.05, P(W/F) = 0.95, P(W/I) = 0.25
(i) won election
P(W) = P(N∩W) + P(F∩W) + P(I∩W) = 0.7 x 0.05 + 0.2 x 0.95 + 0.1 x 0.25= 0.25
(ii) an F. D.C won elections = 0.2 x 0.95 = 0.19

6. A student travels to school by route A or route B. The probability that she uses route A is 0.25.
2
The probability that she is late to school if she uses route A is and the corresponding
3
1
probability if she uses route B is .
3
2 1
P(A) = 0.25, P(B) = 0.75, P(L/A) = , P(L/B) =
3 3
(i) Find the probability that she will be late to school.
2 1
P(L) = P(A∩L) + P(B∩L) = 0.25 x + 0.75 x = 0.417
3 3
(ii) Given that she is late, what is the probability that she used route B
P(B∩L) 0.25
P(L/B) = = = 0.6
𝑃(𝐿) 0.417
7. A student is to travel to school for an interview. The probability that he will be on time for the
interview when he travels taxi and boda respectively are 0.1 and 0.2. The probability that he will
travel by taxi and boda are 0.6 and 0.4 respectively.
Summary
P(T) = 0.6, P(B) = 0.4, P(t/T) = 0.1, P(t’/T) = 0.9, P(t/B) = 0.2, P(t’/B) = 0.8
(i) find the probability that he will be on time
P(on time) = P(t) = P(T∩t) + P(B∩t) = 0.6 x 0.1 + 0.4 x 0.2= 0.14
(ii) given that he is not on time, what is the probability that he travelled by boda
Probability that he s not on time = 1- P(t) = 1 - 0.14 = 0.86
𝑃(𝑡 ′ ∩𝐵) 0.4 𝑥 0.8
P(B/t’) = = = 0.372
𝑃(𝑡 ′ ) 0.86

8. Of the group of students studying A-level in a school, 56% are boys and 44% are girls. The
1
probability that a boy of this group is studying chemistry is and the probability that a girl of
5
1
this group is studying chemistry is :
11

Summary
1
P(B) = 0.56, P(G) = 0.44, P(C/B) = 0.2, P(C/G) =
11

(i) Find the probability that a student selected at random from this group is a girl studying
chemistry.
1
P(G∩C) = P(G)x P(G/C) = 0.44 x = 0.04
11
(ii) Find the probability that a student selected at random from this group is not studying
chemistry.
P(C’) = 1- P(C) = 1 – 0.152= 0.848

(iii) Find the probability that a chemistry student selected at random from this group is
14
male =
19
P(B∩C) 0.56 𝑥 0.2
P(B/C) = = = 0.73684
𝑃(𝐶) 0.152

9. When a school wants to buy chalk, the school phones three suppliers A, B or C of the phone
calls to them. 30% are to A, 10% to B and 60% to C. the percentage of occasions when the
supplies deliver chalk after a phone call to them are 20% for A, 6% for B and 9% for C.
Summary

P(A) = 0.3, P(B) = 0.1, P(C) = 0.6, P(D/A) = 0.2, P(D’/A) = 0.8, P(D/B) = 0.06, P(D’/B) = 0.94,

P(D/C) = 0.09, P(D’/C) = 0.91

(i) Find the probability that the suppler phoned will not deliver chalk on the day if phoning.
P(D’) = P(D’∩A) + P(D’∩B) + P(D’∩C) = 0.3 x 0.8 + 0.1 x 0.94 + 0.6 x 0.91 =0.88
(ii) Given that the school phones a supplier and the supplier can deliver chalk that, find the
probability that the school phoned supplier B.
P(D) = P(D∩A) + P(D∩B) + P(D∩C) = 0.3 x 0.2 + 0.1 x 0.06 + 0.6 x 0.09 = 0.12
Or = 1- P(D’) = 1- 0.88 = 0.12
P(D∩B) 0.1 𝑥 0.06
P(B/D) = = = 0.05
𝑃(𝐷) 0.12

10. In Kampala city, 30% of the people are F.D.C, 50% are N.R.M and 20% are independent.
Records show that in previous elections, 65% of the F.D.C voted, 85% of the N.R.M voted and
50% of the independent voted. A person is randomly selected from the city.
Summary
P(F) = 0.3, P(N) = 0.5, P(I) = 0.2, P(V/F) = 0.65, P(V/N) = 0.85, P(V/I) =0.5
(i) Find the probability that the person voted
P(V) = P(V∩F) + P(V∩N) + P(V∩I) = 0.3 x 0.65 + 0.5 x 0.85 + 0.2 x 0.5 =0.72
(ii) Given that the person didn’t vote, determine the probability that is an F.D.C
P(V’) = P(V’∩F) + P(V’∩N) + P(V’∩I) = 0.3 x 0.35 + 0.5 x 0.15 + 0.2 x 0.5= 0.28
P(V’∩F) 0.3 𝑥 0.35
P(v’/F) = = = 0.375
𝑃(𝑉 ′ ) 0.28
11. A shop stocks two brands of toothpaste, Colgate toothpaste and Fresh up toothpaste, and two
sizes, large and small. Of the stock 70% is Colgate and 30% is Fresh up. Of the Colgate, 30% are
small size and of Fresh up 40% are small size. Find the probability that;
Summary
P(C) = 0.7 P(F) = 0.3, P(s/C) = 0.3, P(s/F) = 0.4
(i) A toothpaste chosen at random from the stalk will be of small size
P(s) = P(C∩s) + P(F∩s) = 0.7 x 0.3 + 0.3 x 0.4 = 0.33
(ii) Small toothpaste chosen at random from the stalk will be of Colgate.
𝑃(𝐶∩𝑠) 0.21 7
P(C/s) = = =
𝑃(𝑠) 0.33 11
12. At a bus park, 60% of the buses are of Teso coaches, 25% are Kakise buses and the rest are Y.Y
buses. Of the Teso coaches 50% have TVs, while for the Kakise and Y.Y buses only 55 and 15
have TVs respectively. If a bus is selected at random from the park, determine the probability
that:

Let T, K, Y stand for Teso, Kakise and Y, Y buses

P(T) = 0.6 P(K) = 0.25 and P(Y) = 0.15

P(R/T) = 0.5, P(R/K) = 0.05, P(R/Y) = 0.01

(i) Has a TV = 0.0315


P(t) = P(T∩t) + P(K∩t) + P(Y∩t)
= P(R/T).P(T) + P(R/K).P(K) . P(R/Y).P(Y)
= 0.6 x 0.5 + 0.25 x 0.05 + 0.15 x 0.01
= 0.314
(ii) Kakise bus is selected that it has TV
𝑃(𝐾∩𝑇) 0.0125
P(K/T) = = = 0.0398
𝑃(𝑇) 0.314
13. On a certain day, fresh fish from lakes, Kyoga, Victoria, Albert and George were supplied to a
market in ratio 30%, 40%, 20% and 10% respectively. Each lake had an estimated ratios of
poisoned fish of 2%, 3%, 3% and 1% respectively. If a health inspector picked a fish at random

(i) What is the probability that the fish was poisoned

P(poisoned) = P(K∩P) + P(V∩P) + P(A∩P) + P(G∩P)

= 0.3 x 0.02 + 0.4 x 0.03 + 0.2 x 0.03 + 0.1 x 0.01

= 0.025
(ii) Given that the fish was poisoned, what is the probability that it was from L. albert. =
0.24
P(A∩P P(A∩P) 0.006
P(A/P) = = = = 0.24
𝑃(𝑃) 𝑃(𝑃) 0.025

14. The chance that a person picked from a Kampala street is employed is 30 in every 48. The
probability that a person is a university graduate is employed is 0.6. Find
30
P(E) = = 0.625 and P(G/E) = 0.6
48
(i) the probability that the person picked at random from the street is a university
graduate and is employed
𝑃(𝐺∩𝐸)
P(G/E) =
𝑃(𝐸)
P(G∩E) = 0.625 x 0.6= 0.375
(ii) number of people that are not university graduates and are employed from a group of
120 people. = 30
From set theory
P(E) = P(G∩E) + P(G’∩E)
P(G’∩E) = 0.625 – 0.375 = 0.25
The number = 0.25 x 120 = 30
15. A mobile phone dealer imports Nokia and Motorola phones. In a given consignment, 55% were
Nokia and 45% were Motorola phones. The probability that a Nokia phone is defective is 4%.
The probability that a Motorola phone is defective is 6%. A phone is picked at random from the
consignment. Determine that it is

(i) defective = P(N∩D) + P(M∩D) = 0.55 x 0.04 + 0.45 x 0.06 = 0.049


(ii) a Motorola given that it is defective
𝑃(𝑀∩𝐷) 0.45 𝑥 0.06
P(M/D) = = = 0.551
𝑃(𝐷) 0.049

Thank you
Dr. Bbosa Science
Discrete probability distribution
A probability density function (p.d.f) if it takes on specific values

Properties of discrete probability density functions

(i) ∑ 𝑃(𝑋 = 𝑥) = 1 𝑜𝑟 ∑ 𝑓(𝑥) = 1


(ii) P(X=x)≥0

Examples 1
2
A discrete random variable has a probability function 𝑃(𝑋 = 𝑥) = {𝑐𝑥 𝑥 = 0, 1, 2, 3, 4
0 𝑜𝑡ℎ𝑒𝑟𝑤𝑖𝑠𝑒
Find the value of c and draw the graph of P(X = x)

Solution

∑ 𝑃(𝑋 = 𝑥) = 1

c(02) + c(12) + c(22) + c(32) + c(42) = 1

c + 4c + 9c + 16c = 2
1
c=
30

Example 2

A discrete random variable has probability function


𝑘𝑥, 𝑥 = 1, 2,3,4
𝑓(𝑥) = { , find the value of k and draw the graph of f(x)
0, 𝑜𝑡ℎ𝑒𝑟𝑤𝑖𝑠𝑒
Solution

∑ 𝑓(𝑥) = 1

k + 2k + 3k + 4k = 1
1
k=
10

Example 3

A random variable X of a discrete probability distribution given by

P(X=1) = 0.2, P(X=2) = P(X=3) = 0.1, P(X=4) = P(X=5) = c

Find the value of the constant c and draw the graph of P(X = x)
Solution

∑ 𝑃(𝑋 = 𝑥) = 1

0.2 + 0.1 + 0.1 + c + c = 2; c = 0.3

Example 4

A discrete random variable has a probability function


2 𝑥
𝑘 ( ) , 𝑥 = 1, 2,3, …
𝑃(𝑋 = 𝑥) = { 3
0, 𝑜𝑡ℎ𝑒𝑟𝑤𝑖𝑠𝑒
Find the value of k

Solution
2 0 2 1 2 2 2 3
𝑘( ) + 𝑘( ) + 𝑘( ) + 𝑘( ) + ⋯.= 1
3 3 3 3

2 1 2 2 2 3
𝑘 (1 + ( ) + ( ) + ( ) + ⋯ . ) = 1
3 3 3

𝑎
Sum to infinity = 𝑆∞ =
1−𝑟

1 1
 𝑘( 2 ) = 1; k =
1− 3
3

Finding probabilities
Example 5

A discrete random variable has a probability distribution

y -3 -2 -1 0 1
P(Y=y) 0.1 0.25 0.3 0.15 a
Find

(i) value of a (ii) P(-3≤ 𝑌 < 0) (iii) P(Y>-1) (iv) P(-1<Y<1) (v) mode

Solution

(i) ∑ 𝑃(𝑌 = 𝑦) = 1

0.1 + 0.25 + 0.3 + 0.15 + a = 1; a = 0.2

(ii) P(-3≤ 𝑌 < 0) = P(Y= -3) + P(Y = -2) + P(Y = -1) = 0.1 + 0.25 + 0.3 = 0.65

(iii) P(Y> -1) = P(Y = 0) + P(Y = 1) = 0.15 + 0.2 = 0.35

(iv) P(-1<Y<1) = P(Y = 0) = 0.15

(v) mode is the value y with the highest probability, mode = -1


Example 6

A discrete random variable X has a probability distribution

X 1 2 3 4 5
P(X = x) 0.15 0.20 0.15 c 0.1
Find
𝑋>2
(i) the value of x (ii) P(X<4) (iii) P(X≤4 (iv) P(2≤ 𝑋 ≤ 4) (v) P( ) (vi) mode
𝑋≤4

Solution

(i) ∑ 𝑃(𝑋 = 𝑥) = 1

0.15 + 0.20 + 0.15 + c + 0.1 = 1; c = 0.4

(ii) P(X<4) = P(X=1) + P(X= 2) + P(X = 3) = 0.15 + 0.20 + 0.15 = 0.5

(iii) P(X<4) = P(X=1) + P(X= 2) + P(X = 3) + P(X= 4) = 0.15 + 0.20 + 0.15 + 0.4 = 0.9

(iv) P(2≤ 𝑋 ≤ 4) = P(X = 2) + P(X = 3) + P(X = 4) = 0.20 + 0.15 + 0.4 = 0.75


𝑃(𝑋>2, 𝑋≤4) 𝑃(𝑋=3)+𝑃(𝑋=4) 0.15 + 0.4
(v) P( 𝑋 > 2⁄𝑋 ≤ 4) = = = = 0.6111
𝑃(𝑋≤4) P(X=1) + P(X= 2) + P(X = 3) + P(X= 4) 0.9

(vi) the mode is a value with highest probability = 4

Example 7

A discrete random variable X has a probability function


𝑘𝑥, 𝑥 = 1, 2,3,4,5
𝑓(𝑥) = {
0, 𝑜𝑡ℎ𝑒𝑟𝑤𝑖𝑠𝑒
𝑋≥1
Find (i) the value of k (ii) P(X = 3) (iii) P(X≥3) (iv) P(X≤ 3) (v) P(1< 𝑋 ≤3) (vi) P( )
𝑋<4

Solution

(i) ∑ 𝑃(𝑋 = 𝑥) = 1
1
k + 2k + 3k + 4k + 5k = 1; k =
15
3 1
(ii) P(X=3) = 3k = =
15 5
12 4
(iii) P(X≥3)= P(X=3) + P(X=4) + P(X = 5) = 3k + 4k + 5k = 12k = =
15 5
6 2
(iv) P(X≤ 3)= P(X= 1) + P(X=2) + P(X = 3) = k + 2k + 3k = =
15 5
5 1
(v) P(1< 𝑋 ≤3) = P(X=2) + P(X = 3) =2k + 3k = =
15 3

(vi) P(𝑋 ≥ 2⁄𝑋 < 4) =


𝑃(𝑋≥2,𝑋<4) 𝑃(𝑋=2)+𝑃(𝑋=3) 2𝑘+3𝑘 5𝑘 5
= = = =
𝑃(𝑋<4) 𝑃(𝑋=1)+𝑃(𝑋=2)+𝑃(𝑋=3) 𝑘+2𝑘+3𝑘 6𝑘 6
Revision exercise 1
1. A discrete random variable X has probability distribution
x 1 2 3 4 5
P(X=x) 0.2 0.25 0.4 a 0.05
Find (i) value of a = 0.1 (ii) P(1≤ 𝑥 ≤ 3)=0.85 (iii) P(X>2) =0.55 (iv) P(2<X<5) = 0.5 (v) mode =3
2. A random variable x of a discrete pdf is given by P(X x) = kx, x = 12, 13, 14
Write the probability distribution and find the value of k
x 12 13 14 1
k=
39
P(X=x) 12k 13k 14k

3. A random variable Y of discrete probability distribution is given by


P(Y = 3) = 0.1, P(Y = 5) = 0.05, P(Y=6) = 0.45 P(Y = 8) = 3P(Y = 10). Find P(Y=10) = 0.1
4. A discrete random variable has a distribution
x 1 2 3 4 5
P(X=x) 0.1 0.3 k 0.2 0.05
Find
7 13
(i) value of k = (ii) P(X≥4) = 0.25 (iii) P(X<1)=0 (iv) P(2≤ 𝑥 < 4) =
20 20
5. Write out the probability distribution for each of these variables
(a) The number of heads X obtained when two fair coins are tossed
x 0 1 2
P(X=x) 0.25 0.5 0.25

(b) The number of tails, X obtained when three fair coins are tossed.
x 0 1 2 3
P(X=x) 0.125 0.375 0.375 0.125

6. A drawer contains 8 brown socks and 4 blue shocks. A sock is taken from the drawer at random,
its colour is noted and it is then replaced. The procedure is performed twice more. X is the
random variable for the number of brown socks taken. Find the probability distribution for X.
x 0 1 2 3
P(X=x) 1 2 4 8
27 9 9 27

7. The discrete random variable R has a p.d.f is given by P(R=r) = c(3 – r), r = 0, 1, 2, 3
1
Find (i) value of c = (ii) P(1≤R<3) = 0.5
6
8. A discrete random variable has probability function
4 𝑥
𝑘 ( ) , 𝑥 = 1, 2,3, … .
𝑃(𝑋 = 𝑥) = { 5 , find the value k = 0.2.
0, 𝑜𝑡ℎ𝑒𝑟𝑤𝑖𝑠𝑒

Solutions to revision exercise 1


5 Write out the probability distribution for each of these variables
(a) The number of heads X obtained when two fair coins are tossed
S =(TT, TH, HT, HH)
1 2 1
P(X= 0) = = 0.25, P(X= 1) = = 0.50, P(X= 2) = = 0.25
4 4 4
Probability distribution table
x 0 1 2
P(X=x) 0.25 0.5 0.25

(b) The number of tails, X obtained when three fair coins are tossed.

S = (TTT, TTH, THT, HTH, THH, HTH, HHT, HHH)

Probability distribution table

number of heads, x 0 1 2 3
P(X=x) 1 3 3 1
= 0.125 = 0.375 =0.375 = 0.125
8 8 8 8
6. A drawer contains 8 brown socks and 4 blue shocks. A sock is taken from the drawer at random,
its colour is noted and it is then replaced. The procedure is performed twice more. X is the
random variable for the number of brown socks taken. Find the probability distribution for X.
Let X’ represent blue shocks
4 4 4 1
P(X = 0) = P(X’∩X’∩X’) = 𝑥 𝑥 =
12 12 12 27
8 4 4 4 8 4 4 4 8 2
P(X=1) = P(X∩X’∩X’)+ P(X’∩X∩X’) + P(X’∩X’∩X) = 𝑥 𝑥 + 𝑥 𝑥 + 𝑥 𝑥 =
12 12 12 12 12 12 12 12 12 9
8 8 4 4 8 8 8 4 8 4
P(X=2) = P(X∩X∩X’)+ P(X’∩X∩X) + P(X∩X’∩X) = 𝑥 𝑥 + 𝑥 𝑥 + 𝑥 𝑥 =
12 12 12 12 12 12 12 12 12 9
8 8 8 8
P(X = 3)= P(X∩X∩X)= 𝑥 𝑥 =
12 12 12 27
Probability distribution table
x 0 1 2 3
P(X=x) 1 2 4 8
27 9 9 27
7. The discrete random variable R has a p.d.f is given by P(R=r) = c(3 – r), r = 0, 1, 2, 3

Find (i) value of c


∑ 𝑃(𝑋 = 𝑥) = 1
3c +2c + c = 3
1
c=
6
1
(ii) P(1≤R<3) = 2c + c = 3c = 3 x =0.5
6
8. A discrete random variable has probability function
4 𝑥
𝑘 ( ) , 𝑥 = 1, 2,3, … .
𝑃(𝑋 = 𝑥) = { 5 , find the value k.
0, 𝑜𝑡ℎ𝑒𝑟𝑤𝑖𝑠𝑒
Solution
4 0 4 1 4 2 4 3
𝑘( ) + 𝑘( ) + 𝑘( ) + 𝑘( ) + ⋯.= 1
5 5 5 5

4 1 4 2 4 3
𝑘 (1 + ( ) + ( ) + ( ) + ⋯ . ) = 1
5 5 5

𝑎
Sum to infinity = 𝑆∞ =
1−𝑟

1 1
 𝑘( 4 ) = 1; k = =0.2
1− 5
5
Expectation of x, E(x) or mean
The expected value of x is given by E(x) = ∑ 𝑥𝑃(𝑋 = 𝑥)

Example 8

A discrete random variable has a probability distribution

x -2 -1 0 1 2
P(X = x) 0.3 0.1 0.15 0.4 0.05
Find expectation, E(x)

Solution

E(X) = (-2 x 0.3) + (-1 x 0.1) + (0 x 0.15) + (1 x 0.4) + (2 x 0.05) = -0.2

Example 9

The discrete random variable Y has a probability distribution is given by

P(Y = y) = cy, y = 1 2,3,

P(Y = y) = c(8-y), y = 4, 5, 6, 7

Find (i) the value of c (ii) mean, μ

Solution

y 1 2 3 4 5 6 7
P(Y = y) c 2c 3c 4c 3c 2c c
(i) ∑ 𝑃(𝑌 = 𝑦) = 1
c + 2c + 3c + 4c + 3c +2c + c = 1
1
c=
16
(ii) E(Y) = ∑ 𝑦(𝑌 = 𝑦) = 1 x c + 2 x 2c + 3 x 3c + 4 x 4c + 5 x 3c + 6 x 2c + 7x c = 64c
1
= 64 x =4
16

Example 10

A fair coin is tossed three times write out the probability distribution for the number of heads, X,
obtained and hence obtain the expected number of heads

Solution

S = (TTT, TTH, THT, HTH, THH, HTH, HHT, HHH)

Probability distribution table

number of heads, x 0 1 2 3
P(X=x) 1 3 3 1
8 8 8 8
1 3 3 1 12
E(X) = ∑ 𝑥(𝑋 = 𝑥) = (0 x ) + 1𝑥 +2𝑥 +3𝑥 = = 1.5
8 8 8 8 8

Example 11

A family plans to have 4children. Given that X is the number of girls in the family. Find the expected
number of girls
Solution

S = (BBBB, BBBG, BBGB, BGBB, GBBB, BBGG, BGGB, BGBG, GGBB, BGGB, GBBG, BGGG, GBGG, GGBG,

GGGB GGGG)

Probability distribution table

Number of girls, x 0 1 2 3 4
P(X=x) 1 4 6 4 1
16 16 16 16 16
1 4 6 4 1
E(X) = ∑ 𝑥(𝑋 = 𝑥) = (0 𝑥 +1𝑥 +2𝑥 +3𝑥 +4𝑥 )=2
16 16 16 16 16

Example 12

A box A contains 4 red sweets and 3 green sweets. Box B contains 5 red sweets and 6 green sweets.
Box A is twice more likely to be picked as Box B. If a box is chosen at random and two sweets are
removed from it, one at a time without replacement.

(a) Find the probability that two sweets removed are of the same colour.
P(same colour) = P(A∩R1∩R2) + P(A∩G1∩G2) + P(B∩R1∩R2) + P(B∩B1∩B2)
2 4 3 2 3 2 1 5 4 1 6 5
= 𝑥 𝑥 + 𝑥 𝑥 + 𝑥 𝑥 + 𝑥 𝑥
3 7 6 3 7 6 3 11 10 3 11 10
24 12 20 30 42 50
= + + + = + = 0.4372
126 126 330 330 126 330
(b) (i) construct a probability distribution table for the number of red sweets removed
Let x = number of red sweets removed
2 3 2 1 6 5
P(X =0) = P(A∩G1∩G2) + P(B∩G1∩G2) = 𝑥 𝑥 + 𝑥 𝑥 = 0.1861
3 7 6 3 11 10
P(X=1) = P(A∩R1∩G2) + P(A∩G1∩R2) + P(B∩R1∩G2) + P(B∩G1∩R2)
2 4 3 2 3 4 1 5 6 1 6 5
= 𝑥 𝑥 + 𝑥 𝑥 + 𝑥 𝑥 + 𝑥 𝑥
3 7 6 3 7 6 3 11 10 3 11 10
24 24 30 30 48 60
= + + + = +
126 126 330 330 126 330
= 0.5628
2 4 3 1 5 4 24 20
P(X=2) = P(A∩R1∩R2)+ P(B∩R1∩R2) = 𝑥 𝑥 + 𝑥 𝑥 = + = 0.2511
3 7 6 3 11 10 126 330

Probability distribution table


x 0 1 2
P(X= x) 0.1861 0.5628 0.2511

(ii) find the mean number of red sweets removed


Mean = ∑ 𝑥(𝑋 = 𝑥) = 0 x0.1861 + 1 x 0.5628 + 2 x 0.2511 = 1.065

Revision exercise 2
1. A discrete random variable X has a probability distribution.

x 0 1 2 3 4
P(X= x) 1 1 1 1 1
6 12 4 3 6
Find E(X) = 2.25

2. A discrete random variable X ha probability distribution


x 5 6 7 8 9
P(X=x) 3 2 1 2 3
11 11 11 11 11
Find the mean = 7
3. A discrete random variable X has a probability distribution

4. A discrete random variable has a probability distribution


x 0 1 2 3
P(X=x) c c2 c2+c 3c2 + 2c
Find (i) the value of c= 0.2 (ii) expectation of c=2.08
5. Find the expected number of heads when two fair coins are tossed (E(x) 1)
6. A family plans to have 3 children. Given that x is the number of boys in the family. Find
the expected number of boys (=1.5)
7. If X is a random variable for the product of the scores on two tetrahedral dice, where the
score is the number on which the die lands, find the expected score for the throw
(=6.25)
8. A bag contains 5 black counters and 6 red counters. Two counters are drawn at random,
12
one at a time without replacement. Find the expected number of red counters. ( = )
11
9. An unbiased tetrahedral die is tossed once. If it lands on a face marked 1, the player has
to pay 10,000/=. If it lands on marked with 2 or 4 the player wins 5000/= and if it lands
on a 3, the player wins 3000/=. Find the expected gain in one throw.
10. A discrete random variable X can take on values 10 and 20 only. If E(X) = 16. Write out
the probability distribution for X (P(X=x) = 0.4 and P(X=x) = 0.6)
11. A discrete random variable X can take on values 0, 1, 2, and 3 only. If E(X) = 1.4, P(X≤ 2)
= 0.9 and P(X≤1) = 0.5. Find (i) P(X=1) = 0.3 (ii) P(X =0) = 0.2
12. The discrete random variable Y has a probability distribution is given by
P(Y=y) = cy, y = 1, 2, 3, 4
Find (i) value of c = 0.1 (ii) E(X) = 3
13. A discrete random variable has p.d.f
𝑘2𝑥 , 𝑥 = 0, 1, 2, 3, 4, 5, 6
𝑃(𝑋 = 𝑥) = { ,
0, 𝑜𝑡ℎ𝑒𝑟𝑤𝑖𝑠𝑒
1
Find (i) value of k = , (ii) mean = 5
127
14. A discrete random variable X has a probability distribution
x 0 1 2 3 4 5
P(X= x) 0.11 0.17 0.2 0.13 p 0.09
Find (i) the value of p = 0.3 (ii) Expected value of X (= 2.6)

Solutions to revision exercise 2

10. A discrete random variable X can take on values 10 and 20 only. If E(X) = 16. Write out the
probability distribution for X
Let P(X=10) = a and P(X=20) = b
a+b = 1
a = (1-b)………………. (i)
10a + 20b = 16 ……. (ii)
Eqn. (i) and eqn. (ii)
10(1-b) + 20b = 16
10 + 20b = 16
b = 0.6
a = 1- 0.6 = 0.4

Probability distribution:(P(X=x) = 0.4 and P(X=x) = 0.6)


11. A discrete random variable X can take on values 0, 1, 2, and 3 only. If E(X) = 1.4, P(X≤ 2) = 0.9
and P(X≤1) = 0.5. Find (i) P(X=1) (ii) P(X =0)
Let P(X = 0) = a, P(X=1)=b, P(X=2)= c p(X = 3) = d
a + b + c + d = 1 ……………. (i)
P(X≤ 2) = a + b+ c = 0.9 … (ii)
Eqn. (i) and eqn. (ii)
d = 0.1
P(X≤1) = a + b = 0.5 ……. (iii)
Eqn. (i) and eqn. (iii)
0.5 + c + 0.1 = 1
c = 0.4
E(X) = 0 x a + 1 x b + 2 x c + 3 x 0.1 = 1.4
= b + 2c + 0.3 = 1.4
b + 2c = 1.1
b + 2 x 0.4 = 1.1
b = 0.3
a = 0.5 – 0.3 = 0.2
Hence, (i) P(X=1) = 0.3 (ii) P(X =0) = 0.2
12. The discrete random variable Y has a probability distribution is given by
P(Y=y) = cy, y = 1, 2, 3, 4
11
Find (i) value of c = 0.1 (ii) E(X) =
3
(i) ∑ 𝑃(𝑋 = 𝑥) = 1

c + 2c + 3c + 4c = 1

10c = 1; c= 0.1

(ii) E(X) =∑ 𝑥𝑃(𝑋 = 𝑥)= 1 x 0.1 + 2 x 0.2 + 3 x 0.3 + 4 x 0.4 =3

13. A discrete random variable has p.d.f


𝑘2𝑥 , 𝑥 = 0,1, 2, 3, 4, 5, 6
𝑃(𝑋 = 𝑥) = { ,
0, 𝑜𝑡ℎ𝑒𝑟𝑤𝑖𝑠𝑒
Find
1
(i) (i) value of k = ,
127
∑ 𝑃(𝑋 = 𝑥) = 1
k(20 + 21 + 22 + 23 + 24 + 25 +26) = 1
1
k=
127
1
(ii) Mean = ∑ 𝑃𝑥(𝑋 = 𝑥) = (0 x 1 + 2 x 2 + 3 x 8 + 4 x 16 + 4 x 32 + 6 x 64) = 5.01
127
14. A discrete random variable X has a probability distribution
x 0 1 2 3 4 5
P(X= x) 0.11 0.17 0.2 0.13 p 0.09
Find
(i) the value of p
∑ 𝑃(𝑋 = 𝑥) = 1
0.11 + 0.17 + 0.2 + 0.13 + p + 0.09 = 1
p = 0.3
(ii) Expected value of X
E(X) =∑ 𝑃𝑥(𝑋 = 𝑥) = 0.11x0 + 0.17x1 + 0.2x2 + 0.13x3 + 0.3 x 4 + 0.09x5= 2.61
Properties of the mean

(i) E(a) = a
(ii) E(ax) = aE(x)
(iii) E(ax + b) = aE(x) + b
(iv) E(ax - b) = aE(x) - b

Example 13

A random variable X of discrete probability distribution is given by

x 1 2 3 4
P(X=x) 0.1 0.2 0.3 0.4
Find

(i) E(x) = ∑ 𝑃𝑥(𝑋 = 𝑥) = 1 x 0.1 + 2 x 0.2 + 3 x 0.3 + 4 x 0.4 = 3


(ii) E(3x) = 3E(x) = 3 x 3 = 9
(iii) E(4x + 6) =4E(x) + 6 = 4 x 3 + 6 = 18

Example 14

A random variable X of discrete probability distribution is given by

x -1 0 1 2
P(X= x) 0.25 0.10 0.45 0.20
Find

(i) P(-1≤ X <1) = P(X= -1) + p(X = 0) = 0.25 + 0.10 = 0.35


(ii) E(X) =∑ 𝑃𝑥(𝑋 = 𝑥) = -1 x 0.25 + 0 x 0.10 + 1 x 0.45 + 2 x 0.20 = 0.6
(iii) E(6x – 2) = 6E(X) – 2 = 0.6 x 6 – 2 = 1.6

Variance, Var(x)
Var(x) = E(X2) = [E(x)]2 where E(X2) = ∑ 𝑥 2 𝑃(𝑋 = 𝑥)

Example 15

A discrete random variable X has a probability distribution

x 1 2 3 4 5
P(X=x) 0.1 0.3 0.2 0.3 0.1
Find

(i) The mean = ∑ 𝑃𝑥(𝑋 = 𝑥) = 1 x 0.1 + 2 x 0.3 + 3 x 0.2 + 4 x 0.3 + 5 x 0.1= 3


(ii) Var(x)
E(X2) =∑ 𝑃𝑥 2 (𝑋 = 𝑥) = 12 x 0.1 + 22 x 0.3 + 32 x 0.3 + 42 x 0.3 + 52 x 0.1 = 10.4
Var(X) = 10.4 – (3)2 = 1.4

Example 16

The discrete random variable Y has a probability distribution is given by P(Y =y), y= -3, -2, -1, 0, 1, 2,3

Find: (i) value of c (ii) mean (iii) standard deviation


Solution

y -3 -2 -1 0 1 2 3
P(Y= y) 3c 2c c 0 c 2c 3c
(i) ∑ 𝑃(𝑋 = 𝑥) = 1
1
3c +2c + c+ c + 2c + 3c = 1; c =
12
(ii) Mean = ∑ 𝑃𝑥(𝑋 = 𝑥) = -3 x 3c + -2 x 2c + -1 x c + 0 x 0 + 1 x c + 2 x 2c + 3 x 3c = 0
1
(iii) E(X2) = (-3)2 x 3c + (-2)2 x 2c + (-1)2 x c + (0)2 x 0 + (1)2 x c + (2)2 x 2c +(3)2 x 3c = 72 x =6
12
Var (x) = E(X2) – (E(x))2 = 6 – (0)2 = 6
S.D = √𝑉𝑎𝑟 (𝑋) = √6 = 2.45

Example 17

Two marbles are drawn without replacement from a box containing 3 red marbles and 4 white
marbles. The marbles are randomly drawn. If X is the random variable for the number of red marble
drawn find

(i) Expected number of red marbles


4 3 2
P(X= 0) = P(W∩W) = 𝑥 =
7 6 7
4 3 3 4 4
P(X= 1) = P(W∩R) + P(R∩W) = 𝑥 + 𝑥 =
7 6 7 6 7
3 2 1
P(X= 2) = P(R∩R) = 𝑥 =
7 6 7
The probability distribution table
x 0 1 2
P(X=x) 2 4 1
7 7 7
2 4 1 6
E(x) = ∑ 𝑃𝑥(𝑋 = 𝑥) = 𝑥0+ 𝑥1+ 𝑥2=
7 7 7 7
(ii) Standard deviation of X
2 4 1 8
E(x2) = ∑ 𝑃𝑥 2 (𝑋 = 𝑥) == 𝑥0+ 𝑥 12 + 𝑥 22 =
7 7 7 7
8 6 2 20
Var (x) = E(x2) – (E(X))2 = − ( ) =
7 7 49
20
S.D = √𝑉𝑎𝑟 (𝑋) = √ = 0.6389
49

Example 19

A vendor stocks 12 copies of a magazine each week and the probability for each possible total
number of copies sold is shown below

Number of copies 9 10 11 12
probability 0.2 0.35 0.30 0.15
(a) Estimate the mean and variance of the number of copies
Mean = ∑ 𝑃𝑥(𝑋 = 𝑥) = 9 x 0.2 + 10 x 0.35 + 11 x 0.3 + 12 x 0.15 = 10.4
E(X2) = 92 x 0.2 + 102 x 0.35 + 112 x 0.3 + 122 x 0.15 =109.1
Var(x) = 109.1 – (10.4)2 = 0.94
(b) The vendor buys the magazine at 8,500/= and sells at 14,500/=. Any copies not sold are
destroyed. Construct a probability distribution table for vendor’s weekly profit and hence
find the expected weekly profit
Profit = S.P – C.P
Profit for 9 copies = 9 x 14,500 – 12 x 8500 = 28500
Profit for 10 copies= 10 x 14,500 – 12 x 8500 =43000
Profit for 11 copies= 11 x 14,500 – 12 x 8500 = 57500
Profit for 12 copies= 12 x 14,500 – 12 x 8500 = 72000

y 28500 43000 57500 72000


P(Y=y) 0.2 0.35 0.30 0.15
E(Y) = 0.2 x 28500 + 0.35 x 43000 + 0.30 x 57500 + 0.15 x 72000 = 48000/=

Example 20

The table below shows the number of red and green balls put in three identical boxes A, B and C.

Boxes A B C
Red balls 4 6 3
Green balls 2 7 5
A box is chosen at random and two balls are then drawn from it successively without
replacement. If the random variable X is “the number of green balls drawn”.
(a) Draw a probability distribution table for X (06marks)
Using combination
4 6 3
1 𝐶2 𝐶2 𝐶2
P(X = 0) = [ 6 + 13 + 8 ]
3 𝐶2 𝐶2 𝐶2
1 2 5 2 1273
= [ + + ]=
3 5 26 28 5460
1
2
𝐶1 𝑥 4𝐶1 7
𝐶1 𝑥 6𝐶1 3
𝐶1 𝑥 3𝐶1
P(X = 1) = [ 6 + 13 + 8 ]
3 𝐶2 𝐶2 𝐶2
1 8 7 15 8777
= [ + + ]=
3 15 13 28 16380
2 7 5
1 𝐶 𝐶2 𝐶2
P(X = 2) = [6 2 + 13 + 8 ]
3 𝐶2 𝐶2 𝐶2
1 1 7 5 946
= [ + + ]=
3 15 26 14 4095
x 0 1 2
P(X =x) 1273 8777 946
5460 16380 4095

(b) Calculate the mean and variance of X (06marks)


1 0 1 2
P(X =x) 1273 8777 946
5460 16380 4095
xP(X =x) 0 8777 1892
16380 4095
x2P(X =x) 0 8777 3784
16380 4095
8777 1892
E(X) = + = 0.9979
16380 4095
8777 3784
E(X2) = + = 1.4599
16380 4095
Var(X) = 1.4599 – 0.9979
= 0.4642
Properties of the variance
(i) Var(a) = 0
(ii) Var(aX) = a2Var(X)
(iii) Var(aX + b) = a2Var(X)
(iv) Var(aX – b) = a2Var(X)

Example 21

A discrete random variable X has a probability distribution

x 1 2 3 4 5
P(X = x) 0.2 0.25 0.4 0.1 0.05
Find

(i) Mean = ∑ 𝑃𝑥(𝑋 = 𝑥) = 1 x 0.2 + 2 x 0.25 + 3 x 0.4 + 4 x 0.1 + 5 x 0.05 = 2.55


(ii) The variance
E(X2) = 12 x 0.2 + 22 x 0.25 + 32 x 0.4 + 4 2 x 0.1 + 52 x 0.05 = 7.65
Var(x) = E(X2) – (E(X))2 = 7.65 – (2.55)2 = 1.148
(iii) Var (3x -2) = 32Var(x) = 9 x 1.148 = 10.332

Example 22

A random variable X of a discrete probability distribution given by

x 10 20 30
P(X = x) 0.2 0.3 0.5
Find

(i) E(X) = 10 x 0.2 + 20 x 0.3 + 30 x 0.5 =22


(ii) Var (X) = E(X2) – (E(X))2
E(X2) = 102 x 0.2 + 202 x 0.3 + 302 x 0.5 = 520
Var(x) = 520 – 222 = 36
(iii) Var(4X + 3) = 42Var(x) = 16 x 36 = 576

Revision exercise 3
1. A random variable X of discrete probability distribution is given by
x 1 2 3
P(X = x) 0.2 0.3 0.5
Find (i) E(X) = 2.3 (ii) E(X2) = 5.9 (iii) Var (X) = 0.61

2. A random variable X of discrete probability distribution is given by

x -1 0 1 2
P(X = x) 0.25 0.1 0.45 0.2
Find: (i) P(-1≤ X < 2) = 0.8 (ii) E(X) = 0.6 (iii) E(2x + 3) = 4.2

3. A random variable X of a discrete probability distribution


P(X = 0) = 0.05, P(X = 1) = 0.45 P(X = 2) = 0.5
Find: (i) E(X) = 1.45, (i) E(X2) = 2.45 (iii) Var (X) = 0.348
4. A random variable X of discrete probability distribution is given by
P(X = 1) 0.1, P(X= 2) = 0.2, P(X = 3) = 0.3, P(X =4)= 0.4
2
Find (i) E(X) = 3 (ii) Var (X) = 1 (iii) P(X = 2/X≥2) =
9
5. The discrete random variable Y has a probability distribution P(Y = y) = k =1, 2, 3, 4, 5,6
1
Find (i) mean, μ = 3.5 (ii) E(3X +4) = 15 (iii) E(X2)=14.5 (iv) standard deviation = 1.708
6
6. The discrete random variable R has a probability distribution is given by
3𝑟+1
P(R = r) = ; r = 0, 1, 2, 3
22
24 61 50
Find (i) mean, μ= , E(R2) = (iii) E(3R-2) =
11 11 11
7. The discrete random variable R has a probability distribution given by
2𝑟+1
; 𝑟 = 0, 1, 2,3
20
𝑃(𝑅 = 𝑟) = { 11−𝑟
, 𝑟 = 4, 5
20
Find (i) E(R) = 2.55, (ii) Var(R) = 1.45
8. The discrete random variable X has a probability distribution given by
𝑘𝑥, 𝑥 = 1, 2, 3, 4, 5
𝑃(𝑋 = 𝑥) = {
𝑘(10 − 𝑥), 𝑥 = 6, 7,8, 9
Find (i) constant, k = 0.04, (i) E(X) = 5 (iii) Var(X) = 4
9. The discrete random variable X has a probability distribution is given by
P(X =x) = kx, x = 1, 2, 3,…..n ; where k is a constant
2 1
Show that k = , hence find in terms of n the mean X = (2𝑛 + 1)
𝑛(𝑛+1) 3
10. A random variable X of a discrete probability distribution given by
P(X = 0) = P(X=1) = 0.1, P(X = 2) = 0.2, P(X= 3) = P(X = 4) = 0.3. Find Var (X) = 1.64
11. A random variable X of a discrete probability distribution given by
P(X = 2) = 0.1; P(X= 4) = 0.3; P(X = 6) = 0.5; P(X = 8) = 0.1. Find Var (X) 2.56

Cumulative distribution function F(X)


F(X) is given by FX = ∑ 𝑃(𝑋 = 𝑥)

Note F(+) = 1 where + is the upper limit.

Example23

A discrete random variable has a probability distribution

x 1 2 3 4 5
P(X = x) 0.2 0.25 0.4 0.1 0.05
Find the cumulative distribution function

Solution

x 1 2 3 4 5
F(X)) 0.2 0.45 0.85 0.95 1

Example 24

The random variable X has a cumulative function below


X -1 0 1 2
F(X) 0.25 0.35 0.80 1
Find the probability distribution function

X -1 0 1 2
F(X) 0.25 0.1 0.45 0.2

Example 25

A discrete random variable has a cumulative distribution

x 1 2 3 4 5
F(X) 0.2 0.32 0.67 0.91 1
Find (i) probability distribution function

x 1 2 3 4 5
F(X) 0.2 0.12 0.35 0.24 0.09
(ii) P(X = 3) =0.35

(iii) P(X>2) = P(X= 3) + P(X =4) + P(X=5) = 1- 0.12 = 0.68

Example 26

The random variable X has a cumulative function

X 1 2 3 4
F(X) 0.1 0.5 0.8 1
Find (i) mean (ii) Var(X) (iii) mode

Solution

X 1 2 3 4
P(X = x) 0.1 0.4 0.3 0.2
(i) ∑
Mean = 𝑥𝑃(𝑋 = 𝑥) = 1 x 0.1 + 2 x 0.4 + 3 x 0.3 + 4 x 0.2 = 2.6
(ii) Var(X)
E(X2) = 12 x 0.1 + 22 x 0.4 + 32 x 0.3 + 42 x 0.2 = 5.92
Var (X) = E(X2) – (E(X))2 = 5.92 – (2.6)2 = 0.84
(iii) Mode 2

Revision Exercise 4
1. A discrete random variable has a cumulative distribution
x 0 1 2 3 4
F(X) 01 0.3 0.6 0.8 1
Find (i) E(X) = 15.2 (ii) Var(X)=1.56 (iii) Var(6X + 2) = 56.16
2. The random variable X has a cumulative function below
x 1 2 3 4
F(X) 0.13 0.54 0.75 1
Find (i) P(X=2) = 0.41 (ii) P(X>1) = 0.87 (iii) P(X ≥ 3) = 0.46 (iv) P(X<2) = 0.13 (V)E(X) = 2.58
3. A discrete random variable X has a cumulative distribution
x 3 4 5 6 7
F(X) 0.01 0.23 0.64 0.85 1
Find (i) probability distribution function (ii) Var (X) = 0.9724)
𝑥2
4. A discrete random variable has a cumulative probability function F(X) = , x = 1, 2, 3.
9
4 1 17
Find (1) F(2) = (ii) P(X = 2)= (iii) E(2X -3) =
9 3 9
5. A discrete random variable has a cumulative probability function. F(X) = k, x = 1, 2,3
Find the
1
(i) constant k =
3
2
(ii) P(X<3) =
3
(iii) Standard deviation, σ = 0.816
6. A discrete random variable has a cumulative probability function
𝑥 𝑥
F(X) = 1 − (1 − ) x = 1, 2, 3, 4
4
Find the
63
(i) F(3) =
64
3
(ii) F(2) =
4
(iii) Var (X) = 0.547

Thank you
Dr. Bbosa Science
A-level math paper 2: Index numbers
This the percentage ratio of one quantity to the other, e.g. price index

Index number is a technique of measuring changes in a variable or group of variables with respect to
time, geographical location or other characteristics. It is a statistical measure of change in a
representative group of individual data points.

For example, if the price of a certain commodity rises from shs. 100 in the year 2007 to shs. 150 in
the year 2017, the price index number will be 150 showing that there is a 50% increase in the prices
over this period

Simple index numbers


The simple index numbers include;
(i) Price index or price relative
𝑃1
Price relative = 𝑥 100 where P1= price the current year and P0 – price in the base year
𝑃𝑜
(ii) Wage index
𝑊1
Wage index = 𝑥 100
𝑊𝑜
(iii) Quantity index (quantum) index
𝑄1
Quantum index = 𝑥 100
𝑄𝑜
Example 1
A loaf of bread costed shs. 1200/= in 2008 and shs. 1800/= in 2014. Taking 2008 as the base year,
find the price relative in 2014
Solution
𝑃1 1800
Price relative = 𝑥 100 = 𝑥 100 = 150
𝑃𝑜 1200
Example 2
In 2020, the price index of a commodity using 2019 as the base was 180. In 2021, the price index
using 2020 as the base year was 150. What is the price index in 2021 using 2019 as the base year?
Solution
𝑃2020
𝑥 100 = 180
𝑃2019
𝑃2020
= 1.80 ……….. (i)
𝑃2019
𝑃2021
𝑥 100 = 150
𝑃2020
𝑃2021
= 1.50 ………. (ii)
𝑃2020
Eqn. (i) x Eqn. (ii)
𝑃2020 𝑃2021
. = 1.8 x 1.5
𝑃2019 𝑃2020
𝑃2021
= 2.7
𝑃2019
𝑃2021
𝑥 100 = 2.7𝑥 100 = 270
𝑃2019
∴ the price index in 2021 using 2019 as the base year = 270

Example 3
The wage of a nurse in Uganda in 2010 was 350,000/=, the wage of the nurse in 2015 was increase
by shs. 150,000/=. Using 2010 as the a base year calculate the nurses wage index in 2015.
𝑊1 500,000
Wage index = 𝑥 100 = 𝑥 100 = 142.9
𝑊𝑜 350,000

Price indices
Price indices are divided into
(a) Simple price index
(b) Simple Aggregate price index
(c) Weighed price index

(a) Simple price index


This is the average of the price relative
𝑃
∑ 1
𝑃0
It is given by simple price index = x100
𝑛
Where n = number of items.
(b) Simple aggregate price index
∑ 𝑃1
It is given by simple aggregate price index = (∑ 𝑥 100)
𝑃0
Example 4
The table shows the prices 0f bread and meat per kg in 2000 and 2008
Year
item 2000 2008
Beans 700 1200
Meat 2500 4500
Using 2000 as the base year, find
(a) Price relatives of each commodity
(b) Simple price index
(c) Simple aggregate price index
Solution
𝑃1
(a) Price index = 𝑥 100
𝑃𝑜
1200
For beans: P.R = 𝑥 100 = 171.43
700
4500
For meat: P.R = 𝑥 100 = 180
2500

171.43+180
(b) Simple price index (S.P.I) = = 175.72
2
1200+450
(c) Simple aggregate price index (S.A.P.I) = 𝑥 100 = 178.13
700+2500
Example 5
In 2014 the price of a shirt, a dress and a pair of shoes were shs. 20,000; shs. 35,000 and shs. 45,
000 respectively. Given that in 2017 the prices were shs. 25,000, shs. 50,000 and shs. y
respectively. Find the value of y if the aggregate price index was 130 while taking 2014 as the
base year.

Solution
∑ 𝑃1
Simple aggregate price index = (∑ 𝑥 100)
𝑃0
25,000+50,000+𝑦
( ) 𝑥 100 = 130
20,000+35,000+45,000

75.000+𝑦
( ) = 130
1000
𝑦 = 𝑠ℎ𝑠. 55,000

Weighted price index (composite index)


If the weight or quantity in the base year and current year are the same, we use
(i) Weighted aggregate price index
∑ 𝑃1 𝑤
Weighted aggregate price index = (∑ ) 𝑥 100
𝑃0 𝑤
Example 6
The table below shows the prices (shs.) and amounts of items bought for assembling a phone in
2012 and 2015
Prices (shs
Items 2012 2015 Quantity
Transistor 12,000 18,000 8
Resistor 16,500 21,000 22
Capacitor 15,000 17,000 9
Diode 16,000 18,000 2
Circuit 20,000 25,000 1
Calculate the composite price index for a phone taking 2012 as the base year
18,000 𝑥 8 +21,000 𝑥 22+17,000 𝑥 9 +18,000 𝑥 2+25,000 𝑥 1
W.A.P.I = ( ) 𝑥100 = 126.94
12,000 𝑥 8+16,500 𝑥 22+15,000 𝑥 9+16,000 𝑥 2+20,000 𝑥 1

Example 7
The table below shows the prices (shs) and amount of items bought for making a cake in 2008
and 2009.
Prices (shs
Items 2008 2009 Quantity
Flour per kg 6,000 7,800 3
Sugar per kg 5,000 4,000 1
Milk per litre 1,000 1,500 2
Eggs per egg 200 300 8
(a) Calculate the weighted aggregate price index taking 2008 as the base year
(b) In 2009, the cost of making a cake was shs. 80,000/=. Using the weighted aggregate price
index above, find the cost of the coke in 2008.
Solution
7800 𝑥 3+4000 𝑥 1+1500 𝑥 2+3 𝑥 8
(a) W.A.P.I = ( ) 𝑥 100 = 123.3083
6000 𝑥 3+5000 𝑥 1+1000 𝑥 2+200 𝑥 8
𝑃1
(b) 𝑥 100 = 123.3083
𝑃𝑜

80,000
𝑥 100 = 123.3083
𝑃𝑜
𝑃0 = 𝑠ℎ𝑠. 64,878.0333

(ii) Average weighted price index

𝑃1
∑ 𝑤
𝑃0
Average weighted price index =
∑𝑤
𝑥 100
When the price relative (P.R) is given then
∑(𝑃.𝑅 𝑥 𝑤)
Average weighted price index =
∑𝑤
Example 8
The table shows the expenditure (Ug. Shs.) of a student during the first and second terms
Expenditure (shs.)
Items 1st term 2nd term Amount
clothing 46,500 49,350 5
Pocket money 55,200 37,500 3
Books 80,000 97,500 8
Using the first term expenditure as the base, find the average weighted price index
𝑃1
∑ 𝑤
𝑃0
Average weighted price index =
∑𝑤
𝑥 100
49,350 37,500 97,500
𝑥 5+ 𝑥 3+ 𝑥 8
= (46,500 55,200 80,000
) 𝑥 100
5+3+8

= 106.841
Example 9
The table below shows the price relatives together with their weights for a certain family
Item Weight Price relative
Food 172 120
Water 160 124
Housing 170 125
Electricity 210 135
Clothing 140 104
Find the:
(i) simple price index
(ii) cost of living
Solution
𝑃
∑( 1 ) 120+124+125+135+104
𝑃0
(i) S.P.I = = = 121.6
𝑛 5
∑(𝑃.𝑅 𝑥 𝑤) 120 𝑥 172 +124𝑥 160 +125 𝑥 170 +135 𝑥 210+104 𝑥 140
(ii) Cost of living =
∑𝑤
= = 122.82
172+160+170+210+140
Weighted aggregate price indices/ Paache’s theory/value index
If the weight or quantity in the base year and current year are different, we use
∑ 𝑃1 𝑊1
Weighted aggregate price index = ∑ 𝑥 100
𝑃0 𝑊0
Example 10
The table below shows the prices of items per kg in the year 2001 and 2002
2001 = 100 2002
Item Price (shs.) Quantity (kg) Price (shs.) Quantity (kg)
Rice 2800 20 3200 30
Millet 1500 10 1900 10
Beans 2000 5 2500 70
Calculate for 2002
(i) Price index
(ii) Simple aggregate price index
(iii) Simple aggregate quantity index
(iv) Weighted aggregate price index

Solution
𝑃2002
(i) Price index = 𝑥 100
𝑃2001
3200
For rice, price index = 𝑥 100 = 114.29
2800
1900
For millet, price index = 𝑥 100 = 126.67
1500
2500
For beans, price index = 𝑥 100 = 125
2000
3200+1900+250
(ii) S.A.P.I = [ ] 𝑥 100 = 120.63
2800+1500+2000
∑ 𝑄2002
(iii) S.A.Q.I = ∑ 𝑄2001
𝑥 100
30+10+70
=[ ] 𝑥 100 = 314.29
20+20+5
∑ 𝑃1 𝑊1
(iv) W.A.P.I = ∑ 𝑃0 𝑊0
𝑥 100
3200 𝑥 30 +1900 𝑥 10+2500 𝑥 70
=[ ] 𝑥 100
2800 𝑥 20+1500 𝑥 10+2000 𝑥 5
= 358.02
Revision exercise
1. UNEB 2020/2/5
The table below shows the price indices of beans, maize, rice and meat with corresponding weights
Item Price index 2008 Weight
(2007 = 100%)
Beans 105 4
Maize x 7
Rice 104 2
Meat 113 5
Calculate the;
(a) Value of x given that the price indices of maize in 2007 and 2008 using 2006 as the base year are
112 and 130 respectively. (116.0714)
(b) Weighted price index for 2008 using 2007 as the base year (111.4167)

2. UNEB 2018/2/5
The price index of an article in 2000 based on 1998 was 130. The price index for the article in 2005
based on 2000 was 80. Calculate
(a) Price index of the article in 2005 based on 1998. (104)
(b) Price of the article in 1998 if the price of the article was 45,000 in 2005.
UNEB 2017/2/7
The table below shows the price (shs.) and amount of items bought weekly by a restaurant in 2002
and 2003.
Price (shs.)
Items 2002 2003 Amount
Mil k per litre 400 500 200
Eggs per tray 2500 3000 18
Cooking oil per litre 2400 2100 2
Flour per packet 2000 2200 15
Calculate
(a) the weighted aggregate price index taking 2002 as the base year. (119.63)
(b) In 2003, the restaurant spent shs. 450,000/=. Using the weighted aggregate price index, find how
the restaurant could have spent in 2002. (376,096.95)

3. The table below shows the prices (shs.) an amount of items bought in 2006 and 2007
Price (shs.)
Items 2006 2007 Amount
Mil k per litre 300 400 1
Eggs per tray 2500 3000 3
Cooking oil per litre 3000 8000 4
Flour per packet 1500 1800 15
Taking 2006 as the base year
(a) Calculate the simple aggregate price index (180.92)
(b) Calculate the weighted aggregate price index(161.702)
4. The table below shows the prices in US dollars and weights of five components of an engine, in
1998 and 2005
Prices (US D)
components weight 1998 2005
A 6 35 60
B 5 70 135
C 3 43 105
D 2 180 290
E 1 480 800
(a) Taking 1998 as the base year, calculate the
(i) Simple aggregate price index (172.03)
(ii) Price relative of each component (171.4, 192.9, 244.2, 161.1, 166.7)
(iii) Weighted price index (178.55)
(b) Using the price index in (a)(i) estimate the cost of the engine in 1998 if the cost of the engine
in 2005 was 1600 USD (896.11)
5. The table below shows the prices and amounts of items bought in 2004 and 2005
Prices (shs)
Item 2004 2005 Amount
A 635 887.5 6
B 720 815 4
C 730 1045 3
D 362 503 7
(a) Calculate the simple aggregate price index (132.836)
(b) Calculate the weighted aggregate price index (133.52)
(c) Calculate the price of an item costing 500 in 2004 using weighted aggregate price index
above. (667.64)
6. The table below shows the prices of items per kg in the year 2005 and 2007
Item Posho Beans Rice Beef Chicken
Price in 2005 1200 2000 1200 4000 8000
Price in 2007 1600 2500 1600 6000 9500
Calculate for 2007 using 2005a as the base year
(a) Simple price index (132.0833)
(b) Simple aggregate price index (129.2683)
7. The table below shows the prices in the year 2010 and 2018
Item Price in 2010 Price in 2018
Flour per kg 3000 5400
Eggs per dozen 5,000 7800
Calculate for 2018 using 2010 as the base year
(i) Simple price index (168)
(ii) Simple aggregate price index (165)
8. The table below shows the prices and quantities of four items in the year 2020 and 2021
Item Price per unit Quantities
2020 2021 2020 2021
A 100 120 36 42
B 110 100 96 88
C 50 65 10 12
D 80 85 11 10
(a) Calculate the price index (120, 90.91, 130, 106.25)
(b) Simple aggregate price index (108.82)
(c) Weighted aggregate price index (99.55)
(d) Cost in 2021, A, B, C and Dare ingredients to make chapatti and in 2020a price of chapatti
costed shilling 600 using index in (iii) above. (shs. 597.3)
Thank You

Dr. Bbosa Science


Complex numbers Where p = and q =

Equations ax2 + bx + c = 0 where a ≠ 0 and Example 1


b2 < 4ac can only be solved by introducing
√ = i or j; and this is the basis of (a) x2+ 4 = 0
complex numbers. Solution
x2 = -4
Solving equations involving complex x=√
numbers (b) x2 + 2x + 5 = 0
Solution
Given ax2 + bx + c = 0

From
( ) ( ) √ √

( ) ( ) x = -1 ± 2i
(c) 2x2 – 3x + 4 = 0
√ √


The above simple examples reveal the
√ following

(a) A complex number Z may be written as


If b2 > 4ac; the equation has two roots which Z = x + iy, where x and y are real numbers.
are both real. The real part of Z is x and the imaginary
part is y. i.e. x = Re(Z) and y = im(Z).
If b2 = 4ac; the equation has got two roots
(b) Solution of such equations whose roots
which are repeated and real.
are complex occur in pairs, called
If b2< 4ac; the equation has got two roots conjugates. For instance, compare
which are not real. Z= , -1 ± 2i and
In complex numbers equations whose roots (i) If Z = -1 + 2i is one root of an
are not real are considered equation, then its conjugate
̅ = -1 – 2i is another root.
Hence (ii) An equation cannot have odd number
√ of complex roots
(c) If x + iy = 0, then x = 0 and y = 0
We let x = p ± iq

digitalteachers.co.ug
This implies that two complex numbers Geometrically, the operations of addition and
are equal if and only if their corresponding subtraction of complex numbers are diagonals
real and imaginary parts are equal of a parallelogram. (see the Argand diagrams)
i.e. Z1 = x1 + iy1 and Z2 = x2 + iy2, then
Multiplication of complex numbers
Z1 = Z2, only and only if x1 = x2 and y1 = y2.
Proof: If Z1 = Z2 When multiplying complex numbers the
x1 + iy1 = x2 + iy2 following properties should be observed
i.e. (x1 – x2) + (y1 – y2) = 0 i2= √ x√ = -1 ;
=> x1 – x2 = 0 and y1 – y2 = 0 i3 = -1 x i = -1
x1 = x2 and y1 = y2 i4 = i2 x i2 = -1 x -1 = 1
Algebra of complex numbers i5 = i3 x i2 =-1 x -1 = 1
Like in the real plane, complex numbers can i6 = i3 x i3 =-1 x -1 = i2 = -1
be subjected to four basic operations of
addition, subtraction, multiplications and i8 = i4 x i4 =1 x 1 = 1
division. Now
Using Z1 = x1 + iy1 and Z2 = x2 + iy2 Z1Z2 = (x1 + iy1)(x2 + iy2)
Addition of complex number = x1x2 + ix1y2 + ix2y1 + i2y1y2
When adding two or more complex numbers, = (x1x2 – y1y2) + i(x1y2 + x2y1)
real parts are added separately and imaginary
parts are added separately Division of complex numbers

Z1 + Z2 = (x1 + x2) + i(y1 + y2) When diving a complex number with another,
the result is also a complex number. It can be
Illustration done in two ways
If Z1 = 2 + 3i and Z2 = 1 – 2i Either:
Z1 + Z2 = (2 + 1) + i(3 + -1)
Let for some real numbers p and
= 3 + 2i q.
Subtraction of complex numbers  Z1 = (p + iq)Z2
When subtracting two or more complex x1 + iy1 = (p + iq)(x2 + iy2)
numbers, real parts are subtracted from real
parts and imaginary parts subtracted from = px2 + ipy2 + iqx2 +i2qy2
imaginary parts. = px2 – qy2 + i(py2 + qx2)
Z1 – Z2 = (x1 – x2) + i(y1 – y2) Equating the corresponding real and
Illustration imaginary parts of the resulting complex
number
If Z1 = 2 + 3i and Z2 = 1 – 2i
x1 = px2 – qy2 and x2= py2 + qx2
Z1 + Z2 = (2 – 1) + i(3 – -1)
Solving these equation simultaneously for p
= 1 + 4i and q

digitalteachers.co.ug
p= and q = When given roots to a complex number, the
quadratic equation say Z can be obtained by
Or: finding the sum and the products of the roots

‘Realizing’ the denominator by multiplying i.e. the equation is given by


through by it conjugate Z2 – (sun of roots)Z + product of roots

Not that if Z is a root of the equation, then its


conjugate ̅ is also a root

Example 3

Example 2 Form a quadratic equation in Z given the


following roots
Given that Z1 = 3 – 2i and Z2= 2 + 3i, by
expressing your answer in the form a + bi, find (a) 1 + 5i
Solution
(a) Z1 + Z2
Let Z = 1 + 5i, the Z* = 1 – 5i
Solution
Sum of roots (1 + 5i) + (1 - 5i) = 2
Z1 + 3Z2 = (3 – 2i) + 3(2 + 3i) = 9 + 7i
Product of roots (1 + 5i)(1 – 5i)= 26
(b) 2Z1 – Z2
Equation: Z2 – 2Z + 26 = 0
Solution
Or
2(3 – 2i) – (2 + 3i) = 4 – 7i
Let Z = 1 + 5i => Z – 1 – 5i = 0
(c) If Z1 = 2 – 3i and Z2 = 4 + i, find the values
Z = 1 – 5i => Z – 1 + 5i = 0
of
 (Z – 1 – 5i)( Z – 1 + 5i) = 0
(i) Z13Z2
Z2 – 2Z + 26 = 0
Solution
(b) 4 – 3i
Z13Z2 = (2 – 3i)3(4 + i) Solution
Let Z = 4 – 3i, the Z* = 4 + 3i
Using Pascal’s triangle Sum of roots (4 - 3i) + (4 + 3i) = 8
= [23 + 3(2)2(-3i) + 3(2)(-3i)2 + (-3i)3](4+ i) Product of roots (4 - 3i)( 4 - 3i)= 41
Equation: Z2 – 8Z + 41 = 0
=(8 – 36i – 54 + 27i)(4 + i) Or
=(-46 – 9i)(4 + i) Let Z = 4 - 3i => Z – 4 + 3i = 0
Z = 4 + 3i => Z – 4 - 3i = 0
= -184 – 46i – 36i + 9  (Z – 4 + 3i)( Z – 4 - 3i) = 0
Z2 – 8Z + 41 = 0
= -175 – 82i
Solving cubic equations
(ii)
It is quite difficult to solve cubic equations,
Solutions however, we use the approach of inspection,
i.e. we substitute the assumed factors into the
given equation. If the unction is f(x) = 0 and
x – a = 0 is a root, then f(a) = 0.

We then get another equation (quadratic)


using long division which can easily be solved
to obtain other factors.
Forming quadratic equations given roots

digitalteachers.co.ug
Example 4 Substituting for x;

Solve the following equations 9 + y2 = 13; y = ±2

(a) x3 – 4x2 + x + 26 = 0 The roots of the equation are x = -2, 3 ± 2i

Solution (b) 2x3 – 12x2 + 25x – 21 = 0

Let f(x)= x3 – 4x2 + x + 26 = 0 Solution

f(1) = 2 – 4 + 1 + 26 ≠ 0; => x – 1 is not a factor Let f(x) = 2x3 – 12x2 + 25x – 21 = 0

f(2) = 8 – 8 + 2 + 26 ≠0; => x – 2 is not a factor f(1) = 2 – 12 + 25 – 12 = -6 ≠ 0;x – 1 is not a


factor
f(-2) = -8 – 16 – 2 + 26 = 0; x +2 is a factor
f(2) = 16 – 48 + 50 – 21 = -3 ≠ 0;x – 2 is not a
Using long division
factor
x2 – 6x + 13
f(3) = 54 – 108 + 75 – 21 = 0; x – 3 is a factor
3 2
(x + 2) x – 4x + x + 26
Using long division
3 2
x + 2x
2x2 – 6x + 7
- 6x2 + x + 26
(x – 3) 2x3 – 12x2 + 25x – 21
- 6x2 – 12x
- 2x2 - 6x2
13x + 26
-6x2 + 25x – 21
- 13x + 26
-6x2 + 18x
0 +0
7x – 21
3 2 2
x – 4x + x + 26 = (x+2)( x – 6x + 13)= 0
7x – 21
Either x + 2 = 0; x = -2
0+0
2
Or x – 6x + 13= 0
2x3 – 12x2 + 25x – 21
Solving x2 – 6x + 13= 0
= (x-3)( 2x2 – 6x + 7) = 0
√ √
= 3 ± 2i Either x – 3 = 0; x = 3

The roots of the equation are x = -2, 3 ± 2i Or 2x2 – 6x + 7 = 0


√ √ √
Alternatively =
Given the equation x2 – 6x + 13= 0 √
The roots of the equation are x = 3,
Let the roots be x ± 2i
Alternatively
Sum of roots x + iy + x – iy = 6
Given the equation 2x2 – 6x + 7 = 0
2x = 6; x = 3
Or x2 – 3 + = 0
Product of roots = (x + iy)(x – iy) = 13

x2 + y2 = 13 Let the roots be x ± 2i

Sum of roots x + iy + x – iy = 3

digitalteachers.co.ug
2x = 3; x = When k = -4, y2 = -4 which is inadmissible
since y must be real.
Product of roots = (x + iy)(x – iy) =
When k = 9; y2 = 9 => y = ±3
x2 + y 2 =  x= = ±2
Substituting for x;
√ = 2 + 3i or -2 – 3i
2 √
+y = ;y= Approach 2
√ Let √ = x + iy
The roots of the equation are x = 3,
x2 – y2 + i2xy = -5 + 12i
Finding the square root of a complex
number Equating corresponding parts

Suppose that the square root of Z is a + bi x2 - y2 = -5 ………. (i)

Then a + bi = => (a + bi)2 = Z 2xy = 12…………… (ii)

Example 5 Eqn. (i)2 + eqn. (ii)2

Find the square roots of x4 - 2x2y2 + y4 = 25

(a) -5 + 12i + 4x2y2 = 144

Solution x4 + 2x2y2 + y4 = 169

Approach 1
x2 + y2 = 13 …………….(iii)
Let √ = x + iy
Eqn.(i) + eqn. (iii)
By squaring both sides
2x2 = 8; x = ±2
-5 + 12i = x2 - y2 + i2xy
Substituting for x in eqn. (iii)
Equating corresponding parts
4 + y2 = 13; y = ±3
x2 - y2 = -5 ………. (i)

2xy = 12 √ = 2 + 3i or -2 – 3i

(b) 3 + 4i
x = ……………… (ii)
Let √ = x + iy
Substituting eqn.(ii) into eqn. (i)
x2 – y2 + i2xy = 3 + 4i
( )
Equating corresponding parts
4 2
36 – y = -5y x2 - y2 = 3 ………. (i) and 2xy = 4…………… (ii)
Let k = y2 Eqn. (i)2 + eqn. (ii)2
k2 – 5k – 36 = 0 x4 - 2x2y2 + y4 = 9
(k – 9)(k + 4) = 0 i.e. k = 9 or k = -4 + 4x2y2 = 16

x4 + 2x2y2 + y4 = 25

digitalteachers.co.ug

=
x2 + y2 = 5 …………….(iii)
But √
Eqn.(i) + eqn. (iii)

2x2 = 8; x = ±2

Substituting for x in eqn. (iii) Either or

4 + y2 = 5; y = ±1 Example 8

√ = 2 + i or -2 – i Solve the equation( ) , If Z is a


Other equations complex number in form a + bi

Example 6 Solution

Solve the simultaneous equations (Z – 1)2 = i(Z + 1)2

Z1 + Z2 = 8 Z2 – 2Z + 1 = iZ2 + 2iZ + i

4Z1 – 3iZ2 = 26 + 8i (i – 1)Z2 + 2(i + 1)Z + i = 0

Solution  Z2 – 2iZ + 1 = 0

Z1 + Z2 = 8 √ √
Z= √
Z1 = 8 – Z2 ……………………… (i)
Z = (1 ± √ )i
4Z1 – 3iZ2 = 26 + 8i…………. (ii)
Revision exercise 1
Substituting eqn. (i) into eqn. (ii)
1. Simplify each of the following and
4(8 – Z2) +3iZ2 = 26 + 8i express your answer in the form a + bi
(a) (3 + 4i) + (2 + 3i) [5 + 7i]
32 – 4Z2 – 3iZ2 = 26 + 8i
(b) (3 + 4i) – (2 – 3i) [1+ 7i]
(4 + 3i)Z2 = 6 – 8i (c) (2i)2 [-4]
(d) (2 + 3i)(2 – 3i) [13]
(e) (3 + 4i)(1 – 2i) [11 – 2i]
2. If Z is a complex number in form (a + bi)
Substituting for Z2 into (i) solve
Z1 = 8 + 2i (a) ( ) [Z = (1 ± √ )i]
Z1 = 8 + 2i and Z2 = -2i (b) * +
(c) Express each of the following complex
Example 7
numbers in the form a + bi
Solve the equation 6x2 – 2(1+ 2i)x – 1 = 0 (a) Z1 = (1 – i)(1 + 2i) [3 + i]
Solution (b) Z2 = [2i]
(c) Z3 = [2 – 2i]

Using (d) Find the square root of
(a) 12i – 5 [2 +3i or -2 – 3i]

(b) 5 + 2i [±(3 +2i)]
(c) 15 + 8i [±(4 + i)]

digitalteachers.co.ug
(d) 7 – 24i [±(4 – 3i)] complex number is represented by a line of
(e) Given that Z = 3 + 4i, find the value of the certain length in a particular direction. Thus
expression [6] each complex number is shown as a vector on
(f) Given that the complex number Z* and its Argand diagram.
complex conjugate satisfy the equation The sum and the difference of the two
ZZ* + 2iZ = 12 + 6i, find the possible values complex numbers can be shown on an Argand
of Z [3 + 3i; 3 – i] diagram in the same way as we show vectors
(g) Solve the simultaneous equation which are normally added or subtracted.
Z1 + Z2 = 8 The complex number x = x + iy is represented
4Z1 – 3iZ2 = 26 + 8i as shown below

[Z1 = 8 + 2i and Z2 = -2i]


Im
(h) Express each of the following in the form y
a + bi
x Re
(a) * +
(b) * +
Example 8
(c) [-1 + i]
Given Z1 = -1 + 2i and Z2 = 3 – i show Z1 + Z2
(d) * + and Z2 – Z1 using Argand diagram
(i) Solve the following equations
(a) 2x2 +32 = 0 [±4i] (a) Z1 +Z2 = -1 + 2i + 3 – i = 2 + i
(b) 4x2 + 9 = 0 * +
2 Im
(c) x + 2x + 5 = 0 [-1 ± 2i]
M3
(d) x2 – 4x + 5 = 0 [2 ± i]

(e) 2x2 + x + 1 = 0 * + 2
(j) Form quadratic equations having roots
(a) 3i, -3i [x2 + 9 = 0] Z1 1
(b) 1 + 2i, 1- 2i [x2 – 2x + 5 = 0]
(c) 2 + I, 2 – i[x2 – 4x + 5 = 0] -2 -1 1 2 3 Re
(d) 3+4i, 3 – 4i [x2 – 6x + 25 = 0] Z2
-1
(k) Solve the following equations if each has
at least one real root
(a) x3 – 7x2 + 19x – 13 = 0 [1, 3 ± 2i]
(b) Z2 - Z1 = (3 – i) – (-1 + 2i) = 4 – 3i
(b) 2x3 – 2x2 – 3x – 2 = 0 * +
(c) x3 + 3x2 + 5x + 3 = 0 [-1, ±√ ] Im

The Argand diagram 1 2 3 4 Re


Complex numbers can be represented -1
graphically just as coordinates are. However, -Z1
instead of the x – axis and the y – axes, thereal -2
(Re) and the imaginary (Im) exes are used Z2
respectively instead. This representation was -3
first suggested by a mathematician named R.
Argand, hence the Argand diagram. Each

digitalteachers.co.ug
The modulus and argument of a complex Arg(Z3) = ( ) = 1800 - 53.10 = 126.90
number
Note: Due care must be taken in obtaining the
From the above diagram argument of complex numbers as above
r2 = | | example shows. Although the moduli for Z1
and Z2 are the same, the arguments differ
i.e. r = | | √ which is modulus Z = because the fall in different quadrants. A
mod(Z) and tanθ = , i.e. , which sketch may be thus may be a necessary
pre-requisite.
is the argument, arg(Z) or amplitude, amp(Z).
The polar (modulus – argument) form of a
Note: for principal values, -π ≤ Arg(Z) ≤ π
complex number.
Example 9
Given Z = x + iy (the Cartesian form)
Find the modulus and argument of the
following
Im
(a) Z1 = 1 + i
y
Solution
| | √ =√ O x Re

Im x = rcosθ and y = rsinθ


1 Z= x + iy = rcosθ + rsinθ
θ
O 1 Re i.e. Z = rcosθ + rsinθ or simply Z = rcis. This
form is referred to as polar form of Z.
Arg (Z1) = ( )
Example 10

(b) Z2 = 1 – i Express Z1 = -1 – i and Z2 = -√ + 3i


Solution
Solution
| | √ =√
Im | | √ =√
1 Im
O θ Re -1
1 θ Re
√ -1

Arg(Z1) = ( )
Arg(Z2) =
(c) Z3 = -3 + 4i √
Solution √ ( )
| | √ =
Im | | √( √ ) √

4 5
θ
-3 O Re

digitalteachers.co.ug
(iv)
Im
3 Example 11
2√
Given that Z1 = 3 + 4i and Z2 = 1 – i, find
θ
O (i) | |
-√ Re
(ii)
Solution
Arg(Z2) = ( )= 1200 =

| | √
0 0
Z2 = 2√ (cos120 + isin120 )
arg(Z1) = ( )
Z2 = 2√ (cos + isin )
| | √ √
Multiplication and division of polar form
arg(Z2) = ( )
The two operations offer valuable results in
complex plane (i) | | | || | √
(ii)
Let | | and arg(Z1) = θ1 i.e.

| | Example 12
Given that Z1= 3(cos + isin )
Let | | and arg(Z2) = θ2 i.e.
And Z2 = 5(cos + isin ) find
| |
(i) | |
(a) Multiplication of polar form (ii)
Solution
Z1Z2 = For
=r1r2(cosθ1cosθ2 + i cosθ1sinθ2 + isinθ1cosθ2 – Z1= 3(cos + isin ); | |
sinθ1cosθ2)
For
= r1r2,(cosθ1cosθ2 - sinθ1cosθ2)+ i(cosθ1sinθ2 +
sinθ1cosθ2)} Z2 = 5(cos + isin ); | |
= r1r2{cos(θ1 + θ2) + isin(θ1 + θ2)} (i) | | | || |
Deductions: (ii) = =
(b) Division of polar form
(i) | | | || |
(ii)

(iii) | | | || | | | { }
(iv)
= { }

In general = { }
(i) | | | || || | | |
Deduction
(ii) | | | || || | | | | |
(iii) | |
(a)
| |

(b) ( )

digitalteachers.co.ug

arg(Z1) = ( )
Example 13
| | √
(a) | | | |
=
Given that Z1 = 3 + 4i and Z2 = 1 – i, find
(b) ( )
(i) | |

(ii) ( )
| | √
Solution (c) | | | | | | √

(d) ( )
| | √

arg(Z1) = ( ) = ( ) ( )

=3
| | √ √
= 3(1200) – 2(300) + 1350 = 750 =
arg(Z2) = ( )
Note: the reader should correlate the
| | √
(i) | | | |
= computations of arguments Z1Z2 and with

(ii) ( ) the laws of logarithms

Example 15
Example 14 (a) Given that Z1 = 3 + 4i, Z2 = 12 + 5i find
Three complex numbers are given as (i) | |
(ii)
Z1 = -1 – i; Z2 = 3 -i√ ; Z3 = -1 + i√ (iii) | |
Find (iv) arg

(a) | | Solution

(b) ( ) | | √

(c) | | Arg(Z1) = ( )

(d) ( ) | | √
Solution Arg(Z1) = ( )
| | √ √ (i) | | | || |
arg(Z1) = ( )
(ii)

| | √ √ √ = 53.10 + 22.60 =75.70


| |

(iii) | | | |
=
arg(Z2) = ( )
(iv) ( )

| | √ √ =53.10 - 22.60 = 30.50

digitalteachers.co.ug
(b) Express Z = -1 -i√ in modulus-urgument (i) Since 1 + 3i and 2 – I are roots of the
form. Hence find in form a + bi where a equation, then their conjugates are also
roots of the same equations. Hence the
and b are real numbers
other two roots are 1 – 3i and 2 + i.
Solution (ii) Sum of roots = (1 + 3i) + (1 – 3i) + (2 – i) +
(2 + i) = 6
| | √ √ (iii) Products of roots
= (1 + 3i)(1 – 3i)(2 – i (2 + i) = 6
Arg(Z) = (

) = (1 – 9i2)(4 – i2)
=(1 + 9)(4 + 1)= 50
Z =2( ) Example 17

Show that Z1 = 2 + 3i is a root of the equation


Hence, ( )
Z4 – 5Z3 + 18Z2 – 17Z + 13 = 0

. ( ( ) ( )) Hence find the other remaining roots

Solution
( ( ) ( ))
Approach 1
√ Given 2 + 3i is a root, its conjugate 2 - 3i is also
=
a root of the equation
Solving equations of higher order
The equation of these two roots
When one or more roots of a complex
Sum of roots 2 + 3i + 2 – 3i = 4
number is(are) given, their conjugates are also
the roots of the same equation Product of roots = (2 + 3i)(2 – 3i) = 13
To prove that a certain root is a factor of a The equation is Z2 – 4Z + 13 = 0
complex function, we either use long division
to show that the quotient of division is zero or Using long division
we substitute the factor given into the Let f(Z) = Z4 – 5Z3 + 18Z2 – 17Z + 13
equation to note the remainder or we get the
product of the already existing roots and then Z2 – Z + 1
carry on long division to observe the Z2 – 4Z + 13 Z4 – 5Z3 + 18Z2 – 17Z + 13
remainder or we use the synthetic approach
Z4 – 4Z3 + 13Z2
Example 16
-Z3 + 5Z2 -17Z + 13
Given that (1+ 3i) and (2 – i) are roots of the
equation -Z3 + 4Z2– 13Z

aZ4 + bZ3 + cZ2 + dZ + e = 0, find Z2 - 4Z + 13

(i) The other two roots Z2 - 4Z + 13


(ii) The sum of the four roots 0+0+0
(iii) The product of the four roots
Since the remainder is zero, Z2 – Z + 1is also a
Solution factor of f(Z)

 f(Z) = (Z2 – 4Z + 13)( Z2 – Z + 1)

digitalteachers.co.ug
Either Z2 – 4Z + 13 = 0 or Z2 – Z + 1= 0 Since the last value in the table is zero,
therefore 2 + 3i is a root of the equation.
Factors of Z2 – Z + 1
Other roots can be obtained as shown above.
√ √
Illustration of the synthetic method

√ Procedure
The roots are 2 ± 3i and
(a) Write down the root and coefficients of
Approach 2 the expression in the first row (as shown
Taking Z = 2 + 3i and substituting in the given above)
equation (b) Write zero immediately below the first
coefficient (this is the only entry in the
LHS = (2 + 3i)4 – 5(2 + 3i)3 + 18(2 + 3i)2 – 17(2+ second row that is simply written, the
3i) + 13 = 0 (RHS) other are to be obtained by
multiplication)
Z1 = 2 + 3i is a root
(c) At the first coefficient to zero to get the
This means Z2 = 2 – 3i is also a root of the first entry in the third row
equation as it is a conjugate of the given root. (d) Obtain the second entry in the second
row by multiplying 1 by 2 + 3i, then add to
Given f(Z) = Z4 – 5Z3 + 18Z2 – 17Z + 13 = 0
-5 to get the second entry in the third row
Sum of roots = 5 and i.e. -3 + 3i
(e) Repeat (d) by multiplying -3 + 3i to get the
Product of roots = 13 third entry in the second row, this entry is
Let the other roots be α and β added to 18 to get to get the third row i.e.
3 – 3i. continue with this trend up to the
Sum of the roots = 2 + 3i + 2 – 3i + α + β = 5 last entry. If the last entry in the table is
4+α+β=5 zero, then the value being tested is a root.

α+β=1 Example 18

α = (1 – β) …………………………….. (i) Show that Z1 = 1 + I is a root of the equation


Z4 – 6Z3 + 25Z2 – 34Z + 26 = 0
Product of roots = (2 + 3i)(2 – 3i)αβ = 13
Solution
13αβ = 13
Using synthetic division
αβ = 1 ………………………………….. (ii)
1+i 1 -6 25 -34 26
Substituting (i) into (ii) 0 2+i -6-4i 21+13i -26
1 -5 + i 17 - 4i -3+ 13i 0
(1 – β)β = 1 i.e. β2 – β + 1 = 0
Since the last entry in the table is zero, 1 + I is
√ √ a root.

The conjugate 1 – i is also a root



The roots are 2 ± 3i and
Let the other roots be α and β
Approach 3: using synthetic method Sum of roots = 1 + i + 1 – i + α + β = 6
2 + 3i 1 -5 18 -17 13 2+α+β=6
0 2+3i -15-3i 15+3i -13
1 -3 + 3i 3 -3i -2+3i 0 α+β=4

digitalteachers.co.ug
α = 4 – β ……………… (i) 7. Show that z = 1 is a root of the equation
z3 -5z2 + 9z – 5 = 0 [2 – i, 2 + i]
Product of roots (1 + i)(1 – i)αβ = 26
8. The complex number satisfies
2αβ = 26 find the real and imaginary parts of z, and
Αβ = 13 ……………… (ii) the modulus and argument of z.
[Re(z) = -3,Im(z) = -1, √ , -2.82rads]
Substituting eqn.(i) into eqn. (ii) 9. One root of the equation z2 + az + b = 0
(4 – β)β = 13 where a and b are real constants is 2 + 3i.
Find the values of a and b [a = -4, b = 13]
β2 - 4β + 13 = 0 10. If z1 = ( ) and
√ √
z2 = ( ( ) ( )) find

= 2 ± 3i (a) | | * +
(b) ( )* +
Hence the other roots are 2 + 3i and 2 – 3i
(c) | |[3]
Revision exercise 2
(d) ( )* +
1. Given Z1 = 7 – 4i and Z2 = -1 + 3i, express in
the modulus-argument form the following 11. If z1 and z2 are two complex numbers
(a) Z1Z2 such that | | | |. Show that
,r = 25.45 *cos(0.437π) + isin(0.437π)+- the difference of their arguments is or
(b) | | .
12. Find the modulus and argument of
{ [ ]
[5, 0.6435 rad]
2. Find the modulus and the principal
13. If and find the
arguments of
moduli of
(a) * +
(a) z1 [5]
(b) *√ + (b) z2 [6.5]
√ (c) z1 + z2 [2.061]
(c) * +
(d) z1z2 [32.5]
(d) [ √ ]
Demoivre’s theorem
3. If z1 and z2 are complex numbers, solve
the simultaneous equation, giving your This states that for all rational integers
answer in the form a + bi.
r(cosθ + isinθ)n = r(cosnθ + isinnθ)
4z1 + 3z2= 23
z1 + iz2 = 6 [2 +3i, 5 – 4i] The theorem can be proved by induction
4. Given that 2 + I is a root of the equation approach or otherwise
z3 – 11z + 20 = 0. Find the remaining roots
[2 – i, -4] By induction
5. Show that 2+ i is a root of the equation Given that (cosθ+ isinθ)n = cosnθ + isinnθ
2z3 – 9z2 + 14z – 5 = 0. Hence find other
roots [2- i, ½ ] Suppose n = 1
6. Find equation whose roots are -1 ± i, cosθ+ isinθ = cos1θ + isin1θ
where i = √ [z2 +2z + 2 = 0]
 cosθ+ isinθ = cosθ+ isinθ i.e. the proof
holds for n = 1

digitalteachers.co.ug
Suppose n = k Hence the theorem holds for all values of
n.
(cosθ+ isinθ)k = coskθ + isinkθ

For n = k + 1
Applications of Demoivre’s theorem
k+1 k 1
(cosθ+ isinθ) = (cosθ+ isinθ) (cosθ+ isinθ)
It is mainly employed in proving trigonometric
= (coskθ + isinkθ)( cosθ + isinθ) functions and finding roots of complex
function.
= coskθcosθ + icoskθsinθ + isinkθcosθ + i2sinkθsinθ
Proving identities
= coskθcosθ – sinkθsinθ + i(coskθsinθ + sinkθcosθ)
Example 19
= cos(k+1)θ + isin(k+1)θ
By using Demoivre’s theorem, show that
which is true when n = k + 1

Therefore the proof holds for all values of n (a) tan5x =

Using otherwise Solution

Given z = cosθ + isinθ cos5x + isin5x = (cosx + sinx)5

Z2 = (cosθ + isinθ)2 = cos5x + 5isinxcos4x – 10sin2xcos3x –


10isin3xcos2x + sin4xcosx + isin5x
= cos2θ + 2isinθcosθ –sin2θ
Equating real parts
= cos2θ – sin2θ + i2sinθcosθ
Cos5x = cos5x – 10cos3xsin2x + cosxsin4x
= cos2θ + isin2θ
Equating imaginary parts
Z3 = (cos2θ + isin2θ)2(cosθ + isinθ)
Sin5x = 5cos4xsinx – 10cos2xsin3x + sin5x
= cos2θcosθ - sin2θsinθ + i(sin2θcosθ + cos2θsinθ)
tan5x =
= cos3θ + isin3θ

 (cosθ+ isinθ)n = cosnθ + isinnθ hold for all Dividing terms of the R.H.S by cos5x
positive values of n
tan5x =
Also, (cosθ+ isinθ)n = cosnθ + isinnθ holds for
fractional and negative indices (b)

(a) z-1 = Solution


=( )( )

= =
=
 z-1 =
=
(b) suppose n = -m
(c)
z-m =
=( )( )


=
=

digitalteachers.co.ug
=
= Im
4
(d) 16sin5x = sin5x -5sin3x + 10sinx 530
O 3 Re
Given Z = cosx + isinx
̅ = cosx – isinx
z = 5[ ]
(Z – ̅ ) = 2isinx
This means that (Z – ̅ )n = 2isinnx
(Z – Z*)5 = 2isin5x = 32isin5x
= * ( ) ( )+
(Z – ̅ )5
= Z5 -5Z4 ̅ + 10Z3 ̅ 2 + 10Z2(- ̅ )3+ 5Z ̅ 4 - ̅ 5 Substituting for k = 0
= Z5 -5Z3 + 10Z - 10( ̅ )+ 5 ̅ 3 - ̅ 5
=[ ̅ ] [ ̅ ] [ ̅] = * ( ) ( )+= -2 - i
= 2isin5x – 10isin3x + 20isinx
√ 2±i
 32isin5x = 2isin5x – 10isin3x + 20isinx
(b) -3 – 4i
16sin5x = sin5x -5sin3x + 10sinx
Solution
Finding roots

Suppose that z = r(cosθ + isinθ) | | √

Then zn = rn(cosnθ + isinnθ)

( ) The principal angle lies in the third quadrant

The general expression for finding the roots of Im


-3
a complex function is given by 530 Re
-4
[ ] where
k = 0, 1, 2, …k and θ is usually in radians.
z = 5[ ]
th
Now for n root, we have
= * ( ) ( )+
zn = rn[ ]
Substituting for k = 0
( )
= * ( ) ( )+= -2 - i
Example 20
Substituting for k = 1
Use Demoivre’s theorem to find the square
root of = * ( ) ( )+
(a) 3 + 4i
= ±(1-2i)
Solution
√ ±(1 – 2i)
| | √ (c) 5 – 12i

Solution

The principal angle lies in the first quadrant | | √

digitalteachers.co.ug

z3 = )=

The principal angle lies in the third quadrant √


the cube roots are 1,
Im
5 Using otherwise
O 67.40 Re
-12 Taking
13
z= , then z3 – 1 = 0

Let f(z) = z3 – 1
z = 13[ ]
f(1) = 13 – 1 = 0
= * ( ) ( )+
z – 1 is a factor.
Substituting for k = 0 Using long division

= * ( ) ( )+ z2 + z +1

= 3 - 2i z-1 z3 – 1

Substituting for k = 1 z3 – z2

= z2 – 1
* ( ) ( )+ z2 – 1

= -3 + 2i z3 – 1 =(z – 1)( z2 + z +1)

√ ±(3 – 2i) Either z – 1 = 0, z = 1

Example 21 Or z2 + z +1 = 0

(a) Use roots 1, α and α2 of unity. Hence √

show that 1 + α + α2 = 0

Solution the cube roots are 1,

Let √ Hence

z= (In mod-Arg form Letting α = , the

√ √
=[ ] , k = 0,1,2 α2=( ) =

= ) √
Letting α = , the
When k = 0
√ √
α2=( ) =
z1 = =1
Hence the roots can be written as 1, α + α2
When k = 1

Adding these roots
z2 = )=
√ √
When k = 2

digitalteachers.co.ug
(b) Evaluate * ( ) ( )+

Solution
* ( ) ( )+
Let z = =2 When k = 0

=[ ] * ( ) ( )+
= * ( ) ( )+, k = 0, 1, 2, 3 =1.2160 +1.0204i
When k = 0 When k = 1
z1 = [ ]=√ √ * ( ) ( )+
When k = 1
= -1.4917 + 0.5429i
Z2 = [ ]=-√ √
When k = 2
When k = 2
* ( ) ( )+
Z3 = [ ]=√ √
When k = 3
= 0.2756 – 1.5633i

Z3 = [ ]=-√ √ ( √ ) =1.2160 +1.0204i,


-1.4917 + 0.5429i, 0.2756 – 1.5633i
(√ √ )
Example 24
Example 23
Find the root of z4 + 4 = 0 using Demoivre’s
Evaluate ( √ ) theorem

z4 = -4 = 4(-1 + 0i)
Solution
arg(z) = ( )
( √ ) *( √ ) +
 z4 = [ ]
[( √ )( √ )]
* ( ) ( )+

[ √ ] When k = 0
Let z = -2 + 2√ i
* ( ) ( )+

| | √ ( √ ) √ ( )
√ √

√ When k = 1
Arg(z) = ( )

The principal angle lies in the second quadrant * ( ) ( )+


Im
2√ √ ( )
4 √ √

1200 When k = 2
- O Re

digitalteachers.co.ug
* ( ) ( )+ (c) 2| | | |
(d) arg( )
√ ( )
√ √ Solution
When k = 3
Let z = x + iy
* ( ) ( )+ (a) | | i.e. √

√ ( )
√ √
The locus of z is a circle with centre (0, 0) and
the roots of z4 + 4 = 0 are 1 ± i and -1 ± i
radius 2units
The locus of a complex number Im
2
This is an equation of a set of point
representing a variable complex number or a 1
path traced. It may be in form of an equation
of a circle or a straight line.
-2 -1 0 1 2 3
Re
Example 25 -1
Determine the locus of z if
-2
(a) | |
(b) Arg(z+2) =
(b) Arg[(x – 1) + iy] =
Solution
( )
Let z = x + iy

(a) | |
| |
The locus is a straight line
Im
The locus of z is a circle with centre (4, 0) 2
(b) Arg(x + iy +2) = Y=x-1
1
Arg((x+2) + iy) =
( ) -1 0 1 2 3
-2

Re
-1

-2

The locus of z is a straight line (c) | | | |


Example 26
* √ + [√ ]
Determine the locus of z if:
3x2 + 3y2 - 8x - 2y + 3 = 0
(a) | | and sketch it
(b) and sketch it x2 + y 2 - x - y + 1 = 0

digitalteachers.co.ug
( ) ( ) Hence z = 2* ( ) ( )+

or z = 2[ ]
( ) ( )
(i) On the same Argand diagram plot ̅ and
The locus of Z is a circle with the centre ( ) 2 ̅

and radius √ ̅ √

(d) arg( ) | | √(√ ) =√ =2

arg( ) ̅ (√ ) = 2√ + i

arg[ ] – arg[ ]= | ̅ | √( √ ) =√

( ) ( )= Finding arg( ̅ :

( ) ( ) Arg( ̅ ( ) = -300
√ √
( )( )
Finding arg ̅ :

Arg ̅ = ( )


√ √ (√ ) (√ ) =

√ √ √
( ) ( )

The locus is a circle with centre at

* ( √ ) ( √ )+and radius √

Example 27

The complex number z = √ . (b) What are the greatest and least values of
̅ | | | | ?

(a) Express Z in the modulus argument form | |


| |
| | √(√ ) =√ =2

This is an equation of the circle with centre


(4, 0) and radius 3.

tanθ =

θ = Arg (z) = ( )=

digitalteachers.co.ug
Greatest value of | |

Lowest value of | | =1 z + 1= 0
z = -1
Example 28
So the roots are 2±3i and -1
(a) Given that the complex number Z and its
conjugate ̅ satisfy the equation Example 29
Z ̅ . Find Z.
(a) If z1 = and z2 = , find | |
Let z = x + iy then ̅
(x + iy)(x – iy) +2i(x + iy) = 12 + 6i Solution

Equating imaginary parts


2xi = 6i [ ]
x=3 = =
Equating real parts
= =
By substituting for x
9 + y2 -2y = 12 = =
y2 -2y -3 = 0
(y – 3)(y+1) = 0 | | √ ( ) =
y = 3 or y = -1
when y = 3; z = 3 + 3i Alterative 2
when y = -1; z = 3 - i
the possible values of z are 3 +3i and 3- i.
(b) One root of the equation

Determine other roots

Solution

Given root 2 + 3i; its conjugate is also a root of


| | √ ( ) =
the equation

The equation of these two is (b) Given the complex number z = x + iy

z2 – (sum of roots)z + product of roots = 0 (i) Find

[ ][ ]
Sum = 2 + 3i + 2 – 3i = 4 =[ ][ ]

Product of roots =(2 + 3i)(2 – 3i) = 2 + 9 = 13 [ ] [ ]


=

=
-
(ii) Show that the locus of is a straight
line when its imaginary part is zero.
- State the gradient of the line.

digitalteachers.co.ug
If imaginary part is zero | | | |

0
x2 + y2 – 6x + 12y + 45 4x2 + 4y2

y= +1 x2 + y2 + 2x – 4y – 15 = 0

Comparing with y = mx + c Hence sketching the region satisfying


| | | |
The gradient =
i.e. x2 + y2 + 2x – 4y – 15 ≥ 0
Example 30 substituting (0, 0)
(a) Given that a complex number Z* and its LHS = - 15 ; RHS = 0
conjugate satisfy the equation
Since LHS< RHS, the point (0, 0) does not lie in
ZZ* + 2iZ = 12 + 6i, find the possible values the region| | | | therefore it lies
of Z. in unwanted region.
Solution Now x2 + y2 + 2x – 4y – 15 ≥ 0
Let z* = x + iy, then z = x –iy =
(x + iy)(x – iy) + 2i(x – iy) + 6i = 12 + 6i The locus is a circle centre (-1, 2) and radius
x2 + y2 + 2y + 2xi = 12 + 6i √

Equating real parts The plot of | | | |


Im
x2 + y2 + 2y = 12
8
Equating imaginary parts

2x = 6 => x = 3 4
9 + y2 + 2y = 12

y2 + 2y – 3 = 0 -8 -4 0 4 8
Re
(y – 1)(y + 3) = 0 => y = 1 or y = -3 -4
When y = -3; z = 3 + 3i and

When y = 1, z = 3 –i Example 31
If n is a variable and z = 4n + 3i(1 – n), show
The possible values of z are 3 + 3i and 3 - i that the locus of Z is a straight line. Determine
the minimum value of | |
(b) Find the Cartesian equation, in its simplest
Solution
form of the curve described by
| | | | where z is the complex Let z = x = iy
 x + iy = 4n +3i(1 – n)
number x + iy.
Equating real parts
Hence sketch an Argand diagram the
region satisfying | | | | x = 4n => n =
Solution
Equating imaginary parts
Substituting z = x = iy

digitalteachers.co.ug
y = 3(1 – n) If the above expression is wholly imaginary,
then the real part must be zero
 y= ( )
i.e.
y= which is a straight line with
=> x2 + y2 – 3x + y = 0
gradient and intercept 3

Given z = 4n + 3i(1 – n) ( ) ( )
| |
The locus is a circle with centre with
| | and radius √
The minimum value can be obtained by
completing squares or differentiation Revision exercise 4
By completing squares
| | 1. Use Demoivre’s theorem to show that
(a) cos6x = cos6x -15cos4xsinx + 15coxsin4x –
[( ) ] sin6x

[( ) ] (b) tan3x =

( ) 2. (a) Evaluate ( √ )
[1.2160 + 1.0204i, -1.4917 +0.5429i]
| | is minimum when
(b) Find the roots of z4 + 4 = 0 using
| | = Demoivre’s theorem
 | | [1± i, -1± i]
3. (a) Given that the complex number Z and
By differentiation its conjugate satisfy the equation
Z . Find possible
| || |
values of Z. (06marks)
[Z = ]
n=
(b) Prove that if is real, then the
| | ( ) ( ) locus of the point representing the
complex number Z is a straight line.
 | | (06marks) [y = + 6]
4. (a)Express 5 + 12i in polar form
Example 32 [5 + 12i = 13(cos 67.380 + isin67.380)]
Show that the locus of z if is wholly Hence, evaluate√ , giving your
answer in the form a + ib where a and b
imaginary is a circle with centre at and are corrected to two decimal places.
radius √ (12marks)

Solution [z = 2.17 + 0.90i, -1.86 + 1.43i, -0.31 – 2.33i]

Let z = x + iy
5. (a) The total impedance z in an electric
circuit with two branches z1 and z2 is given
by . Given that z1= 3 + 4i and z2
[
=

digitalteachers.co.ug
= 5 + 5i where i =√ , calculate the total (ii) * +
impedance z in form a + bi (iii) Show in an Argand diagram the
* + points representing complex
(b) If n is a variable and z = 4n + 3i(1 – n), numbers z, z2 and .
show that the locus of Z is a straight (b) In an Argand diagram, P represents a
line. Determine the minimum value of complex number z such that
| | *| | + | | | | show that P lies
on a circle, find
6. Given that z1 = 3 + 4i and z2 = -1 + 2i (i) The radius of this circle[4.2164]
(a) Express z1, z2, z1 + z2 and z1 – z2 in the (ii) The complex number
form r(cosθ + isinθ) represented by it centre
[z1= 5(scos53.10+ isin53.10), * +
z2 =√ (cos116.60 + isin116.60), 11. (a) given the complex number z1 = 1 – i,
z1 + z2 = 2√ (cos71.60 + isin71.60) z2 = 7 + I represent z1z2 and z1 – z2 on the
z1 – z2 = 2√ (cos-153.40 + isin-153.40)] Argand diagram.
(b) Determine the angle between z1 + z2 Determine the modulus and argument of
and z1- z2 [2250] [0.6325, -124.70]
7. Solve the simultaneous equation (b) If z is a complex number of form
z1 + z2 = 8
4z1 – 3iz2 = 26 + 8i a + bi, solve ( ) [z = 1 ±i√ ]xz
Using the values of z1 and z2, find the
modulus and argument of z1 + z2 –z1z2. 12. (a) show the region represented by
[z1 = 8 +2i, z2 = -2i; 6.49, 75.960] | | on an Argand diagram
8. (a) Given that z = 3 + 4i, find the value of [it is a circle of centre(2, -1) and radius 1
expression z + [6] unit, the circle is drawn with dotted line
since the equation is less than, and the
Given that | | , show that the locus
unwanted region is the area outside the
of the complex number is circle]
x2 + y 2 + . Sketch the locus (b) Express the complex number
[The centre of the circle is ( ) it z = 1 - i√ in modulus argument form

passes through -3 but does not touch the and hence find z2 and in the form
y-axis] a + bi
9. (a)(i) Express each of the following ( ( ) ( ))
complex numbers in the form a + bi [ ]

z1 = (1 –i)(1 + 2i) [3 + i] √
z2 = [2i] 13. (a)Given that z1 = -i + 1, z2 = 2 + i, and
z3 = [2 – 2i] z3 = 1 + 5i, represent z2z3, z2-z1 and
(b) Find the modulus and argument of o, Argand diagram
z1z2z3 [17.889, 63.43490] (b) Prove that for positive integers
(c) Find the square root of 12i – 5 .
[1 + 3i or -2 – 3i] Deduce that this formula is also true
10. (a) given that z = √ + i, find the modulus for negative values of n.
and argument 14. If z is a complex number, describe and
(i) z2 * + illustrate on the Argand diagram the locus
given by each of the following

digitalteachers.co.ug
(a) | | 21. Given the complex number z =
(i) Determine z in form a + bi where a and
[it is a circle centre ( ) radius 0. 8385] b are constant [-4,- 3i]
(ii) arg(z) [-143.130]
(ii) Arg (z + 3) = [is it a straight line

represented by equation y = ] 22. (a) Express the complex numbers z1 =4i
and z2 = 2 – 2i in trigonometric form
15. (a) Use Demoivre’s theorem or otherwise r(cosθ + isinθ).
to simplify [z1 =4(cos900 + isin900),
z2 = 2√ (cos-450 + isin-450)]
* ( ) ( )+
Hence or otherwise evaluate * +
(b) Express in modulus-argument (b) Find the values of x and y in
form. [x = 2.8, y =0.4]
,z = 0.1387*cos(0.187π)+ isin(0.187π)+- 23. Find the fourth root of 4 + 3i
(c) Solve (z + zz*)z = 5 + 2z where z* is [±(1.4760 + 0.23971i), ±(0.2397 – 1.4760i)]
the complex conjugate of z.
[z = 1 + 2i, z* = 1 -2i] 24. (a) Given that ; find the
16. Show that 2 + I is a root of the equation values of x and y
2z3 – 9z2 + 14 – 5 = 0. Hence find the other [x = 2, y= 1.5 or x = -2 and y = -1.5]
roots 2 – i and ½ (b) If z = x +iy, find the equation of the
17. (a) find the equation whose root are -1 ± i locus | |
where i = √ [z2 + 2z + 2 = 0]
(b) Find the sum of the first 10 terms of * +
the series 1 + 2i+ -4 – 8i + 16 + …. 25. (a) given that the complex number z and
[205 + 410i] its conjugate Z*, satisfy the equation
(c) Prove by induction that zz*+ 3z* = 34 – 12i, find the value of z.
. [z = 3 - 4i and z = -6 + 4i]
18. Show that z = 1 is a root of the equation (b) Find the Cartesian equation of the
z3 – 5z2 + 9z – 5 = 0. locus of a point P represented by
Hence solve the equation for other roots equation| |
[2 ± i] [the locus P is an equation of line
19. (a) Use Demoivre’s theorem to express 8y + 2x = 11]
tan5 in terms of tan . 26. (a) form a quadratic equation having
(b) Solve the equation z3 + 1 = 0 -3 + 4i as one of its roots. [z2 + 6z + 25]
√ √
* + (b) Given z1 = -1 + i√ and z2 = -1 - i√
20. (a) Without using tables or calculators, (i) Express in form a + i√ , where
( ) a and b are real number
simplify [-1]
( ) √
* +
(b) Given that x and y are real, find the
(ii) Represent on an Argand
values of x and y which satisfy the
diagram
equation: [x = -1
(iii) find| | [1]
when y = -2 and x = 1 when y = 2]
27. If z =
(a) Find the

digitalteachers.co.ug
(i) Modulus of z [5.814] (b) Given the complex number z = x + iy
(ii) Argument z [-86.0550] (i) Find * +
(b) Represent z on a complex plane
(c) Write z in the polar form (ii) Show that the locus of is a
[ z = 5.814(cos(-86.0550) + isin(- straight line when its imaginary
86.0550) or z = 5.814(cos (0.47 ) + part is zero. State the gradient of
isin(0.47 )] the line. [y = + 1]

28. (a) the complex number z = √ . 31. (a) Given that the complex number Z and
̅
its conjugate satisfy the equation
(i) Express Z in the modulus argument
Z . Find possible
form
values of Z. [Z = ]
[z = 2* ( ) ( )+ or z =
2[ ]] (b) Prove that if is real, then the locus of
(ii) On the same Argand diagram plot ̅
the point representing the complex
and 2 ̅
number Z is a straight line. [y = + 6]
(b) What are the greatest and least values
of | | | | ? 32. (a) Express 5 + 12i in polar form
[Greatest value of | | , Lowest value of Hence, evaluate√ , giving your
| | = 1] answer in the form a + ib where a and
b are corrected to two decimal places.
29. (a) Given that the complex number Z and (12marks)
its conjugate ̅ satisfy the equation
Z ̅ . Find Z.
[possible values of z are 3 +3i and 3- i] [√ √ * ( )
(b)One root of the equation
( )+ ;
Determine
other roots 2.17 + 0.90i; -1.86 + 1.43i; -0.31 – 2.33i]
[other roots are 2-3i and -1]
33. Show that the modulus of =4√

30. (a) If z1 = and z2 = , find | |


Thank you
[ ] Dr. Bbosa Science

digitalteachers.co.ug
Binomial theorem Example 1

Pascal’s triangle Use Pascal’s triangle to expand

The Pascal’s triangle below and its further (a) (p + q)4


extension is used to determine the The coefficients are 1 4 6 4 1
coefficients of the expansion of (p + q)n
Terms are 1p4q0 + 4p3q1 + 6p2q2 + 4p1q3 + 1q4

= p4 +4p3q + 6p2q2 + 4pq3 + q4

(b) (2x + 3y)3

The coefficients are 1 3 3 1

Terms are
3 0 2 1 1 2 0 3
1(2x) (3y) + 3(2x) (3y) + 3(2x) (3y) + 1(2x) (3y)
Observations
= 8x3+ 36x2y + 54xy2 + 27y3
- The coefficients are symmetrical; they are
the same irrespective of which side they (c) (a + b + c)2
are read from, for instance the The terms can be grouped into two ways:
coefficients of (p + q)6 are
1 6 15 20 15 6 1 Either a and (b + c) or (a + b) and c
- The coefficient of the 2nd term in the The coefficients are 1 2 1
expansion is the index of a given
expansion. E.g. in the expansion of Either
(p + q)6, the coefficient of the 2nd term is
Terms are: 1a2(b + c)0 + 2a1(b + c)1 + 1a0(b + c)2
6.
- The number of terms in the expansion = a2 + 2a(b + c) + (b + c) 2
exceeds the index by one, e.g. (p + q)4
= a2 + 2ab + 2ac + b2 + 2bc + c2
with index 4, has 5 terms
- The index of the first term of the Or
expansion decreases by one, from the
index given till zero, whereas, the index of (a + b + c)2
the second term increases by one from = 1(a + b)2c0 + 2(a + b)1c1 + 1c2
zero to the given index
For (p + q)3 = 1p3q0 + 3p2q1 + 3p1q2 + 1p0q3 = (a + b)2 + 2c(a + b) + c2
=p3 + 3p2q + 3pq2 + q3 =a2 + 2ab + b2 + 2ac + 2bc + c2
- The sum of indices in each term s constant
and equal to the index of the expansion.

digitalteachers.co.ug
Example 2 = 1.0612 (5 significant figures)

Expand (a + b)4 using Pascal’s triangle. Hence The idea of factorial and combination can also
find (1.996)4 correct to 3 decimal places be used to determine the coefficients of the
expansions
Solution
Example 3
From Pascal’s triangle the coefficients are
1 4 6 4 1 Expand

(a + b)4 = a4 + 4a3b + 6a2b2 + 4ab3 + b4 (a) (p + 3q)3

(1.996)4 = (2- 0.004)4 Solution

Substituting a = 2 and b = -0.004 The coefficients are 3C0 3C1 3


C2 3
C3

24 + 4(2)3(-0.004) + 6(2)2(-0.004)2 + Terms are p3 p2(3q) p(3q)2 (3q)3


4(2)(-0.004)3 + (-0.004)4
(p + 3q)3 = 3C0p3 + 3C1p2(3q) + 3C2p(3q)2 +
16 – 0.128 + 0.000384 = 15.872 (3D) 3
C3(3q)3
Example 3
= P3 + 9P23q + 27pq2 + 27q3
(a) Expand (2 – 3x)4 using Pascal’s triangle.
(b) (1 – x)4. Hence evaluate (0.99)4 correct to
Hence evaluate (1.97)4 correct to 3
four decimal places
decimal places.
Solution
Solution
Coefficients are 4C0 4C1 4C2 4C3 4C4
Coefficients: 1 4 6 4 1

(2 – 3x)4 = 1 (2)4(-3x)0 + 4(2)3(-3x)1 + 6(2)2(-3x)2 Or simply ( ) ( ) ( ) ( ) ( )

+ 4(2)1(-3x)3 + 1(2)0(-3x)4 1 – x)4 = 4C014(-x)0 + 4C113(-x)1 + 4C212(-x)2 +


= 16 – 96x + 216x2 – 216x3 + 81x4 4
C311(-x)3 +4C410(-x)40
Now (1.97)4 = (2 – 0.03)4 = (2 – 3(0.01))4 = 1 – 4x + 6x2 – 4x3 + x4
 x = 0.01 Now 0.99 = 1 – 0.01) = x = 0.01
(1.97)4 = 16 – 96 (0.01) + 216(0.01)2 - (0.99)4
216(0.01)3 + 81(0.01)4 = 15.0613848 = 1 – 4(0.01) + 6(0.01)2 – 4(0.01)3 + (0.01)4
= 15.061 (3d.p) = 0.96059601
(b) Use Pascal’s triangle to evaluate (1.02)3 = 0.9606 (4d.p)
correct 5 significant figures
The binomial theorem for positive
Solutions integral index
(1.02)3 = (1 + 02)3 Consider the expansion of (1 + x)4, the result is
Coefficients are : 1 3 3 1 (1 + x)4 = 1 + 4x + 6x2 + 4x3 + x4
(1 + 02)3 = 13 + 3(1)2(0.02) + 3(1)(0.02)2 + Observations
3
(0.02) = 1.061208

digitalteachers.co.ug
(i) The indices of x increase by 1 from term Expand (3 – 2x) 12 in ascending powers of x up
to term to and including the term x3. Hence evaluate
(ii) The index of the last term being the same (2.998)12 correct to the nearest whole
as the power to which (1 + x) s raised number.
(iii) The coefficients of the terms of expansion
Solution
are 4C0 4C1 4C2 4C3 4C4

Hence the expansion of (1 + x)n is (3 – 2x) 12 = 312 + ( )

(1 + x)n = 1 + ( ) ( ) +( ) + …. xn ( ) +…

The expansion of (a + x)n is = 531441 – 4251528x + 1588936x2

Now (2.998)12 = (3 – 0.002)12 => x = 0.001


( )
(2.998)12 =531441 – 4251528(0.001) +
= [ ( )( ) ( )( ) ( ) ] 1588936(0.001)2

= ( ) ( ) = 527205.0609

=527205 (nearest whole number)


Example 4

Expand (1 + 4x)14 in ascending power of x up Particular terms of binomial expansion


to and include the 4th term. Hence evaluate As earlier seen, the expansion of
(1.0004)14 correct to four decimal places
(a + x)5 = a5x0 + 5a4x1 + 10a3x2 + 10a2x3 + 5ax4 +
Solution
x5
14
(1 + 4x)
In general if r is the power of x in the
=1+( ) ( ) ( ) expansion (a + x)n, then

the Ur+1 term of x = nCran-rxr


= 1 + 56x + 1456x2 + 23296x3

Now (1.0004)14 = (1 + 0004)14 => x = 0.0001 Example 6

Find the term of x4 in the expression of


(1 + 4x)14

= 1 + 56(0.0001) + 1456(0.0001) 2 + (i) (1 + x)9

23296(0.0001) 3 By using Un+1; term of x = nCran-rxr

n = 9, r = 4, a = 1
= 1.005614583

= 1.0056 (4d.p) U5 = 9C419-4x4 = 9C4x4 = 126x4

(ii) (3 + x)7
Note the next term in the expansion is
n = 7, a = 3, r = 4
( ) = 256256x4
U5 = 7C437-4x4 = 7C4(3)3x4 = 945x4
4 -11
Its value = 256256(0.0001) = 2.56256 x 10
(iii) ( )
Which when added to the above answer there
will negligible change in value

Example 5

digitalteachers.co.ug
By using Ur+1 = nCran-rxr
U5 = 12C4212-4( ) = 12C4(2)8( ) =7920x4
The term independent of x is got by equating
Example 7
the index of x to zero
Find the term indicated in expansion of the
following expression Ur +1 = 12Cr(2x)12-r( )

= 12Cr(2x)12-rx-2r
(i) ( ) [x3]
= 12Cr. 212-r .x12-3r
( ) ( )
Equating the x12-3r = 1

The term in x3 = .5C114( )  12 – 3r = 0


r=4
= .5C1( ) =-810x3 Term independent of x = 12C4. 212-4

(ii) ( ) [x4] =12C4. 28

= 126720
( ) ( )
(b) ( )
The term in x3 = .6C115( ) Solution

= .6C1( ) 960x4 ( ) ( )
Finding terms independent of x Un+ r = 12Cr(2x2)12-r(3x-1)r
The term is said to be independent of x if the = 12Cr(2x2)12-r(3r)x-r
power of x is zero
= 12Cr. 212-r(3r)x24-3r
Example 8
Equating the x12-3r = 1
Find the term independent of x in the
following expansion  24 – 3r = 0
r=8
(a) ( )
Term independent of x = 12C8. 212-4.38
Solution
=12C8. 24.38
( ) ( ) = 51, 963,120
The term independent of x = 212x12 multiplied Binomial expansion of terms with
by the term in x-12 in the expansion fractional or negative powers
( )
As noted earlier
 The term independent of x
(1 + x)n = 1 + ( ) ( ) +( ) + …. xn
= 212x12.12C4( ) = 212x12 x 12C4 x The following is noted
=28 x 495 = 126720 (i) For positive integral value of n, i.e. n ≥ 1,
the series above terminates at the term xn
Alternatively
and its sum is (1 + x)n.

digitalteachers.co.ug
(ii) For fractional or negative values of n, the
√( )( ) √
series above does not terminate but
instead converges to (1 + x)n as the limit of
its sum only -1 < x< 1 or | |

Example 9 But =1+( ) + …..

Expand √ up to the term x3. Hence


evaluate √ correct to four decimal places.

Solution √( ) ( )

Using
Putting x =
n n
(1 + x) = 1 + ( ) ( ) +( ) + …. x
√( ) ( )
Comparing with => x ≡ -2x and
n=½

( )



( )( )

Example 11
=1 – x -
Expand √( ) up to the term x3
Now √ √
Use your expansion to evaluate √ correct
Comparing with √ ; x = 0.01 to 3 decimal places taking x = .
Substituting
Solution
√ 1 – (0.01) - By rationalizing the denominator
= 0.9899495
√( )( )
√ (4d.p)
But

Example 10
√( ) ( )
Given that x is very small that its cube and
higher powers can be neglected, show that

√( ) Hence putting x =

By putting x = show that √


√( ) ( ) ( )
Solution

By rationalizing the denominator

digitalteachers.co.ug
( )
√ =1+

√ ( )( ) ( )( )( )

√ (4d.p)
=1
Example 12

Use the binomial theorem to expand Now √ √ √ ( )


3
√ up to x . Use your expansion to
evaluate √ correct to four decimal places
( )
Solution

√ Comparing ( ) with

( ) ( )( )
+…  x=

Substituting for x

Hence √ √ √ ( ) ( )

( ) ( ) ( )
( ) => x = 1 ( )

√ * ( ) ( ) ( )+ =

√ (4 S.F)
=1.9130(4d.p)
Not the termx3 has been neglected as it
Example 13
does not affect the answer to 4 significant
Write down the expansion of √ in figured
ascending powers of x as far as the term
Example 14
x4. Use your expansion to find √ correct
to four significant figures. John operates an account with a certain
bank which pays a compound interest rate
Solution of 13.5% per annum. He opened the
account at the beginning of the year with

sh. 500,000 and deposits the same
For binomial expansion amount of money at the beginning of
every year. Calculate how much he will
(1 + x)n = 1 + ( ) ( ) +( ) + …. xn accumulate at the end of 9 years. After
how long will the money have
Comparing with(1 + x)n accumulated to sh. 3.32 millions?
n = and x = -x Solution
The 1st deposit will grow to

digitalteachers.co.ug
500000( )= 500000 x 1.135 [ ( ) ]

2nd deposit will grow to 500000 x 1.1352


= * +
th n
n deposit will grow to 500000 x 1.135
= [ ]
9th deposit will grow to 500000 x 1.1359
=
The total = 500,000[1.135 +1.1352 + … +
1.1359] If x = 9

(1 + x)-2=
= * ( )+
= (using the first 2 terms)
= * ( )+
=
=8,936,381
-2
The exact value is (1 + 9) =
Finding how long it will take the money to
accumulate to sh. 3,320,000
Error =
* ( )+
%error = = 3.978%(3d.p)
n = 4.6years
Example 16
Example 15
(a) Find the three terms of the expansion (2 –
-2
Expand (1 + x) in descending powers of x x)6 and use it to find (1.998)6 correct to two
including the term x-4. If x = 9 find the decimal places (06 marks)
percentage error in using the first two
(2 – x)6 = ( ) ( )
terms of the expression.
= 64 -192x + 240x2
Solution
From

Now (1 + x)-2 = x-2( )

x-2( )

Example 17

(a) Prove by induction

1.3 + 2.4 + … + n(n + 2) = for all values of n.


Suppose n = 1
L.H.S = 1 x 3 = 3
R.H.S =
L.H.S = R.H.S, hence the series holds for n =1

digitalteachers.co.ug
Suppose n = 2
L.H.S= 1 x 3 + 2 x 4 = 11
R.H.S =
L.H.S = R.H.S, hence the series holds for n =2
Suppose n = k
1.3 + 2.4 + … + k(k + 2) =
For n = k + 1
1.3 + 2.4 + … + k(k + 2), (k + 1)(k + 3) = +(k +1)(k + 3)
=(k + 1)* +
=
=
=
= [ ]
Which is equal to R.H.S when n = k + 1
It holds for n = 1, 2, 3 …, hence it holds for all integral values of n.

(b) A man deposits Shs. 150,000 at the beginning of every year in a micro finance bank with
the understanding that at the end of the seven years he is paid back his money with 5%
per annum compound interest. How much does he receive?
Using amount, A = P( )

=150000( ) =211,065.06
Alternatively
1st year
P = 150,000
He is paid back principal plus interest; P( ) = 150,000( )=157,500
nd
2 year
P = 157, 500
He is paid back principal plus interest; P( ) = 157500( )=165375
rd
3 year
P = 165375
Interest =
He is paid back principal plus interest; P( ) = 165375 ( )=173643.75
th
4 year
P=173643.75
He is paid back principal plus interest; P( ) = 173643.75( )=182325.94
th
5 year
P =182325.94
His paid back principal plus interest; P( ) = 182325.94( )=191442.23
th
6 year
P = 191442.23
He is paid back principal plus interest; P( ) = 191442.23( )=201014.35

digitalteachers.co.ug
7th year
P = 201014.35
He is paid back principal plus interest; P( ) = 201014.35( )=211,065.06
th
by the 7 year he has accumulated shs. 211,065.06

Example 18

Expand√( ) up to the term x2. Hence find the value of √( ) to four significant figures.
(12marks)

√( )

√( ) ( )( )

=1+
=1+ +

√( ) 1+ +

√( ) √

=1+ = 1.030

Revision exercise of √ to 3 significant figures


[1.0202, 7.14]
1. Given that the ratio of the 3rd to the 4th
5. Five millions shillings are invested each
term of the expansion (2+3x)n is 5:14, find
year at a rate of 15%. In how many years
the value of n when x = . [n= 16] will it accumulate to more than
2. Expand (3 – 4x)5 in ascending order of x up 50millions? [6years]
to and including the term x3. Hence
evaluate (4.96)5 correct to 2 d.p. [3001.98] 6. Expand ( ) as far as x3. Hence
3. (a) Find the coefficient of x2 in the evaluate √ correct to 3 decimal places
expansion of (1 – 2x)n is 24 [4] [2.829]
(b) Find the term independent of x in the 7. A man deposits sh. 800,000 into his saving
expansion of ( ) [-496128] account on which interest is 15% per
annum. If he makes no withdraws, after
(c) Use the binomial expansion to expand
how many years will his balance exceed
√ up to the term x3. Hence sh. 8millions? [16.5 years]
evaluate√ correct to three decimal 8. Determine the binomial expansion of
places [3.018]
( ) . Hence evaluate (2.1)4 correct to
4. Expand √( )up to and including the 2 decimal places.[19.45]
9. Determine the binomial expansion of
term x3. Hence find the value of √ to
( ) . Hence evaluate (0.875)5 correct
four significant figures. Deduce the value
to four decimal places [0.5129]

digitalteachers.co.ug
10. A financial credit society give a compound (b) show that, when x = ;
interest of 2% per annum to its members. √
If Bbosa deposits sh. 10000 at the
beginning of every year. How much would (c) substitute x = into your expansion
he accumulate at the end of the fifth year and hence obtain an approximation of
if no withdraws within this period
√ , give your answer to five decimal
[sh. 530812] places [1.41415]
11. Expand √( ) in ascending powers of x
14. (a) show that ( )

2
up to a term x . * + (c) Write the first three terms in the

12. (a) Using the expansion up to binomial expansion of ( ) in


3
the term x , find the value of √ to ascending power of x stating the
four decimal places [1.0392] range for which this expansion is
(b) Express √ in the form √ . Hence
valid. * | | +
evaluate √ correct to 3 significant figure (d) Find the first three terms in the
* √ + expansion of in ascending

13. (a) obtain the first four non – zero terms power of x for small values of
of the binomial expansion in ascending
x.* +
powers of x of give
that| | * +
Thank you

Dr. Bbosa Science

digitalteachers.co.ug
Polynomials with its appropriate sign is the product
of the roots
A polynomial in x is a function in the form
Example 1
f(x) = a0 + a1x + a2x2 + a3x3 + … + anxn where
a 0 Find the sums and products of the roots of
the following equations
(a) Quadratics
(a) 5x5 + 4x4 – 3x3 + 2x2 – x + 6 = 0
If α and β are the two roots, then, x=α Solution
and x =β Dividing through by 5
x2 – (α + β)x + αβ = 0 x5 + x4 – x3 + x2 – x + = 0

(b) Cubic sum of roots =


If α, β and λ are the three roots, then, product of roots =
x = α, x =β and x = λ
 (x – α)(x – β)(x – λ) = 0 (b) 3x4 - 5x3 + 2x2 + 0x + 9 = 0
Dividing through by 3
[x2 – (α + β)x + αβ]( x – λ) = 0
x4 - x3 + x2 + 0x + 3 = 0
3 2 2
x – λx – (α + β)x + (α + β)λx +αβx-αβλ = 0

x3 – (α + β + λ)x2 + (αβ + αλ + βλ)x -αβλ = 0 Sum of roots =


(c) Qurtic Product of roots = 3

If α, β, λ and ρ are the four roots, then


Addition and subtraction of polynomial
x = α, x =β, x = λ and x = ρ
Polynomial are added or subtracted if they are
 (x – α)(x – β)(x – λ)(x – ρ) = 0 of the same degree. This is done by adding or
3 2
[x – (α + β + λ)x + (αβ + αλ + βλ)x –αβλ] (x – ρ) = 0 subtracting the coefficients of the
4 3 2
corresponding term
X – (α + β + λ+ρ)x + (αβ + αλ + αρ + βλ +βρ +λρ )x
– (αβλ + αβρ + αλρ +βλρ)x + αβλρ = 0 Example 2

The above illustration shows that: (a) Given the polynomial


f(x) = 5x5 + 4x4 – 3x3 + 2x2 – x + 6 and
(i) The signs of the terms alternate in the
g(x) = 3x4 – 5x3 + 2x2 + 9
order: +, - , + ….. starting with the first
Find (i) f(x) + g (x) (ii) f(x) – g(x)
term.
(ii) The coefficient of the second term is –
(sum of the roots) and the last term

digitalteachers.co.ug
Solution Find coefficient of x3 in the expansion
(i) f(x) + g(x)
(2x3 + x2 – 5x + 6)(2x + 4)
5x5 + 4x4 – 3x3 + 2x2 – x + 6
+ 3x4 – 5x3 + 2x2 + 0x + 9 Solution
= 5x5 + 7x4 – 8x3 + 2x2 – x + 15
(2x3 + x2 – 5x + 6)(2x + 4)
f(x) - g(x) = 2x(2x3 + x2 – 5x + 6) + 4(2x3 + x2 – 5x + 6)
5x5 + 4x4 – 3x3 + 2x2 – x + 6
- 3x4 – 5x3 + 2x2 + 0x + 9 = (2x4 + 2x3 – 10x2 + 12x)+ 8x3 + 4x2 – 20x + 16

= 5x5 + x4 + 2x3 + 0x2 – x – 3 =2x4 + 10x3 – 6x2 – 8x + 16

(b) Given polynomials The coefficient of x3 is 10


f(x) = 2x3 + 4x2 – 2x – 8 and Division of polynomials
g(x) = x3 -2x2 + 3x + 5
The division of polynomial may be done by
Find (i) f(x) + g (x) (ii) f(x) – g(x)
long division as follows.
Solution
Example 5
(i) f(x) + g (x)
(a) Divide x3 – 7x – 6 by (x+ 1)
= (2 + 1)x3 + (4 – 2)x2 + (-2 + 3)x + (-8 + 5)
Solution
= 3x3 + 2x2 + x – 3
x2- x - 6
(ii) f(x) – g(x) (x + 1) )x3 – 7x - 6
= (2 - 1)x3 + (4 – -2)x2 + (-2 - 3)x + (-8 - 5) – x3 – x2
= x3 + 6x2 – 5x -13
x2 – 7x – 6
Multiplication of polynomials – x2 – x
When multiplying two functions together, the -6x – 6
terms of the first function are multiplied by
the terms of the second function – -6x – 6

Example 3 0+0

(a) Given the polynomial Example 6


f(x) = 5x3 + 2x2 + 9 and Find the remainder when 2x3 + x2 + 5x – 4 is
g(x) = 4x4 – 3x3 + 2x2 – x + 6 divided by 2x - 1
Find f(x) x g (x)
Solution x2+ x + 3
(2x - 1) )2x33 + x22 + 5x - 4
= 5x3 (4x4 – 3x3 + 2x2 – x + 6) – 2x – x
+ 2x2 (4x4 – 3x3 + 2x2 – x + 6)
+ 9 (4x4 – 3x3 + 2x2 – x + 6) 2x2 + 5x – 4
= (20x7 – 15x6 + 10x5 – 5x4 + 30x3) – 2x2 – x
+ (8x6 – 6x5 + 4x4 - 2x3 + 12x2)
+ (36x4 – 27x3 + 18x2 -9x + 54) 6x – 4
= 20x7 – 7x6 + 4x5 + 35x4 + x3 + 30x2 – 9x + 54
– 6x – 3
Hence f(x) .g(x) =
-1
20x7 – 7x6 + 4x5 + 35x4 + x3 + 30x2 – 9x + 54
Hence the remainder is -1
Example 4

digitalteachers.co.ug
Example 7 (d) 2x4 + 10x3 – 6x2 – 8x + 16 [4]
2. Given that
Show that x = -2 is a root of the equation
(a) f(x) = x3 + 2x2 – 3x + 2 and
2x3 – x2 – 8x + 4 = 0. Hence find the other
g(x) = 2x3 – x2 + 5x – 4, find f(x) – g(x)
roots
[-x3 + 3x2 – 8x + 6]
Solution (b) f(x) = x3 + 2x2 – 3x + 2 and
g(x) = 2x3 – x2 + 5x – 4, find g(x) – f(x)
If x =-2 is a root of the function, then its [x3 - 3x2 + 8x - 6]
Remainder must be equal to zero (c) f(x) = 2x3 – 5x2 + 6x and
g(x) = x3 – 6x2 + 5x + 1, find f(x) – g(x)
Hence x = -2 is a root of 2x3 – x2 – 8x+ 4 = 0 [x3 + x2 + x – 1 ]
x = -2 => x + 2 =0 (d) f(x) = 2x3 – 5x2 + 6x and
g(x) = x3 – 6x2 + 5x + 1, find 2f(x) + g(x)
[5x3 – 16x2 + 7x + 1 ]
3. Given that
2x2 - 5x + 2 (a) f(x) = x2 – 2x + 5 and
(x - 2) )2x33 - x2 2- 8x + 4 g(x) = x3 + 6x – 4, find xf(x) + 3g(x)
– 2x + 4x
[4x3 – 2x2 + 23x – 12]
-5x2 - 8x + 4 (b) f(x) = x3 + 6x – 4 and
g(x) = x2 – 2x + 5, find 3f(x) + xg(x)
– 5x2 – 10 x
[2x3 + 2x2 + 13x – 12]
2x + 4

– 2x + 4 Other methods of finding the remainder


0 Apart from using long division, the remainder
Since a cubic equation has at most three when a function is divided by a certain factor
roots; the remaining two roots are obtained can be obtained by remainder theorem and
by solving the quadratic equation by synthetic approach.
factorization
(a) The remainder and factor theorems
2x2 – 5x + 2 = 0
When a number say 186 is divided by 4, this
2x2 – 4x – 2x + 2 =0 can be represented simply as follow
2x(x – 2) – 1(x – 2) = 0 46
4 )186
(2x – 1)(x – 2) = 0 – 16
2x – 2 = 0 or x – 2 = 0 26
x= or x = 2 - 24

the other roots are x = and x = 2 2


the quotient is 46 and the remainder is 2
Revision exercise 1 The above alogarithm can be written as
1. Find the degree of each of the following
polynomials Or simply 186 = 4Q + R, where the quotient
(a) x6 + 4x4 – 2 [6] Q = 46 and the remainder, R = 2. This is
(b) 2x3 – x2 – 8x+ 4 [3] referred to as the remainder theorem
(c) 5x3 + 2x2 + 9 [3]

digitalteachers.co.ug
The remainder theorem states that, when a (iv) The first digit in the second row comes
function f(x) is divided by by (x – a) and leaves under the second digit in the first row
a remainder, then the remainder of the and this digit is equal to a0 x a, where a0
function is f(a) is the coefficient of xn.
From f(x) = (x –a)Q(x) + R (v) The second digit in the 3rd row is the same
When we substitute for x = a; f(a) = R as the sum of the digits in the 1st and 2nd
rows
When R = 0, => f(a) = 0. This is referred to as (vi) The next digit in the 2nd row is equal to
the factor theorem, a0 x the 2nd digit in the 3rd row
(vii) The corresponding numbers in the 1st and
Example 8 2nd row are added to give the digit in the
3rd row and the process continues
Find the remainder when
(viii) The last digit in the 3rd row is the
(a) f(x) = x3 + 3x2 – 4x + 2 is divided by x – 1 remainder of the polynomial and the
Solution digit to the left of the remainder are
Let x3 + 3x2 – 4x + 2 = (x – 1)Q(x) + R coefficients of the quotient provided the
Putting x = 1: 1 + 3 – 4 + 2 = R divisor is in the form tx + b, where t = 1.
2=R If t ≠ 0, then we divide the digit to left of
(b) f(x) = 3x3 + 2x – 4 is divided by x – 2 the remainder by t to obtain the
Solution coefficients of the quotient.
Let 3x3 + 2x – 4 = (x – 2)Q(x) + R
Example 9
Putting 2: 24 + 4 – 4 = R
R = 24 Find the remainder and quotient when the
(c) f(x) = 2x + 4x2 – 6x + 5 is divided by x – 1
3
function
Let 2x3 + 4x2 – 6x + 5 =(x – 1)Q(x) + R
(a) f(x) = x3 + 3x2 – 4x + 2 is divided by x – 1
Putting 1: 2 + 4 – 6 + 5 = R
from f(x) = x3 + 3x2 – 4x + 2, the
R=5
coefficients of x in a descending order are
(d) 8x3 + 4x + 3 is divided by 2x -1
1, 3, -4 and 2
Let 8x3 + 4x + 3 = (2x – 1)Q(x) + R
From x – 1 = 0, => x = a = 1
Putting : 1 + 2 + 3 = R 1st row 1 3 -4 2
R=6 2nd row 1 4 0 +; a=1
3rd row 1 4 0 (2)←remainder
Synthetic approach for finding the The remainder is 2
remainder Quotient x2 + 4x
The synthetic approach can be illustrated as (b) x5 + x – 9 is divided by x + 1
follows
x5 + x – 9 x5 + 0x4 + 0x3 +0x2 + x - 9
f(x) = a0xn + a1xn-1 + a2xn-2 + …. + an is divided by
x – a, then the following steps are taken using 1st row 1 0 0 0 1 -9
a table +; x = -1
(i) The factor in question must be linear 2nd row -1 1 -1 1 -2
(ii) The 1st row of the table contains the
3rd row 1 -1 1 -1 2 (-11) ←remainder
coefficients of x in f(x) in descending
order. Remainder: -11
(iii) The at most left hand side of the 3rd row
contains a value a from the division, x – a Quotient: x4 – x3 + x2 –x + 2
i.e. if x – a = 0, then x = a
(c) 4x5 – 3x3 + 2x + 7is divided by 2x – 1

digitalteachers.co.ug
4x5 – 3x3 +2x+ 7 4x5+ 0x4– 3x3 +0x2 +2x+7 By long division to find other factors

1st row 4 0 -3 0 2 7 x2 + x – 6
+; x=
2nd row 2 1 -1 - x2 + 3x + 2 x4 + 4x3 –x2 - 16x – 12

– x4 + 3x3 + 2x2
3rd row 4 2 -2 -1 ( ) ←remainder
x3 – 3x2 – 16x
The quotient is [4x4 + 2x3 -2x2 – x + ] =
– x3 + 3x2 + 2x
4 3 2
2x + x – x - x + and remainder is
– 6 x2 –18x – 12
Example 10
– – 6 x2 –18x – 12
The polynomial x4 + px3 –x2 + qx – 12 has
0 + 0 + 0
factors x + 1 and x + 2.

Find the values of p and q, hence factorize the x4 + 4x3 –x2 + 16x – 12
polynomial completely. =( x2 + 3x + 2)( x2 + x – 6)
Solution Now x2 + x – 6 = (x + 3)(x – 2)
Let f(x) = x4 + px3 –x2 + qx – 12 Hence x4 + 4x3 –x2 + 16x – 12
By factor theorem = (x + 1)(x + 2) (x + 3)(x – 2)
Putting x = -1 Example 11
f(-1) = 1 – p - 1 – q – 12 =0 When f(x) = x3 – ax + b is divided by x + 1, the
p + q = -12 …………………………. (i) remainder is 2 and x + 2 is a factor. Find a and
b.
Putting x = -2
Solution
f(-2) = 16 – 8p – 4 – 2q – 12 = 0
By substitution for x = -1 in the function
4p + q = 0 ………………………….(ii)
(-1)3 – a(-1) + b = 0
Eqn. (ii) – eqn. (i)
-1 + a + b = 2
3p = 12
a + b = 3 ………………. (i)
p=4
since x + 2 is a factor, substituting for x = -2 in
substituting for p into eqn. (i) the function gives zero
4 + q = -12 (-2)3 – a(-2) + b = 0
q = -16 -8 + 2a + b = 0
f(x) = x4 + 4x3 –x2 + 16x – 12 2a + b = 8 …………….(ii)
Now (x + 1)(x + 2) = x2 + 3x + 2 Eqn. (ii) – eqn. (i)
Since (x + 1)(x + 2) is a factor, then x2 + 3x + 2 a=5
is also a factor of f(x)
substituting for a in eqn. (i)

5 + b = 3 => b = -2

digitalteachers.co.ug
Example 12 4p – q = 0 …………….(ii)

The function f(x) = x3 + px2 – 5x + q has a Eqn. (i) + eqn. (ii)


factor x – 2 and has a value of 5 when x = -3.
3p = 12
Find the values of p and q
p=4
Solution
From eqn. (i)
By substitution for x = 2 in the function
q = -4 – 12 = -16
23 + p(2)2 – 5(2) + q = 0
Hence p = 4 and q = -16
4p + q = 2 ………….(i)
4 3 2 4 3 2
x + px – x + qx – 12= x + 4x – x – 16x – 12
By substitution for x = -3 in the function
By long division to find other factors
(-3)3 + p(-3)2 – 5(-3) + q = 5

9p + q = 17 ………. (ii) x2 + x – 6

Eqn. (ii) – eqn.(i) x2 + 3x + 2 x4 + 4x3 –x2 - 16x – 12

5p = 15 – x4 + 3x3 + 2x2

p=3 x3 – 3x2 – 16x

From eqn. (i) – x3 + 3x2 + 2x

4x3+q=2 – 6 x2 –18x – 12

q = -10 – – 6 x2 –18x – 12

Hence p = 3 and q = -10 0 + 0 + 0

Example 13 x4 + 4x3 –x2 + 16x – 12


The polynomial x4 + px3 – x2 + qx – 12 has a =( x2 + 3x + 2)( x2 + x – 6)
factor x2 + 3x + 2.
Now x2 + x – 6 = (x + 3)(x – 2)
Find the value of p and q and hence factorise
completely Hence x4 + 4x3 –x2 + 16x – 12

Solution = (x + 1)(x + 2) (x + 3)(x – 2)

x2 + 3x + 2 = (x + 1) (x + 2) Example 14

Let f(x) = x4 + px3 – x2 + qx – 12 If 4x3 + ax2+ bx + 2 is divisible by x2 + k2, show


that ab = 8
f(-1) = (-1)4 + p(-1)3 – (-1)2 + q(-1) – 12 = 0
Solution
1 – p – 1 + q – 12 = 0

-p + q = 12……………..(i)

f(-2) =(-2)4 + p(-2)3 – (-2)2 + q(-2) – 12 = 0

16 – 8p – 4 – 2q – 12 = 0

-8p – 2q = 0

digitalteachers.co.ug
leaves a remainder of 18 and when
By long division divided by x + 3, it leaves a remainder -50.
Determine the values of a, b, and c. Hence
4x + a
factorize the polynomial completely
(x2 + k2) 4x3 + ax2 + bx + 2 [ a = 6, b = 13, c = -1; (x + 2)(2x +1)(3x-1)]
4. Factorize completely
– 4x3 + 4 k2
f(xyz) = (x + y)3(x – y) + (y + z)3(y – z)+
ax2 + (b – 4 k2)x + 2 (z + x)3(z – x)
[(x + y + z)(x – y)(y – z)(z – x)
– ax2+ a k2 5. When the quadratic expression ax2 + bx +
2
(b – 4 k )x + 2 - ak 2 c is divided by x – 1, x – 2 and x + 1, the
remainders are 1, 1, and 25 respectively,
determine the factors of the expression
Since the remainder is zero,
[(2x – 3) and (2x – 3)]
(λ – 4k2)x + 2 - ak2 = 0 6. The remainder when px3 + 2x2 – 5x + 7 is
divided by x – 2 is equal to the remainder
Comparing coefficients of x:
when the same expression is divided by x
b – 4 k2 = 0 …………………. (i) + 1. Find the value of p [p = 1]
7. Given that x – 4 is a factor of
2 - ak2 = 0 2x3 – 3x2 – 7x + k, where k is a constant,
k2 = ……………….……… (ii) find the remainder when the expression is
divided by 2x -1 [k = - 52, remainder = -56]
Eqn. (ii) into eqn. (i) 8. The expression px3 + qx2 + 3x + 8 leaves a
remainder of -6 when divided by x – 2
b - 4( ) = 0 and a remainder of -34 when divided by
x + 2. Find the values of constants p and q.
 ab – 8 = 0 [p = 1, q = -7]
ab = 8 9. Show that x – 2 is a factor of
x3 – 9x2 + 26x – 24. Find the set of values
Revision exercise 2 of x for which x3 – 9x2 + 26x – 24< 0
[x < 2 and 3< x < 4]
1. Given that f(x) 3x3 – 4x2 -5x + 2, factorize
10. The remainder when x3 – 2x2 + kx + 5 is
f(x) completely and hence solve the
divided by x – 3 is twice when the same
equation f(x) = 0 * + expression is divided by x + 1. Find the
2. Prove that a – b is a factor of value of the constant k [k = -2]
a2(b – c) + b2(c – a) + c2(a – b) and write
down two other factors of the expression.
Hence or otherwise factorize the Thank you
expression completely
[-(a – b)(b – c)(c – a) or (b – a) )(b – c)(c – a)] Dr. Bbosa Science
3 2
3. The polynomial ax + bx – cx – 2 is
divisible by x +2. When divided by x – 1 it

digitalteachers.co.ug
Inequalities (b) 3(2 – x) > 5(3 + 2x)
Solution
An inequality is a logical statement that states 6 – 3x > 15 + 10x
relationship between two mathematical -9 > 13x
expressions. x<
The basic inequalities commonly used are (c)
- Less than (<) Solution
- More than (>) Multiply both sides by 12
- Less than or equal ( ) ( ) ( )
- Greater than or equal ( ) 3(x – 2) < 4(2x – 3)
When solving for equations, the solutions 3x – 6 < 8x – 12
or answers are individual values but when -5x < -6
solving inequalities, the solutions are a x>
range of possible real values.
(d) ( ) ( ) ( )
Linear inequalities
Multiply both sides by 12
Linear inequalities in one unknown given 6(x – 1) + 4(x – 2) 3(x – 3)
one in one equation 6x – 6 + 4x – 8 3x – 9
Solving linear inequalities in one unknown 7x 5
given one in one equation is done in the x
same way as solving for linear equation
Linear inequalities involving indices
except
When solving inequalities involving indices
- The inequality symbols must be
such as ax > b, where a and b are positive
maintained
integers, introduce natural logarithms to both
- The inequality symbol changes when
sides of the inequality. i.e.
dividing both sides of inequality
equation by a negative number. Inax > b
Example 1 xlna > lnb
Solve the following inequalities x>
(a) 4x – 2 > x + 7
Example 2
Solution
4x – x > 7 + 2 Solve the following inequalities correct to 3
3x > 9 decimal places
x>3

digitalteachers.co.ug
(a) 52x > 8 Example 3
Solution
Find the set of integers which satisfy
In52x > In8
simultaneously both of the following
2xln5 > ln8
equations

x > 0.646 (a) 4x + 3 2x + 5; x+4 7


(b) 20-3x < 15 Solution
Solution 4x + 3 2x + 5; x+4 7
In20 -3x
> In15 2x 2 x 3
x 1
-3xln20 > ln15

x > -0.301

(c) (0.8)-3x > 2.4 The number line show that the set of integers
that satisfies the two equations are {1, 2, 3}
Solution (b) 5 – 2x 3–x; 1 -2x 11 – 4x
In(0.8)-3x > In2.4 Solution
-3xln0.8- > ln2.4 5 – 2x 3 – x ; 1 -2x 11 – 4x
-x -2 2x 10
x > x 2 x 5
(d) (0.8)3x > 2.4

Solution

In(0.8)3x > In2.4


The number line show that the set of integers
3xln0.8 > ln2.4 that satisfies the two equations are {x:x ≤ 2}
Note that logarithm of any number
between 0 and 1 is negative; so ln0.8 is (c) 5x – 4 ≥ 4x -3,
negative Solution
5x – 4 ≥ 4x -3,
x < x≥1 x<3

Linear inequalities in one unknown given two


inequalities equations.

Linear inequalities in one unknown given two


The number line show that the set of integers
inequalities equations
that satisfies the two equations are {1, 2} (3 is
The solution to two linear inequalities can be not included
best handled by use of a number line. When
Example 4
finding a set of integers that satisfy the
equations, we only take on integral (discrete) Show that there is just one integer which
values. simultaneously satisfies the three inequalities
and find that number

digitalteachers.co.ug
( ) Solution

Solution Let y = 2x2 – 3x + 1

The curve cuts the x – axis when y = 0


( )
 2x2 – 3x + 1 = 0
x–1≥2 -3x ≤ 5 x≤3 (x – 1)( 2x – 1) = 0
x≥3 x≥- x≤3 Either x – 1 = 0 or 2x – 1 = 0;

x = 1 or x =
x≥3
x≤3 the curve cuts the y – axis when x = 0
x≥
 y=1
-2 -1 0 1 2 3 4 5 6 7 8 9 Since the coefficient of x2 is positive, that the
From the number line, there is only one point curve is U shaped
of intersection of the three inequalities, which
is 3

Hence the set of integers that satisfy the three


inequalities is {3}

Non-linear inequalities in one unknown


The following methods are employed

- Sign change
- Graphical method The solution set is 0.5 ≤ x ≤ 1
When using graphical method, the set of (b) 7x2 > 1- 6x
values above the axis are positive and those
below are negative Solution

When using sign change method, a table 7x2 > 1- 6x


describing specific regions of inequalities is 7x2 + 6x -1 > 0
used and the necessary tests are performed
Let y =7x2 + 6x -1
If the inequality symbol is ≥ or ≤, care must be
taken, because the critical values of the The curve cuts the x – axis when y = 0
function and the numerator in case of
 7x2 + 6x -1 = 0
fractions will always satisfy the inequalities
(x + 1)(7x – 1) = 0
Before solving inequality, all terms must be
taken to one side preferably the LHS Either x + 1 = 0 or 7x – 1 = 0

Method I: Graphical method x = -1 or x =


Example 5 the curve cuts the y – axis when x = 0
Solve the following inequalities  y = -1
(a) 2x2 – 3x + 1 ≤ 0

digitalteachers.co.ug
Method II: sign change
Example 6

(a) 2x2 – 3x + 1 ≤ 0
(2x -1)(x-1) ≤ 0
The critical values of (2x -1)(x-1)= 0 are
x = 1 and x = respectively
The above illustration shows that the
numbers and 1 subdivide the number
From the graph, the solution is x < -1 and x >
line into three regions namely
3 2
(c) 2x + 3x 2x x≤ , , x≥1
Solution The corresponding sign in the respective
regions can be analysed by choosing any
2x3 + 3x2 – 2x 0 random value in each region, substitute it
in the equation (2x -1)(x-1) and put the
Let y = 2x3 + 3x2 – 2x
sign of the answer on the following
The curve cuts the x – axis when y = 0 number line

 2x3 + 3x2 – 2x = 0 + - +
x(2x – 1)(x + 2) = 0

Either x = 0, 2x -1 = 0 or x + 2 = 0 1
Note that the solution for (2x -1)(x-1) ≤ 0 is
x = 0, x = or x = -2 equal or less than zero or negative.
Closed circles indicate that the critical values
the curve cuts y – axis when x = 0
are part of the solution.
 y=0
Hence the solution set for (x – 1)(2x – 1) ≤ 0

is

(b) 7x2 + 6x -1> 0

7x2 +7x – x – 1 = 0
(x + 1)(7x -1) = 0

The critical values x = -1 and


+ - +

-1
From the graph the solution
-2 ≤ x ≤ 0 and x ≥ 0.5 The solution for (x + 1)(7x -1) > 0 is
positive . Open circles indicate that the
critical values are not part of the solution
(d) Solve the inequality
Hence the solution set for (x + 1)(7x -1) > 0 is
2
4x + 2x < 3x + 6 (06marks)
x < -1 and x >

digitalteachers.co.ug
(c) 2x3 + 3x2- 2x ≥ 0 (x + 1) = 0; x = -1

x(2x2 +3x -2) ≥ 0 (x + 2) = 0, x = -2

x(x + 2)(2x – 1) ≥ 0 Testing for correct region

Critical values x = 0, x =-2, and x =


- + - +
- + - +
-2 -1
-2 0
The solution for is positive
The solution for x(x + 2)(2x – 1) ≥ 0 is and the critical values are part of the solution
positive and the critical values are part of
the solution Hence the solution for is
Hence the solution for x(x + 2)(2x – 1) ≥ 0 -2 ≤ x ≤ -1 and x ≥

-2 ≤ x ≤ 0 and x ≥ The modulus of inequalities


The modulus of a number is the magnitude of
that number (absolute value) which is always
(d) 4x2 + 5x – 6 < 0 positive, e.g. | | | | = 1

4x2 + 5x – 6= 0 When finding modulus of an inequality, the


Critical values following must be considered
- The modulus on one side of the linear
√ ( )( ) √ inequality is removed by introducing a
x= ( )
=
negative number of the given value on the
x = -2, other side
i.e. if | | , then -3 < x < 3
+ - + - The modulus on both sides of the linear
inequality is removed by squaring both
-2 sides
The solution 4x2 + 5x – 6 < 0 is negative and - When the terms under modulus are
the critical values are not part of the solution fractional, square both sides of the
inequality
the solution is -2 < x <
Example 7

(e) Solve the following inequalities

(a) | |
( ) -4< x – 6 < 4
-4 + 6 < x 4 + 6
2 < x < 10
( )( ) (b) | |
-6 < 3x + 4 < 6
Finding critical values -6 – 4 < 3x < 6 – 4
-10 < 3x < 2
3x – 5 = 0; x =

digitalteachers.co.ug
(c) | | | |
2 2 + - +
(2x – 3) > (x + 3)
4x2 – 12x + 9 > x2 + 6x + 9
3x2 – 18x > 0
3x(x – 6) = 0
Critical values are x = 0 and x = 6 Solution for (x – 2)(x – 6) > 0 (positive)

Hence the solution x < 2 and x > 6


+ - +

0 6
Limits of inequality
The solution x < 0 and x > 6
This refers to interval within which the
(d) | | | | inequalities lies or does not lie.
2 2
(2x + 5) < (x – 3)
This is done by expressing the function given
4x2 + 20x + 25 < x2 – 6x + 9
as quadratic equation in x.
3x2 + 26x + 16 < 0
(3x + 2)(x + 8) < 0 For real values of x, b2 ≥ 4ac
Critical values are x = -8 and x = -
Example 9
+ - +
(a) Given the function y = , find the
-8 - range of values within which y does not
lies

The solution is -8 < x < - Solution

Example 8 y=

Find the range of value of x can take for the y(x2 + 6x) = 3x – 6
following inequality to be true
yx2 + (6y – 3)x + 6 = 0
| |
For real values of x, b2 ≥ 4ac
Solution (6y – 3)2 = 24y
Squaring both sides 36y2 – 60y + 9 ≥ 0

Dividing through by 3

x2 < 4(x2 – 6x + 9) 12y2 – 2y + 3 ≥ 0

x2 < 4x2 – 24x + 36 (6y – 1)(2y – 3) ≥ 0

0 < 3x2 – 24x + 36 The critical values are y = and y =

Divide through by 3 + - +
x2 – 8x + 12 > 0

(x – 2)(x – 6) > 0 Since the solution of the equation is positive;


Critical values are x = 2 and x = 6 the required range

digitalteachers.co.ug
(b) Find the range of values within which the For y ≥ 3 boundary line is y = 3
function y = lies

Solution

y(4 + x2) ≥ 3 – 2x

yx2 + 2x + 4y – 3 ≥ 0

For real values of x, b2 ≥ 4ac

22 ≥ 4y(4y – 3)

1 ≥ 4y2 – 3y Show by shading the unwanted regions; the


region satisfying the inequalities x +2y ≥ 6,
0 ≥ 4y2 – 3y – 1
y > x, x< 5 and 3x + 5y ≤ 30
4y2 – 3y – 1 ≤ 0
Solution
(y – 1)(4y + 1) ≤ 0
For x +2y ≥ 6 the boundary line is
Critical values y = 1 and y =
x + 2y = 6

+ - + x 0 6
y 3 0
- 1 For x> y

The boundary line is x = y


Solution for (y – 1)(4y + 1) ≤ 0 is negative and
critical values are part of the solution x 0 5
y 0 5
Hence range of values is For x < 5

Simultaneous inequalities The boundary line is x = 5

Solving two simultaneous inequalities is best For 3x + 5y ≤ 30


done by representing the inequalities on the The boundary line
graph. The unshaded (feasible) region
represents the solution to the inequalities. 3x + 5y = 30

Example 10 x 0 5
y 6 3
Show by shading the unwanted regions; the
region satisfying the inequalities y ≤ 2x +1 and
y≥3

Solution

For y ≤ 2x +1, the boundary line is y = 2x +1

If x= 0, y= 1, (x, y) = (0, 1)

If x= 2, y= 5, (x, y) = (2, 5)

Testing for wanted region using point (0,0);


0 ≤ 1. Hence this point is in wanted region.

digitalteachers.co.ug
Revision exercise 5. Solve the following inequalities
(a)
1. Solve the following inequalities
(a) 7x – 3 2x – 1 * + * +
(b) 5(2 – x) – 2(3 – 6x) + 2(x – 1) > 0 (b) [ ]
* + (c)
(c) ( ) ( )[ ] * +
(d) ( ) ( ) * + 6. Solve the following inequalities
2. Solve the following inequalities. Correct 2 (a) | | | |
[ ]
decimal places
(a) (0.8)-3x > 4.0 [ x > 2.07 (2dp)] (b) | | [x < -4 and x> 0]
(b) (0.6)-2x< 3.6 [x< 1.25 (2dp)] (c) | | * +
3. Find the integers which simultaneously
satisfy the following inequalities (d) | | [x < 2, and x > 6]
(a) 3x + 2 ≥ 2x – 1, 7x + 3 < 5x + 2 7. If y = , find the range of possible
{-3, -2, -1}
values of y for which x is real * +
(b) ( ) , 5x + 1 < 4(x + 2)
8. Find the range of values of x can take for
{2, 3, 4, 5, 6} the following inequalities to be true
4. Find the set of values of x for which
(a) | | [ x < -4, x > 0]
(a) * + (b) | | | | [x > 1]
(b) [ x < -1, x > 1]
(c) 2(x – 5) < x2 + 6 [-4 < x < 4]
2
Thank you
(d) x2 – x – 12 > 0 [ x < -3, x > 4]
(e) 2x(x + 3) > (x + 2)(x – 3) [ x < -6, x > -1] Dr. Bbosa Science

digitalteachers.co.ug
Permutations and combinations Example 1

Permutation Find the values of the following expression

A permutation is an ordered arrangement of a (a) 5!


number of objects Solution
5!= 5 x 4 x 3 x 2 x 1 = 120
Consider digits 1, 2 and 3; find the possible (b)
arrangements of the digits
Solution
123, 132, 321
The total of six
231, 321, 321
(c)
This problem may also be solved as follows:
Solution
Given the three digits above, the first position =
can take up three digits, the second position
can take up two digits and the third position
(d) Four different pens and 5 different books
can take up 1 digit only
are to be arranged on a row. Find
1st position 2nd position 3rd position (i) The number of possible arrangements
3 2 1 of items
The total is thus 3 x 2 x 1 = 6 Solution
Total number of items = 4 + 5 = 9
If the digits were four say 1, 2, 3, 4 the Total number of arrangements = 9!
arrangement would be =9x8x7x6x5x4x3x2x1
1st 2nd 3rd 4th = 362,880 ways
4 3 2 1 (ii) The number of possible arrangements
The total is thus 4 x 3 x 2 x 1 = 24 if three of books must be kept together
Solution
In summary the number of ways of arranging The pens are taken to be one since
n different items in a row is given by they are to be kept together. So we
n(n – 1)(n – 2)(n -3) x ……….2 x 1 and can be consider total number of items to six.
expressed as n! The number of arrangements of six
If the total number of books is 6 items = 6! = 6 x 5 x 4 x 3 x 2 x 1
= 720 ways
The total number of arrangements = 6! The arrangement of 4 pens = 4!
= 6 x 5 x 4 x 3 x 2 x 1 = 720 ways = 4 x 3 x 2 x 1 =24
Total number arrangements of all the
items = 720 x 24 = 17, 280

digitalteachers.co.ug
Multiplication principle of permutation Number of ways in which David arranges = 5

If one operation can be performed Number of ways in which John arranges = 3


independently in a different ways and the
Number of ways in which either David or John
second in b different ways, then either of the
arrange the items = 5 + 3 = 8 ways
two events can be performed in (a + b) ways
The number of permutation of r objects
Example 2
taken from n unlike objects
There are 6 roads joining P to Q and 3 roads
The permutation of n unlike objects taking r at
joining Q to R. Find how many possible routes
a time is denoted by nPr which is defined as
are from P to R
n
From P to Q = 6 ways Pr = ( )
, where r ≤ n.

From Q to R = 3 ways In case r = n, we have nPn which is interpreted


as the number of arranging n chosen objects
Number of routes from P to R = 6 x 3 = 18
from n objects denoted by n!
Example 3 n
Pn = ( )
Peter can eat either matooke, rice or posh on
any of the seven days of the week. In how Example 6
many ways can he arrange his meals in a week
How many three letter words can be formed
Solution from the sample space {a, b, c, d, e, f}

For each of the 7 days, there are 3 choices Solution

Total number of arrangements Total number of letters = 6 and r = 3

= 3 x 3 x 3 x 3 x 3 x 3 x 3 = 37 = 2187 ways Total number of worms = 6P3

Example 4 =( ways
)
There are four routes from Nairobi to
Example 7
Mombasa. In how many different ways can a
taxi go from Nairobi to Mombasa and Find the possible number of ways of arranging
returning if for returning: 3 letters from the word MANGOES

(a) any of the route is taken Solution


= 4 x 4 = 16 ways
Total number of letter in the word = 7
(b) the same route is taken
= 4 x 1 = 4 ways and r = 3
(c) the same route is not taken
= 4 x 3 = 12 ways Number of ways 7P3 = ( )

Example 5 = ways
David can arrange a set of items in 5 ways and
Example 8
John can arrange the same set of items in 3
ways. In how many ways can either David or Find number of ways of arranging six boys
John arrange the items? from a group of 13
Solution Solution

digitalteachers.co.ug
Number of arrangements = 13P6 Solution

=( The word ‘MISSISSIPPI’ has 10 letters with 4I,


)
4S, and 2P

The number of ways = 34650


= 1235520 ways
The number of permutations of the like and
The number of permutations of n objects of unlike objects with restrictions
which r are alike
One should be cautious when handling these
The number of permutations of n objects of problems
which r are alike is given by Example 12
Example 9 Find the possible number of ways of arranging
Find the number of arranging in a line the The letters of the word MINIMUM if the
letters B, C, C, C, C, C, C arrangement begins with MMM?
The number of ways of arranging the seven Solution
letters of which of which 6 are alike
There is only one way of arranging MMM
ways
The remaining contain four letters with 2I can
The number of ways of permutations on n be arranged in
objects of which p of one type are alike, q of
the second type are alike, r of the third type ways
are alike, and so on.
Example 13
The number of ways of permutations on n
objects of which p of one type are alike, q of (a) How many 4 digit number greater than
the second type are alike, r of the third type 6000 can be formed from 4, 5, 6, 7, 8 and
9 if:
are alike given by
(i) Repetitions are allowed
Example 10 Solution
The first digit can be chosen from 6, 7,
Find the possible number of ways of arranging 8 and 9, hence 4 possible ways, the
the letter of the word MATHEMATICS in line 2nd, 3rd and 4th are chosen from any of
Solution the six digits since repetitions are
allowed
The word MATHEMATICS has 11 letters and position 1st 2nd 3rd 4th
contains 2 M, 2A and 2T repeated selections 4 6 6 6
Number of ways
The number of ways = = 4 x 6 x 6 x 6 = 864 ways

= (ii) Repetition are not allowed


= 4,989,600 The first can be chosen from 6, 7, 8
and 9, hence 4 possible ways, the 2nd
Example 11 from 5, 3rd from 4 and 4th from 3 since
no repetitions are allowed
Find the possible number of ways of arranging
position 1st 2nd 3rd 4th
the letter of the word ‘MISSISSIPPI’ in line
selections 4 5 4 3

digitalteachers.co.ug
Number of ways (d) How many odd numbers greater than
= 4 x 5 x 4 x 3 = 240 ways 60000 can be formed from 0, 5, 6, 7, 8, 9,
if no number contains any digit more than
(b) Find how many four digit numbers can be once
formed from the six digits 2, 3, 5, 7, 8 and Solution
9 without repeating any digit. Considering six digits
Find also how many of these numbers Taking the first digit to be odd, the first
(i) Are less than 7000 digit is selected from 3 digits (5, 7, 9) and
(ii) Are odd the last is selected from 2 digits
1st 2nd 3rd 4th 5th 6th
Solution
3 4 3 2 1 2
position 1st 2nd 3rd 4th
selections 6 5 4 3 Number of ways = 3 x 4 x 3 x 2 x 1 x 2
Total number of ways = 6 x 5 x 4 x 3 = 360 = 144
Taking the first digit to be even, the first
(i) The 1st number is selected from three digit is selected from 2 digits (6, 8) since
(2, 3, 5), the 2nd number from 5, the 3rd the number should be greater than 60000
from 4 and the 4th from 3 digits and the last is selected from 2 odd digits
position 1st 2nd 3rd 4th 1st 2nd 3rd 4th 5th 6th
selections 6 5 4 3 2 4 3 2 1 3
Total number of less than 7000
Number of ways = 3 x 4 x 3 x 2 x 1 x 2
= 3 x 5 x 4 x 3 = 180 = 144
(ii) The last number is selected from four Considering five digits
odd digits (3, 5, 7, and 9), the 1st Taking the first digit to be odd, the digit
number selected from five remaining, greater than 6 are 7 and 9 so first digit is
2nd from 4 and 3rd from 3 selected from 2 digits and the last is
selected from 2 digits
position 1st 2nd 3rd 4th 1st 2nd 3rd 4th 5th
selections 5 4 3 4 2 4 3 2 2
Total number of odd numbers formed
Number of ways = 2 x 4 x 3 x 2 x 2
= 5 x 4 x 3 x 4 = 240
= 96
(c) How many different 6 digit number Taking the first digit to be even, the first
greater than 500000 can be formed by digit is selected from 2 digits (6, 8) since
using the digits 1, 5, 7, 7, 7, 8 the number should be greater than 60000
Solution and the last is selected from 3 odd digits
The 1st digit is selected from five (5, 7, 7, (5, 7, 9)
7, 8), the 2nd from remaining five, 3rd from 1st 2nd 3rd 4th 5th
four, 4th from three, 5th from two and 6th 2 4 3 2 3
from one
1st 2nd 3rd 4th 5th 6th Number of ways = 2 x 4 x 3 x 2 x 3
5 5 4 3 2 1 = 144
The total number of selections
Total number = = 100 = 144 + 144 + 96 + 144 = 528
NB. The number is divided by 3! Because 7
appears three times

digitalteachers.co.ug
Example 14 The number of permutation of n different
The six letter of the word LONDON are objects taken r at a time, if repetition are
each written on a card and the six cards permitted
are shuffled and placed in a line. Find the
Example 16
number of possible arrangements if
(a) The middle two cards both have the How many four digit numbers can be formed
letter N on them from the sample space {1, 2, 3, 4, 5} if
Solution repetitions are permissible
If the middle letter are NN, the we
need to find the number of different Solution
arrangements of the letter LODO. The 1st position has five possibilities, the 2nd
With the 2O’s, the number of five, the 3rd five, the 4th five
arrangements =
Number of permutations = 5 x 5 x 5 x 5 = 625

(b) The two cards with letter O are not


adjacent and the two cards with letter
Circular permutations
N are also not adjacent
Solution Here objects are arrange in a circle
If the two cards are not adjacent, the
number of arrangements = Total The number of ways of arranging n unlike
number of arrangements of the word objects in a ring when clockwise and
LONDON – number of arrangements anticlockwise are different.
when the two letters are adjacent Consider four people A, B, C and D seated at a
== round table. The possible arrangements are as
shown below
Example 15

In how many different ways can letters of the


word MISCHIEVERS be arranged if the S’s
cannot be together

Solution

There are 11 letters in the word MISCHIEVERS


with 2S’s, 2I’s and 2E’s

Total number of arrangements

= = 4989600
With circular arrangements of this type, it is
If S’s are together, we consider them as one, the relative positions of the objects being
so the number of arrangements arranged which is important. The
arrangements of the people above is the
= = 907200 same. However, if the people were seated in a
line the arrangements would not be the same,
the number of possible arrangements of the i.e. A, B, C, D is not the same as D, A, B, C.
word MISCHIEVERS when S’s are not together When finding the number of different
= 4989600 – 907200 = 4082400 arrangements, we fix one person say A and
find the number of ways of arranging B, C and
D.

digitalteachers.co.ug
Therefore, the number of different Then the above arrangements are the same
arrangements of four people around the table since one is the other viewed from the
is 3! opposite side

Hence the number of different arrangements The number of arrangements = = 3ways


of n people seated around a table is (n – 1)!
Hence the number of ways of arranging n
Example 17 unlike objects in a ring when clockwise and
(a) Seven people are to be seated around a anticlockwise arrangements are the same
( )
table, in how many ways can this be done =
Solution
The number of ways = (7 – 1)! = 6! Example 18
= 720 A white, a blue, a red and two yellow cards at
(b) In how many ways can five people A, B, C, arranged on a circle. Find the number of
D and E be seated at a round table if arrangements if red and white cards are next
(i) A must be seated next to B to each other.
Solution Solution
If A and B are seated together, they are If red and white cards are next to each other,
taken as bound together. So four people they are considered as bound together. So we
are considered have four cards. Since anticlockwise and
The number of ways = (4 – 1)! = 3! = 6 clockwise arrangements are the same and
there are two yellow cards, the number of
The number of ways in which A and B can ( )
arrangements =
be arranged = 2

The total number of arrangements The number of ways of arranging red and
white cards = 2
= 6 x 2 = 12 ways
(ii) A must not seat next to B Total number of ways of arrangements

If A and B are not seated together, then = =3


the number of arrangements = total
number of arrangements – number of Revision exercise 1
arrangements when A and B are seated 1. In how many ways can the letters of the
together words below be arranged
= (5 – 1)! – 12 = 12 ways (a) Bbosa (5!]
(b) Precious [8!]
The number of ways of arranging n unlike 2. How many different arrangements of the
objects in a ring when clockwise and letters of the word PARALLELOGRAM can
anticlockwise arrangements are the same be made with A’s separate *83160000]
3. How many different arrangements of the
Consider the four people above, if the
letters of the word CONTACT can be made
arrangement is as shown below
with vowels separated? [900]
4. How many odd numbers greater than
6000 can be formed using digits 2, 3, 4, 5
and 6 if each digit is used only once in
each number [12]

digitalteachers.co.ug
5. Three boys and five girls are to be seated Example 19
on a bench such that the eldest girl and
A committee of four people is chosen at
eldest boy sit next to each other. In how
random from a set of seven men and three
many ways can this be done [2 x 7!]
women
6. A round table conference is to be held
between delegates of 12 countries. In How many different groups can be chosen if
how many ways can they be seated if two there is at least one
particular delegates wish to sit together
[2 x 10!] (i) Woman on the committee
7. In how many ways can 4 boys and 4 girls Solution
be seated at a circular table such that no Possible combinations
two boys are adjacent [144] 7 men 3 women
8. How many words beginning or ending 3 1
2 2
with a consonant can be formed by using
1 3
the letters of the word EQUATION? [4320]
The number of ways of choosing at least one
woman

Combinations =( ) ( ) ( ) ( ) ( ) ( )
A combination is a selection of items from a
=
group not basing on the order in which the
items are selected (ii) Man on the committee
Consider the letters A, B, C, D 7 men 3 women
1 3
The possible arrangements of two letters 2 2
chosen from the above letters are 3 1
4 0
AB, AC, AD, BA, BC, BD, CA, CB, CD, DA, DB,
The number of ways of choosing at least one
DC. AS seen earlier, the total number of
man
arrangements of the above letters is
expressed as ( )
= ( ) ( ) ( ) ( ) ( ) ( ) ( ) ( )

However, when considering combinations, the =


grouping such as AB and BA are said to be the
same groupings such as CA and AC, AD and =210
DA, etc. Example 20
So the possible combinations are AB, AC, AD, A group of nine has to be selected from ten
BC, BD, CD which is six ways. men and eight women. It can consist of either
Thus the number of possible combinations of five men and four women or four men and
n items taken r at a time is expressed as nCr or five women. How many different groups can
be chosen?
( ) which is defined as nCr = ( )
where r ≤ n Solution

Hence the number of combinations of the Possible combination


above letters taken two at a time is
10 men 8 women
5 4
( )
4 5

digitalteachers.co.ug
Number of groups = ( ) ( ) ( ) ( ) Example 23

(a) Find the number of different selections of


= x x = 29400 4 letters that can be made from the word
UNDERMATCH.
Example 21
Solution
A team of six is to be formed from 13 boys There are 10 letters which are all different
and 7 girls. In how many ways can the team Number of selections of 4 letters from 10
be selected if it must consist of is given by ( ) ( )
(a) 4 boy and 2 girls (b) How many selections do not contain a
13 boys 7 girls vowel?
4 2 Solution
( ) ( )= = 15015 Number of vowels in the word = 2
Number of letters not vowels = 8
(b) At least one member of each sex
Number of selections of 4 letters from 10
Possible combinations without containing a vowel = selecting 4
letters from 8 consonants =
13 boys 7 girls
( )
5 1 ( )
4 2
Example 24
3 3
2 4 In how many ways can three letters be
1 5 selected at random from the word BIOLOGY is
=( ) ( ) ( ) ( ) ( ) ( ) selection

( ) ( ) ( ) ( ) (a) Does not contain the letter O


Solution
= Number of selections without the letter O
= number of ways of choosing three
letters from B, I, L, G, y
= 37037 ( )
Example 22 (b) Contain only the letter O

A team of 11 players is to be chosen from a Solution


group of 15 players. Two of the 11 are to be
Number of selections with one letter O =
randomly elected a captain and vice-captain
number of ways of choosing two letters
respectively. In how many ways can this be
from B, I, L, G, y
done?

Number of ways of choosing 11 players from ( )

15 = ( ) (c) Contains both of the letters O

Solution
A captain will be elected from 11 players and
a vice-captain from 10 players Number of selections with two letter O
= number of ways of choosing one letter
Total number of selection =( ) from B, I, L, G, y

= x 11 x 10 = 150150 ( )

digitalteachers.co.ug
Example 25 Example 26

In how many ways can four letters be selected How many different selections can be made
at random from the word BREAKDOWN if the from 26 different letters of the alphabet?
letters contain at least one vowel?
Number of selection = 226 – 1
Solution
= 67,108,863
Vowels: E, A, O (3)
Cases involving repetitions
Consonants: B, R, K, D, W, N (6)
Suppose we need to find the number of
Consonants (6) Vowels (3) possible selections of letters from a word
3 1 containing repeated letters, we take the
2 2 selections mutually exclusive
1 3
Number of selection of four letters with at Example 27
least one vowel
How many different selections can be made
from the letters of the word CANADIAN?
=( ) ( ) ( ) ( ) ( ) ( )
Solution
Example 26
There are 3A’s, 2N’s and 3 other letters
How many different selections can be made
from the six digits 1, 2, 3, 4, 5, 6 The A’s can be dealt with in 4 ways (either no
A, 1A’s, 2A’s or 3A’s)
Solution
The N’s can be dealt in 3 ways (no N, 1N, or
Note: this an open questions because 2N’s)
selections can consist of only one digit, two
digits, three digits, four digits, five digits or six The C can be dealt with in 2 ways (no C, 1C)
digits The D can be dealt with in 2 ways (no D, 1D)
6
Number of selection of 1 digit = C1 = 6 The I can be dealt with in 2 ways (no I, 1I)
Number of selection of 2 digits = 6C2 = 15 The number of selections
6
Number of selection of 3 digits = C3 = 20 = 4 x 3 x 2 x 2 x 2 – 1 = 95
6
Number of selection of 4 digits = C4 = 15 Example 28
6
Number of selection of 5 digits = C5 = 6 How many different selections can be made
Number of selection of 6 digits = C1 = 1 6 from the letters of the word POSSESS?

Total number of selections Solution

= 6 + 15 + 20 + 15 +6 + 1 = 63 There are 4S’s and 3 other letters

This approach is tedious for a large group of The S’s can be dealt in 5 ways (no S, 1S, 2N’s,
objects. 3S’s, 4S’s, or 5S’s)

The general formula for selection from n The P can be dealt with in 2 ways (no P, 1P)
unlike objects is given by 2n – 1. The O can be dealt with in 2 ways (no O, 1O)
For the above problems, number of selections The E can be dealt with in 2 ways (no E, 1E)
= 26 – 1 = 63

digitalteachers.co.ug
Total number of selections = 5 x 2 x 2 x 2 – 1 (a) Number of ways = = 24310
= 39 (b) Number of ways = = 2858856

Cases involving division into groups (c) Number of ways = =190590400


(d) Number of ways = = 14702688
The number of ways of dividing n unlike
objects into say two groups of p and q where Example 31
p + q = n is given by
(a) Find how many words can be formed
For three groups of p, q and r provided using all letters in the word MINIMUM.
p+q+r=n
Solution
Number of ways of division = Number of ways of arranging the letters = 7!

However, for the two groups above, if p = q There are 3M’s and 2I’s
then the number of ways of division =
Number of words formed =
For three groups where p = q = r (b) Compute the sum of four-digit numbers
then the number of ways of division = formed with the four digits 2, 5, 3, 8 if
each digit is used only once in each
Example 29 arrangement

The following letters a, b, c, d, e, f, g, h, I, j, k, l Solution


are to be divided into groups containing Number of ways of arranging a four digit
(a) 3, 4, 5 number = 4!
(b) 5, 7 Sum of any four digit number formed
(c) 6, 6 = 2 + 5 + 3 + 8 = 18
(d) 4, 4, 4 letters. In how many ways can this
be done? Total sum of four digit numbers formed

Solution = 18 x 4! = 432

(a) Number of ways = = 27720 (c) A committee consisting of 2 men and 3


women is to be formed from a group of 5
(b) Number of ways = = 792 men and 7 women. Find the number of
(c) Number of ways = = 462 different committees that can be formed.
If two of the women refuse to serve on
(d) Number of ways = =5775
the same committee, how many
Example 30 committees can be formed?

Find the number of ways that 18 objects can Solution


be arranged into groups if there are to be The committees formed = 5C2.7C3
(a) Two groups of 9 objects each = 10 x 35 = 350
(b) Three groups of 6 objects each
(c) 6 groups of 3 objects each Suppose two women are to serve together,
(d) Three groups of 5, 6 and 7 objects each we take them as glued together, so the
number of committees =5C2.6C3 = 200
Solution

digitalteachers.co.ug
Number of committees in which two women girls to join a discussion group
refuse to serve together = 350 – 200 = 150 [15 ways]
4. Find in how many ways 11 people can be
divided into three groups containing 3, 4,
Revision exercise 2 4 people each. [5775]
5. A group of 5 boys and 8 girls. In how many
1. (a) Find the number of different selection ways can a team of four be chosen, if the
of 3 letters that can be made from the team contains
word PHOTOGRAPH. [53] (a) No girl [5]
(b) How many of these selections contain (b) No more than one girl [85]
no vowel [18] (c) At least two boys [365]
(c) How many of these selections contain 6. Calculate the number of 7 – letter
at least one vowel? [35] arrangements which can be made with
2. (a) find the number of different selections the letters of the word MAXIMUM. In how
of 3 letters that can be made from the many of these do all the 4 consonants
letters of the word SUCCESSFUL.[36] appear next to each other? [840, 96]
(c) How many of these selections contain 7. In how many ways can a club of 5 be
only consonants [11] selected from 7 boys and 3 girls if it must
(d) How many of these selections contain contain
at least one vowel [25] (a) 3 boys and 3 girls [105]
3. (a) Find the value of n if nP4 = 30nC5 [8] (b) 2 men and 3 girls [21]
(b) How many arrangement can be made (c) At least one girl [231]
from the letters of the name 8. How many different 6 digit numbers
MISSISSIPPI greater than 400,000 can be formed form
(i) when all the letters are taken at a the following digits 1, 4, 6, 6, 6 7? [100]
time [34650]

(ii) If the two letters PP begin every Thank you


word [630 ways]
Dr. Bbosa Science
(c) Find the number of ways in which a
one can chose one or more of the four

digitalteachers.co.ug
A-level math paper 2: Correlations and scatter diagrams
Correlation refers to the degree of correspondence or relationship between two variables.
Correlated variables tend to change together. If one variable gets larger, the other one
systematically becomes either larger or smaller.

The degree of correlation/association is determined by rank correlation coefficients


There are two types
1. Spearman rank correlation coefficient (ρ)
2. Kenddall’s rank correlation coefficient (τ

Interpretation of the rank correlation coefficients


A rank correlation coefficient measures the degree of similarity between two rankings
The table below is used
Correlation coefficient Interpretation
0-0-19 Very low correlation
0.2-0.39 Low correlation
0.4-0.59 Moderate correlation
0.6 – 0.79 High correlation
0.8 – 1.0 Very high correlation

Note: the positive or negative signs indicate positive or negative relationships respectively. Or they
the relationships are directly or inversely related.
The closer to zero the lower the relationship

Spearman rank correlation (ρ)


6 ∑ 𝑑2
It is given by ρ = 1- [ ]
𝑛(𝑛2 −1)
Where d = difference between ranks
n = total number of pairs
Example 1
Two examiners marked the scripts of 8 candidates. The table shows the marks awarded by two
examiners x and y.
x 72 60 56 76 68 52 80 64
y 56 44 60 74 66 38 68 52
Calculate the rank correlation coefficient and comment on your reults
Solution
Rx Ry d d2
3 5 2 4 6 ∑ 𝑑2
ρ = 1- [ ]
𝑛(𝑛2 −1)
6 7 1 1 6 𝑥 18
7 4 3 9 = 1- [ ] = 0.786
8(82 −1)
2 1 1 1 There is a high positive correlation
4 3 1 1 between x and y
8 8 0 0
1 2 -1 1
5 5 1 1
∑ 𝑑 2 = 18

Example 2
The following shows the marks obtained by 10 students in mathematics and physics exams.
Mathematics 80 80 70 60 65 80 68 90 95 50
Physics 50 45 70 80 70 90 70 80 70 95
Calculate the ranks correlation coefficient and comment on your results
Solution
RM RP d d2
6 ∑ 𝑑2
4 9 -5 25 ρ = 1- [ ]
𝑛(𝑛2 −1)
6 6.5 0.5 0.25 = 1- [
6 𝑥 211.5
] = -0.282
10(102 −1)
9 3.5 5.5 30.25
There is a low negative correlation
8 6.5 1.5 2.25
between mathematics and physics
4 2 2 4
7 6.5 0.5 0.25
2 3.5 1.5 2.25
1 6.5 -5.5 30.25
10 1 9 81
∑ 𝑑 2 = 211.5

Example 3
The following table gives the order in which six candidates were ranked in two tests x and y
x E C B F D A
y F A D E C C
Calculate the rank correlation coefficient and comment on your results
Solution
Rx Ry d d2
6 ∑ 𝑑2
5 6 1 1 ρ = 1- [ ]
𝑛(𝑛2 −1)
3 1 2 4 6 𝑥 14.5
= 1- [ ] = -0.586
2 4 2 4 6(62 −1)
There is a moderate negative correlation
6 5 1 1
between x and y
4 2.5 1.5 2.25
1 2.5 -1.5 225
∑ 𝑑 2 = 14.5

Kendall’s Rank correlation coefficient


It is a coefficient that represents the degree of concordance/agreement between toe columns of
ranked data. The greater the ‘inversions’ the smaller the coefficient will
𝐶−𝐷
Kendall’s Rank correlation coefficient, τ (tau) =
𝐶+𝐷
Where C = number of concordant pairs or pairs in agreement
D = number of disconcordant pairs or pairs in disagreement

The Tau correlation coefficient returns a value of 0 to 1, where: 0 is no relationship, 1 is a perfect


relationship
- Concordant pairs are the number of observed ranks below a particular rank which are larger
than that particular rank.
- Disconcordant pairs are the number of observed rank below a particular rank which are smaller
than that particular rank

Example 4
Two examiners marked the scripts of 8 candidates. The table shows the marks awarded by two
examiners x and y.
x 72 60 56 76 68 52 80 64
y 56 44 60 74 66 38 68 52
Calculate the rank correlation coefficient and comment on your results
Solution
Ranking values
The subscripts are the ranks
x 723 606 567 762 684 528 801 645
y 565 447 604 741 663 388 682 526

- The ranks of x and y are filled in the table as below those of x in ascending order and those of y
correspondingly as shown in the table below.
- The values of C are bigger values in the column Ry bigger than and below a particular value in
that column. While the values of D are sallerr values in the column Ry bigger than and below a
particular value in that column
- For instance, the first value of C in the table below is the number of values bigger than and
below 2 in column Ry i.e. (5, 3, 6, 7, 4, 8) =6; the first value D = the number of values smaller
than and below 2 in column Ry; i.e. (1) = 1

Rx Ry C D 𝐶−𝐷
τ (tau) =
𝐶+𝐷
1 2 6 1 22−5
τ (tau) = = 0.63
2 1 6 0 22+5

3 5 3 2 moderate positive correlation


4 3 3 0
5 6 2 1
6 7 1 1
7 4 1 0
8 8
∑ 𝐶 = 22 ∑𝐷 = 5

Example 5
The height (cm) an ages (years) of random sample of ten farmers are given in the table below

Height 156 151 152 160 146 157 149 142 158 140
(cm)
Ages 47 38 44 55 46 49 54 52 45 30
(years)
(a)(i) Calculate the Kendall rank correlation coefficient
(ii) comment on your result (06marks)

Let the farmers be A, B, C, D, E, F, G, H, I, J (the subscripts are the rank)


Farmers A B C D E F G H I J
Height 1564 1516 1525 1601 1468 1573 1497 1429 1582 14010
Age 475 389 448 551 466 494 542 523 457 3010

By re-arranging the findings we have


Farmers D I F A C B G E H J
Height 1 2 3 4 5 6 7 8 9 10
Age 1 7 4 5 8 9 2 6 3 10
Agreements 9 3 5 4 2 1 3 1 1 =29
(C)
Disagreements 0 4 2 2 3 3 0 1 0 =15
(D)
𝐶−𝐷
τ (tau) =
𝐶+𝐷
29−15
τ (tau) = = 0.32
29+15
low positive correlation
Significance of ranks correlation coefficients
The calculated ranks correlation coefficients to be statistically significant; the calculated value should
be greater than that from the table critical values associated with the various sample sizes and
significance levels (α).
That is
 If the |𝜌𝑐 | > |𝜌𝑇 | 𝑜𝑟 |𝜏𝑐 | > |𝜏 𝑇 | , a significant relationship exists
 If the |𝑃𝑐 | < |𝑃𝑇 | 𝑜𝑟 |𝜏𝑐 | < |𝜏 𝑇 | , no significant relationship exists
Where 𝜌𝑐 = calculated Spearman’s correlation coefficient
𝜌𝑇 = Table Spearman’s correlation coefficient at either 1% (α=0.01) or 5% (α =0.05)
𝜏𝑐 = calculated Kendall’s correlation coefficient
𝜏 𝑇 = Table Kendall’s correlation coefficient at either 1% (α=0.01) or 5% (α =0.05)
Example 6
The following shows the marks obtained by 8students in mathematics and physics exams
Mathematics 65 65 70 75 75 80 85 85
Physics 50 55 58 55 65 58 61 65
Calculate the ranks correlation coefficient and comment of the significance of your results at 5%
level (Spearman’s 𝜌 = 0.71), 𝐾𝑒𝑛𝑑𝑎𝑙𝑙 ′ 𝑠 𝜏 = 0.64
(a) Using Spearman’s correlation coefficient 6 ∑ 𝑑2
2 ρ = 1- [ 2 ]
RM RP d d 𝑛(𝑛 −1)
6 𝑥 21
7.5 8 0.5 0.25 = 1- [ ] = 0.75
8(82 −1)
7.5 6.5 1 1 Since ρC(0.75)> ρT (0.71), a
6 4.5 1.5 2.25 significant relationship exist
4.5 6.5 -2 4
4.5 1.5 3 9
3 4.5 -1.5 2.25
1.5 3 -1.5 2.25
1.5 1.5 0 0
∑ 𝑑 2 = 21

Using Kendall’s correlation coefficient


RM RP C D 𝐶−𝐷
τ (tau) =
1.5 1.5 6 0 𝐶+𝐷
22−3
1.5 3 5 1 τ (tau) = = 0.76
22+3
3 4.5 3 1 Since τC(0.75)> τT (0.64), a
4.5 1.5 4 0 significant relationship exist
4.5 6.5 1 1
6 4.5 2 0
7.5 6.5 1 0
7.5 8 =22 3
Scatter graphs
They are graphs showing the relationship between two variables

Example 7
The heights and masses of ten students are given in the table below
Height 156 152 152 146 160 157 149 142 158 68
(cm)
Mass 62 58 63 58 70 60 55 57 68 56
(kg)
(a)(i) Plot the data on a scatter diagram
(ii) Draw the line of the best fit. Hence estimate the mass corresponding to height of 155cm
(b) (i) Calculate the rank correlation coefficient for the data.
(ii) Comment on the significance of the heights on the masses of the students.[Spearman’s ρ =
0.79 and Kendall’s τ = 0.64at 1% level of significance based on 10 observations]
Solution
(a)(i)

The weight corresponding to height 155cm is 65kg


Note: this value may vary from 63 to 67kg depending on how one has drawn the line of the best fit.
(b)(i) Using Spearman’ ran correlation coefficient
Heights (x) Mass (y) Rx Ry d d2 6 ∑ 𝑑2
ρ = 1- [ ]
156 62 4 4 0 0 𝑛(𝑛2 −1)
6 𝑥 21.5
151 58 6 6.5 0.5 0.25 = 1- [ ] = 0.8697
10(102 −1)
152 63 5 3 2 4 Since ρC(0.87)> ρT (0.79), a
146 58 8 6.5 1.5 2.25 significant relationship
160 70 1 1 0 0 exist between height and
weight of students at 1%
157 60 3 5 -2 4
level
149 55 7 10 -3 9
142 57 9 8 1 1
158 68 2 2 0 0
141 56 10 9 1 1
∑ 𝑑 2 = 21.5

Using Kendall’s method


By naming the pairs we have
A(156, 62), B(151, 58), C(152, 63), D(146, 58), E(160, 70), F(157, 60), G149, 55), H(142, 57), I(158, 68),
J(141, 56)
E I F A C B G D H J
x 1 2 3 4 5 6 7 8 9 10
y 1 2 5 4 3 6.5 10 6.5 8 9
C 9 8 5 5 5 3 0 2 1 =38
D 0 0 2 1 0 0 3 0 0 =6
𝐶−𝐷
τ (tau) =
𝐶+𝐷
38−6
τ (tau) = = 0.73
38+6
Since τC(0.73)> τT (0.64), a significant relationship exist between the heights and masses of student.

Revision questions
1. UNEB 1990/212
Eight candidate seeing admission to a University sat for written and oral test. The scores were
shown below
Written 55 54 35 62 87 53 71 50
(x)
Oral (y) 57 60 47 65 83 56 74 63
(a) Plot the result on a scatter diagram. Comment on the relationship between the written test
and oral test
(b) Draw the line of the best fit on your graph and use it to estimate y when x = 70.
(c) Calculate the rank correlation coefficient. Comment on your results
2. UNEB 1990/2/12
The pairs of observation have been made on two random variables x and y. the ten (x, y) are
(0, 20), (-7, 12), (-10, 15), (-12, 22), (-17, 5), (-30, -5), (-32, 13), (10, 30), (15, 40), and (-12, 8).
(a) Draw the results on a scatter diagram
(b) Draw the line of the best fit
(c) Estimate the expected value of y corresponding to x = -7
(d) Calculate the rank correlation coefficient and comment on the significance of the results at
1% significance level. (ρ = 0.894, τ= 0.778)
3. UNEB 1991/2/12
Three examiners X, Y and Z each marked scripts of ten candidates who sat for mathematics
examination. The table below shows the examiner’s ranking of candidates.
A B C D E F G H I J
X 8 5 9 2 10 1 7 6 3 4
Y 5 3 6 1 4 7 2 10 8 9
Z 6 3 7 2 5 4 1 10 9 8
Calculate the coefficient of rank correlation of the rankings
(i) X and Y
(ii) Y and Z
(iii) Comment on the significance of each at 5% significant level
4. UNEB 1992/2/13
Three weighing scales from three different shops W, X and Y in a market were used to weigh 10
bags of beans (A, B, C……) and the results in (kg) were given in the table below
A B C D E F G H I J
W 65 68 70 63 64 62 73 75 72 78
X 63 68 68 60 65 60 72 73 70 66
Y 63 74 78 75 64 73 79 70 67 79
Determine the rank correlation coefficient for the performance of the scales
(i) W and X
(ii) X and Y
(iii) Which of the three scales W, X and Y were in good working conditions
5. UNEB 1994/2/14
(a) In many government institution, officers complain about typing errors. A test was designed
to investigate the relationship between typing speed and errors made. Twelve typist A, B, C
…L were picked at random to type a text. The table below shows the rankings of the typist
according to speed and errors made. (N.B lowest ranking in error implies the least errors
Typist A B C D E F G H I J K L
Speed 3 4 2 1 8 11 10 6 7 12 5 9
Errors 2 6 5 1 10 9 8 3 4 12 7 11
(i) Calculate the coefficient of rank correlation
(ii) Comment on the significance at 1% significance level
(b) The cost of travelling at a certain distance away from the city centre is found to depend on
the route and distance a given place is away from the centre. The table below gives average
rates of travel charged for distances to be travelled away from the city centre
Distance 9 12 14 21 24 30 33 45 46 50
(s km)
Rate 750 1000 1150 1200 1350 1250 1400 1750 1600 2000
charged
(r shs)
(i) Plot the above data on a scatter diagram and draw a line of best fit through the points of the
scatter diagram
(ii) Estimate the expected value r corresponding to s = 40km
6. UNEB/1995/2/13
In a certain commercial institution, a speed and error typing examination was administered to 12
randomly selected candidates A, B, C, …, L of the institution. The table below shows their speed (y) in
seconds and the number of errors in their typed scripts (x)
A B C D E F G H I J K L
No. of 12 24 20 10 32 30 28 15 18 40 27 35
errors
(x)
Speed 130 136 124 120 153 160 155 142 145 172 140 157
(y) in
seconds
(i) Calculate the coefficient of rank correlation of the ranking
(ii) Comment on your results
(iii) Plot the above data on a scatter diagram and draw a line of the best fit through the points of
the scatter diagram
7. UNEB 1996/2/16
The following table gives the marks obtained in calculus, physics and statistics by seven students
Calculus 72 50 60 55 35 48 82
Physics 61 55 70 50 30 50 73
Statistics 50 40 62 70 40 40 60
Determine the rank correlation coefficient for the performance of students in
(i) Calculus and physics
(ii) Calculus and statistics
8. UNEB/1999/2/8
Given the table below
x 80 75 86 60 75 92 86 50 64 75
y 62 58 60 45 68 68 81 48 50 70
Determine the rank correlation coefficient between the variable x and y, comment on your
results
9. UNEB2003/2/15
The table below shows the percentage of sand y in the soils at different depth x (in cm)
Soil 35 65 55 25 45 75 20 90 51 60
depth
(x)(cm)
% of 86 70 84 92 79 68 96 58 86 77
sand, y
(a) Plot the results on a scatter diagram. Comment on the relationship between the depth of
the soil and the percentage of sand in the soil
(b) Draw the line of the best fit on you graph and use it to estimate
(i) The percentage of sand in the soil at a depth of 31cm
(ii) Depth of the soil with 54% sand
(iii) Calculate the rank correlation coefficient
10. UNEB 2004/2/7
Eight applicants for a certain job obtained the following marks in aptitude and written test
Applicants A B C D E F G H
Aptitude 33 45 16 42 45 35 40 48
test
Written 57 60 40 75 68 48 54 68
test
(i) Calculate the coefficient of rank correlation of applicant’s performance in the two tests
(ii) Comment on your results
11. UNEB 2005/2/7
The table below shows the marks scored by students in mathematics and fine art tests
students A B C D E F G H I J
Mathematics, 40 48 79 26 55 35 37 70 60 40
Fine art 59 62 68 47 46 39 63 29 55 67
Calculate the coefficient of rank correlation for the students’ performance in the two subjects
and comment on your results.
12. UNEB 2007/2/12
Below are marks scored by 8 students A, B, C … H in Mathematics, Economics and geography in
the end of term examination.
A B C D E F G H
Math 52 75 41 60 81 31 65 52
Economics 50 60 35 65 66 45 60 48
Geography 59 62 68 54 63 40 55 72
Determine the rank correlation coefficient for the performance of students in
(i) Math and economics
(ii) Geography and math
(iii) Comment on the significance of the math in performance of economics and geography.
(ρ = 0.86, τ = 0.79 based on 8 observations at a 1% level of significance)
13. UNEB 2011/2/12
The heights and ages of ten students are given in the table below
Height, 156 151 152 146 160 157 149 142 158 140
cm
Mass, 62 58 63 58 70 60 55 57 68 56
kg
(a) Plot the data on a scatter diagram
(b) Draw the line of best fit on you graph and use it to estimate the mass corresponding to a
height of 155cm
(c) Calculate the rank correlation coefficient for the data. Comment on the significance of the
height on masses of students (ρ = 0.79, τ 0.64 based on 10 observations at 1% level of
significance.
14. UNEB 2013/2/9
The heights and ages of ten farmers are given in the table below
Height, 156 151 152 160 146 157 149 142 158 140
cm
Age, 47 38 44 55 46 49 45 30 45 20
years
(a) Plot the data on a scatter diagram
(b) Draw the line of best fit on your diagram and use it to estimate
(i) Age when height = 147
(ii) Height when the age is 43
(c) Calculate the rank correlation coefficient for the data. Comment on your results (ρ=0.752, τ= 0.6)
15. UNEB 2015/2/12
The table gives the points awarded to eight schools by three judges, J1, J2 and J3 during a music
competition. J1 was the chief judge.
J1 72 50 50 55 35 38 82 72
J2 60 55 70 50 50 50 73 70
J3 50 40 62 70 40 48 67 67
(a) Determine the rank correlation coefficients between the judges of
(i) J1 and J2
(ii) J1 and J3
(b) Who of the two judges had a better correlation with the chief judge? Give a reason.

Answers to the revision exercise

1. UNEB 1990/212
Eight candidate seeing admission to a University sat for written and oral test. The scores were
shown below
Written 55 54 35 62 87 53 71 50
(x)
Oral (y) 57 60 47 65 83 56 74 63
(a) Plot the result on a scatter diagram. Comment on the relationship between the written test
and oral test

(b) Draw the line of the best fit on your graph and use it to estimate y when x = 70. The
value of y when x = 70 is 71
(c) Calculate the rank correlation coefficient. Comment on your results
Using Spearman’s rank correlation method
x y Rx Ry d d2 6 ∑ 𝑑2
ρ = 1- [ ]
55 57 4 6 -2 4 𝑛(𝑛2 −1)
6 𝑥 14
54 60 5 5 0 0 = 1- [ ] = 0.833
8(82 −1)
35 47 8 8 0 0
There is very high correlation
62 65 3 3 0 0 between x and y
87 83 1 1 0 0
53 56 6 7 -1 1
71 74 2 2 0 0
50 63 7 4 3 9
2
∑ 𝑑 =14

Using Kendall’ Rank correlation coefficient method


Rx Ry C D 𝐶−𝐷 25−3
τ= = = 0.786
1 1 7 0 𝐶+𝐷 25+3

2 2 6 0 there is high correlation between x


and y
3 3 5 0
4 6 3 1
5 5 2 1
6 7 1 1
7 4 1 0
8 8 =25 =3

2. UNEB 1990/2/12
The pairs of observation have been made on two random variables x and y. the ten (x, y) are
(0, 20), (-7, 12), (-10, 15), (-12, 22), (-17, 5), (-30, -5), (-32, 13), (10, 30), (15, 40), and (-12, 8).
(a) Draw the results on a scatter diagram
(b) Draw the line of the best fit

(c) Estimate the expected value of y corresponding to x = -7


The value of y corresponding to x = -7 is 15.7
(d) Calculate the rank correlation coefficient and comment on the significance of the results at
1% significance level. (ρ = 0.894, τ= 0.778)
Using Spearman’s rank correlation coefficient
X Y Rx Ry d d2 6 ∑ 𝑑2
ρ = 1- [ ]
0 20 3 4 -1 1 𝑛(𝑛2 −1)
6 𝑥 42.5
-7 12 4 7 -3 9 = 1- [ ] = 0.742
10(102 −1)
-10 15 5 5 0 0
Since ρC (0.742)<ρT (0.894)
-12 22 6.5 3 3.5 12.25
There is no significant
-17 5 8 9 -1 1 relationship between X and Y at
-30 -5 9 10 -1 1 1% significance level
-32 13 10 6 4 16
10 30 2 2 0 0
15 40 1 1 0 0
-12 8 6.5 8 -1.5 2.25
∑ 𝑑 2 =42.5

Using Kendall’s rank correlation coefficient


Rx Ry C D
1 1 9 0 𝐶−𝐷 37−8
τ= = = 0.644
𝐶+𝐷 37+8
2 2 8 0
Since τC (0.644)<τT (0.778)
3 4 6 1
There is no significant relationship
4 7 3 3
between X and Y at 1% significance
5 5 4 1 level.
6.5 3 4 0
6.5 8 2 1
8 9 1 1
9 10 0 1
10 6 =37 =8

3. UNEB 1991/2/12
Three examiners X, Y and Z each marked scripts of ten candidates who sat for mathematics
examination. The table below shows the examiner’s ranking of candidates.
A B C D E F G H I J
X 8 5 9 2 10 1 7 6 3 4
Y 5 3 6 1 4 7 2 10 8 9
Z 6 3 7 2 5 4 1 10 9 8
Calculate the coefficient of rank correlation of the rankings
(i) X and Y
Spearman’s rank correlation
6 ∑ 𝑑2
Rx Ry d d2 ρ = 1- [ ]
𝑛(𝑛2 −1)
8 5 3 9 6 𝑥 186
= 1- [ ] = -0.127
5 3 2 4 10(102 −1)
9 6 3 9 There is low negative
2 1 1 1 relationship between X and Y
10 4 6 36
1 7 -6 36
7 2 5 25
6 10 -4 16
3 8 -5 25
4 9 -5 25
∑ 𝑑 2 =186

Kendall’s rank correlation coefficient

Rx Ry C D 𝐶−𝐷 22−23
τ= = = -0.022
𝐶+𝐷 22+23
1 7 3 6
There is very low negative
2 1 8 0
relationship between X and Y at 1%
3 8 2 5
significance level.
4 9 1 5
5 3 4 1
6 10 0 4
7 2 3 0
8 5 1 1
9 6 0 1
10 4 =22 =23

(ii) Y and Z
Spearman's rank correlation coefficient
Ry Rz d d2 6 ∑ 𝑑2
5 6 -1 1 ρ = 1- [ ]
𝑛(𝑛2 −1)
3 3 0 0 6 𝑥 16
= 1- [ ] = 0.903
10(102 −1)
6 7 -1 1
There is high positive
1 2 -1 1
relationship between Y and Z
4 5 -1 1
7 4 3 9
2 1 1 1
10 10 0 0
8 9 -1 1
9 8 1 1
2
∑ 𝑑 =16
Kendall’s rank correlation coefficient
Rz Ry C D
1 2 9 1 𝐶−𝐷 41−5
τ= = = 0.7826
𝐶+𝐷 41+5
2 1 8 0
There is very high relationship
3 3 7 0
between X and Y
4 7 3 3
5 4 5 0
6 5 4 0
7 6 3 0
8 9 1 1
9 8 1 0
10 10 41 5

(iii) Comment on the significance of each at 5% significant level


(From the tables of critical values at 5% level of significance based on 10 observations, ρ
= 0.648 , τ = 0.467)
(i) For X and Y since|𝜌𝐶 (−0.127)| < |𝜌𝑇 (0.648)| and |𝜏𝐶 (−0.022)| < |𝜏𝜌𝑇 (0.467)| ;
there is no significant relationship between X and Y at 5% significance level.
(ii) For Y and Z since|𝜌𝐶 (0.903)| > |𝜌𝑇 (0.648)| and |𝜏𝐶 (0.7826)| > |𝜏𝜌𝑇 (0.467)| ;
there is significant relationship between Y and Z at 5% significance level.

4. UNEB 1992/2/13
Three weighing scales from three different shops W, X and Y in a market were used to weigh 10
bags of beans (A, B, C……) and the results in (kg) were given in the table below
A B C D E F G H I J
W 65 68 70 63 64 62 73 75 72 78
X 63 68 68 60 65 60 72 73 70 66
Y 63 74 78 75 64 73 79 70 67 79
Determine the rank correlation coefficient for the performance of the scales
(i) W and X (ρ = 0.8)
Spearman’s rank correlation
W X RW RX d d2 6 ∑ 𝑑2
65 63 7 8 -1 1 ρ = 1- [ ]
𝑛(𝑛2 −1)
68 68 6 4.5 1.5 2.25 6 𝑥 33
= 1- [ ] = 0.8
10(102 −1)
70 68 5 4.5 0.5 0.25
There is high positive
63 60 9 9.5 -0.5 0.25
relationship between Y and Z
64 65 8 7 1 1
62 60 10 9.5 0.5 0.25
73 72 3 2 1 1
75 73 2 1 1 1
72 70 4 3 1 1
78 66 1 6 -5 25
2
∑ 𝑑 =33
Kendall’s correlation coefficient
Rw Rx C D
1 6 4 5 𝐶−𝐷 37−6
τ= = = 0.721
𝐶+𝐷 37+6
2 1 8 0
There is high relationship positive
3 2 7 0 between X and Y
4 3 6 0
5 4.5 4 0
6 4.5 4 0
7 8 2 1
8 7 2 0
9 9.5 0 0
10 9.5 =37 =6

(ii) X and Y (ρ = 0.185)


Spearman’s correlation coefficient
X Y Rx Ry d d2 6 ∑ 𝑑2
ρ = 1- [ ]
63 63 8 10 -2 4 𝑛(𝑛2 −1)
6 𝑥 134.5
68 74 4.5 4 0.5 0.25 = 1- [ ] = 0.185
10(102 −1)
68 78 4.5 3 1.5 2.25
There is very low positive
60 75 9.5 4 5.5 30.25 relationship between Y and Z
65 64 7 9 -2 4
60 73 9.5 6 3.5 12.25
72 79 2 1.5 0.5 0.25
73 70 1 7 -6 36
70 67 3 8 -5 25
66 79 6 1.5 4.5 20.25
2
∑ 𝑑 =134.5
Kendall’s rank coefficient
Rx Ry C D 𝐶−𝐷 25−18
1 7 3 6 τ= = = 0.163
𝐶+𝐷 25+18
2 1.5 7 0 There is very low relationship positive
3 8 2 5 between X and Y
4.5 4 3 2
4.5 3 4 1
6 1.5 4 0
7 9 1 2
8 10 0 2
9.5 4 1 0
9.5 6 =25 =18

(iii) Which of the three scales W, X and Y were in good working conditions
W and X are in good working conditions because the show high positive correlation
5. UNEB 1994/2/14
(a) In many government institution, officers complain about typing errors. A test was designed
to investigate the relationship between typing speed and errors made. Twelve typist A, B, C
…L were picked at random to type a text. The table below shows the rankings of the typist
according to speed and errors made. (N.B lowest ranking in error implies the least errors
Typist A B C D E F G H I J K L
Speed 3 4 2 1 8 11 10 6 7 12 5 9
Errors 2 6 5 1 10 9 8 3 4 12 7 11
(i) Calculate the coefficient of rank correlation
Spearman’s rank coefficient
Rs Re d d2
6 ∑ 𝑑2
3 2 1 1 ρ = 1- [ ]
𝑛(𝑛2 −1)
4 6 -2 4 = 1- [
6 𝑥 52
] = 0.8182
12(122 −1)
2 5 -3 9
There is very low positive
1 1 0 0
relationship between Y and Z
8 10 -2 4
11 9 2 4
10 8 2 4
6 3 3 9
7 4 3 9
12 12 0 0
5 7 -2 4
9 11 -2 4
2
∑ 𝑑 =52

Kendall’s rank correlation coefficient

Rs Re C D 𝐶−𝐷 55−11
1 1 11 0 τ= = = 0.667
𝐶+𝐷 55+11
2 5 7 3 There is very low relationship positive
3 2 9 0 between X and Y
4 6 6 2
5 7 5 2
6 3 6 0
7 4 5 0
8 10 2 2
9 11 1 2
10 8 2 0
11 9 1 0
12 12 55 11

(ii) Comment on the significance at 1% significance level


(From the tables of critical values at 1% level of significance based on 12 observations, ρ
= 0.727 , τ = 0.545)
Since|𝜌𝐶 (0.8182)| > |𝜌𝑇 (0.727)| and |𝜏𝐶 (0.667)| > |𝜏𝜌𝑇 (0.545)| ; there is significant
relationship between speed and errors at 1% significance level.
(b) The cost of travelling at a certain distance away from the city centre is found to depend on
the route and distance a given place is away from the centre. The table below gives average
rates of travel charged for distances to be travelled away from the city centre
Distance 9 12 14 21 24 30 33 45 46 50
(s km)
Rate 750 1000 1150 1200 1350 1250 1400 1750 1600 2000
charged
(r shs)
(i) Plot the above data on a scatter diagram and draw a line of best fit through the points of the
scatter diagram

(ii) Estimate the expected value r corresponding to s = 40km


Value of r corresponding to s = 40 is shs 1610

6. UNEB/1995/2/13
In a certain commercial institution, a speed and error typing examination was administered to 12
randomly selected candidates A, B, C, …, L of the institution. The table below shows their speed (y) in
seconds and the number of errors in their typed scripts (x)
A B C D E F G H I J K L
No. of 12 24 20 10 32 30 28 15 18 40 27 35
errors
(x)
Speed 130 136 124 120 153 160 155 142 145 172 140 157
(y) in
seconds
(i) Calculate the coefficient of rank correlation of the ranking
Spearman’s rank correlation
Errors speed Re Rs d d2
12 130 11 10 1 1 ρ = 1- [ 6 ∑ 𝑑2 ]
𝑛(𝑛2 −1)
24 136 7 9 -2 4
6 𝑥 46
20 124 8 11 -3 9 = 1- [ ] = 0.839
12(122 −1)
10 120 12 12 0 0 There is high positive
32 153 3 5 -2 4 relationship between speed
30 160 4 2 2 4 and errors made
28 155 5 4 1 1
15 142 10 7 3 9
18 145 9 6 3 9
40 172 1 1 0 0
27 140 6 8 -2 4
35 157 2 3 -1 1
46
Kendall’s rank correlation
Re Rs C D
𝐶−𝐷 57−9
1 1 11 0 τ= = = 0.72
𝐶+𝐷 57+9
2 3 9 1 There is high positive relationship
3 5 7 2 between speed and errors made
4 2 8 0
5 4 7 0
6 8 4 2
7 9 3 2
8 11 2 2
9 6 3 0
10 7 2 0
11 10 1 0
12 12 57 9
(ii) Comment on your results
(iii) Plot the above data on a scatter diagram and draw a line of the best fit through the points of
the scatter diagram
7. UNEB 1996/2/16
The following table gives the marks obtained in calculus, physics and statistics by seven students
Calculus 72 50 60 55 35 48 82
Physics 61 55 70 50 30 50 73
Statistics 50 40 62 70 40 40 60
Determine the rank correlation coefficient for the performance of students in
(i) Calculus and physics
Spearman’s correlation coefficient
C P Rc Rp d d2 6 ∑ 𝑑2
ρ = 1- [ ]
72 61 2 3 -1 1 𝑛(𝑛2 −1)
6 𝑥 5.5
50 55 5 4 1 1 = 1- [ ] = 0.902
7(72 −1)
60 70 3 2 1 1
There is high positive
55 50 4 5.5 -1.5 2.25 relationship between calculus
35 30 7 7 0 0 and physics
48 50 6 5.5 0.5 0.25
82 73 1 1 0 0
2
∑ 𝑑 =5.5

Kendall’ correlation coefficient


Rc Rp C D 𝐶−𝐷 18−2
τ= = = 0.8
1 1 6 0 𝐶+𝐷 18+2

2 3 4 1 There is high positive relationship


3 2 4 0 between calculus and physics
4 5.5 1 1
5 4 2 0
6 5.5 1 0
7 7 =18 =2

(ii) Calculus and statistics (ρ = 0.64)


Spearman’s correlation
C S Rc Rs d d2 6 ∑ 𝑑2
ρ = 1- [ ]
72 50 2 4 -2 4 𝑛(𝑛2 −1)
6 𝑥 20
50 40 5 6 -1 1 = 1- [ ] = 0.643
7(72 −1)
60 62 3 2 1 1
55 70 4 1 3 9
35 40 7 6 1 1
48 40 6 6 0 0
82 60 1 3 -2 4
2
∑ 𝑑 =20
Kendall’s rank correlation coefficient
Rc Rs C D 𝐶−𝐷 13−5
1 3 4 2 τ= = = 0.444
𝐶+𝐷 13+5
2 4 3 2
3 2 3 1
4 1 3 0
5 6 0 0
6 6 0 0
7 6 =13 =5

8. UNEB/1999/2/8
Given the table below
x 80 75 86 60 75 92 86 50 64 75
y 62 58 60 45 68 68 81 48 50 70
Determine the rank correlation coefficient between the variable x and y, comment on your
results (ρ = 0.715)
Spearman’s correlation
x y Rx Ry d d2
80 62 4 5 -1 1
6 ∑ 𝑑2
75 58 6 7 -1 1 ρ = 1- [ ]
𝑛(𝑛2 −1)
86 60 2.5 6 -3.5 12.25 = 1- [
6 𝑥 47
] = 0.715
10(102 −1)
60 45 9 10 -1 1
75 68 6 3.5 2.5 6.25
92 68 1 3.5 -2.5 6.25
86 81 2.5 1 1.5 2.25
50 48 10 9 1 1
64 50 8 8 0 0
75 70 6 2 4 16
47
Kendall’s correlation coefficient
Rx Ry C D
1 3.5 6 2
𝐶−𝐷 32−13
2.5 6 4 4 τ= = = 0.422
𝐶+𝐷 32+13
2.5 1 7 0
4 5 4 2
6 7 3 2
6 3.5 3 2
6 2 3 0
8 8 2 0
9 10 0 1
10 9 =32 =13
9. UNEB2003/2/15
The table below shows the percentage of sand y in the soils at different depth x (in cm)
Soil 35 65 55 25 45 75 20 90 51 60
depth
(x)(cm)
% of 86 70 84 92 79 68 96 58 86 77
sand, y
(a) Plot the results on a scatter diagram. Comment on the relationship between the depth of
the soil and the percentage of sand in the soil

(b) Draw the line of the best fit on you graph and use it to estimate
(i) The percentage of sand in the soil at a depth of 31cm (91%)
(ii) Depth of the soil with 54% sand (105cm)
(iii) Calculate the rank correlation coefficient
Spearman’s rank correlation coefficient

soil depth %of sand Rx Ry d d2 6 ∑ 𝑑2


(x) (y) ρ = 1- [ ]
𝑛(𝑛2 −1)
35 86 8 3.5 4.5 20.25 6 𝑥 321.5
= 1- [ ] = -0.948
65 70 3 8 -5 25 10(102 −1)

55 84 5 5 0 0
25 92 9 2 7 49
45 79 7 6 1 1
75 68 2 9 -7 49
20 96 10 1 9 81
90 58 1 10 -9 81
51 86 6 3.5 2.5 6.25
60 77 4 7 -3 9
∑ 𝑑 2 =321.5
Kendall’s correlation coefficient
Rx Ry C D
𝐶−𝐷 2−42
1 10 0 9 τ= = = -0.91
𝐶+𝐷 2+42
2 9 0 8
3 8 0 7
4 7 0 6
5 5 1 4
6 3.5 1 2
7 6 0 3
8 3.5 0 2
9 2 0 1
10 1 2 42

10. UNEB 2004/2/7


Eight applicants for a certain job obtained the following marks in aptitude and written test
Applicants A B C D E F G H
Aptitude 33 45 16 42 45 35 40 48
test
Written 57 60 40 75 68 48 54 68
test
(i) Calculate the coefficient of rank correlation of applicant’s performance in the two tests
Aptitude (A) Written (W) RA RW d d2
33 57 7 5 2 4 6 ∑ 𝑑2
ρ = 1- [ ]
𝑛(𝑛2 −1)
45 60 2 4 -2 4
6 𝑥 21.5
16 40 8 8 0 0 = 1- [ ] = 0.744
8(82 −1)
42 75 4 1 3 9 (ii) There is high positive
45 68 3 2.5 0.5 0.25 relationship between
35 48 6 7 -1 1 Aptitude and written
40 54 5 6 -1 1
48 68 1 2.5 -1.5 2.25
21.5
Kendall’s correlation coefficient
Rx Ry C D
𝐶−𝐷 21−6
1 2.5 5 1 τ= = = 0.555
𝐶+𝐷 21+6
2 4 4 2 (ii) There is moderate positive
3 2.5 4 1 relationship between Aptitude and
4 1 4 0 written
5 6 2 1
6 7 1 1
7 5 1 0
8 8 =21 =6

(ii) Comment on your results


11. UNEB 2005/2/7
The table below shows the marks scored by students in mathematics and fine art tests
students A B C D E F G H I J
Mathematics, 40 48 79 26 55 35 37 70 60 40
Fine art 59 62 68 47 46 39 63 29 55 67
Calculate the coefficient of rank correlation for the students’ performance in the two subjects
and comment on your results.
Math Fine Art RM RF d d2
40 59 6.5 5 1.5 2.25 6 ∑ 𝑑2
ρ = 1- [ ]
48 62 5 4 1 1 𝑛(𝑛2 −1)
6 𝑥 146.5
79 68 1 1 0 0 = 1- [ ]
10(102 −1)
26 47 10 7 3 9
= 0.112
55 46 4 8 -4 16
35 39 9 9 0 0
37 63 8 3 5 25
70 29 2 10 -8 64
60 55 3 6 -3 9
40 67 6.5 2 4.5 20.25
∑ 𝑑 2 =146.5
Kendall’s correlation coefficient

RM RF C D
1 1 9 0 𝐶−𝐷 23−22
2 10 0 8 τ= = = 0.022
𝐶+𝐷 23+22
3 6 3 4
4 8 1 5
5 4 3 2
6.5 5 2 2
6.5 2 3 0
8 3 2 0
9 9 0 1
10 7 23 22

12. UNEB 2007/2/12


Below are marks scored by 8 students A, B, C … H in Mathematics, Economics and geography in
the end of term examination.
A B C D E F G H
Math 52 75 41 60 81 31 65 52
Economics 50 60 35 65 66 45 69 48
Geography 35 40 60 54 63 40 65 72
Determine the rank correlation coefficient for the performance of students in
(i) Math and economics
Spearman’s correlation coefficient
Math, M Economic C RM RE d d2 6 ∑ 𝑑2
52 50 5.5 5 0.5 0.25 ρ = 1- [ ]
𝑛(𝑛2 −1)
75 60 2 4 -2 4 6 𝑥 12.5
= 1- [ ]
10(102 −1)
41 35 7 8 -1 1
= 0.851
60 65 4 3 1 1
81 66 1 2 -1 1
31 45 8 7 1 1
65 69 3 1 2 4
52 48 5.5 6 -0.5 0.25
2
∑ 𝑑 =12.5
Kendall’s correlation coefficient
RM RE C D
1 2 6 1 𝐶−𝐷 25−4
τ= = = 0.724
2 4 4 2 𝐶+𝐷 25+4

3 1 5 0
4 3 4 0
5.5 5 3 0
5.5 6 2 0
7 8 1 1
8 7 =25 =4

(ii) Geography and math


Spearman’s rank correlation coefficient
Geog, G Math, M RG RM d d2
6 ∑ 𝑑2
35 52 8 5.5 2.5 6.25 ρ = 1- [ ]
𝑛(𝑛2 −1)
40 75 6.5 2 4.5 20.25 6 𝑥 64
= 1- [ ]
60 41 4 7 -3 9 8(82 −1)

54 60 5 4 1 1 = 0.238
63 81 3 1 2 4
40 31 6.5 8 -1.5 2.25
65 65 2 3 -1 1
72 52 1 5.5 -4.5 20.25
64
Kendall’s rank coefficient
RG RM C D
1 5.5 2 4 𝐶−𝐷 16−11
τ= = = 0.185
2 3 4 2 𝐶+𝐷 16+11

3 1 5 0
4 7 1 3
5 4 2 1
6.5 2 2 0
6.5 8 0 1
8 5.5 16 11
(iii) Comment on the significance of the math in performance of economics and geography.
(ρ = 0.86, τ = 0.79 based on 8 observations at a 1% level of significance)
Since|𝜌𝐶 (0.851)| < |𝜌𝑇 (0.86)| and |𝜏𝐶 (0.724)| < |𝜏𝜌𝑇 (0.79)| ; there is no significant
relationship between math and economics at 1% significance level.

13. UNEB 2011/2/12


The heights and ages of ten students are given in the table below
Height, 156 151 152 146 160 157 149 142 158 140
cm
Mass, 62 58 63 58 70 60 55 57 68 56
kg
(a) Plot the data on a scatter diagram

(b) Draw the line of best fit on you graph and use it to estimate the mass corresponding to a
height of 155cm(63kg)
(c) Calculate the rank correlation coefficient for the data. Comment on the significance of the
height on masses of students (ρ = 0.79, τ =0.64 based on 10 observations at 1% level of
significance.)
Method I: Using Spearman’s rank correlation coefficient
Height (x) Mass (y) Rx Ry Rx – Ry = d d2
156 62 4 4 0 0
151 58 6 6.5 -0.5 0.25
152 63 5 3 2 4
146 58 8 6.5 1.5 2.25
160 70 1 1 0 0
157 60 3 5 -2 4
149 55 7 10 -3 9
142 57 9 8 1 1
158 68 2 2 0 0
140 56 10 9 1 1
∑ 𝑑 2 = 21.5
6 ∑ 𝑑2 6 𝑥 21.5
ρ = 1- [ ] = 1- [ ] = 0.87
𝑛(𝑛2 −1) 10(102 −1)
Since|𝜌𝐶 (0.0.87)| > |𝜌𝑇 (0.79)| there is no significant relationship between height and age at 1%
significance level.

Since |𝜏𝐶 (0.6 )| < |𝜏𝜌𝑇 (0.0.64 )| ;

14. UNEB 2013/2/9


The heights and ages of ten farmers are given in the table below
Height, 156 151 152 160 146 157 149 142 158 140
cm
Age, 47 38 44 55 46 49 45 30 45 20
years
(a) Plot the data on a scatter diagram

(b) Draw the line of best fit on your diagram and use it to estimate
(i) Age when height = 147 (39)
(ii) Height when the age is 43 (152)
(c) Calculate the rank correlation coefficient for the data. Comment on your results
Method I: Using Spearman’s rank correlation coefficient
Height (x) Age (y) Rx Ry Rx – Ry = d d2
156 47 4 3 1 1
151 38 6 8 -2 4
152 44 5 7 -2 4
160 55 1 1 0 0
146 46 8 4 4 16
157 49 3 2 1 1
149 45 7 5.5 1.5 2.25
142 30 9 9.5 -0.5 0.25
158 45 2 5.5 -3.5 12.25
140 30 10 9.5 0.5 0.25
∑ 𝑑 2 = 41
6 ∑ 𝑑2 6 𝑥 41
p =1 - =1− = 0.7515(4𝐷)
𝑛(𝑛2 −1) 10(102 −1)

Method II: using Kendall’s rank correlation coefficient


Let the farmers be A, B, C, D, E, F, G, H, I, J
Farmers A B C D E F G H I J
Height 156 151 152 160 146 157 149 142 158 140
Age 47 38 44 55 46 49 45 30 45 30

By re-arranging the findings we have

Farmers D I F A C B G E H J
Height 1 2 3 4 5 6 7 8 9 10
Age 1 5.5 3 3 7 8 5.5 4 9.5 9.5
agreements 9 4 7 6 3 2 2 2 0 =35
Disagreements 0 3 0 0 2 2 1 0 0 =8
s = total agreements – total disagreements
= 35 – 8 = 27
2𝑠 2 𝑥 27 54 35−8
𝜏= = = = 0.6 Or τ = = 0.63
𝑛(𝑛−1) 10(10−1) 90 35+8
Comment
Since|𝜌𝐶 (0.7515)| < |𝜌𝑇 (0.79)| and |𝜏𝐶 (0.6 )| < |𝜏𝜌𝑇 (0.0.64 )| ; there is no significant
relationship between height and age at 1% significance level.

Kendall’s rank correlation coefficient


By naming the pairs we have
A(156, 62), B(151, 58), C(152, 63), D(146, 58), E(160, 70), F(157, 60), G149, 55), H(142, 57), I(158, 68),
J(141, 56)
E I F A C B G D H J
x 1 2 3 4 5 6 7 8 9 10
y 1 2 5 4 3 6.5 10 6.5 8 9
C 9 8 5 5 5 3 0 2 1 =38
D 0 0 2 1 0 0 3 0 0 =6
𝐶−𝐷
τ (tau) =
𝐶+𝐷
38−6
τ (tau) = = 0.73
38+6
Since τC(0.73)> τT (0.64), a significant relationship exist between the heights and masses of student.

15. UNEB 2015/2/12


The table gives the points awarded to eight schools by three judges, J1, J2 and J3 during a music
competition. J1 was the chief judge.
J1 72 50 50 55 35 38 82 72
J2 60 55 70 50 50 50 73 70
J3 50 40 62 70 40 48 67 67
(a) Determine the rank correlation coefficients between the judges of
(i) J1 and J2
(ii) J1 and J3
(b) Who of the two judges had a better correlation with the chief judge? Give a reason.

Solution
(i) J1 and J2
J1 J2 𝑅𝐽1 𝑅𝐽2 𝐷1 𝐷12
72 60 2.5 4 -1.5 2.25
50 55 5.5 5 0.5 0.25
50 70 5.5 2.5 3 9
55 50 4 7 -3 9
35 50 8 7 1 1
38 50 7 7 0 0
82 73 1 1 0 0
72 70 2.5 2.5 0 0
∑ 𝐷12 = 21.5
6 ∑ 𝐷12
𝜌1 = 1 −
𝑛(𝑛2 −1)
6 𝑥 21.5
=1−
8(82 −1)
125
= = 0.7440
168
(ii) J1 and J3 (10marks)
J1 J3 𝑅𝐽1 𝑅𝐽3 𝐷2 𝐷22
72 50 2.5 5 -2.5 6.25
50 40 5.5 7.5 -4 4
50 62 5.5 4 0.5 2.25
55 70 4 1 3 9
35 40 8 7.5 0.5 0.25
38 48 7 6 1 1
82 67 1 2.5 -1.5 2.25
72 67 2.5 2.5 0 0
∑ 𝐷12 = 25
6 ∑ 𝐷22
𝜌2 = 1 −
𝑛(𝑛2 −1)
6 𝑥 25
=1−
8(82 −1)
= 0.7023

(a) Who of the two other judges had a better correlation with the chief judge? Give a reason.
(02marks)
J2 has a better correlation with the Chief Judge because the coefficient of correlation is
smaller showing a stronger mutual relationship
Thank You

Dr. Bbosa Science


Continuous probability distribution
A probability density function (p.d.f) is continuous if it takes on values between an interval.

Properties of a continuous probability density functions

(i) ∫ 𝑓(𝑥)𝑑𝑥 = 1
(ii) f (x) ≥ 0

Example 1
𝑘𝑥 0≤𝑥≤5
A random variable X of continuous p.d.f is given by 𝑓(𝑥) = {
0 𝑒𝑙𝑠𝑒𝑤ℎ𝑒𝑟𝑒
Find the value of k

Solution
5 52 02 2
∫0 𝑘𝑥𝑑𝑥 = 1 𝑘( − )=1 𝑘=
2 2
25
5 25
𝑥2 k =1
𝑘[ ] = 1 2
2 0

Example 2
𝑘𝑥 0≤𝑥≤2
A random variable X of a continuous p.d.f is given by𝑓(𝑥) = { 2𝑘(𝑥 − 1), 2≤𝑥≤4
0, 𝑒𝑙𝑠𝑒𝑤ℎ𝑒𝑟𝑒
Solution
2 4 22 02 42 22
∫0 𝑘𝑥𝑑𝑥 + ∫2 2𝑘(𝑥 − 1)𝑑𝑥 = 1 ( − ) + 2𝑘 {( − 4) − ( − 2)}= 1
2 2 2 2

2 4 1
𝑥2 𝑥2 2k + 8k = 1; k=
𝑘 [ ] + 2𝑘 [ − 𝑥] = 1 10
2 0 2 2

Sketching f(x)

- find the initial and final points of f(x)


- join the initial and final points of f(x) using a line or curve.
Note

- A line is in the form of y = mx + c


- A curve has a power of x being 2 and above or fractional power e.g. y = x2.
- A curve has a positive coefficient of x2 has a minimum turning point while a curve with a
negative coefficient has a maximum turning point

Example 3
𝑘𝑥 0 ≤ 𝑥 ≤ 3
A random variable x of a continuous p.d.f is given by 𝑓(𝑥) = {
0, 𝑒𝑙𝑠𝑒𝑤ℎ𝑒𝑟𝑒
Find the value of the constant k and sketch f(x)

Solution
f(x)
3 2
∫0 𝑘𝑥𝑑𝑥 =1 k= 2
9
3 3
𝑥2 2
𝑘[ ] = 1 When x = 0, f(x) = 𝑥0=0
2 0 9
2 2
32 02 When x = 3, f(x) = 𝑥3=
𝑘( − )=1 9 3
2 2
0 3 x
Example 4 xx

𝑘𝑥, 0≤𝑥≤3
A random variable X of continuous p.d.f is given by 𝑓(𝑥) = { 𝑘(6 − 𝑥), 3≤𝑥≤6
0, 𝑒𝑙𝑠𝑒𝑤ℎ𝑒𝑟𝑒
Find the value of the constant k and sketch x

Solution
3 6 f(x)
∫0 𝑘𝑥𝑑𝑥 + ∫3 𝑘(6 − 𝑥)𝑑𝑥 = 1 When x = 0, f(x) = k(0) = 0
3 6
3k
𝑥2 𝑥2 When x = 3, f(x) = k(3) = 3k
𝑘 [ ] + 𝑘 [6𝑥 − ] =1
2 0 2 3
When x = 6, f(x) = k(6-6) = 0
1
k=
9
0 3 6 x
Example 5
𝑘(𝑥 + 2) , −2≤𝑥 ≤0
A random variable x of a continuous p.d.f is given by 𝑓(𝑥) = { 𝑘(2 − 𝑥), 0≤𝑥≤2
0, 𝑒𝑙𝑠𝑒𝑤ℎ𝑒𝑟𝑒
Find the value of k and sketch f(x) f(x)
0 2 When x = -2, f(x) = k(-2+2) = 0 2k
∫−2 𝑘(𝑥 + 2)𝑑𝑥 + ∫0 𝑘(2 − 𝑥)𝑑𝑥 = 1
When x = 0, f(x) = k(0 +2) = 2k
0 2
𝑥2 𝑥2
𝑘[ + 2𝑥] + 𝑘 [2𝑥 − ] =1 When x = 2, f(x) = k(2-2) = 0
2 −2 2 0

1
k= -2 0 2 x
4
Finding Probabilities
Example 6
𝑘𝑥 0 ≤ 𝑥 ≤ 6
A random variable X of a continuous p.d.f is given by 𝑓(𝑥) = {
0, 𝑒𝑙𝑠𝑒𝑤ℎ𝑒𝑟𝑒
Find

(i) the value of k (ii) P(X > 4) (iii) P(X< 3) (iv) P(1< x < 3) (v) P(𝑋 > 2⁄𝑋 ≤ 4)

Solution
6
(i) ∫0 𝑘𝑥𝑑𝑥 = 1 1 3
6 (iii) 1 < x < 3) = ∫ 𝑥𝑑𝑥
18 1
=1
𝑥2 62 02
𝑘[ ] = 𝑘[ − ]=1
2 0 2 2 3
1 1 𝑥2 1 32 12
k= = [ ] = [ − ]
18 18 2 1 18 2 2
1 6
(ii) P(X > 4) = ∫ 𝑥𝑑𝑥 = 1
18 4 2
6 = = 0.2222
1 𝑥2 1 62 42 5 9
= [ ] = [ − ] = = 0.5556
18 2 4 18 2 2 9
1 3
(iii) P(X < 3) = ∫ 𝑥𝑑𝑥 = 1
18 0
3
1 𝑥2 1 32 02 1
= [ ] = [ − ] = = 0.25
18 2 0 18 2 2 4

1 4
𝑃(𝑋>2 ∩ 𝑋≤4 𝑃(2<𝑋<4) ∫ 𝑥𝑑𝑥=1 3
(iv) P(𝑋 > 2⁄𝑋 ≤ 4) = = = 18 2
1 4 =
𝑃(𝑋≤4) 𝑃(𝑋≤4) ∫ 𝑥𝑑𝑥=1 4
18 0

Example 7
𝑘𝑥(6 − 𝑥) 0 ≤ 𝑥 ≤ 6
A random variable X of a continuous p.d.f is given by 𝑓(𝑥) = {
0, 𝑒𝑙𝑠𝑒𝑤ℎ𝑒𝑟𝑒
Find the (i) value of k and sketch f(x) (ii) P(X ≥ 5)
6
(i) ∫0 𝑘𝑥(6 − 𝑥)𝑑𝑥 = 1 Sketch
6
𝑥3 63 03
𝑘 [3𝑥 2 − ] = 𝑘 [(3𝑥 62 − ) − (3 𝑥 02 − )]=1 f(x)
3 0 3 3 1
1 f(x) = x(6 – x)
k= 36
36
1
When x = 0, f(x) = (0)(6 − 0) = 0
36
0 6 x
1
When x = 6, f(x) = (6)(6 − 6) = 0
36
1 6
(ii) P(X ≥ 5) = ∫ 𝑥(6
36 5
− 𝑥)𝑑𝑥
6
1 𝑥3 1 63 53
= [3𝑥 2 − ] = [(3𝑥 62 − ) − (3 𝑥 52 − ) ] = 0.074
36 3 5 36 3 3
Example 8

𝑘𝑥 2 0 ≤ 𝑥 ≤ 4
A random variable of continuous p.d.f is given by 𝑓(𝑥) = {
0, 𝑒𝑙𝑠𝑒𝑤ℎ𝑒𝑟𝑒
Find (i) value of k and sketch f(x) (ii) P(1 ≤ x ≤ 3)

Solution
4
(i) ∫0 𝑘𝑥 2 𝑑𝑥 = 1 Sketch
𝑥3
4
43 03 f(x)
𝑘[ ] = 𝑘[ − ]=1
3 0 3 3
3 3⁄
k= 4
64
3
When x = 0, f(x) = 02 = 0
64
3 3
When x = 4, f(x) = 42 = 0 4 x
64 4
3 3
(ii) P(1 ≤ x ≤ 3)= ∫ 𝑘𝑥 2 𝑑𝑥 = 1
64 1
3
3 𝑥3 3 33 13
= [ ] = [ − ] = 0.4063
64 3 1 64 3 3

Example 9

𝑘(𝑥 2 + 1) 0 ≤ 𝑥 ≤ 3
A random variable X of a continuous p.d.f is given by 𝑓(𝑥) = {
0, 𝑒𝑙𝑠𝑒𝑤ℎ𝑒𝑟𝑒
Find (i) value of k and sketch f(x) (ii) P(1 ≤ x ≤ 3)
3
(i) ∫0 𝑘(𝑥 2 + 1)𝑑𝑥 = 1 f(x)
3
𝑥3 33 03
𝑘[ + 𝑥] = 𝑘 [( + 3) − ( + 0) ] = 1 10⁄
3 0 3 3
1
12
k=
12

When x = 0, f(x) =
1
(02 + 1) =
1 1
12 12
12
1 10
When x = 3, f(x) = [32 + 1] = 0 3 x
12 12

(ii) P(1 ≤ x ≤ 3)
3
1 3 1 𝑥3 1 33 13
∫ (𝑥 2 + 1)𝑑𝑥 =12 [ 3 + 𝑥] =
12 1 12
[(
3
+ 3) − ( + 1) ] = 0.8889
3
1

Example 10
𝑘, 0≤𝑥≤2
A random variable X of continuous p.d.f is given by 𝑓(𝑥) = { 𝑘(2𝑥 − 3), 2≤𝑥≤3
0, 𝑒𝑙𝑠𝑒𝑤ℎ𝑒𝑟𝑒

Find (i) value of k and sketch f(x) (ii) P(X<1) (ii) P(X > 2.5) (iv) (0 ≤ 𝑋 ≤ 2⁄𝑋 ≥ 1)
Solution
f(x)
2 3 1
∫0 𝑘𝑑𝑥 + ∫2 𝑘(2𝑥 − 3)𝑑𝑥 = 1 When x = 0, f(x) = k =
4 3
𝑘 [𝑥 ]20 + 𝑘 [𝑥 − 2
3𝑥 ]32 =1 1 4
When x = 2, f(x) = k = f(x) =
1
4
1 1⁄ 4
k= 1 4
4
When x = 3, f(x) = (2 x3 - 3)
4

=
3 0 2 3 x
4

1 1 1 1
(ii) P(X<1) = ∫0 𝑑𝑥 = [𝑥 ]10 =
4 4 4
1 3 1
(iii) P(X > 2.5) = ∫2.5(2𝑥 − 3)𝑑𝑥 = [𝑥 2 − 3𝑥 ]32.5 = 0.3125
4 4
1 2 1
𝑃(0≤𝑋≤2) 𝑃((0≤𝑋≤2)∩(𝑋≥1)) ∫ 𝑑𝑥 1
(iv) 𝑃 (0 ≤ 𝑋 ≤ 2⁄𝑋 ≥ 1) = = =1 2
4 1
1 3 = 4
1 1 =
𝑃(𝑋≥1) 𝑃(𝑋≥1) ∫ 𝑑𝑥+ 4 ∫2 (2𝑥−3)𝑑𝑥 + 3
4 1 4 2

Example 11

𝑘(𝑥 + 2)2 , −2≤𝑥 ≤0


4
A random variable X of continuous p.d.f is given by 𝑓(𝑥) = { 4𝑘, 0≤𝑥≤
3
0, 𝑒𝑙𝑠𝑒𝑤ℎ𝑒𝑟𝑒
Find

(i) the value of the constant k and sketch f(x)


(ii) P(-1 < x< 1) (iii) P(X > 1)

Solution Sketch f(x)


1
0 2 2 1 F(x) =
∫−2 𝑘(𝑥 + 2) 𝑑𝑥 + ∫0 4𝑘𝑑𝑥 =1 2
2
0
(𝑥+2)3
𝑘[ ] + 4𝑘 [𝑥 ]20 = 1
3 −2
1
k=
8
4
When x = -2, f(x) = (-2 + 2)2 = 0
1 -2 0 x
3
8
1 1
When x = 0, f(x) = (0+2) 2 =
8 2
4 1 1
When x = , f(x) =4 x =
3 8 2
0 1
(ii) P(-1 < x< 1) =∫−1 𝑘(𝑥 + 2)2 𝑑𝑥 + ∫0 4𝑘𝑑𝑥
0
1 (𝑥+2)3 1 7 1 19
= [ ] + 4𝑥 [𝑥 ]10 = + =
8 3 −12 8 24 2 24

4 4⁄
1 1
(iii) P(X > 1) =∫03 4𝑘𝑑𝑥 =4𝑥 [𝑥 ]1 3 =
8 6
Finding the constant k from a sketch graph

Example 12
𝑘𝑥 0≤𝑥≤4
A random variable X of continuous p.d.f is given by 𝑓(𝑥) = {
0, 𝑒𝑙𝑠𝑒𝑤ℎ𝑒𝑟𝑒
(a) Sketch and find the value of constant k
(b) Find (i) P(X ≤ 1) (ii) P(1 < x < 2)

Solution

(a) When x = 0, f(x) = k x 0 = 0 1 1 1 𝑥2


1
When x = 4, f(x) = k x 4 = 4k (b)(i) P(X ≤ 1) = ∫ 𝑥𝑑𝑥 =8 [ 2 ]
8 0 0
f(x)
1 12 02 1
4k = ( − )=
8 2 2 16

2
1 2 1 𝑥2
(ii) P(1 < x < 2) = ∫1 𝑥𝑑𝑥 = [ ]
8 8 2 1

1 22 12 3
= ( − )=
8 2 2 16
0 4 x

1
Area under the curve = 𝑥 4 𝑥 4𝑘 = 1
2
1
k=
8

Example 13
𝑘𝑥, 0≤𝑥≤2
A random variable X of a continuous p.d.f is given by 𝑓(𝑥) = { 𝑘(4 − 𝑥), 2 ≤𝑥≤4
0, 𝑒𝑙𝑠𝑒𝑤ℎ𝑒𝑟𝑒
(a) Sketch f(x) and find the value of k
(b) Find (i) P(X< 1) (ii) P(X >3) (iii) P(1 ≤ x≤ 3) (iv) 𝑃 (𝑋 ≥ 1⁄𝑋 ≤ 3)

Solution
1
When x = 0, f(x) = 𝑘(0) = 0 1 1 1 𝑥2
(b)(i) P(X< 1) = ∫0 𝑥𝑑𝑥 = [ ]
4 4 2 0
When x = 2, f(x) = 𝑘 𝑥 2 = 2𝑘
1 12 02 1
= ( − )=
When x = 4, f(x) =k (4 – 4) = 0 4 2 2 8

1 4
f(x) (ii) P(X >3) = ∫3 (4 − 𝑥)𝑑𝑥
4
4
2k 1 𝑥2
= [4𝑥 − ] = 0.125
4 2 3

1 2 1 3
(iii) P(1 ≤ x≤ 3)= ∫1 𝑥𝑑𝑥 + ∫2 4 − 𝑥𝑘𝑑𝑥
4 4
0 2 4 x 2 3
1 𝑥2 1 𝑥2 3
x = [ ] + [4𝑥 − ] =
4 2 1 4 2 2 4
1
Area under the curve = 𝑥 4 𝑥 2𝑘 = 1
2
1
k=
4
3
3⁄
𝑃 (𝑋 ≥ 1⁄𝑋 ≤ 3) =
𝑋≥1 ∩𝑋≤3 𝑃(1≤𝑥 ≤3 4 6
(iv) = = 1 2 1 3 = 4⁄7 =
𝑋≤3 𝑋≤3 ∫ 𝑥𝑑𝑥+ ∫ 4−𝑥𝑘𝑑𝑥 7
4 0 4 2
8

Example 14
𝑘, 2≤𝑥≤3
A random variable X of a continuous p.d.f is given by 𝑓(𝑥) = { 𝑘(𝑥 − 2), 3≤𝑥≤4
0, 𝑒𝑙𝑠𝑒𝑤ℎ𝑒𝑟𝑒
Find (i) the value of k and sketch the graph (ii) P(|𝑋 − 2.5| > 0.5) (iii) P(|𝑋 − 2.5| < 0.5)

Solution

(i) When x = 2, f(x) = 𝑘 1


Area under the curve = 1 x k + (𝑘 + 2𝑘)𝑥 1 = 1
2
When x = 3, f(x) = k 2
k=
5
When x = 4, f(x) =k (4 – 2) = 2k
(ii) P(|𝑋 − 2.5| > 0.5) = P(-0.5 < X-2.5<0.5)
f(x)
= P(2 < X <3)
2k
2 3
f(x) = k = ∫2 𝑑𝑥 = [𝑥 ]32
k 5
2
=
5
2 3 4 x

Finding p.d.f from a sketch graph


Example 15

A random variable X of a continuous p.d.f is given by


2
f(x) 𝑦−0 3
−0
grad = =
𝑥−0 1−0
k
2
y= 𝑥
3

For interval 1≤ x≤ 3

0 1 2 3 Coordinates are (3,0) and (1, k)


x 2
𝑦−0 −0
3
1
grad = =
𝑥−3 1−3
(a) Area = 1 = 𝑥3𝑥𝑘
2
2 1
K= y = − (𝑥 − 3)
3 3
(b) Find f(x) 2
Let f(x) = y 𝑥, 0≤𝑥≤1
3
For interval: 0 ≤ 𝑥 ≤ 1 coordinates are 𝑓(𝑥) = 1
(0, 0) and (1, k) (𝑥 − 3), 1≤𝑥≤3
3
{0, 𝑒𝑙𝑠𝑒𝑤ℎ𝑒𝑟𝑒
Example 16

A continuous random variable X has a probability density function (p.d.f) f(x) as shown in the graph
below

f(x) For interval: 1 ≤ 𝑥 ≤ 2 coordinates


k are
(1, k) and (2, 0)
𝑦−1 0−1
grad = =
𝑥−1 2−1

0 1 2 x y=2−𝑥
x 𝑥, 0≤𝑥≤1
𝑓(𝑥) = { (2 − 𝑥), 1≤𝑥≤2
(a) Find the
0, 𝑒𝑙𝑠𝑒𝑤ℎ𝑒𝑟𝑒
(i) value of k
(ii) expression for the probability density (b)(i) E(X) =∑ 𝑥𝑓(𝑥)
function 1 2
(b) Calculate the = ∫0 𝑥. 𝑥𝑑𝑥 + ∫1 𝑥(2 − 𝑥)𝑑𝑥
(i) The mean 𝑥3
1
𝑥3
2
(ii) P(X<1.5/X> 0.5) =[ ] + [𝑥 2 − ]
3 0 3 1

Solution 1
=( − 0) + [(4 − ) − (1 − )]
8 1
3 3 3
(i) Area under the graph = 1 1 4 2
1 = + − =1
𝑥 2 𝑥 𝑘 = 1; k = 1 3 3 3
2
(ii) Let f(x) = y
For interval: 0 ≤ 𝑥 ≤ 1 coordinates are
(0, 0) and (1, k)
𝑦−0 1−0
grad = =
𝑥−0 1−0

y=𝑥

1 1.5
𝑃(𝑥<1.5 ∩𝑥>0.5) 𝑃(0.5<𝑥 1.5) ∫0.5 𝑥𝑑𝑥+∫1 (2−𝑥)𝑑𝑥
(b)(ii) P(X<1.5/X> 0.5) = = = 0.5
𝑃(𝑋>0.5) 𝑃(𝑋>0.5) 1−∫0 𝑥𝑑𝑥

1 1.5
𝑥2 𝑥2
[ ] +[2𝑥− ]
2 0.5 2 1
= 0.5 = 0.8751
𝑥2
1−[ ]
2 0
Example 17

The departure time T of pupils from a certain day primary school can be modelled as in the diagram
below, where t is the time in minutes after the final bell at 5.00pm

f(t)

0 5 10 t
x
Determine the

(i) value of k
Area under the curve = 1
1
𝑥 10 𝑥 𝑘 =1
2
1
𝑘=
5
(ii) equation of the p.d.f
1
C(5, )
5

R(t f(t)) S(t f(t))

A(0, 0) B(0, 10)

Gradient of 𝐴𝐶 = Gradient of 𝐴𝑅
1
−0 𝑓(𝑥)−0
5
=
5−0 𝑡 =0
1 𝑓(𝑥)
=
25 𝑡
1
f(x) = 𝑡
25
Gradient of 𝐵𝐶 = Gradient of 𝐵𝑆
1
−0 𝑓(𝑥)−0
5
=
5−10 𝑡 =10
1 𝑓(𝑥)
− =
25 𝑡−10
10−𝑡
f(x) =
25
1
𝑡, 0≤𝑥≤5
25
Hence 𝑓(𝑥) = {1 (10 − 𝑡), 5 ≤ 𝑥 ≤ 10
25
0, 𝑒𝑙𝑠𝑒𝑤ℎ𝑒𝑟𝑒

(iii) E(T): since the graph is symmetrical about t = 5; Hence E(T) = 5


(iv) Probability that a pupil leaves between 4 and 7 minutes after the bell
1 5 1 7
P(4 < t < 7) = ∫ 𝑡𝑑𝑥 + 25 ∫5 (10 − 𝑡)𝑘𝑑𝑥
25 4
5 7
1 𝑡2 1 𝑡2
= [ ] + [10𝑡 − ] = 0.5
25 2 4 25 2 5

Revision exercise 1
𝑘𝑥 2 0≤𝑥≤2
1. A random variable X of a continuous p.d.f is given by 𝑓(𝑥) = {
0, 𝑒𝑙𝑠𝑒𝑤ℎ𝑒𝑟𝑒
3
(a) Find the value of the constant k (= ) and sketch f(x)
8
3 13
(b) Find (i) P(X≥1) = (ii) P(0.5≤ x ≤ 1.5) =
8 32
𝑘 −2≤𝑥 ≤3
2. A random variable X of a continuous p.d.f is given by 𝑓(𝑥) = {
0, 𝑒𝑙𝑠𝑒𝑤ℎ𝑒𝑟𝑒
(i) Sketch f(x)
1
(ii) Find the value of the constant k = .
5
(iii) Find P(-1.6 ≤ x ≤ 2.1) = 0.74
𝑘(4 − 𝑥) 1≤𝑥≤3
3. A random variable X of a continuous p.d.f is given by 𝑓(𝑥) = {
0, 𝑒𝑙𝑠𝑒𝑤ℎ𝑒𝑟𝑒
(i) Sketch f(x)
1
(ii) Find the value of the constant k = .
4
(iii) Find P(1.2 ≤ x ≤ 2.4) = 0.66
𝑘(𝑥 + 2)2 0 ≤ 𝑥 ≤ 2
4. A random variable X of a continuous p.d.f is given by 𝑓(𝑥) = {
0, 𝑒𝑙𝑠𝑒𝑤ℎ𝑒𝑟𝑒
(a) Sketch f(x)
1
(b) Find the value of the constant k = .
56
19 37
(c) Find (i) P(0 ≤ x ≤ 1) = (ii) P(X ≥ 1) =
56 56
𝑘(𝑥)3 0≤𝑥≤𝑐
5. A random variable X of a continuous p.d.f is given by 𝑓(𝑥) = {
0, 𝑒𝑙𝑠𝑒𝑤ℎ𝑒𝑟𝑒
1
Given that P(X ≤ 0.5) =
16
(i) Find the value of k and c (k = 1 and k = 4)
(ii) Sketch f(x)
𝑘𝑥 0≤𝑥≤4
6. A random variable X of a continuous p.d.f is given by 𝑓(𝑥) = {
0, 𝑒𝑙𝑠𝑒𝑤ℎ𝑒𝑟𝑒
(i) Sketch f(x)
1
(ii) Find the value of the constant k = .
8
(iii) Find P(1 ≤ x ≤ 2.5) = 0.328

𝑘, 0≤𝑥≤2
7. A random variable X of a continuous p.d.f is given by 𝑓(𝑥) = { 𝑘(2𝑥 − 3), 2 ≤ 𝑥 ≤ 3
0, 𝑒𝑙𝑠𝑒𝑤ℎ𝑒𝑟𝑒
(i) Sketch f(x)
1
(ii) Find the value of the constant k = .
4
1
(iii) Find (i) P(X >1) = (ii) P(X > 2.5) = 0.3125 (iii) P(1 ≤ x ≤ 2.3) = 0.3475
4
𝑎, 0 ≤ 𝑥 ≤ 1.5
𝑎
8. A random variable X of a continuous p.d.f is given by 𝑓(𝑥) = { 2 (2 − 𝑥), 2 ≤ 𝑥 ≤ 3
0, 𝑒𝑙𝑠𝑒𝑤ℎ𝑒𝑟𝑒
16
Find (i) value of a= (ii) P(X < 1.6) = 0.9744
25

Expectation or mean of X
Example 18

𝑘𝑥 2 0≤𝑥≤3
A random variable X of a continuous p.d.f is given by 𝑓(𝑥) = {
0, 𝑒𝑙𝑠𝑒𝑤ℎ𝑒𝑟𝑒
Find the

(i) value of the constant k and sketch f(x)


(ii) the mean, μ
(iii) P(X ≤ μ)

Solution
3
(i) ∫0 𝑘𝑥 2 𝑑𝑥 = 1 (ii)
3
E(X) = ∫0 𝑥. 𝑥 2 𝑑𝑥
3
𝑥3 1 1 𝑥4
3
𝑘[ ] = 1,k= = [ ] = 2.25
3 0 9 9 4 0
1
When x = 0, f(x) = (0)2 =0 (iii) P(X ≤ μ)
1 2.25
= ∫0 𝑥 2 𝑑𝑥
9 9
1
When x = 3, f(x) = (3)2 =1 1 𝑥4
2.25
9 = [ ]
9 4 0

f(x) = 0.42

1
f(x) = 𝑥 2
9

0 3 x
x

Example 19

𝑘𝑥 3 0≤𝑥≤2
A random variable X of a continuous p.d.f is given by 𝑓(𝑥) = {
0, 𝑒𝑙𝑠𝑒𝑤ℎ𝑒𝑟𝑒
Find (i) the value of the constant k
3
∫0 𝑘𝑥 3 𝑑𝑥 = 1
2
𝑥4 1
𝑘[ ] = 1,k=
4 0 4
(ii) mean
2
1 3 1 𝑥5
E(X) = ∫0 𝑥. 𝑥 3 𝑑𝑥 = [ ] = 1.6
4 4 5 0
1
11 1 𝑥4
(iii) P(X ≤ 1) = ∫0 𝑥 3 𝑑𝑥 = [ ] = 0.0625
4 4 4 0
Example 20

𝑘(4𝑥 − 𝑥 2 ), 0 ≤ 𝑥 ≤ 2
A random variable X of a continuous p.d.f is given by 𝑓(𝑥) = {
0, 𝑒𝑙𝑠𝑒𝑤ℎ𝑒𝑟𝑒
Find

(i) the value of constant k


2
∫0 𝑘(4𝑥 − 𝑥 2 )𝑑𝑥 = 1
2
𝑥3 3
𝑘 [2𝑥 2 − ] ,k=
3 0 16

(ii) E(X)
2
3 2 3 4𝑥 3 𝑥4
∫ 𝑥(4𝑥 − 𝑥 2 )𝑑𝑥 = 16 [
16 0 3

4 0
] = 0.25

1
3 1 3 𝑥3
(iii) P(X ≤ 1) = ∫ (4𝑥 − 𝑥 2 )𝑑𝑥 = 16 [2𝑥 2 −
16 0
] = 0.3125
3 0

Example 21

3𝑥 𝑘 , 0≤𝑥≤1
A random variable X of a continuous p.d.f is given by 𝑓(𝑥) = {
0, 𝑒𝑙𝑠𝑒𝑤ℎ𝑒𝑟𝑒
(i) Find the value of k
(ii) Find the mean
1
3 ∫0 𝑥 𝑘 𝑑𝑥 =1 1 𝑥4
1
E(X) =∫0 𝑥(3𝑥 2 )𝑑𝑥 = 3 [ ] = 0.75
4 0
1
𝑥 𝑘+1
3[ ] =1
𝑘+1 0 (iii) Find the value of a such that P(X ≤ a) = 0.5
1𝑘+1 0𝑘+1 𝑎
3[ − ]=1 P(X ≤ a) =3 ∫0 𝑥 2 𝑑𝑥 = 0.5
𝑘+1 𝑘+1
𝑎
3 𝑥3
𝑘+1
=1 = 3 [ ] = a3 – 03 = 0.5
3 0

k=2 = a3 = 0.5; a = 0.794

Example 22
1
𝑥, 0≤𝑥≤2
4
A random variable X of a continuous p.d.f is given by 𝑓(𝑥) = {(1 − 1 𝑥) , 2≤𝑥≤4
4
0, 𝑒𝑙𝑠𝑒𝑤ℎ𝑒𝑟𝑒
(i) Sketch f(x)

1
f(x)
When x = 0, f(x) = 𝑥 (0) = 0
4
2k
1
When x = 2, f(x) = 𝑥 (2) = 0.25
4

1
When x = 4, f(x) =(1 − (4)) = 0
4
0 2 4 x
(ii) Mean 4
(iii) P(X > 3) = ∫3 (1 − 𝑥) 𝑑𝑥
1
21 4 1 4
E(X) = ∫0 𝑥. 𝑥𝑑𝑥 + ∫2 𝑥 (1 − 𝑥) 𝑑𝑥
4 4 4
2 4 𝑥2
1 𝑥3 𝑥2 𝑥3 =[𝑥 − ] = 0.125
[ ] +[ − ] =2 8 3
4 3 0 2 12 2

Example 23
𝑘(𝑥 + 2) , −1≤𝑥 ≤0
A random variable X of a continuous p.d.f is given by 𝑓(𝑥) = { 2𝑘(1 − 𝑥), 0≤𝑥≤1
0, 𝑒𝑙𝑠𝑒𝑤ℎ𝑒𝑟𝑒
(i) Sketch f(x) Sketch f(x)
When x = -1, f(x) = k(-1 + 2) = k 2k

When x = 0, f(x) = k(0 + 2) = 2k


k
When x = 1, f(x) =2k(1-1) = 0

-1 0 1 x
(ii) value of k
Area under the graph = 1
1 1
𝑥 1(𝑘 + 2𝑘) + 𝑥 1 𝑥 2𝑘 = 1
2 2
2
𝑘=
5
(iii) 𝑃 (0 < 𝑥 < 0.5⁄𝑋 > 0)
0.5
4 0.5 𝑥2
∫ (1−𝑥)𝑑𝑥 [𝑥− ] 3⁄
𝑃(0<𝑥<0.5) 2 0
𝑃 (0 < 𝑥 < 0.5⁄𝑋 > 0) = = 5 0
4 1 = 1 = 1⁄
8
= 0.75
𝑃(𝑋>0) ∫ (1−𝑥)𝑑𝑥 𝑥2 2
5 0 [𝑥− ]
2 0

(iv) Mean
2 0 4 1
E(X) = ∫−1 𝑥(𝑥 + 2)𝑑𝑥 + ∫0 𝑥(1 − 𝑥)𝑑𝑥
5 5
0 1
2 𝑥3 4 𝑥2 𝑥3 2
= [ + 𝑥2] + [ − ] =-
5 3 −1 5 2 3 0 15

Properties of the mean

(i) E(a) = a
(ii) E(ax) = a.E(x)
(iii) E(ax + b) = aE(x) + b
(iv) E(ax - b) = aE(x) – b
Where a and b are constants

Example 24
1
(𝑥 + 3), 0 ≤ 𝑥 ≤ 4
A random variable X of a continuous p.d.f is given by 𝑓(𝑥) = {20
0, 𝑒𝑙𝑠𝑒𝑤ℎ𝑒𝑟𝑒
(i) Sketch f(x)
(ii) Find E(X)
(iii) Find E(2X + 5)

Solution
1 3 1 4
(i) When x = 0, f(x) = (0 + 3) = (ii) E(X) = ∫ 𝑥(𝑥
20 0
+ 3)𝑑𝑥
20 20
1 7 4
1 𝑥3 3𝑥 2
When x = 4, f(x) = (4 + 3) = = [ − ]
20 20 20 3 2 0
Sketch = 2.266
f(x) (iii) E(2X + 5) = 2 x 2.266 + 5 = 9.533
7
20
3
20

0 4 x

Example 25
3
𝑥2, 0≤𝑥≤2
32
A random variable X of a continuous p.d.f is given by 𝑓(𝑥) = {3 (6 − 𝑥), 2≤𝑥≤6
32
0, 𝑒𝑙𝑠𝑒𝑤ℎ𝑒𝑟𝑒
(i) Sketch f(x)
(ii) Find P(X< 4)
3 2
When x = 0, f(x) = (0) = 0 3 2 2 3 4
32
P(X<4) = ∫ 𝑥 𝑑𝑥
32 0
+ ∫ (6
32 2
− 𝑥)𝑑𝑥
3 3
When x = 2, f(x) = (2)2 = 2 4
32 8 3 𝑥3 3 𝑥2 13
= [ ] + [6𝑥 − ] =
3 32 3 0 32 2 2 16
When x =6, f(x) = (6 − 6) = 0
32
(iii) find the mean
f(x)
3 2 3 4
3 E(X) = ∫ 𝑥. 𝑥 2 𝑑𝑥
32 0
+ ∫ 𝑥(6
32 2
− 𝑥)𝑑𝑥
8 2 6
3 𝑥4 3 𝑥3
= [ ] + [3𝑥 2 − ]
32 4 0 32 3 2

= 2.875

0 2 4 6 x

(iv) Find E(100x -20)

E(100X -20) = 100 x 2.875 – 20 = 267.50

Revision exercise 2
𝑘𝑥 2 , 0≤𝑥≤4
1. A random variable X of a continuous p.d.f is given by 𝑓(𝑥) = {
0, 𝑒𝑙𝑠𝑒𝑤ℎ𝑒𝑟𝑒
(i) Sketch f(x) (ii) Find E(x) = 3 (iii) find E(2X + 5) = 11
𝑘𝑥 2 (10 − 𝑥), 0≤𝑥≤4
2. A random variable X of a continuous p.d.f is given by 𝑓(𝑥) = {
0, 𝑒𝑙𝑠𝑒𝑤ℎ𝑒𝑟𝑒
3
(i) Find value of k = (ii) Find E(x) = 6 (iii) find E(3X - 4) = 14
2500
𝑘𝑥, 5 ≤ 𝑥 ≤ 10
3. A random variable X of a continuous p.d.f is given by 𝑓(𝑥) = {
0, 𝑒𝑙𝑠𝑒𝑤ℎ𝑒𝑟𝑒
2 70
(i) Sketch f(x) (ii) Find value of k = (iii) Find E(x) = (iii) find P(X >8) = 0.48
75 9
𝑘[1 − (𝑥 − 2)2 ], 1 ≤ 𝑥 ≤ 3
4. A random variable X of a continuous p.d.f is given by 𝑓(𝑥) = {
0, 𝑒𝑙𝑠𝑒𝑤ℎ𝑒𝑟𝑒
3
(i) Find value of k = (ii) sketch f(x) (iii) find E(X) = 2
4
𝑘𝑥(5 − 𝑥), 0≤𝑥≤5
5. A random variable X of a continuous p.d.f is given by 𝑓(𝑥) = {
0, 𝑒𝑙𝑠𝑒𝑤ℎ𝑒𝑟𝑒
6
(i) Find value of k = (ii) sketch f(x) (iii) find E(X) = 2.5
125
𝑘(1 − 𝑐𝑜𝑠𝑥), 0≤𝑥≤𝜋
6. A random variable X of a continuous p.d.f is given by 𝑓(𝑥) = {
0, 𝑒𝑙𝑠𝑒𝑤ℎ𝑒𝑟𝑒
1
(i) Find value of k = (ii) sketch f(x) (iii) find mean of x = 0.9342
𝜋
𝑘
𝑥, 0≤𝑥≤3
3
7. A random variable X of a continuous p.d.f is given by 𝑓(𝑥) = {𝑘, 3≤𝑥≤4
0, 𝑒𝑙𝑠𝑒𝑤ℎ𝑒𝑟𝑒
2
(i) Sketch f(x) (ii) find k = (iii) find E(X) = 2.6
5

(iv) find value of c such that P(X>c) = 0.85; c = 1.5


1
𝑘(𝑥 − ), 1≤𝑥≤3
8. A random variable X of a continuous p.d.f is given by 𝑓(𝑥) = { 𝑎
0, 𝑒𝑙𝑠𝑒𝑤ℎ𝑒𝑟𝑒
Given that P(X> 1) = 0.8,
2
Find (i) values of a and k ( , −1) (ii) probability between 0.5 and 2.5 = 0.6667 (iii) E(X) =1.8
15
𝑘(𝑥 + 2) −1≤𝑥 ≤0
2𝑘, 0≤𝑥≤1
9. A random variable X of a continuous p.d.f is given by 𝑓(𝑥) = 𝑘
(5 − 𝑥) 1≤𝑥≤3
2
{ 0, 𝑒𝑙𝑠𝑒𝑤ℎ𝑒𝑟𝑒
(a) Sketch the function f(x)
2 12
(b) Find the value of k (= ) and the mean (= )
13 13
2𝑘𝑥, 0≤𝑥≤1
10. A random variable X of a continuous p.d.f is given by 𝑓(𝑥) = {𝑘(3 − 𝑥) 1 ≤ 𝑥 ≤ 2
0, 𝑒𝑙𝑠𝑒𝑤ℎ𝑒𝑟𝑒
(a) Sketch f(x)
2 17
(b) Find the value of k (= ) and the mean =
5 15
𝜋
𝛼(1 − 𝑐𝑜𝑠𝑥), 0 ≤ 𝑥 ≤
2
11. A random variable X of a continuous p.d.f is given by 𝑓(𝑥) = {𝛼𝑠𝑖𝑛𝑥, 𝜋
≤𝑥≤𝜋
2
𝑒𝑙𝑠𝑒𝑤ℎ𝑒𝑟𝑒 0,
2 𝜋 𝜋 3𝜋
(i) Find value of α (= ) (ii) mean, μ (= 1 + ) (iii) 𝑃 ( < 𝑥 < ) = 0.6982
𝜋 4 3 4
𝑘1 𝑥, 1≤𝑥≤3
12. A random variable X of a continuous p.d.f is given by 𝑓(𝑥) = { 𝑘2 (4 − 𝑥), 3 ≤ 𝑥 ≤ 4
0, 𝑒𝑙𝑠𝑒𝑤ℎ𝑒𝑟𝑒
(a) Show that k2 = 3k1
(b) Find (i) values of k1 and k2 (ii) mean, μ
𝑦+1
1≤𝑦≤𝑘
4
13. A random variable X of a continuous p.d.f is given by 𝑓(𝑥) = {
0, 𝑒𝑙𝑠𝑒𝑤ℎ𝑒𝑟𝑒
Find

(i) Value of k =2
(ii) Expectation Y = 1.6667
(iii) P(1≤ y≤ 1.5) = 0.2813

Solutions to revision exercise 2


1
𝑘(𝑥 − ), 1≤𝑥≤3
8. A random variable X of a continuous p.d.f is given by 𝑓(𝑥) = { 𝑎
0, 𝑒𝑙𝑠𝑒𝑤ℎ𝑒𝑟𝑒
Given that P(X> 1) = 0.8,
Find
2
(i) values of a and k ( , −1) (ii) probability between 0.5 and 2.5
15

3 1 2 2.5
∫0 𝑘 (𝑥 − 𝑎) 𝑑𝑥 = 1 P(0.5 < x < 2.5) = ∫ (𝑥
15 0.5
+ 1)𝑑𝑥

3 2.5
𝑥2 𝑥 2 𝑥2 𝑥
𝑘[ − ] =1 = [ − ] = 0.6667
2 𝑎 0 15 2 𝑎 0.5

9 3 (iii) mean
𝑘( − ) = 1
2 𝑎
2 3
(9a-6)k = 2a …………… (i) E(X) = ∫ 𝑥(𝑥
15 0
+ 1)𝑑𝑥
3
Given P(X>1) = 0.8 2 𝑥3 𝑥2
= [ + ] = 1.8
15 3 2 0
3 1
 ∫1 𝑘 (𝑥 − ) 𝑑𝑥 = 0.8
𝑎
3
𝑥2 𝑥
𝑘[ − ] = 0.8
2 𝑎 1

9 3 1 1
𝑘 [( − ) − ( − )] = 1
2 𝑎 2 𝑎

(8a - 4)k = 1.6a ….. (ii)


2
Eqn.(i) and (ii), a = -1, k =
15

𝑘(𝑥 + 2) −1≤𝑥 ≤0
2𝑘, 0≤𝑥≤1
9. A random variable X of a continuous p.d.f is given by 𝑓(𝑥) = 𝑘
(5 − 𝑥) 1≤𝑥≤3
2
{ 0, 𝑒𝑙𝑠𝑒𝑤ℎ𝑒𝑟𝑒
(a) Sketch the function f(x)
For −1 ≤ 𝑥 ≤ 0, f(x) = 𝑘(𝑥 + 2)
When x = -1, f(x) = k
When x = 0, f(x) =2k
For 0 ≤ 𝑥 ≤ 1, f(x) = 2𝑘,
(b)(i) find value of k
When x = 0, f(x) = 2k
When x = 1, f(x) =2k Area under the graph = 1
𝑘
For 1 ≤ 𝑥 ≤ 3, f(x) = (5 − 𝑥) 1 1
𝑘
2 𝑥 1 (𝑘 + 2𝑘) + 1 𝑥 2𝑘 + 𝑥 2 (𝑘 + 2𝑘)= 1
2 2
When x = 1, f(x) = (5 − 1) =2k
2 2
𝑘 k=
When x = 3, f(x) = (5 − 3) =k 13
2
Sketch or
f(x)
0 1 𝑘 3
2k 𝑘 ∫−1(𝑥 + 2)𝑑𝑥 +2𝑘 ∫0 𝑑𝑥 + ∫1 (5 − 𝑥)𝑑𝑥 = 1
2
k 0 3
𝑥2 𝑘 𝑥2
𝑘[ + 2𝑥] + 2𝑘 [𝑥 ]10 + [5𝑥 − ] =1
k 2 −1 2 2 1

2
A B C k=
13

-1 0 1 2 2

(b) (ii) Find the mean


2 0 4 1 1 3
E(X) = ∫ 𝑥(𝑥 + 2)𝑑𝑥 +13 ∫0 𝑥𝑑𝑥 + 13 ∫1 𝑥(5 − 𝑥)𝑑𝑥
13 −1

0 1 3
2 3 4 𝑥2 1 5𝑥 2 𝑥3 12
= [ + 𝑥2] + [ ] + [ − ] =
13 3 −1 13 2 0 13 2 3 1 13

2𝑘𝑥, 0≤𝑥≤1
10. A random variable X of a continuous p.d.f is given by 𝑓(𝑥) = {𝑘(3 − 𝑥) 1 ≤ 𝑥 ≤ 2
0, 𝑒𝑙𝑠𝑒𝑤ℎ𝑒𝑟𝑒
(a) Sketch f(x)
For 0 ≤ 𝑥 ≤ 1, f(x) = 2kx (b) Find value of k
When x = 0, f(x) = 2k(0) = 0
Area under the graph = 1
When x = 1, f(x) = 2k(1) =2k
1 1
For 1 ≤ 𝑥 ≤ 2, f(x) = k(3-x) 𝑥1 𝑥 2𝑘 + 𝑥 1 (𝑘 + 2𝑘) = 1
2 2
When x= 1, f(x) = k(3 – 1) = 2k
2
When x =3, f(x) = k(3 -2) = k k=
5
Sketch
Alternatively
f(x) 1 2
2𝑘 ∫0 𝑥𝑑𝑥 + 𝑘 ∫1 (3 − 𝑥) 𝑑𝑥 = 1
2k
k 1 2
𝑥2 𝑥2
2𝑘 [ ] + 𝑘 [3𝑥 − ] =1
2 0 2 1
k
2
k=
5

0 1 2 x
(b) Find the mean
4 1 2
E(X) = ∫0 𝑥 2 𝑑𝑥 + 𝑘 ∫1 𝑥(3 − 𝑥) 𝑑𝑥 = 1
5
1 2
4 𝑥3 4 3𝑥 2 𝑥3 17
= [ ] + [ − ] = =1.133
5 3 0 5 2 3 1 15

𝜋
𝛼(1 − 𝑐𝑜𝑠𝑥), 0 ≤ 𝑥 ≤
2
11. A random variable X of a continuous p.d.f is given by 𝑓(𝑥) = {𝛼𝑠𝑖𝑛𝑥, 𝜋
≤𝑥≤𝜋
2
0, 𝑒𝑙𝑠𝑒𝑤ℎ𝑒𝑟𝑒
(i) Find value of α
𝜋
𝜋
𝛼 ∫02 (1 − 𝑐𝑜𝑠𝑥)𝑑𝑥 + 𝛼 ∫𝜋 𝑠𝑖𝑛𝑥𝑑𝑥 = 1
2
𝜋
𝛼[𝑥 − 𝑠𝑖𝑛𝑥 ]0 + 𝛼[−𝑐𝑜𝑠𝑥 ]𝜋𝜋 =12
2
2
α=
𝜋
(ii) mean, μ
𝜋
2 2 𝜋
E(X) = ∫02 𝑥(1 − 𝑐𝑜𝑠𝑥)𝑑𝑥 + ∫𝜋 𝑥𝑠𝑖𝑛𝑥𝑑𝑥
𝜋 𝜋 2
𝜋
2 2 𝜋
= ∫0 (𝑥 − 𝑥𝑐𝑜𝑠𝑥)𝑑𝑥 + ∫𝜋 𝑥𝑠𝑖𝑛𝑥𝑑𝑥
2
𝜋 𝜋 2
𝜋
𝜋
2 𝑥2 2 2 𝜋
= [[ ] − ∫0 𝑥𝑐𝑜𝑠𝑥𝑑𝑥 ] + ∫𝜋 𝑥𝑠𝑖𝑛𝑥𝑑𝑥
2
𝜋 2 0 𝜋 2
𝜋 𝜋
2 𝑥2 2 2
= [[ ] − [𝑥𝑠𝑖𝑛𝑥 + cos 𝑥 ]0 ] + [−𝑥𝑐𝑜𝑠𝑥 + 𝑠𝑖𝑛𝑥 ]𝜋𝜋 2
𝜋 2 0 𝜋 2
𝜋
2 𝑥2 2 𝜋
= [ − (𝑥𝑠𝑖𝑛𝑥 + 𝑐𝑜𝑠𝑥)] = 1 +
𝜋 2 0 4

𝜋 3𝜋
(iii) 𝑃( < 𝑥 < )
3 4
𝜋 3𝜋
𝜋 3𝜋 2 2 2 4
𝑃( < 𝑥 < ) = ∫ (1 − 𝑐𝑜𝑠𝑥)𝑑𝑥 + ∫ 𝑠𝑖𝑛𝑥𝑑𝑥 = 1
𝜋 𝜋
3 4 𝜋 𝜋
3 2
𝜋 3𝜋
𝛼[𝑥 − 𝑠𝑖𝑛𝑥 ]𝜋2 + 𝛼[−𝑐𝑜𝑠𝑥 ]𝜋4 = 0.6982
3 2
𝑘1 𝑥, 1≤𝑥≤3
12. A random variable X of a continuous p.d.f is given by 𝑓(𝑥) = { 𝑘2 (4 − 𝑥), 3 ≤ 𝑥 ≤ 4
0, 𝑒𝑙𝑠𝑒𝑤ℎ𝑒𝑟𝑒
(a) Show that k2 = 3k1
For1 ≤ 𝑥 ≤ 3, f(x) = k1(x)
f(3) = 3k1 ……………………………….(i)
For3 ≤ 𝑥 ≤ 4, f(x) =𝑘2 (4 − 𝑥)
f(3) = k2
Eqn. (i) and eqn. (ii)
k2 = 3k1
(b) Find (i) values of k1 and k2
3 4
𝑘1 ∫1 𝑥𝑑𝑥 + 3𝑘1 ∫3 (4 − 𝑥)𝑑𝑥 = 1
3 4
𝑥2 𝑥2
𝑘1 [ ] + 3𝑘1 [4𝑥 − ] =1
2 1 2 3
2
k1=
11
6
k2 =
11

(c) mean, μ
2 3 2 6 4
E(X) = ∫ 𝑥 𝑑𝑥
11 1 11 3
+
∫ 𝑥(4 − 𝑥)𝑑𝑥
3 3 4
2 𝑥 𝑥3
[ ] + 3𝑘1 [2𝑥 2 − ] = 2.485
11 3 1 3 3
𝑦+1
1≤𝑦≤𝑘
4
13. A random variable X of a continuous p.d.f is given by 𝑓(𝑥) = {
0, 𝑒𝑙𝑠𝑒𝑤ℎ𝑒𝑟𝑒
Find
(a) The value of k (06marks)
𝑘
𝑘 (𝑦+1) 1 𝑦2
∫0 𝑑𝑦 = [ + 𝑦] = 1
4 4 2 0
1 𝑘2
[( + 𝑘) − 0] = 1
4 2
𝑘 2 + 2𝑘 − 8 = 0
(k + 4)(k-2) = 0
Either
k + 4 = 0; k = -4
Or
k-2 = 0; k = 2
∴ 𝑘 = 2 (since k is greater than zero)
(b) The expectation of Y (03marks)
2
E(Y) = ∫0 𝑦𝑑𝑦
2 𝑦+1
=∫0 𝑦 [ ] 𝑑𝑦
4
2 𝑦 2 +𝑦
=∫0 ( ) 𝑑𝑦
4
2
1 𝑦3 𝑦2
= [ − ]
4 3 2 0
1 8 4 7
= [( − ) − 0] = = 1.166
4 3 2 6
(c) P(1≤ 𝑌 ≤ 1.5) (03marks)
1.5 𝑦+1
P(1≤ 𝑌 ≤ 1.5) = ∫1 [ ] 𝑑𝑦
4
1.5
1 𝑦2
= [ + 𝑦]
4 2 1
1 (1.5)2 1
= [( + 1.5) − ( + 1)]
4 2 2
1
= (2.625 − 1.5)
4
= 0.28125

Variance of X
For a continuous random variable with p.d.f, f(x)

Var(X) = EX2 – [E(X)]2 or Var(X) = E(X2) – μ2

Where E(X2) = ∫ 𝑥 2 (𝑥)𝑑𝑥 and μ = mean


Properties of variance

(i) Var(a) = 0
(ii) Var(ax) = a2Var(x)
(iii) Var(ax + b) = a2Var(x)
(iv) Var(ax – b) = a2Var(X)

Where a and b are constants

Example 26

𝑘(1 − 𝑥 2 ), 0≤𝑥≤1
A random variable x of a continuous p.d.f is given by 𝑓(𝑥) = {
0, 𝑒𝑙𝑠𝑒𝑤ℎ𝑒𝑟𝑒
Find

(i) the value of k


1 (iii) Var(X)
𝑘 ∫0 (1 − 𝑥 2 ) 𝑑𝑥 = 1
1
1 E(X2) = 1.5 ∫0 𝑥 2 (1 − 𝑥 2 ) 𝑑𝑥
𝑥3
𝑘 [𝑥 − ] =1
3 0 1
𝑥3 𝑥5 1
= 1.5 [ − ] =
3 5 0 5
k = 1.5

(ii) E(X) Var(X) = EX2 – [E(X)]2

1 1 3 2 19
E(X) =1.5 ∫0 𝑥(1 − 𝑥 2 ) 𝑑𝑥 = −( ) =
5 8 320

1
𝑥2 𝑥4 3
=1.5 [ − ] =
2 4 0 8

Example 27
1
𝑥, 0≤𝑥≤4
A random variable x of a continuous p.d.f is given by 𝑓(𝑥) = { 8
0, 𝑒𝑙𝑠𝑒𝑤ℎ𝑒𝑟𝑒
Find

(i) E(X)
4 (iii) Standard deviation
1 4 1 𝑥3
E(X) = ∫0 𝑥. 𝑥 𝑑𝑥 = [ ] = 2.667
8 8 3 0
s.d = √𝑉𝑎𝑟(𝑋)
(ii) Var(X)
=√0.887 = 0.942
4
1 4 2 1 𝑥4
E(X2) = ∫ 𝑥 . 𝑥 𝑑𝑥 = [ ] =8 (iv) Var(3x + 2) = 0.887 x 3 = 7.983
8 0 8 4 0

Var(X) = EX2 – [E(X)]2

= 8 – (2.667)2 = 0.887

Example 28
4
(5 − 2𝑥), 0 ≤ 𝑥 ≤ 2.5
A random variable x of a continuous p.d.f is given by 𝑓(𝑥) = {25
0, 𝑒𝑙𝑠𝑒𝑤ℎ𝑒𝑟𝑒
Find

(i) Mean
2.5
4 2.5 4 5𝑥 2 2𝑥 3
E(X) = ∫ 𝑥(5 − 2𝑥) 𝑑𝑥 = 25 [
25 0 2

3
] = 0.833
0
(ii) Standard deviation
2.5
4 2.5 4 5𝑥 3 2𝑥 4
E(X2) = ∫ 𝑥 2 (5 − 2𝑥) 𝑑𝑥 = 25 [
25 0 3

4
] = 1.041
0
Var(X) = EX2 – [E(X)]2 = 1.041 – (0.5625)2 = 0.347
s.d = √𝑉𝑎𝑟(𝑋)= √0.347 = 0.59

Example 29
3
(1 + 𝑥 2 ), 0≤𝑥≤1
A random variable x of a continuous p.d.f is given by 𝑓(𝑥) = { 4
0, 𝑒𝑙𝑠𝑒𝑤ℎ𝑒𝑟𝑒
Find

(i) Mean
1
3 1 3 𝑥2 𝑥4
E(X) = ∫0 𝑥(1 + 𝑥 2 ) 𝑑𝑥 = [ + ] = 0.5625
4 4 2 4 0
(ii) Standard deviation
1
3 1 3 𝑥3 𝑥5
E(X2) = ∫0 𝑥 2 (1 + 𝑥 2 ) 𝑑𝑥 = [ + ] = 0.4
4 4 3 5 0
Var(X) = 0.4 – (0.525)2 = 0.835
s.d = √0.0835 = 0.289
(iii) P(|𝑋 − 𝜇| < 𝜎)
P(|𝑋 − 𝜇| < 𝜎) = P(|𝑋 − 0.5625 |< x < 0.289)
= P(0.2735 < x < 0.8515
0.8515
3 0.8515 3 𝑥3
∫ (1 + 𝑥 2 ) 𝑑𝑥 = [𝑥 + ] = 0.583
4 0.2735 4 3 0.2735

Revision exercise 3
𝑘𝑥 2 , 0≤𝑥≤4
1. A random variable x of a continuous p.d.f is given by 𝑓(𝑥) = {
0, 𝑒𝑙𝑠𝑒𝑤ℎ𝑒𝑟𝑒
(a) Sketch f(x)
3 7
(b) Find (i) value of k (= ) (ii) E(X) = 3 and var (X) = 0.6 (iii) P(1<X<2) =
64 64
𝑘𝑥, 0≤𝑥≤1
2. A random variable x of a continuous p.d.f is given by 𝑓(𝑥) = {𝑘(2 − 𝑥) 0≤𝑥≤1
0, 𝑒𝑙𝑠𝑒𝑤ℎ𝑒𝑟𝑒
1 19
Find (i) constant k = 1 (ii) E(X) = 1 (iii) var(X) = (iv)P(0.75 < X < 1.5) = (v) mode =1
6 32
1
𝑥2, 0≤𝑥≤3
27
3. A random variable x of a continuous p.d.f is given by 𝑓(𝑥) = { 1 3≤𝑥≤5
3
0, 𝑒𝑙𝑠𝑒𝑤ℎ𝑒𝑟𝑒
(a) Sketch f(x)
(b) Find (i) E(X) = 3417 (ii) standard deviation = 1.008
𝑘
, 1≤𝑥≤3
4. A random variable x of a continuous p.d.f is given by 𝑓(𝑥) = {𝑥(4−𝑥)
0, 𝑒𝑙𝑠𝑒𝑤ℎ𝑒𝑟𝑒
3
(i) Show that k =
𝐼𝑛𝑥
4
(ii) Find (i) E(X) = 2 (ii) Var(X) = 4-
𝐼𝑛𝑥
𝑘(𝑎𝑥 − 𝑥 2 ), 0≤𝑥≤2
5. A random variable x of a continuous p.d.f is given by 𝑓(𝑥) = {
0, 𝑒𝑙𝑠𝑒𝑤ℎ𝑒𝑟𝑒
8
(i) Show that k =
6𝑎−8
(ii)Given that E(X) = 1, find the values of a (=2) and k(=0.75)
(iii)
For the above values of a and k, find Var(X) = 0.2
12(𝑥 2 − 𝑥 3 ), 0≤𝑥≤1
6. A random variable x of a continuous p.d.f is given by 𝑓(𝑥) = {
0, 𝑒𝑙𝑠𝑒𝑤ℎ𝑒𝑟𝑒
Find the (i) mean = 0.6 (ii) standard deviation = 0.2
𝑘
, 0≤𝑥≤𝛽
7. A random variable x of a continuous p.d.f is given by 𝑓(𝑥) = {𝛽
0, 𝑒𝑙𝑠𝑒𝑤ℎ𝑒𝑟𝑒
𝛽 𝛽
Find (i) value of k (=1) (ii) mean = (iii) standard deviation =
2 2√3

1
(𝑥 + 1), 2≤𝑥≤4
8. A random variable x of a continuous p.d.f is given by 𝑓(𝑥) = { 8
0, 𝑒𝑙𝑠𝑒𝑤ℎ𝑒𝑟𝑒
37 47
Find (i) mean = (ii) var(X) = (iii) P(2.5 < x< 3) = 0.234
12 144
𝑘(1 − 𝑥)2 , 2≤𝑥≤4
9. A random variable x of a continuous p.d.f is given by 𝑓(𝑥) = {
0, 𝑒𝑙𝑠𝑒𝑤ℎ𝑒𝑟𝑒
3 1
Find (i) constant k = (ii) mean = (iii) standard deviation = 0.94
26 4
𝑘𝑥, 0≤𝑥≤2
10. A random variable x of a continuous p.d.f is given by 𝑓(𝑥) = {𝑘(4 − 𝑥) 2 ≤ 𝑥 ≤ 4
0, 𝑒𝑙𝑠𝑒𝑤ℎ𝑒𝑟𝑒
1 2 1 3
Find (i) value of k = (ii) E(X) = 2 (iii) Var(X) = (iv) P(X< 1) = (iv) P(X<X<3) =
4 3 8 8

Mode
This is the value of f(x) is maximum in the given range of x.
𝑑
(i) The mode is obtained from (𝑓𝑥) = 0
𝑑𝑥
𝑑2
The maximum value is confirmed if (𝑓𝑥) = negative
𝑑𝑥 2
(ii) When a sketch of f(x) is drawn, the value of x for which f(x) is maximum gives the mode.

Note: for any line the mode can be determined from a sketch of f(x)

Example 30
𝑘(2 + 𝑥)(4 − 𝑥), 0≤𝑥≤4
A random variable x of a continuous p.d.f is given by 𝑓(𝑥) = {
0, 𝑒𝑙𝑠𝑒𝑤ℎ𝑒𝑟𝑒
Find

(i) Value of k
4
𝑘 ∫0 (2 + 𝑥)(4 − 𝑥)𝑑𝑥 = 1
4
𝑘 ∫0 (8 + 2𝑥 − 𝑥 2 )𝑑𝑥 = 1
4
𝑥3 3
[8𝑥 + 𝑥 − ] = 1; k =
3 0 80
(ii) Mode
𝑑
(𝑓𝑥) = 0
𝑑𝑥
𝑑 3
(8 + 2𝑥 − 𝑥 2 ) = 0
𝑑𝑥 80
3
(2 − 2𝑥) = 0; x = 1
80
∴mode = 1

Example 30
1
𝑥(6 − 𝑥)2 , 0≤𝑥≤6
A random variable x of a continuous p.d.f is given by 𝑓(𝑥) = {108
0, 𝑒𝑙𝑠𝑒𝑤ℎ𝑒𝑟𝑒
Find

(i) Mean
6 1 (iii) mode
E(X) = ∫0 𝑥 2 (6 − 𝑥)2 𝑑𝑥
108
1 6 𝑑
= ∫ (36𝑥 2 − 12𝑥 3 + 𝑥 4 )𝑑𝑥 (𝑓𝑥) = 0
108 0 𝑑𝑥
𝑑 1
1 𝑥5
6 𝑥(6 − 𝑥)2 = 0
= [12𝑥 3 − 3𝑥 4 + ] = 2.4 𝑑𝑥 108
108 5 0 𝑑 1
(36𝑥 − 12𝑥 2 + 𝑥 3 )=0
(ii) Standard deviation 𝑑𝑥 108
1
E(X2) = ∫0
6 1
𝑥 3 (6 − 𝑥)2 𝑑𝑥 (36 − 24𝑥 + 3𝑥 2 ) = 0
108
108
1 6 1 (6 − 𝑥)(2 − 𝑥) = 0
= ∫ (36𝑥 3 − 12𝑥 4 + 𝑥 5 )𝑑𝑥
108 0 108 x = 6 or x = 2
6
1 12𝑥 5 𝑥6 ∴ mode = 2 or 6
= [9𝑥 4 − + ] = 7.2
108 5 6 0

s.d = √7.2 − (2.4)2 = 1.2

Example 31
𝑘𝑠𝑖𝑛𝑥 , 0≤𝑥≤𝜋
A random variable x of a continuous p.d.f is given by 𝑓(𝑥) = {
0, 𝑒𝑙𝑠𝑒𝑤ℎ𝑒𝑟𝑒
Find

(i) value k
𝜋 Sign derivative integral sign
∫0 𝑘𝑠𝑖𝑛𝑥 𝑑𝑥 = 1
𝑘[−𝑐𝑜𝑠𝑥 ]𝜋0 = 1 + x sinx
k[-cosπ- -cos0) = 1
1 - 1 -cosx
k=
2
𝜋 + 0 -sinx
(ii) P(X ≥ )
3
𝜋 1 𝜋 3 1 𝜋
(iii) P(≥ ) = ∫𝜋 𝑠𝑖𝑛𝑥 𝑑𝑥 =𝑘 [−𝑐𝑜𝑠𝑥 ]𝜋𝜋 =  E(x) = ∫0 𝑥𝑠𝑖𝑛𝑥 𝑑𝑥
2
3 2 3 4 3
(iv) Mean 1
1 𝜋
= [−𝑥𝑐𝑜𝑠𝑥 + 𝑠𝑖𝑛𝑥 ]𝜋0
2
E(x) = ∫ 𝑥𝑠𝑖𝑛𝑥
2 0
𝑑𝑥
𝜋
=
2
(v) Var (X)
1 𝜋
E(X2) = ∫0 𝑥 2 𝑠𝑖𝑛𝑥 𝑑𝑥
2

Sign Derivative Integral sign


+ x2 sinx
- 2x -cosx
+ 2 -sinx
- 0 cosx
1 𝜋 1 𝜋2 −4
 E(X2) = ∫0 𝑥 2 𝑠𝑖𝑛𝑥 𝑑𝑥 = [−𝑥 2 𝑐𝑜𝑠𝑥 + 2𝑥𝑠𝑖𝑛𝑥 + 2𝑐𝑜𝑠𝑥 ]𝜋0 =
2 2 2
𝜋2 −4
∴Var(X) =
2
(vi) Mode
𝑑 1
( 𝑠𝑖𝑛𝑥) =0
𝑑𝑥 2
1
𝑐𝑜𝑠𝑥 = 0
2
𝑥 = 900
𝜋
∴mode =
2

Example 32
𝑘𝑥 , 0≤𝑥≤6
A random variable x of a continuous p.d.f is given by 𝑓(𝑥) = {
0, 𝑒𝑙𝑠𝑒𝑤ℎ𝑒𝑟𝑒
(a) Sketch f(x) (b) value of k

When x= 0, f(x) = k(0) = 0 Area under the graph = 1


When x= 6, f(x) = k(6) = 6k 1
𝑥𝑘𝑥6𝑥6=1
2
f(x) 1
k=
6k 18

k (c) mode = 6

0 6 x

Median
𝑚
This is the value of f(x) for which ∫𝑎 𝑓(𝑥) = 0.5; where m is the median, and a is the lower limit.

Example 33
1
𝑥, 0≤𝑥≤4
A random variable x of a continuous p.d.f is given by 𝑓(𝑥) = { 8
0, 𝑒𝑙𝑠𝑒𝑤ℎ𝑒𝑟𝑒
Find the median
𝑚1
∫0 𝑥𝑑𝑥 = 0.5
8
1 𝑚
[ 𝑥2] = 0.5
16 0

𝑚2
= 0.5; m = √8 = ±2.828
16

Median = 2.828 (since it falls in the range)

Example 34
2
(𝑥 + 2) , −1≤𝑥 ≤0
5
A random variable x of a continuous p.d.f is given by 𝑓(𝑥) = {4 (1 − 𝑥) 0 ≤ 𝑥 ≤ 10
5
0, 𝑒𝑙𝑠𝑒𝑤ℎ𝑒𝑟𝑒
Find the median

Solution

We need to first integrate the first interval to check if it is ≥ 0.5. if not the median lies in the second
interval
0
0 2 2 𝑥2
∫−1 5 (𝑥 + 2)𝑑𝑥 = 5
[
2
+ 2𝑥] = 0.6
−1

It shows that the median lies in the first interval


𝑚
𝑚2 2 𝑥2
Then ∫−1 (𝑥 + 2)𝑑𝑥 = [ + 2𝑥] = 0.5
5 5 2 −1

m = -0.129 or m = -3.871

the median = -0.129 since it lies in the range

Example 34
2
(𝑥 + 1) , −1≤𝑥 ≤0
3
A random variable x of a continuous p.d.f is given by 𝑓(𝑥) = {1 (2 − 𝑥) 0≤𝑥≤2
3
0, 𝑒𝑙𝑠𝑒𝑤ℎ𝑒𝑟𝑒
Find the median

We need to first integrate the first interval to check if it is ≥ 0.5. if not the median lies in the second
interval
0
0 2 2 𝑥2 1
∫−1 3 (𝑥 + 1)𝑑𝑥 = 5
[
2
+ 𝑥] =
3
−1

It shows that the median lies in the second interval


1 1 𝑚 1
Then + ∫ (2 − 𝑥)𝑑𝑥 =
3 3 0 2
𝑚
1 𝑥2 1
[2𝑥 − ] = ; m =0.268
3 2 0 6
Revision exercise 4
𝑘𝑥(4 − 𝑥 2 ) , 0≤𝑥≤2
1. A random variable x of a continuous p.d.f is given by 𝑓(𝑥) = {
0, 𝑒𝑙𝑠𝑒𝑤ℎ𝑒𝑟𝑒
Find
(i) value of the constant =0.25 (iii) mean = 1.067
(ii) median x =2.613 (iv) standard deviation = 0.442
𝑘𝑥, 0≤𝑥≤1
2. A random variable x of a continuous p.d.f is given by 𝑓(𝑥) = {𝑘(2 − 𝑥) 1≤𝑥≤2
0, 𝑒𝑙𝑠𝑒𝑤ℎ𝑒𝑟𝑒
Find
(i) constant k = 1 (ii) median = 1 (iii) mode = 1
𝑘𝑥(4 − 𝑥 2 ) , 0≤𝑥≤2
3. A random variable x of a continuous p.d.f is given by 𝑓(𝑥) = {
0, 𝑒𝑙𝑠𝑒𝑤ℎ𝑒𝑟𝑒
Find
1
(i) value of the constant = (iii) mean = 1.0667
4
(ii) median x =2.6131 (iv) standard deviation = 0.4422
𝛼, 2≤𝑥≤3
4. A random variable x of a continuous p.d.f is given by 𝑓(𝑥) = {𝛼(𝑥 − 2) 3 ≤ 𝑥 ≤ 4
0, 𝑒𝑙𝑠𝑒𝑤ℎ𝑒𝑟𝑒
(a) sketch f(x)
(b) find (i) constant α = 0.4 (ii) median, m = 3.225 (iii) P(2.5 < x < 3.5) =0.65
𝛽, 0≤𝑥≤2
5. A random variable x of a continuous p.d.f is given by 𝑓(𝑥) = {𝛽(3 − 𝑥) 2 ≤ 𝑥 ≤ 3
0, 𝑒𝑙𝑠𝑒𝑤ℎ𝑒𝑟𝑒
19 5
Find (i) value of β =0.4 (ii) mean = (iii) standard deviation = (iv) P(X<μ –σ) =0.207
15 4
𝑥2, 0≤𝑥≤1
1
6. A random variable x of a continuous p.d.f is given by 𝑓(𝑥) = { 1≤𝑥≤𝑘
2
0, 𝑒𝑙𝑠𝑒𝑤ℎ𝑒𝑟𝑒
(i) Sketch f(x)
7 49 4
(ii) Find (i) value of k = (ii) mean = (iii) median =
3 36 3
𝑘𝑥, 0≤𝑥≤1
7. A random variable x of a continuous p.d.f is given by 𝑓(𝑥) = { 𝑘 1≤𝑥≤2
0, 𝑒𝑙𝑠𝑒𝑤ℎ𝑒𝑟𝑒
(i) Sketch f(x)
2 49 17
(ii) (iii) median = 1.25 (iv) P(|𝑋 − 𝑚|>0.5) =
Find (i) value of k = (ii) mean =
3 36 48
2𝑘(𝑥 + 1) , −1≤𝑥 ≤0
8. A random variable x of a continuous p.d.f is given by 𝑓(𝑥) = {𝑘(2 − 𝑥) 0≤𝑥≤2
0, 𝑒𝑙𝑠𝑒𝑤ℎ𝑒𝑟𝑒
(i) Sketch f(x)
1 1 5
(ii) Find (i) value of k = (ii) mean = (iii) Var(X) = (iv) mode =0
3 3 18
Cumulative distribution function, F(x)
𝑥
The cumulative distribution function F(x) is defined by F(x) = ∫𝑎 𝑓𝑥)𝑑𝑥

Steps in finding F(x)

- For each interval, integrate its function from lower limit to x with respect to x.
- Substitute the upper limit in the integral and carry it forward to the next interval
- Continue the process until when the last upper limit has been substituted to get a 1.

Example 35
1
(𝑥 + 1) , 1≤𝑥≤3
A random variable x of a continuous p.d.f is given by 𝑓(𝑥) = {6
0, 𝑒𝑙𝑠𝑒𝑤ℎ𝑒𝑟𝑒
Find F(x)

Solution
𝑥
1 𝑥 1 𝑥2 1 𝑥2 12
F(x) = ∫1 (𝑥 + 1)𝑑𝑥 = [ + 𝑥] = {( + 𝑥) − ( + 1)}
6 6 2 1 6 2 2

1 𝑥2 3
F(x) = ( +𝑥− )
6 2 2

1 32 3
F(3) = ( + 3 − ) =1
6 2 2

0 𝑥≤1
1 𝑥2 3
∴ F(x) = { ( +𝑥− ) , 1≤𝑥≤3
6 2 2
1, 𝑥≥3
Example 36
3
(1 − 𝑥)2 , 2≤𝑥≤4
A random variable x of a continuous p.d.f is given by 𝑓(𝑥) = {26
0, 𝑒𝑙𝑠𝑒𝑤ℎ𝑒𝑟𝑒
Find F(x)
𝑥
3 𝑥 3 𝑥 3 𝑥3
F(x) = ∫ (1 − 𝑥)2 𝑑𝑥 = 26 ∫2 (1 − 2𝑥 + 𝑥 2 )𝑑𝑥 = 26 [𝑥 − 𝑥 2 +
26 2 3 2
]

3 𝑥3 23 3 𝑥3 2
= {(𝑥 − 𝑥 2 + ) − (2 − 22 + )} = (𝑥 − 𝑥 2 + − )
26 3 3 26 3 3

43 2
F(4) =(4 − 42 + − )=1
3 3

0 𝑥≤2
3 𝑥3 2
∴ F(x) = { (𝑥 − 𝑥 2 + − ), 2≤𝑥≤3
26 3 3
1, 𝑥≥4
Example 37
𝑥, 0≤𝑥≤1
A random variable x of a continuous p.d.f is given by 𝑓(𝑥) = { (2 − 𝑥) , 1≤𝑥≤2
0, 𝑒𝑙𝑠𝑒𝑤ℎ𝑒𝑟𝑒
Find F(x)
𝑥
𝑥 𝑥2 𝑥2 02 𝑥2
For 0 ≤ 𝑥 ≤ 1, F(x) = ∫𝑜 𝑥𝑑𝑥 = [ ] = ( − )=
2 0 2 2 2

12 1
F(1) = =
2 2
𝑥
1 𝑥 1 𝑥2 1 𝑥2 12
For 1 ≤ 𝑥 ≤ 2; F(x) = + ∫1 (2 − 𝑥)𝑑𝑥 = + [2𝑥 − ] = + {(2𝑥 − ) − (2 − )}
2 2 2 1 2 2 2

𝑥2
=(2𝑥 − )−1
2

22
F(x) = (2𝑥2 − ) − 1= 1
2

0 𝑥≤1
𝑥2
, 0≤𝑥≤1
2
∴ 𝐹(𝑥) = 𝑥2
(2𝑥 − )−1, 1≤𝑥≤2
2
{ 1, 𝑥≥2

Example 38
2
, 0≤𝑥≤2
5
A random variable x of a continuous p.d.f is given by 𝑓(𝑥) = {2 (3 − 𝑥) , 2≤𝑥≤3
5
0, 𝑒𝑙𝑠𝑒𝑤ℎ𝑒𝑟𝑒
Find F(x)
2 𝑥 2 2 2
For 0 ≤ 𝑥 ≤ 2, F(x) = ∫0 𝑑𝑥 = [𝑥 ]0𝑥 = {𝑥 − 0} = 𝑥
5 5 5 5
2 4
F(2) = 𝑥2 =
5 5
𝑥
4 2 𝑥 4 2 𝑥2 4 2 𝑥2 22
For 2 ≤ 𝑥 ≤ 3, F(X) = +
5
∫ (3 − 𝑥)𝑑𝑥 = 5 +
5 0 5
[3𝑥 −
2 2 5
] = +
5
(3𝑥 −
2
) − (3𝑥2 −
2
)

2 𝑥2 4
F(x) = (3𝑥 − )−
5 2 5

2 32 4
F(3) = (3𝑥3 − )− =1
5 2 5

0 𝑥≤0
2
𝑥, 0≤𝑥≤2
5
∴ 𝐹(𝑥) = 2 𝑥2 4
(3𝑥 − )− , 2≤𝑥≤3
5 2 5
{1, 𝑥≥3
Finding the median, quartiles and probability from F(x)

- The median is the value of m for which F(m) = 0.5


- The lower quartile is the value q1 for which F(q1) = 0.25
- The upper quartile is the value q3 for which F(q3) = 0.75

Example 39

The continuous random variable X has a cumulative distribution function given below
0 𝑥≤0
𝑥2
𝐹(𝑥) = { 0≤𝑥≤4
16
1 𝑥≥4
Find

(i) P(0.3≤ X ≤ 1.8)


1.82 0.32
P(0.3≤ X ≤ 1.8) = F(1.8) – F(0.3) = − = 0.197
16 16

(ii) Median, m
F(m) = 0.5
𝑚2
= 0.5; 𝑚 = ±2.828
16
𝑚𝑒𝑑𝑖𝑎𝑛 = 2.828 (𝑠𝑖𝑛𝑐𝑒 𝑖𝑡 𝑖𝑠 𝑤𝑖𝑡ℎ𝑖𝑛 𝑡ℎ𝑒 𝑟𝑎𝑛𝑔𝑒
(iii) Interquartile range
F(q1) = 0.25
𝑞12
= 0.25; 𝑞1 = 2
16
F(q3) = 0.75
𝑞32
= 0.75; 𝑞3 = 3.464
16
Interquartile range = 3.464 – 2 = 1.464

F(x)

1 F(x) = 1
k
𝑥2
F(x) =
16

0 4 x

Example 40
0 𝑥≤0
The continuous random variable X has a c.d.f given by 𝐹(𝑥) = { 2𝑥 − 𝑥 2 0≤𝑥≤1
1 𝑥≥4
Find

(i) F(X≤0.5)
F(X≤0.5) = F(0.5) – F(0) =(2(0.5) − (0.5)2 ) − (2(0) − (0)2 )=0.75
(ii) Median, m
F(m) = 0.5
(2(𝑚) − (𝑚)2 ) = 0.5
𝑚2 − 2𝑚 + 0.5 = 0
m = 1.71 or m = 0.293
m = 0.293 (since it is in the range)
(iii) Interquartile range
F(q1) = 0.25
2𝑞1 − 𝑞12 = 0.25; 𝑞1 = 0.134
F(q3) = 0.75
2𝑞3 − 𝑞32 = 0.75; 𝑞3 = 0.5
Interquartile range = 0.5 – 0.134 = 0.366

Example 40
0 𝑥≤0
𝑥2
0≤𝑥≤2
6
The cumulative distribution function is given by 𝐹(𝑥) = 𝑥2
− + 2𝑥 − 2 2≤𝑥≤3
3
{ 1 𝑥≥3
Find

(i) P(1 ≤ x ≤ 2.5)


P(1 ≤ x ≤ 2.5) = P(2.5) – P(1)
2.52 12
− + 2𝑥 2.5 − 2 − =0.75
3 6
(ii) Median, m
P(0 ≤ x ≤ 2) = F(2) –F(0)
22 02 2
= − =
6 6 3
2
Since > 0.5 the median lies between 0 ≤ 𝑥 ≤ 2
3
F(m) =0.5
𝑚2
= 0.5
6
𝑚 = ±1.73
Median = 1.73

Revision exercise 5
3
𝑥2 0≤𝑥≤2
1. The random variable X has a probability density function f(x) ={ 8
0 𝑒𝑙𝑠𝑒𝑤ℎ𝑒𝑟𝑒
Find
(i) Sketch F(X)
0 𝑥<2
1 3
(ii) Cumulative distribution function; F(x) ={ 𝑥 0≤𝑥≤2
8
1 𝑥≥0
(iii) Median, m = 1.59
1
(4 − 𝑥) 1≤𝑥≤3
2. The random variable X has a probability density function f(x) ={4
0 𝑒𝑙𝑠𝑒𝑤ℎ𝑒𝑟𝑒
Find
0 𝑥≤1
1
(i) cumulative mass function; F(x) ={8 (8𝑥 − 𝑥 2 − 7) 1≤𝑥≤3
1 𝑥≥3
9
(ii) P(1.5≤x≤2)=
32
(iii) median, m = 1.764
(iv) sketch F(x)
𝑘 1≤𝑥≤6
3. The random variable X has a probability density function f(x) ={
0 𝑒𝑙𝑠𝑒𝑤ℎ𝑒𝑟𝑒
Find
1
(i) Value of k =
5
0 𝑥≤1
1
(ii) Cumulative function, F(x) ={ 5 (𝑥 − 1) 1≤𝑥≤6
1 𝑥≥6
(iii) Interquartile range =2.5
1
0≤𝑥≤2
4
4. The random variable X has probability density function f(x) ={1 (2𝑥 − 3) 2 ≤ 𝑥 ≤ 3
4
0 𝑒𝑙𝑠𝑒𝑤ℎ𝑒𝑟𝑒
Find
0 𝑥≤1
𝑥
0≤𝑥≤2
4
(i) Cumulative function, F(x) = 1
(𝑥 2 − 3𝑥 + 4) 2≤𝑥≤3
4
{ 1 𝑥≥3
(ii) Median, m= 2
(iii) Sketch F(x)
0 𝑥≤0
5. The random variable X has a cumulative distribution function, F(x) ={ 𝑥 4 1≤𝑥≤1
1 𝑥≥1
Find
(i) P(0.3 ≤ x ≤ 0.6) =0.1215
(ii) Median, m = 0.841
(iii) The value of a such that P(X>a) = 0.88

1
0≤𝑥≤3
6. The random variable X has a probability density function f(x) ={3
0 𝑒𝑙𝑠𝑒𝑤ℎ𝑒𝑟𝑒
Find (i) E(x) = 1.5 (ii) Var (X) = 0.75 (iii) P(X> 1.8) =0.4 (iv) P(1.1 < x <1.7) =0.2
0 𝑥≤0
1
(v) cumulative distribution function, F(x) ={ 3 𝑥 0≤𝑥≤3
1 𝑥≥3
2
7. The random variable X has a probability density function f(x) ={𝑘𝑥 1≤𝑥≤2
0 𝑒𝑙𝑠𝑒𝑤ℎ𝑒𝑟𝑒
Find
3
(i) Value of k = (ii) standard deviation = 0.272 (iii) median, m = 1.65
7
0 𝑥≤1
1
(ii) Cumulative mass function, F(x) ={ (𝑥 3 − 1)
7
1≤𝑥≤2
1 𝑥≥2
2
8. The continuous random variable X has a p.d.f given by f(x) ={𝑘(4 − 𝑥 ) 0≤𝑥≤2
0 𝑒𝑙𝑠𝑒𝑤ℎ𝑒𝑟𝑒
3 3 19
Find (i) constant k (= ) (ii) E(x) = (iii) Var(X) = (iv) median = 0.695
16 4 80
0 𝑥≤0
3 1 3
(v) cumulative distribution function, F(X) = { 𝑥 − 𝑥4 16
0≤𝑥≤2
1 𝑥≥2
(vi) = P(0.69 ≤ x ≤ 0.7)= 0.007
1+𝑥
1≤𝑥≤3
9. The continuous random variable X has a p.d.f given by f(x) ={ 6
0 𝑒𝑙𝑠𝑒𝑤ℎ𝑒𝑟𝑒
(i) Sketch f(x)
19
(ii) Find the mean =
9
(iii) Find m such that P(X≤ m) 0.5; m = 2.16
(iv) Determine cumulative function, F(X) and sketch it

0 𝑥≤0
1 1 1
F(X) = {5 𝑥 + 12 𝑥 2 − 4 1≤𝑥≤3
1 𝑥≥3
10. A factory is supplied with flour at the beginning of each week. The weekly demand, X thousand
tones for flour from this factory is a continuous random variable having a probability density
𝑘 1≤𝑥≤3
function f(x) ={
0 𝑒𝑙𝑠𝑒𝑤ℎ𝑒𝑟𝑒
Find
(i) Value of k = 5
1
(ii) Mean of x =
6
5
(iii) Variance of x =
252
1
0≤𝑥≤1
4
11. The continuous random variable X has a p.d.f given by f(x) ={𝑥 3 1≤𝑥≤2
5
0 𝑒𝑙𝑠𝑒𝑤ℎ𝑒𝑟𝑒
Find
0 𝑥<0
1
𝑥 0≤𝑥≤1
4
(i) Cumulative mass function, F(x) and sketch it F(x) = 1 𝑥4
+ 1≤𝑥≤2
5 20
{1 𝑥≥2
(ii) Median, m=1.565 (iii) interquartile range = 0.821
𝑘(𝑥 + 3) −3≤𝑥 ≤3
12. The continuous random variable X has a p.d.f given by f(x) ={
0 𝑒𝑙𝑠𝑒𝑤ℎ𝑒𝑟𝑒
1
(a) Show that k =
18
(b) Find (i) E(x) =1, (ii) Var (x) = 2 (iii) Lower quartile, q1 = 0
1
(c) Given that E(ax + b) = 0 and Var(ax+b) = 1, find the values of a and b where a>0 (a = b) =
√2
𝑘𝑥 0≤𝑥≤8
13. The continuous random variable X has a p.d.f given by f(x) ={8𝑘 8≤𝑥≤9
0 𝑒𝑙𝑠𝑒𝑤ℎ𝑒𝑟𝑒
(a) Sketch f(x)
(b) Find value of k = 0.025 (ii) P(X>6) =0.55
0 𝑥<0
0.0125𝑥 0≤𝑥≤8
(c) Find F(X) =={
0.2𝑥 − 0.8 8≤𝑥≤9
1 𝑋≥9
2
14. The continuous random variable X has a p.d.f given by f(x) ={𝑎𝑥 − 𝑏𝑥 0≤𝑥≤2
0 𝑒𝑙𝑠𝑒𝑤ℎ𝑒𝑟𝑒
If E(X) =1, find
(i) values of a and b (a= 1.5, b= 0.75) (ii) Var(x) = 0.2
0 𝑥<0
(ii) F(X) ={0.75𝑥 2 − 0.25𝑥 3 0≤𝑥≤2
1 𝑥≥2
𝑘
1≤𝑥≤9
15. The continuous random variable X has a p.d.f given by f(x) ={ 𝑥
0 𝑒𝑙𝑠𝑒𝑤ℎ𝑒𝑟𝑒
Find (i) value of k =0.455, (ii) median =3 (iii) mean =3.64 (iv) Var(X) =4.95
0 𝑥<1
1
(v) F(X) ={ 𝐼𝑛𝑥
𝐼𝑛 9
1≤𝑥≤9
1 𝑥≥9

20
𝑤 3 (5 − 𝑤) 0≤𝑤≤5
16. The continuous random variable X has a p.d.f given by f(x) = {55
0 𝑒𝑙𝑠𝑒𝑤ℎ𝑒𝑟𝑒
Find (i) P(2<w<5) =0.5 (ii) mean = 3.33 (iii) Var(X) = 0.794 (iv) mode = 3.5
0 𝑤<0
𝑤4
(v) F(X) = = { (25 − 𝑤) 0≤𝑥≤5
55
1 𝑤≥5
𝑘𝑥 0≤𝑥≤1
17. The continuous random variable X has a p.d.f given by f(x) = {𝑘(4 − 𝑥 2 ) 1≤𝑥≤2
0 𝑒𝑙𝑠𝑒𝑤ℎ𝑒𝑟𝑒
6
Find (i) value of k = (ii) E(X) = 1.1923 (iii) Var(x) =0.1399
13
0 𝑥<0
3
𝑥 0≤𝑥≤1
(iv) F(x) = 113
(24𝑥 − 2𝑥 3 − 19) 1≤𝑥≤2
13
{ 1 𝑥≥2
18. The probability density function f(x) of a random variable x takes on the form shown in the
diagram below

Find

(i) Expression for f(x)


(ii) F(x), cumulative distribution function
2 2
(iii) Mean = and Var(x) =
3 9
Finding f(x) from F(X)
𝑑
f(x) can be obtained from; f(x) = 𝐹(𝑋)
𝑑𝑥

Example 41
0 𝑥<0
𝑥3
The continuous random variable X has a c.d.f F(X) = { 0≤𝑥≤3
27
1 𝑥≥3
Find the probability density function f(x) and sketch f(x)
𝑑 𝑑 𝑥3 3𝑥 2 𝑥2
f(x) = 𝐹(𝑋) = ( )= = f(x)
𝑑𝑥 𝑑𝑥 27 27 9

𝑥2 1
f(x) = { 9 0≤𝑤≤3
0 𝑒𝑙𝑠𝑒𝑤ℎ𝑒𝑟𝑒 𝑥2
f(x)=
02 9
When x = 0, f(x) = =0
9

32 0 3 x
When x = 3, f(x) = =1
9

Example 42
0 𝑥<0
The continuous random variable X has a c.d.f F(X) = { 𝑘𝑥 3 0≤𝑥≤4
1 𝑥≥4
Find

(i) Value of k
F(4) – F(0) = 1
1
K(43) = 1; k =
64
(ii) Probability density function, f(x)
𝑑 𝑑 𝑥3 3𝑥 2
f(x) = 𝐹(𝑋) = =
𝑑𝑥 𝑑𝑥 64 64
3𝑥 2
f(x) = { 64 0≤𝑤≤4
0 𝑒𝑙𝑠𝑒𝑤ℎ𝑒𝑟𝑒

Example 43
0 𝑥<0
2𝑥 − 2𝑥 2 0 ≤ 𝑥 ≤ 0.25
The continuous random variable X has a c.d.f F(X) = 𝑎 + 𝑥 0.25 ≤ 𝑥 ≤ 0.5
𝑏 + 2𝑥 2 − 𝑥 0.5 ≤ 𝑥 ≤ 0.75
{ 1 𝑥 ≥ 0.75
Find

(i) Value of constants a and b


For 0 ≤ 𝑥 ≤ 0.25, F(X) =2𝑥 − 2𝑥 2
F(0.25) =2𝑥0.25 − 2(0.25)2 = 0.375
For 0.25 ≤ 𝑥 ≤ 0.5; F(X) = a + x
F(0.25) = a + 0.25 = 0.375
a = 0.125
For 0.5 ≤ 𝑥 ≤ 0.75; F(X) = 𝑏 + 2𝑥 2 − 𝑥
F(0.75) = b + 2(0.75)2 -0.75 = 1; b = 0.625
(ii) Probability density function f(x)
𝑑
f(x) = 𝐹(𝑋)
𝑑𝑥
2 − 4𝑥 0 ≤ 𝑥 ≤ 0.25
1 0.25 ≤ 𝑥 ≤ 0.5
f(x) ={
4𝑥 − 1 0.5 ≤ 𝑥 ≤ 0.75
0 𝑒𝑙𝑠𝑒𝑤ℎ𝑒𝑟𝑒
(iii) Mean = 0.375

Revision exercise 6
1. The continuous random variable X has cumulative distribution function
0 𝑥<2
F(X) = = {0.25𝑥 − 0.5 2≤𝑥≤6
1 𝑥≥6
Find the
1
2≤𝑥≤6
(i) probability density function f(x); f(x)= { 4
0 𝑒𝑙𝑠𝑒𝑤ℎ𝑒𝑟𝑒
(ii) E(X) = 4 (iii) interquartile range = 2 (iv) sketch f(x)
2. The continuous random variable X has cumulative distribution function
0 𝑥<0
F(X) = = { 𝑥 3 0≤𝑥≤1
1 𝑥≥1
Find (i) median (m=0.794) (ii) mean (μ =0.75)
3. The continuous random variable X has cumulative distribution function
0 𝑥<0
F(X) = {𝑥 − 𝑘𝑥 2 0≤𝑥≤2
1 𝑥≥2
2
Find the (i) value of k= 0.25, (ii) median (m= 0.586) (iii) variance of x (Var(x) = )
9
1 − 0.5𝑥 0≤𝑥≤2
(iv) probability density function; f(x) ={
0 𝑒𝑙𝑠𝑒𝑤ℎ𝑒𝑟𝑒
4. The continuous random variable X has cumulative distribution function
0 𝑥<0
2𝑥
0≤𝑥≤1
F(X) = 𝑥3
+𝑘 1≤𝑥≤2
3
{1 𝑥≥2
1 5
Find (i) value of k = (ii) mean (μ = ) (iii) standard deviation =0.5528
3 6
(iv) P(|𝜇 − 𝜎| < 𝜎) =0.608
2
0≤𝑥≤2
3
(v) p.d.f; f(x) = { 1
1≤𝑥≤2 (vi) sketch f(x)
3
0 𝑒𝑙𝑠𝑒𝑤ℎ𝑒𝑟𝑒
5. The continuous random variable X has cumulative distribution function
0 𝑥<1
(𝑥−1)2
1≤𝑥≤3
12
F(X) = (14𝑥−𝑥 2 −25)
3≤𝑥≤7
24
{1 𝑥≥7
Find
1
(𝑥 − 1) 1≤𝑥≤3
6
(i) probability density function, f(x) = {1 (7 − 𝑥) 3≤𝑥≤7
12
0 𝑒𝑙𝑠𝑒𝑤ℎ𝑒𝑟𝑒
11 14
(ii) sketch f(x) (iii) mean of X (μ = ) (iv) Var (x) = (v) median of X (m= 3.45)
3 9

(vi) P(2.8 < x < 5.2) = 0.595

6. The continuous random variable X has cumulative distribution function


0 𝑥 < −1
𝑥+1
−1≤𝑥 ≤0
8
3𝑥+1
F(X) = 0≤𝑥≤2
8
𝑥+5
2≤𝑥≤3
8
{1 𝑥≥3
Find (i) probability density function, f(x) (ii) P(3≤ 2𝑥 ≤ 5) (iii) mean and variance
7. The continuous random variable X has cumulative distribution function
0 𝑥<0
𝛼𝑥 0≤𝑥≤1
F(X) = { 𝑥 + 𝛽 1≤𝑥≤1
3
1 𝑥≥2
2 1 5 19
Find (i) values of α and β (α= ; 𝛽 = ) (ii) mean (μ = ) (iii) Var(X) =
3 3 6 36
(iv) 𝑃 (𝑋 < 1.5⁄𝑋 > 1)= 0.4998 (v) probability density function, f(x) and sketch it
8. The continuous random variable X has cumulative distribution function
0 𝑥<1
𝑥 2 −1
−𝑥 1≤𝑥≤2
2
F(X) = 𝑥2
3𝑥 − 2≤𝑥≤3
2
{1 𝑥≥3
Find
(i) Probability density function, f(x) and sketch it
(ii) P(1.2 < x< 2.4) = 0.8
(iii) Mean (μ = 2)
9. The continuous random variable X has cumulative distribution function
0 𝑥<0
𝑘 2
𝑥 0≤𝑥≤2
F(X) = 2
𝑘(6𝑥 − 𝑥 2 − 6) 2≤𝑥≤3
{ 1 𝑥≥3
1
(a) Determine the value of k = . Hence sketch graph of F(X)
3
(b) Find the probability density function.
Uniform or rectangular distribution
A continuous random variable X is said to be uniformly distributed over the interval a and b, if the
1
𝑎≤𝑥≤𝑏
p.d.f is given by f(x) ={ 𝑏−𝑎
0 𝑒𝑙𝑠𝑒𝑤ℎ𝑒𝑟𝑒
Graph of f(x)
f(x)

1
𝑏−1

a b x

Example 44

X is uniformly distributed between 6 and 9.

(i) Write the probability density function


1
6≤𝑥≤9
f(x) ={ 9−6
0 𝑒𝑙𝑠𝑒𝑤ℎ𝑒𝑟𝑒
(ii) Find P(7.2 < x < 8.4)
8.4 1 1
P(7.2 < x < 8.4)=∫7.2 𝑑𝑥 = [𝑥 ]8.4
7.2 = 0.4
3 3

Example 45
𝜋
X is uniformly distributed between 0 and .
2

(i) Write the probability density function


1 𝜋
𝜋 0≤𝑥≤
f(x) ={ −0 2
2
0 𝑒𝑙𝑠𝑒𝑤ℎ𝑒𝑟𝑒
𝜋 𝜋
(ii) Find P( < x < )
3 2
𝜋 𝜋
𝜋 𝜋 2 2 2 1
(iii) P( < x < =∫ 𝜋 𝑑𝑥 = [𝑥 ]𝜋2 =
3 2 𝜋 𝜋 3
3 3

Expectation of X, E(x)
𝑏 𝑏 1 1 1 (𝑏+𝑎)(𝑏−𝑎) (𝑏+𝑎)
E(x)= ∫𝑎 𝑥𝑓(𝑥)𝑑𝑥 =∫𝑎 𝑥𝑑𝑥 = [𝑥 2 ]𝑏𝑎 = (𝑏 2 − 𝑎2 ) = =
𝑏−𝑎 2(𝑏−𝑎) 2(𝑏−𝑎) 2(𝑏−𝑎) 2

Variance of x, Var(X)
𝑏 𝑏 1 (𝑏+𝑎) 2 1 (𝑏+𝑎) 2
Var(x) =∫𝑎 𝑥 2 𝑓(𝑥)𝑑𝑥 − [𝐸(𝑥)]2 =∫𝑎 𝑥 2 𝑑𝑥 − [ ] = [𝑥 3 ]𝑏𝑎 − [ ]
𝑏−𝑎 2 3(𝑏−𝑎) 2

(𝑏−𝑎)(𝑏 2 +𝑎𝑏+𝑎2 ) (𝑏+𝑎) 2 (𝑏−𝑎)(𝑏 2 +𝑎𝑏+𝑎2 ) 𝑏 2 +2𝑎𝑏+𝑎2


= −[ ] = −
3(𝑏−𝑎) 2 3(𝑏−𝑎) 4

4𝑏 2 +4𝑎𝑏+4𝑎2 −3𝑏 2 −6𝑎𝑏−3𝑎2 𝑏 2 −2𝑎𝑏 + 𝑎2 (𝑏−𝑎)2


= = =
12 12 12
Example 45
𝜋 𝜋
X is a rectangular distribution between - 𝑎𝑛𝑑
2 2
1 𝜋 𝜋
𝜋 𝜋 − ≤𝑥≤
(i) Write the probability density function; f(x) = { −(− ) 2 2
2 2
0 𝑒𝑙𝑠𝑒𝑤ℎ𝑒𝑟𝑒
𝜋 𝜋
(𝑏+𝑎) ( +(− ))
2 2
(ii) Find the mean = = =0
2 2
𝜋 𝜋 2
(𝑏−𝑎)2 [ −(− )] 𝜋2
2 2
(iii) Find the variance of x = = =
12 12 12

Example 46

X is a rectangular distribution between over the interval −3 ≤ 𝑥 ≤ −1

Find
−1.5 1 1 1
(i) P(-2 ≤ X ≤ - 1.5) = ∫−2 𝑑𝑥 = (𝑥)−1.5
−2 =
2 2 4
(𝑏+𝑎) (−1+(−3))
(ii) Mean = = = −2
2 2
(𝑏−𝑎)2 (−1− −3)2 1
(iii) Var(x) = = =
12 12 3

Revision exercise 7
𝑘 3≤𝑥≤6
1. X follows a uniform distribution with probability density function f(x) ={
0 𝑒𝑙𝑠𝑒𝑤ℎ𝑒𝑟𝑒
1 1
Find (i) value of k = (ii) E(X) = 4.5 (iii) var(X) = 0.75 (iv) P(X>5) =
3 3
2. X is distributed uniformly over -5 ≤ x ≤ -2
Find (i) P(-4.3 ≤ X≤ -2.8) = 0.5 (ii) E(X) = -2.5 (iii) standard deviation = 0.865
1
1≤𝑥≤𝑘
3. The continuous random variable has a probability density function f(x) ={ 4
0 𝑒𝑙𝑠𝑒𝑤ℎ𝑒𝑟𝑒
1
Find (i) value of k = 5 (ii) P(2.1 ≤ X≤ 3.4) =0.325 (iii) E(X) = 3 (jv) Var (X) = 1
3
1
32 ≤ 𝑥 ≤ 37
4. The continuous random variable has a probability density function f(x) ={ 5
0 𝑒𝑙𝑠𝑒𝑤ℎ𝑒𝑟𝑒
Find the probability that y lies within one standard deviation of the mean= 0.577
5. The continuous random variable X has cumulative distribution function
0 𝑥<2
𝑥−2
𝐹(𝑋) = { 2≤𝑥≤7
5
1 𝑥≥7
1
Find (i) E(X) = 4.5 (ii) Var(X) = 2
12
6. The continuous random variable X is uniformly distributed in the interval a ≤ x ≤ b. the lower
quartile is 5 and the upper quartile is 9. Find
(i) Values of a and b (a= 3, b = 11)
(ii) P(6 ≤ X ≤ 7) = 0.125
0 𝑥<3
𝑥−3
(iii) Cumulative distribution function; 𝐹(𝑋) = { 3 ≤ 𝑥 ≤ 11
8
1 𝑥 ≥ 11
7. The number of patients visiting a certain hospital is uniformly distributed between 150 and 210
(i) Write down the probability density function of the number of patients
1
150 ≤ 𝑥210
f(x) = {210−150
0 𝑒𝑙𝑠𝑒𝑤ℎ𝑒𝑟𝑒
(ii) Find P(170< x< 194) = 0.4

Thank you
Dr. Bbosa Science
Trapezium rule
It is used for estimating an integral area under a curve of continuous function over a given interval
[a, b]

if y = f(x)
𝑏
Using several strips between x = a and x = b of equal width,
A = ∫𝑎 𝑦𝑑𝑥 trapezium rule can be used to determine the area.
1
A ≈ ℎ[(𝑓𝑖𝑟𝑠𝑡 + 𝑙𝑎𝑠𝑡 𝑜𝑟𝑑𝑖𝑛𝑎𝑡𝑒𝑠) + 2(𝑠𝑢𝑚 𝑜𝑓 𝑡ℎ𝑒 𝑚𝑖𝑑𝑑𝑙𝑒 𝑜𝑟𝑑𝑖𝑛𝑎𝑡𝑒𝑠)]
2
𝑏−𝑎
where h =
𝑠𝑢𝑏𝑖𝑛𝑡𝑒𝑟𝑣𝑎𝑙𝑠

Note

(i) sub-intervals, subdivision and strips are the same


(ii) subinterval = ordinates
(iii) when dealing with a trigonometric function, calculators must be in radian mode
(iv) when the final answer is required to a specific number of d.p’c, the working’s should be
done at least a d.p higher but the final answer rounded to the required d.p’s

Example 1
1.0 2𝑥+1
Use the trapezium rule with four-intervals to estimate∫0.2 ( ) 𝑑𝑥 . Correct to two decimal places.
𝑥 2 +𝑥

2𝑥+1
Let y = ( )
𝑥 2 +𝑥
1.0−0.2
h= = 0.2
4
x 2𝑥+1 1.0 2𝑥+1 1
y= ∫0.2 (𝑥 2 +𝑥) 𝑑𝑥 = 𝑥 0.2(7.3333 + 7.3116 )
𝑥 2 +𝑥 2
0.2 5.8333 = 2.1955
0.4 3.2143 = 2.20 (2D)
0.6 2.2917
0.8 1.8056
1.0 1.5000
Sum 7.3333 7.3116

Example 2

Use the trapezium rule with seven coordinates to estimate


1
3
∫0 [(1.2)𝑥 − 1]2 dx correct to 2 decimal places (05marks)
Solution
For 7 ordinates, there are 6 subintervals
𝑏− 𝑎 3−0
Width, h = = = 0.5
𝑠𝑢𝑏𝑖𝑛𝑡𝑒𝑟𝑣𝑎𝑙 6
Let y = √(1.2)𝑥 − 1
x y
0 0
0.5 0.309
1 0.447
1.5 0.561
2 0.663
2.5 0.760
3 0.853
Sum 0.853 2.74

Using the trapezium rule


1
3 0.5
∫0 [(1.2)𝑥 − 1]2 dx = 2
[0.853 + 2(2.74)] = 1.58

Example 3
1
(a) Use the trapezium rule with 6-ordinated to estimate the value of∫02(𝑥 + 𝑠𝑖𝑛𝑥)𝑑𝑥 , correct to
three decimal places.
1
−0 𝜋
2
ℎ= =
5 10
x y
0 0
𝜋 0.6232
10
2𝜋 1.2161
10
3𝜋 1.7515
10
4𝜋 2.2077
10
𝜋 2.5708
2
Sum 2.5708 5.7985
1
1 𝜋
∫0 (𝑥 + 𝑠𝑖𝑛𝑥)𝑑𝑥 =
2
2
𝑥
10
(2.5708 + 2 𝑥 5.7985)

= 2.225
1
(b)(i) Evaluate∫02(𝑥 + 𝑠𝑖𝑛𝑥)𝑑𝑥 , correct to three decimal places
1
1
𝑥2 2
∫0 (𝑥 + 𝑠𝑖𝑛𝑥)𝑑𝑥 = | 2 − cos 𝑥|
2
0

1 𝜋2 𝜋
= ( − 0) − (cos − cos 0)
2 4 2

𝜋2
= +1
8

= 2.234
(ii) Calculate the error in your estimation in (a) above
Error = |2.234 − 1.225| = 0.009
(iii) Suggest how the error may be reduced (06marks)
Increasing on number of strips or subintervals

Example 4

A student used the trapezium rule with five sub-intervals to estimate


3 𝑥
∫2 (𝑥 2 −3)
𝑑𝑥 correct to three decimal places

Determine;

(a) The value the student obtained (06marks)


3−2
h= =0.2
5
X y1, y6 y2 ……y5
2.0 2.0
2.2 1.1956
2.4 0.8696
2.6 0.6915
2.8 0.5785
3 0.5
Sum 2.5 3.3352
3 𝑥 1
∫2 (𝑥 2−3) 𝑑𝑥 =
2
𝑥 0.2[2.5 + 2(3.3352)]

= 0.91704 = 0.917 (3D)

(b) The actual value of the integral (03marks)


3 𝑥 1 3
∫2 (𝑥 2 −3)
𝑑𝑥 = [ 𝐼𝑛𝑥 2 − 3]
2 2
1
= (𝐼𝑛 6 − 𝐼𝑛 1)
2
= 0.896
(c) (i) the error the student made in the estimate
Error = |0.896 − 0.917| = 0.021
(ii) how the student can reduce the error(03marks)
Increasing on the number of sub-intervals or ordinates or reducing the width of h

Example 5
𝜋
Use trapezium rule with 4 subintervals to estimate to 3 decimal places ∫02 cos 𝑥𝑑𝑥
Solution
𝜋
−0 𝜋
2
h= =
4 8

x f(x) = cos x
0 1.0000
𝜋
0.9239
8
2𝜋 0.7071 𝜋
1 𝜋
8 ∫02 cos 𝑥𝑑𝑥 = 2 𝑥 [1 + 2 𝑥 2.0137]
3𝜋 0.3827 8
8
4𝜋 0.0000 =0.987
8
sum 1.0000 2.0137

Example 6
3 1
Use trapezium rule with 7 ordinates to estimate ∫0 𝑑𝑥 correct to 3dp
1+𝑥

Solution
3−0
h= = 0.5
7−1

x f(x) = cos x
0 1.0000
0.5 0.6667
1.0 0.5000 3 1 1
∫0 𝑑𝑥 = 𝑥 0.5[1.25 + 2 𝑥 2.1857]
1.5 0.4000 1+𝑥 2
2.0 0.3333 = 1.405
2.5 0.2757
3.0 0.2500
sum 1.2500 2.157

Example 7
3 𝑥
(a) Use the trapezium rule to estimate the integral value of ∫2 𝑑𝑥 using five subinterval and
1+𝑥 2
correct to 3d.p.
3 𝑥
(b) (i) find the exact value of ∫2 𝑑𝑥
1+𝑥 2
(ii) suggest how the error may be reduced.
3−2
(a) h = = 0.2
5
𝑥
x f(x) =
1+𝑥 2
2.0 0.40000
2.2 0.37671
2.4 0.35503 3 𝑥 1
∫2 𝑑𝑥 = 𝑥 0.2[0.7 + 2 𝑥 1.38353]
1+𝑥 2 2
2.6 0.33505
2.8 0.31674 = 0.3467
3.0 0.30000
sum 0.70000 1.3353
3 𝑥 1 3 1
(b)(i) ∫2 𝑑𝑥 = [ 𝐼𝑛(1 + 𝑥 2 )] = (𝐼𝑛10 𝐼𝑛5) = 0.3466
1+𝑥 2 2 2 2

(ii) error = |𝑒𝑥𝑎𝑐𝑡 𝑣𝑎𝑙𝑢𝑒 – 𝑎𝑝𝑝𝑟𝑜𝑥𝑖𝑚𝑎𝑡𝑒 𝑣𝑎𝑙𝑢𝑒| = |0.3466 − 0.3467| = 0.0001

(iii) the error can be reduced by reducing h or increasing the number of sub-intervals.

Example 8
1
(a) Use trapezium rule to estimate the integral value of ∫0 𝑥 2 𝑒 𝑥 𝑑𝑥
1
(b) (i) find exact value of ∫0 𝑥 2 𝑒 𝑥 𝑑𝑥
(ii) determine the percentage error in your estimation

1−0
(a) h = = 0.2
5

x f(x) = 𝑥 2 𝑒 𝑥 1
(b)(i) ∫0 𝑥 2 𝑒 𝑥 𝑑𝑥 = [𝑥 2 𝑒 𝑥 − 2𝑥𝑒 𝑥 + 2𝑒 𝑥 ]10
0 0
0.2 0.0489 = 0.718
0.4 0.2387
0.6. 0.6560 (ii) error = |0.718 − 0.745|= 0.027
0.8 1.4243 Percentage error =
𝑒𝑟𝑟𝑜𝑟
𝑥 100%
1.0 2.7183 𝑒𝑥𝑎𝑐𝑡 𝑣𝑎𝑙𝑢𝑒

sum 2.7183 2.3679 =


0.027
𝑥 100 = 3.8%
1 1 0.718
∫0 𝑥 2 𝑒 𝑥 𝑑𝑥 = 𝑥 0.2[2.713 + 2 𝑥 2.3679]
2

=0.74541≈ 0.745

Revision Exercise
𝜋
1. (a) Use trapezium rule with six strips to estimate ∫0 𝑥𝑠𝑖𝑛𝑥𝑑𝑥 [3.069]
(b) Determine the percentage error in your determination. [2.3%]
1 1
2. Use the trapezium rule to estimate the approximate value of ∫0 using 6 ordinates and
1+ 𝑥 2 𝑑𝑥
correct to 3 decimal places. [0.784]
4 10
3. (a) Use trapezium rule with six strips to estimate ∫2 𝑑𝑥 correct 4dp. [2.9418]
2𝑥+1
(b) Determine the percentage error in your estimation and suggest how this error may be
reduce. [0.098%]
4. (a) Use trapezium rule to estimate the area of y = 3x between x-axis, x = 1 and x = 2, using five
subintervals. Give your answer correct to four significant figures. [5.483]
2
(b) Find the exact value of ∫1 3𝑥 𝑑𝑥 [5.461]
(c) Find the exact percentage error in calculations (a) and (b) above. [0.4028%]
3 1
5. Use trapezium rule with 7 ordinates to estimate ∫0 𝑑𝑥 , correct to 3 decimal places [1.405]
1+𝑥
1 2
6. Use the trapezium rule with 6 ordinates to evaluate∫0 𝑒 −𝑥 correct to 2 decimal place. [ 0.74]
2 𝐼𝑛𝑥
7. Use the trapezium rule with 6 ordinates to estimate ∫1 𝑑𝑥.Give your answer correct to 3
𝑥
decimal places [0.237]
1 𝑑𝑥
8. Find the approximate value to one decimal place of ∫0 ’ using the trapezium rule with five
1+𝑥
strips. [ 0.7]
𝜋
9. (a) Use trapezium rule with five subintervals to estimate ∫03 tan 𝑥 𝑑𝑥 correct to 3dp. [0.704]
𝜋
(b) (i) Find the exact value of ∫03 tan 𝑥 𝑑𝑥 to 3 d.p. [0.693]
(ii) Calculate the percentage error in your estimation in (a) above [1.587%]
(iii) Suggest how the percentage error in (b)(ii) may be reduced.

𝜋
1
10. Use the trapezium rule with four subdivisions to estimate∫02 𝑑𝑥 . Give your answer correct
1+𝑠𝑖𝑛𝑥
to three decimal places. [1.013]
2 1
11. Find the approximate value of ∫0 𝑑𝑥 using trapezium rule with 6 ordinates. Give your
1+ 𝑥 2
answer to 3 decimal places (05marks)[1.105]
12. Use the trapezium rule with five subintervals to estimate
4 5
∫2 𝑑𝑥. Give your answer correct to 3 decimal places (05marks)[2.559]
(𝑥+1)
13. A student used the trapezium rule with five sub-intervals to estimate
3 𝑥
∫2 (𝑥 2 −3)
𝑑𝑥 correct to three decimal places

Determine;

(a) The value the student obtained (06marks) [0.917]


(b) The actual value of the integral (03marks) [0.896]
(c) (i) the error the student made in the estimate [0.021]
(ii) how the student can reduce the error(03marks)
1
14. (a) Use the trapezium rule with 6-ordinated to estimate the value of ∫02(𝑥 + 𝑠𝑖𝑛𝑥)𝑑𝑥 , correct to
three decimal places, [2.225]
1
(b)(i) Evaluate∫02(𝑥 + 𝑠𝑖𝑛𝑥)𝑑𝑥 , correct to three decimal places [2.234]
(ii) Calculate the error in your estimation in (a) above [0.009]
(iii) suggest how the error may be reduced (06marks)

Thank you
Dr. Bbosa Science
Location of real roots
The range where the root of an equation lie can be located using the following methods

(i) sign change


(ii) Graphical method
(a) Sign change method

Example 1

Show that equation x3 + 6x2 + 9x +2 = 0 has a root between -1 and 0

Solution

f(x) = x3 + 6x2 + 9x +2

f(-1) = (1)3 + 6(1)2 + 9(-1) +2 = -14

f(0) = (0)3 + 6(0)2 + 9(0) +2 = 2

Since there is a sign change the root lies between 0 and -1.

Example 2

Show that the equation 𝑒 2𝑥 sin 𝑥 − 1 = 0 has a root between 0 and 1

Solution

Note that in trigonometric function the calculator must be in radian mode

f(x) =𝑒 2𝑥 sin 𝑥 − 1

f(0) =𝑒 2(0) sin 0 − 1 = -1

f(1) =𝑒 2 sin 1 − 1 = 5.2177

Since there is a sign change the root lies between 0 and 1.

(b) Using graphical method


One or two graph(s) can be drawn to locate the root.

(i) Single graph method


When one graph is drawn then the root lies between the two points where the curve crosses
the xaxis.
Example 3

Using a suitable graph locate the interval over which the root of the equation 3x2 + x – 4 = 0 lie.

x -3 -2 -1 0 1 2 3
f(x) 20 6 -2 -4 0 10 26

The root lies between -1 and 1

Example 4

Show graphically that there is positive real root of equation x3 – 5x + 1 = 0

x -3 -2 -1 0 1 2 3
f(x) -11 3 5 1 -3 -1 13

(ii) Double graph method

When two graphs are drawn, the root lies between the points where the two curves meet.

Note

(i) Both curves must have a consistent scale and should be labelled.
(ii) A line must be drawn using a ruler while a curve must be drawn using a freehand
(iii) Both graphs must be labelled
(iv) The initial approximation of the root must be located and indicated in the graph
Example 5

By plotting graph of y = 𝑒 𝑥 and y = 4 - x on the same axes, show the root of the equation 𝑒 𝑥 +x - 4 = 0
lie between 1 and 2

x 1 1.2 14 1.6 1.8 2.0


y = 𝑒𝑥 2.7 3.3 4.1 5.0 6.0 7.4
y=4-x 3.0 2.8 2.6 2.4 2.2 2

Therefore the root(1.07) lies between 1 and 2.

Example 6

Show that the equation Inx + x – 2 = 0 has a real root between x = 1 and x = 2

x 1 1.2 1.4 1.6 1.8 2.0


y = Inx 0 0.1823 0.3365 0.4700 0.5878 0.6731
y = 2-x 1 0.8 0.6 0.4 0.2 0

Therefore the root lies between x =1 and x =2


Example 7

By plotting graphs y = ex -2 an y = x sin x on the same axis show that the root of the equation
ex – 2 –xsinx = 0 lies between x = 0.5 and x = 1.5

x 0.5 0.75 1.00 1.25 1.5


y = xsinx 0.240 0.511 0.841 1.186 1.496
y = ex-2 -0.351 0.117 0.718 1.490 2.481

Example 8

Show graphically the equation x + logx = 0.5 has only one real root that lie between 0.5 and 1.

Solution
let y = x + logx - 0.5
x 0.25 0.5 0.75 1.00 1.25
y -.852 -0.301 0.125 0.5 0.847

Therefore the root (0.66) lies between 0.5 and 1


Revision exercise 1
1. By sketching graphs of y = 2x and y = tanx show that the equation 2x = tanx has only one
root between x = 1.1 and 1.2. Use linear interpolation to find the value of the root correct to
2dp.
𝑥
2. Given the equation y = sinx - , show by plotting two suitable graphs on the same axis that
3
2𝜋 5𝜋
positive root lies between and .
3 6
3. Show graphically that the positive real root of the equation 2x2 + 3x -3 = 0 lies between 0
and 1 [0.7]
4. Use a graphical method to show that the equation ex –x - 2 = 0 has only one real root
between 2 and -1 by drawing two graphs y = ex and y = x + 2 [-1.8]
5. On the same axes, draw graphs of y = 3 – 3x and y = 2x2 to show that the root of the
equation 2x2 + 3x 3 = 0 lies between -3 and -2 [-2.2]
6. Show graphically that the positive real root of the equation x3 -3x – 1 = 0, lies between 1 and
2 [1.6]
7. on the same axes, draw graph y = 3x -1 and y = x3 to show that the root of the equation
X3 – 3x -1 = 0lies between 0 and 1.[0.35]
8. Using suitable graphs and plotting them on the same axes. Find the root of the equation
e2xsinx -1 = 0, in the interval x = 0.1 and x = 0.8. [0.44]
9. Show graphically that equation 𝑒 −𝑥 = 𝑥 has only one real root between 0.5 and 1. [0.56]
10. Show graphically that equation ex = -2x + 2 has only one real root between 0 and 1.0.
11. on the same axes, draw graphs of y = 9x -4 and y = x3 show that the root of equation
x3-9x + 4 = 0 lie between 2.5 and 3
12. Show that the positive real root of equation 4 + 5x2- x3 = 0 lies between 5 and 6.
13. On the same axes, draw graphs of y = x +1 and y = tanx to show that the equation
tanx - x -1 = 0 lie between 1 and 1.5.
14. Using suitable graphs and plotting them on the same axes, find the roots of the equation 5ex
= 4x + 6 in the interval x = 2 and x = -1.
15. On the same axes, draw graphs of y = 2x + 1 and y = log 𝑒 (𝑥 + 2) to show that the root of
equation log 𝑒 (𝑥 + 2) − 2𝑥 − 1 = 0 lies between 1 and 0.
16. Using suitable graphs and plotting them on the same axes, find the real root of the equation
9log10 𝑥 = 2(X -1) in the interval x = 3 and x =4.

Method of solving for roots


The following methods can be used

(a) Interpolation

Example 9

Show that the equation x4 – 12x2 + 12 = 0 has root between 1 and 2. Hence use linear interpolation
to get the first approximation of the root.

Solution

f(x) = x4 – 12x2 + 12
f(1) = 14 – 12(1)2 + 12 = 1 x 1 x0 2
f(x) 1 0 -20
f(2) = 24 – 12(2)2 + 12 = -20 𝑥0 −1 2−1
=
0−1 −20−1
Since there is a sign change,
x0 = 1.05
then the root lies between

1 and 2.

Example 10
𝑥
Show that the equation 2x – 3cos ( ) = 0 has a root between 1 and 2.Hence use linear interpolation
2
twice to get the approximation of the root.

solution

Note: for trigonometric functions the x 1 x0 2


f(x) -0.633 0 2.379
calculator must be strictly in radian mode 𝑥0 −1
=
2−1
0− −0.633 2.379−−0.633
𝑥
f(x) =2x – 3cos ( ) x0 = 1.2102
2

1
f(1) = 2 x 1 -3cos ( ) = -0.633 x 1.2102 x0 2
2
f(x) -0.047 0 2.379
2 𝑥0 −1 2−1
f(2) = 2 x 1 -3cos ( ) = 2.379 =
2 0− −0.047 2.379−−0.047

Since there is a sign change, x0 = 1.226

then the root lies between

1 and 2

Example 11

Show that the equation 3x2 + x – 5 =0 has a real root between x = 1 and x = 2. Hence use linear
interpolation twice to calculate the root to 2 dp.

Solution
x0 = 1.1
f(x) = 3x2 + x – 5
x 1.1 x0 2
f(1) = 3(1)2 + 1 – 5 = 1
f(x) -0.27 0 9
𝑥0 −1.1 2−1.1
f(2) =3(2)2 + 2 – 5 = 9 =
0− −0.27 9−−0.27

Since there is a sign change, x0 = 1.13


then the root lies between

x 1 x0 2
f(x) -1 0 9
𝑥0 −1 2−1
=
0− −1 9−−1
(b) General iterative method

This involves generating equation by splitting the original equation into several equations by
making x the subject.

Example 12

Given x2 + 4x – 2 = 0. Find the possible equations for estimating the roots

Solution

Let xn+1 be a better approximation

xn be the next approximation


2 2−𝑥 2
𝑥𝑛+1 = − 4 𝑥𝑛+1 = √(2 − 4𝑥𝑛 ) 𝑥𝑛+1 =
𝑥𝑛 4

Example 13

Given f(x) = x3 – 3x – 12 = 0. Generate equations in form of 𝑥𝑛+1 = 𝑔(𝑥𝑛 ) that can be used to
solve the equation f(x) =0

Solution

Let xn+1 be a better approximation

xn be the next approximation


3 −12
𝑥𝑛 2 12
𝑥𝑛+1 = 𝑥𝑛+1 = √(3𝑥𝑛 + 12) = 2 12 3𝑥𝑛 +12
3 𝑥 −3 𝑛 𝑥𝑛+1 = √(3 + ) = 2
𝑥 𝑛 𝑥𝑛

Testing for convergence


From the several iterative equations obtained, the equation whose |𝑓 1 (𝑥𝑛 | < 1 is the one which
converges the correct root.

Example 14

Given the two iterative formulas


3 −1
𝑥𝑛 1
(i) 𝑥𝑛+1 = (ii) 𝑥𝑛+1 = √(5 + )
5 𝑥𝑛

Using x0 = 2 deduce a more suitable formula for solving the equation. Hence find the root correct to
2dp
3 −1
𝑥𝑛
𝑥𝑛+1 =
5
3 −1
𝑥𝑛 2
3𝑥𝑛
f(𝑥𝑛 ) = 𝑥𝑛+1 = ; 𝑓 1 (𝑥𝑛 ) =
5 5

3(2)2
𝑓 1 (2) = = 2.4
5

since |𝑓 1 (2)|>1 it will not converge


1
𝑥𝑛+1 = √(5 + )
𝑥𝑛

1 1 1
f(xn) =√(5 + ); 𝑓 1 (𝑥𝑛 ) = − 𝑥𝑛 −2 (5 + )
𝑥𝑛 2 𝑥𝑛

1 1
𝑓 1 (2) = − (2)−2 (5 + ) = -0.0533
2 2

since |𝑓 1 (2)|<1 it will converge so this equation gives the root

1
𝑥𝑛+1 = √(5 + ) , |𝑒|= 0.005, x0 = 2
𝑥𝑛

1
x1 = √(5 + ) = 2.3452
2

|𝑥1 − 𝑥0 | = 2.3452 – 2 = 0.3452 > 0.005

1
x2 = √(5 + ) = 2.3295
2.3452

|𝑥2 − 𝑥1 | = 2.3452 – 2.3295 = 0.0157 > 0.005

1
x3 = √(5 + ) = 2.3301
2.3295

|𝑒|=|2.3301 − 2.3295|= 0.0006 < 0.005

Hence root is 2.33

Example 15

1
Show that the iterative formula for solving the equation x3 = x + 1 is 𝑥𝑛+1 = √(1 + ) starting with
𝑥𝑛
x0 = 1 find the solution of the equations to 3sf.

Solution

1
𝑥𝑛+1 = √(1 + ) |𝑒|= 0.005, x0 = 1
𝑥𝑛

1
𝑥1 = √(1 + ) =1.41421
1

|𝑥1 − 𝑥0 | = |1.41421 – 1| = 0.41421 > 0.005

1
𝑥2 = √(1 + ) =1.30656;
1.41421

|𝑥2 − 𝑥1 | = |1.30656 − 1.41421| = 0.10765 > 0.005

1
𝑥3 = √(1 + ) = 1.32869
1.30656

|𝑥3 − 𝑥2 | = |1.32869 − 1.30656| =0.03691 > 0.005

1
𝑥4 = √(1 + ) =1.32389
1.32869
|𝑒|=|1.32389 − 1.32869|= 0.0048 < 0.005

Hence the root is 1.32

Example 16

Given two iterative formulae I and II (shown below) for calculating the positive root of the quadratic
equation f(x) = 0
1 2 +1
1 𝑥𝑛
𝑥𝑛+1 = (𝑥𝑛2 − 1) and 𝑥𝑛+1 = ( ) for n = 1, 2, 3 ………….
2 2 𝑥𝑛 −1

Taking x0 = 2.5, use each formula thrice to two decimal places to decide which is the more suitable
formula. Give a reason for your answer.

Solution
2 +1
1 𝑥𝑛
1 Iterative formula 𝑥𝑛+1 = ( )
Iterative formula 𝑥𝑛+1 = (𝑥𝑛2 − 1) 2 𝑥𝑛 −1
2
x0 = 2.5
x0 = 2.5
1 2.52 +1
1
(2.52 𝑥1 = ( ) = 2.416666667
𝑥1 = − 1) = 2.625 2 2.5−1
2

|𝑥1 − 𝑥0 | = 0.125 |𝑥1 − 𝑥0 | = 0.083333

1 1 2.4166666672 +1
𝑥2 = (2.6252 − 1) 2.99453125 𝑥2 = ( ) = 2.414215686
2 2 2.416666667−1

|𝑥2 − 𝑥1 | = 0.3200125 |𝑥2 − 𝑥1 | = 0.002450781

1 1 2.4142156862 +1
𝑥3 = (2.994531252 − 1) = 3.837432861 𝑥2 =
2
(
2.414215686−1
) = 2.414215686
2

|𝑥3 − 𝑥2 | = 0.89212036 |𝑥3 − 𝑥2 | = 0.000002124

2 +1
1 𝑥𝑛
The more suitable formula is 𝑥𝑛+1 = ( ).
2 𝑥𝑛 −1

Because the absolute difference between 𝑥3 − 𝑥2 is less than absolute error, where as in the first
formula the absolute difference between𝑥3 − 𝑥2 is greater than absolute error. In all the 2nd
formula converge whereas the first formula diverges.

Example 17

(a) (i) Show that the equation ex – 2x – 1 = 0 has a root between x = 1 and x = 1.5.
(ii) Use linear interpolation to obtain an approximation for the root
(b) (i) Solve the equation in (a)(i), using each formula below twice
Take the approximation in (a)(i) as the initial value
1
Formula I: 𝑥𝑛+1 = (𝑒 𝑥𝑛 + 1).
2
𝑒 𝑥𝑛 (𝑥𝑛 −1) + 1
Formula II: 𝑥𝑛+1 =
𝑒 𝑥𝑛 −2
(ii) Deduce with a reason which of the two formulae is appropriate for solving the given
equation in (a)(i). Hence write down a better approximate root, correct to two decimal
places
Solution

(a) (i) using sign change method Since f(1).f(1.5)< 0, the root lies
let f(x) = ex – 2x – 1 between x = 1 and x = 1.5
f(1) = e1 – 2(1) – 1 = -2.817
f(1.5) = e1.5 – 2(1.5) – 1 = 0.4817

(a)(ii) Extract

1 x0 1.5
-0.2817 0 0.4817
𝑥0 −1 1.5−1
= ; x0 = 1.1845
0−−0.2817 0.4817−−0.2817

Hence the approximation to the root is 1.18 (2 dp)

(b)(i)

Solution
𝑒 𝑥𝑛 (𝑥𝑛 −1) + 1
1 formula 2: 𝑥𝑛+1 =
(𝑒 𝑥𝑛 𝑒 𝑥𝑛 −2
formula 1: 𝑥𝑛+1 = + 1)
2
x0 = 1.18
x0 = 1.18
𝑒 1.18 (1.18−1) + 1
1 𝑥1 = = 1.2642
𝑥1 = (𝑒 1.18 + 1)= 2.1272 𝑒 1.18 −2
2
|𝑥1 − 𝑥0 | = 0.0842
|𝑥1 − 𝑥0 | = 0.9472
𝑒 1.2642 (1.2642−1) + 1
1
(𝑒 2.127187 + 1)= 4.6956 𝑥2 = = = 1.2565
𝑥2 = 𝑒 1.2642 −2
2
|𝑥2 − 𝑥1 | = 0.0077
|𝑥2 − 𝑥1 | = 2.5684

Formula 1, the sequence 1.18, 2.1272, 4.6956 diverge, hence the formula is not suitable

Formula 2, the sequence 1.18, 1.2642, 1.2565 converge, hence the formula is suitable solving the
equation

A better approximation = 1.26 (2 dp)

Revision exercise 2
1. Given the following iterative formula
3 1
(i) 𝑥𝑛+1 = 5 − (ii) 𝑥𝑛+1 = (𝑥𝑛2 + 3)
𝑥𝑛 5

Taking x0 = 5 deduce a more suitable iterative formula for solving the equation

2. Show that the iterative formula for solving the equation x2 -5x + 2 = 0 can be written in two
2 2 +2
𝑥𝑛
ways as 𝑥𝑛+1 = 5 − or 𝑥𝑛+1 = .
𝑥𝑛 5
Starting with x0 = 4, deduce the more suitable formula for the equation and hence find the root
correct to 2 dp [4.56]
1
3. Show that the iterative formula for solving the equation x3 – x – 1 = 0 is 𝑥𝑛+1 = √(1 + ).
𝑥𝑛
Starting with xo = 1 find the root of the equation correct to 3 s.f. [1.33]
2 +3
2𝑥𝑛
4. (a) Show that the iterative formula for solving the equation 2x2 – 6x - 3 = 0 is 𝑥𝑛+1 =
4𝑥𝑛 +6
2
(b) Show that the positive root for 2x – 6x - 3 = 0 lies between 3 and 4. find the root correct to
2 decimal places [3.44]
5. (a) If b is the first approximation to the root of equation x2 = a, show that the second
𝑎
𝑏+
𝑏
approximation to the root is given by . Hence taking b = 4, estimate √17 correct to 3 dp
2
[4.123]
(b) Show that the positive real root of the equation x2 – 17 = 0 lies between 1.5 and 1.8. Hence
use the formula in (a) above to determine the root to 3 dp

(c) Newton Raphson’s Method


𝑓(𝑥𝑛 )
It is given by 𝑥𝑛+1 = 𝑥𝑛 − [ ] n = 1, 2, 3 …
𝑓 1 (𝑥𝑛 )

Example 18

Use Newton Raphson’s method to find the root of equation x3 + x – 1 = 0 using x0 = 0.5 as the initial
approximation, correct your answer to 2 decimal places

Solution

f(x) = x3 + x – 1, f1(x) = 2x2 + 1 |𝑥2 − 𝑥1 | = |0.6831 − 0.7142|


(𝑥 3 +𝑥−1) = 0.0311 > 0.005
𝑥𝑛+1 = 𝑥𝑛 − [ ]
3𝑥 2 +1
((0.6831)3 + 0.6831−1)
((0.5)3 + 0.5−1) 𝑥2 = 0.6831 − [ ] =0.6824
𝑥1 = 0.5 − [ ] =0.7142 3(0.6831)2 +1
3(0.5)2 +1
|3 − 𝑥2 | = |0.6824 − 0.6831|
|𝑥1 − 𝑥0 | = 0.7142 − 0.5 = 0.2142 > 0.005
= 0.0007 < 0.005
((0.7142)3 + 0.7142−1)
𝑥2 = 0.7142 − [ ] =0.6831
3(0.7142)2 +1 ∴ Root = 0.68

Example 19

Show that the equation 5x - 3cos2x = 0 has a root between 0 and 1. Hence use Newton Raphson’s
method to find the root of equation correct to 2 decimal places using x0 = 0.5 .

Solution
f(x) =5x - 3cos2x
Using sign change method to locate
f(0) = 5(0) – 3cos2(0) = -3
The roots. Note for trigonometric
f(1) = 5(1) – 3cos2(1) = 2.455
functions the calculator is used
Since there is change sign the root lies between x = 0
in radians mode
and x = 1
f(x) = 5x - 3cos2x, f1(x) = 5 + 6sin2x
(5x𝑛 − 3cos2x𝑛 )
𝑥𝑛+1 = 𝑥𝑛 − [ ]
5 + 6sin2x𝑛

x0 = 0.5, |𝑒| = 0.005


(5(0.5) − 3cos2(0,5)
𝑥1 = 0.5 − [ ] = 0.4125
5 + 6sin2(0.5)

|𝑥1 − 𝑥0 | = |0.4125 − 0.5| = 0.0875 > 0.005


(5(0.4125) − 3cos2(0.4125)
𝑥1 = 0.4125 − [ ] =0.4096
5 + 6sin2(0.4125)

|𝑥2 − 𝑥1 | = |0.4096 − 0.4125| = 0.0029 < 0.005

∴ Root = 0.41

Example 20

Use Newton Raphson’s iterative formula to show that the cube root of a number N is given by
1 𝑁 3
(2𝑥𝑛 + 2 ). Hence taking x0 = 2.5 determine √10 correct to 3 dp.
3 𝑥𝑛

Solution
1
x = 𝑁3 1 𝑁
𝑥1 = (2(2.5) + ) = 2.2
3 2.52
3
x –N=0
|𝑥1 − 𝑥0 | = |2.2 − 2.5| = 0.3 > 0.005
f(x) = x3 – N; f1(x) = 3x2
1 𝑁
𝑥2 = (2(2.2) + ) = 2.1554
(x𝑛 3 – N) 𝑥𝑛 (3x𝑛 2 )− (x
𝑛
3 – N) 3 2.22
𝑥𝑛+1 = 𝑥𝑛 − [ ]=
3x𝑛 2 3x𝑛 2
|𝑥2 − 𝑥1 | = |2.1554 − 2.2| = 0.0446 > 0.005
2 x𝑛 3+ N 1 𝑁
= = (2𝑥𝑛 + ). 1
𝑥3 = (2(2.1554 ) +
𝑁
) = 2.1544
3x𝑛 2 3 2
𝑥𝑛
3 2.1554 2
x0 = 2.5, N = 10, |𝑒| = 0.005 |𝑥3 − 𝑥2 | = |2.1544 − 2.1554| = 0.001 < 0.005
1 𝑁
𝑥𝑛+1 = (2𝑥𝑛 + 2 ) ∴ Root = 2.154
3 𝑥𝑛

Example 21

(a) Show that the equation x -3sinx = 0 has a root between 2 and 3. (03marks)

f(x) = x – 3sinx
f(2) = 2 – 3sin 2 = -0.7279
f(3) = 3 – 3sin3 = 2.5766
since f(2).f(3) = -1.8755<0
there exist a root of x-3sinx =0 between 2 and 3

(b) Show that Newton- Raphson iterative formula for estimating the root of the equation in (a) is
given by
3(sin 𝑥𝑛 − 𝑥𝑛 cos 𝑥𝑛 )
𝑋𝑛+1 = , 𝑛 = 0, 1, 2 ….
1 − 3 cos 𝑥𝑛
Hence find the root of the equation corrected to 2 decimal places (09 marks)
f'(x) = 1 – 3cos x
𝑓(𝑥)
xn+1 = xn -
𝑓′(𝑥)
𝑥𝑛 −3 sin 𝑥𝑛
= xn -
1−3 cos 𝑥𝑛
𝑥𝑛 −3𝑥𝑛 cos 𝑥𝑛 − 𝑥𝑛 + 3 sin 𝑥𝑛
=
1−3 cos 𝑥𝑛
3(sin 𝑥𝑛 −𝑥𝑛 cos 𝑥𝑛
xn+1 =
1−3 cos 𝑥𝑛
2+3
Taking x0 = = 2.5
2
3(sin 2.5−2.5 cos 2.5
x1 = = 2.293
1−3 cos 2.5
Error = |2.293 − 2.5| = 0.207 > 0.005
3(sin 2.293−2.5 cos 2.293
x2 = = 2.279
1−3 cos 2.293
Error = |2.279 − 2.293| = 0.014 > 0.005
3(sin 2.279−2.5 cos 2.279
x3 = = 2.279
1−3 cos 2.279
Error = |2.279 − 2.279| = 0.000 < 0.005
∴ root = 2.279 = 2.28(2D)

Example 22

(a) On the same axis, draw graphs of y = x and y = 4sinx to show that the root of the equation
x-4sinx = 0 lies between x= 2 and x = 3

Therefore the root (2.47) lies between x = 2 and x = 3

(b) Use Newton Raphson’s method to calculate the root of the equation x – 4sinx = 0, taking
approximate root in (a) as the initial approximation to the root. correct your answer to 3
decimal places.
f(x) = x – 4sinx
f’(x) = 1 – 4cos x
𝑥𝑛 −4 sin 𝑥𝑛
𝑥𝑛+1 = 𝑥𝑛 −
1−4 cos 𝑥𝑛
𝑇𝑎𝑘𝑖𝑛𝑔 𝑥0 = 2.47
2.47−4 sin 2.45
𝑥1 = 2.47 − = 2.4746
1−4 cos 2.47
𝐸𝑟𝑟𝑜𝑟 = |2.4746 − 2.47| = 0.0046 > 0.0005
2.4746−4 sin 2.4546
𝑥2 = 2.4746 − = 2.4746
1−4 cos 2.4746
𝐸𝑟𝑟𝑜𝑟 = |2.4746 − 2.4746| = 0.000 < 0.0005 ∴ the root = 2.475 (3D)
Example 23

(a) Draw on the same axes the graphs of the curves y = 2 – e-x and y = √𝑥values 2≤ x ≤ 5. (04marks)

x y = 2 – e-x y = √𝑥
2.0 1.86 1.41
2.5 1.92 1.58
3.0 1.95 1.73
3.5 1.97 1.87
4.0 1.98 2.00
4.5 1.99 2.12
5.0 1.99 2.24

(b) Determine from your graph the interval within which the roots of the equation
𝑒 −𝑥 + √𝑥 − 2 = 0 lies

Hence, use Newton-Raphson’s method to find the root of the equation correct to 3
decimal places (07marks)

Root lies between 3.9 and 4


f(x) = 2 – e-x - √𝑥
1
f’(x) = 𝑒 −𝑥 −
2√𝑥
1
f(xn) = 𝑒 −𝑥𝑛 −
2√𝑥𝑛
2− 𝑒 𝑥𝑛 − √𝑥𝑛
xn+1 =xn -
2𝑒 −𝑥𝑛 √𝑥𝑛 −1
3.9+4
x0 = = 3.95
2
2√3.95(2− 𝑒 −3.95 −√3.95)
𝑥1 = 3.95 − = 3.9211
2𝑒 −3.95 √3.95−1
𝐸𝑟𝑟𝑜𝑟 = |3.9211 − 3.95| = 0.0289
2√3.9211(2− 𝑒 −3.9211 −√3.9211)
𝑥2 = 3.9211 − = 3.9211
2𝑒 −3.9211 √3.9211−1
∴ Root = 3.921 (3dp)
Example 24

Given the equation X3 – 6x2 + 9x +2 = 0


(a) Show that the equation has a root between -1 and 0.
Let f(x) = X3 – 6x2 + 9x +2
f(-1) = (-1)3 – 6(-1)2 + 9(-1) +2
= -1 – 6 -9 +2 = -14
f(0) = 0 + 0 + 0 + 2
=2
f(-1).f(0) = -14 x 2 = -28
since f(-1).f(0) < 0; the root exist between -1 and 0.
(b) (i) Show that the Newton Raphson formula approximating the root of the equation is
3 −3𝑥 2 −1
2 𝑥𝑛
given by 𝑋𝑛+1 = [ 2 𝑛 ]
3 𝑥𝑛 −4𝑥𝑛 +3
f(x) = X3 – 6x2 + 9x +2
f(xn) = 𝑥𝑛3 − 6𝑥𝑛2 + 9𝑥𝑛 + 2
f’(xn) =3𝑥𝑛2 − 12𝑥𝑛 + 9
3 −6𝑥 2 +9𝑥 +2
𝑥𝑛 𝑛 𝑛
𝑥𝑛+1 = 𝑥𝑛 − ( 2 −12𝑥 +9 )
3𝑥𝑛 𝑛
2 −12𝑥 +9)− (𝑥 3 −6𝑥 2 +9𝑥 +2)
𝑥𝑛 (3𝑥𝑛 𝑛 𝑛 𝑛 𝑛
= 2 −12𝑥 +9
3𝑥𝑛 𝑛
3 −12𝑥 2 +9𝑥 )− (𝑥 3 −6𝑥 2 +9𝑥 +2)
(3𝑥𝑛 𝑛 𝑛 𝑛 𝑛 𝑛
= 2 −12𝑥 +9
3𝑥𝑛 𝑛
3 −6𝑥 2 −2
2𝑥𝑛 𝑛
= 2 −12𝑥 +9
3𝑥𝑛 𝑛
2 𝑥𝑛3 −3𝑥 2 −1
𝑛
= [2 −4𝑥 +3 ]
3 𝑥𝑛 𝑛
(ii) Use the formula in (b)(i) above, with initial approximation of x0 = -0.5, to find the root
of the given equation correct to two decimal places
Taking x = -0.5
2 (−0.5)3 −3(−0.5)2 −1
𝑥1 = [ ] = -0.2381
3 (−0.5)2 −4(−0.5)+3
|𝑒| = |0.2381 − (−0.5)| = 0.2619
2 (−0.2381)3 −3(−0.2381)2 −1
𝑥2 = [ ] = -0.1968
3 (−0.2381)2 −4(−0.2381)+3
|𝑒| = |0.1968 − (−0.2381)| = 0.0.0413
2 −0.19683 −3(−0.1968)2 −1
𝑥3 = [ ] = -0.1958
3 (−0.1968)2 −4(−0.1968)+3
|𝑒| = |−0.1958 − (−0.1968)| = 0.001< 0.005

Hence the root = -0.20 (2D)

Revision Exercise 3
1. Using the Newton Raphson’s formula, show that the reciprocal of a number N is
𝑥𝑛 (2 − 𝑁𝑥𝑛)
2. Use Newton Raphson’s iterative formula to show that the cube root of a number N is
1 𝑁 3
given by (2𝑥𝑛 + ). Hence use the iterative formula to find √96 correct to3 decimal
3 𝑥𝑛 2
places. use 𝑥0 = 5. [4.579]
3. (a) Show that the equation 3x3 + x -5 = 0 has real root between x = 1 and x = 2.
(b) Using linear interpolation, find the first approximation for this root to 2dp. [1.04]
(c) Using Newton Raphson’s method twice find the value of this root correct to 2 dp.
[1.09]
4. (a) Show graphically that there is a positive real root of equation xe-x -2x + 5 = 0 between
x = 2 and x = 3
(b) Using Newton Raphson’s method, find this root correct to 1 dp. [2.6]
5. Using the iterative formula for NRM, show that the fourth root of a number N is
1
3 𝑁
(𝑥𝑛 + ). Starting with x0 = 2.5 show that (45.7)4 = 2.600 (3dp)
4 3𝑥𝑛 3
6. On the same axes, draw graphs of y = x3 and y = 2x + 5. Using NRM twice find the
positive root of the equation x3 - 2x – 5 = 0 correct to 2 decimal places. [2.09]
7. (a) Show that the Newton Raphson’s formula for finding the smallest positive root of the
6𝑥𝑛 −3 sin 2𝑥𝑛
equation 3tanx + x = 0 is
6+ 2 cos 2𝑥𝑛
−𝑥
(b) By sketching the graphs of y = tans, y = 0r otherwise, find the first approximation
3
to the required root and use it to find the actual root correct to 3 dp . [2.456]
8. (a) Show that the root of the equation f(x) = ex + x3 – 4x = 0 has a root between x =1 and
x=2
(b) Use the Newton Raphson’s method to find the root of equation in (a) correct to 2
decimal places. [x0 = 1, root =1.12]
9. (a) Show that the iterative formula for approximation of the root of f(x) = 0 by NRM
𝑥𝑛 2 𝑒 𝑥𝑛 +10
process for the equation xex + 5x – 10 = 0 is 𝑥𝑛+1 = .
𝑥𝑛 𝑒 𝑥𝑛 + 𝑒 𝑥𝑛 +5
(b) Show that the root of the equation in (i) above lies between x = 1 and x = 2. Hence
find the root of the equation correct to 2 dp. [1.20]
10. (a) Use a graphical method to find a first approximation to the real root of x3 + 2x -2 = 0.
(b) Use the Newton Raphson’s method to find the root of the equation in (a) correct to 2
dp. [0.77]
11. (a) Show that equation x = In (8-x) has a root between x= 1 and x = 2.
(b) Use the Newton Raphson’s method to find the root of the equation in (a) correct to 2
decimal places [1.82]
12. (a) Use graphical method to find the first approximation to the root of x3 – 3x + 4 = 0.
[-2]
(b) Use NRM to find the root of the equation in (a) correct to 2 d.p. [-2.20]
13. Show graphically that equation ex + x -4 = 0 has only one root between x = 1 and x = 2.
Use NRM to find the approximation of the equation correct to 3dp. [1.07]
14. Show that the NRM for approximating the Kth root of a number N is given by
1 𝑁
((𝐾 − 1)𝑥𝑛 + ). Hence use your formula to find the positive square root of 67
𝐾 𝑥𝑛 𝐾−1
correct to 4 s.f. [8.185].
15. (a) Show that equation x3 + 3x – 9 = 0 has a root between x= 1 and x = 2.
(b) Use the Newton Raphson’s method to find the root of the equation in (a) correct to 2
0ne places [1.6]
16. (a) Show graphically that there is a positive real root of equation xe-x -2x - 1 = 0 between
x = 1 and x = 2
(b) Using Newton Raphson’s method, find this root for the equation in (a) correct to 2
dp. [1.26]
𝑥
17. (a) Show that equation 2x – 3cos ( ) = 0 has a root between x= 1 and x = 2.
2
(b) Use the Newton Raphson’s method to find the root of the equation in (a) correct to
one places [1.23]
18. (a) If a is the first approximation to the root of the equation x5 – b = 0, show that the
𝑏
4𝑎+ 4
𝑎
second approximation is given by .
5
(b) Show that the positive real root of the equation x5 – 17 = 0 lies between 1.5 and 1.8.
Hence use the formula in (a) above to determine the root to 3 decimal places. [1.762]
𝜋 𝜋
19. (a)(i) On the same axes, draw graphs of y = x2 and y = cox x for 0 ≤ x ≤ at intervals of .
2 8
(ii) Use your graphs, to find to 1 decimal place an approximate root of the equation
x2 – cos x = 0 [0.8]
(b) Use the NRM to calculate the root of the equation x2 – cos x = 0 taking the
approximate root in (a) as the initial approximation. Correct your answer to 3 dp. [0.824]
20. (a) (i) Draw on same axes the graphs of equation y = xsinx and y = ex -2 for 0≤ x ≤1.5.
(ii) Use your graphs to find an approximate root of the equation 2 – ex + xsinx = [1.1]
(c) Use the Newton Raphson’s method to find the root of the equation in (a)(ii) correct to
three decimal places [1.085]
21. Show graphically that equation ex + x -8 = 0 has only one real root between x = 1 and x =
2. Use NRM to find approximation of x = In(x 8) correct to 3 dp [1.821]
𝜋
22. Draw using the same axes, graphs of y = x2 and y = sin2x for 0≤ x ≤ . From the graphs
2
obtain to one decimal place an approximation of the non-zero root of the equation
x2 –sin 2x = 0. Using NRM, calculate to 2 dp a more suitable approximation. [0.97]
23. Given the equation In(1 + 2x) – x = 0.
(i) show the root of the equation above lies between x = 1 and x = 1.5
(ii) Use NRM twice to estimate the root of the equation, correct to 2 dp. [1.26]

Thank you
Dr. Bbosa Science
Errors
An error, commonly known as absolute error is the absolute difference between exact value and
approximate value.

Source of errors
(a) Rounding off

These errors that arise as a result of simply approximating the exact value of different numbers.

Example 1

Round off the following numbers to the given number of decimal places or significant figures.

(i) 3.896234 to 4 dp [3.8962] (iv) 0.00652673bto4 s.f [0.006527]


2
(ii) to 3dp [0.667]
3 (v) 7.00214 to 4 s.f. [7.002]
(iii) 5.002570 to 3s.f [5.00]
(vi) 5415678 to 3 s.f. [5420000]

(b) Truncation

These occur when an infinite number is terminated/cutoff (without rounding off] at some point.

Example 2

Truncate the following number to the given number of decimal places (d.p) or significant figures. s.f.
2
(i) 4.56172 to 2dp [4.56] (ii) to 3dp [0.666] (iii) 1.345618 to 4 s.f. [1.345]
3

Common terms used


(a) Error or absolute error
If x represent an approximate value of X and ∆x is the error approximation
|𝐸𝑟𝑟𝑜𝑟| = |𝑒𝑥𝑎𝑐𝑡 𝑣𝑎𝑙𝑢𝑒 − 𝑎𝑝𝑝𝑟𝑜𝑥𝑖𝑚𝑎𝑡𝑒 𝑣𝑎𝑙𝑢𝑒|
|∆𝑥| = |𝑋 − 𝑥|

Example 3

Round off 32.5263 to 2 dp and determine the absolute error.

Solution

X = 32.5263, x = 32.53
|∆𝑥| = |𝑋 − 𝑥| = |32.5263 − 32.53| = 0.0037

(b) Relative error


𝑎𝑏𝑠𝑜𝑙𝑢𝑡𝑒 𝑒𝑟𝑟𝑜𝑟 |∆𝑥| |𝑋−𝑥|
Relative error = = =
𝑒𝑥𝑎𝑐𝑡 𝑣𝑎𝑙𝑢𝑒 𝑋 𝑋
(c) Percentage error or percentage relative error
𝑎𝑏𝑠𝑜𝑙𝑢𝑡𝑒 𝑒𝑟𝑟𝑜𝑟 |∆𝑥| |𝑋−𝑥|
Percentage relative error = 𝑥 100% = 𝑥 100% = 𝑥 100%
𝑒𝑥𝑎𝑐𝑡 𝑣𝑎𝑙𝑢𝑒 𝑋 𝑋
Example 4
Find the percentage error in rounding off √3 2 dp
Solution
X = √𝑥 = 1.732050808, x = 1.73
|𝑋−𝑥| |1.732050808 −1.73|
Percentage error = 𝑥 100% = 𝑥 100% = 0.118%
𝑋 1.732050808

(d) Error bound or minimum possible error in an approximated number


This depends on the number of decimal places the number is rounded to. If the number is
rounded to n dp, then the maximum possible error in that number is = 0.5 x 10-n.

Example 4

If a student weighs 50kg. Find the range where his weight lies

Solution

n = 0 dp, e = 0.5 x 10-0 = 0.5

Range = 50 ± 0.5 =(49.5, 50.5)

Example 5

If x is given to stated level of accuracy stat the lower and upper bounds of x

(a) 6.45
n = 2 dp, e = 0.5 x 10-2 = 0.005
Lower bound = 6.45 – 0.005 = 6.445
upper bound = 6.45 + 0.005 = 6.455

(b) 0.278
n = 3 dp, e = 0.5 x 10-3 = 0.0005
(c) Lower bound = 0.278 – 0.0005 = 0.2775
upper bound = 0.278 + 0.0005 = 0.2785

Example 6

A value of w = 150.58m was obtained when measuring the width of the football pitch. Given that the
relative error in this value as 0.07%, find the limit within which the value w lies.
|∆𝑤|
% relative error = 𝑥100%
𝑤 Lower limit = 150.58 – 0.105 = 150 .475
|∆𝑤|
0.07 = 𝑥 100 Upper limit = 150.58 + 0.105 = 150 .685
150.58

|∆𝑤| = 0.105
Absolute error in an operation
When the minimum and maximum value is known then.
1
absolute error = [𝑚𝑎𝑥𝑖𝑚𝑢𝑚 𝑣𝑎𝑙𝑢𝑒 − 𝑚𝑖𝑛𝑖𝑚𝑢𝑚 𝑣𝑎𝑙𝑢𝑒]
2

Absolute error in addition

Given two numbers a and b with errors ∆a + ∆b

(a + b)max = amax + bmax = (a + ∆a) + (b +∆b)

(a + b)min = a min + b min = (a - ∆a) + (b - ∆b)

Absolute error in subtraction

Given two numbers a and b with errors ∆a + ∆b

(a - b)max = amax - bmin = (a + ∆a) - (b - ∆b)

(a + b)min = a min - b max = (a - ∆a) + (b + ∆b)

Example 7

Given that a = 2.453, b = 6.79, find the limits and hence absolute error of

(i) a+b

solution

a = 2.453, ∆a = 0.0005 and b = 6.79, ∆b = 0.005

(a + b)max = amax + bmax = (a + ∆a) + (b +∆b)

= (2.453 + 0.0005) + (6.79 + 0.005) = 9.2485

(a + b)min = a min + b min = (a - ∆a) + (b - ∆b)

= (2.453 - 0.0005) + (6.79 - 0.005) = 9.2375

lower limit = 9.2375; upper limit = 9.2485


1
absolute error = [9.2485 − 9.2375] = 0.0055
2

(ii) a-b

solution

(a + b)max = amax - bmin = (a + ∆a) - (b -∆b)

= (2.453 + 0.0005) - (6.79 - 0.005) = -4.3315

(a + b)min = a min - b max = (a - ∆a) - (b + ∆b)

= (2.453 - 0.0005) - (6.79 + 0.005) = -4.3425

lower limit = -4.3425; upper limit = -4.3315


1
absolute error = [−4.3315 − −4.3425] = 0.0055
2
Absolute error in multiplication

Given two numbers a and b with errors ∆a + ∆b


(ab)max = amaxbmax = (a + ∆a)(b+∆b)
(ab)min = aminbmin = (a - ∆a)(b - ∆b)

Example 8

Given that a = 4.617, and b = 3.65 find the absolute error in ab

solution

a = 4.617, ∆a= 0.0005, b = 3.65, ∆b = 0.005

(ab)max = amaxbmax = (a + ∆a)(b+∆b) = (4.617 + 0.0005)(3.65b+ 0.005) =16.87696

(ab)min = aminbmin = (a - ∆a)(b - ∆b) = (4.617 - 0.0005)(3.65b - 0.005)=16.82853


1 1
absolute error = [𝑚𝑎𝑥𝑖𝑚𝑢𝑚 𝑣𝑎𝑙𝑢𝑒 − 𝑚𝑖𝑛𝑖𝑚𝑢𝑚 𝑣𝑎𝑙𝑢𝑒] = (16.87696 − 16.82853) =0.02422
2 2

Example 9

Given that a = 4.617, and b = -3.65 find the

(i) Limits of values where ab lies


Solution

a = 4.617, ∆a= 0.0005, b = -3.65, ∆b = 0.005

(ab)max = amaxbmax = (4.617 + 0.0005)(-3.65b+ 0.005) = -16.83079


(ab)min = aminbmin = (4.617 - 0.0005)(-3.65b - 0.005 = -16.87331
Lower limit = -16.87331; upper limit = -16.83079
(ii) the interval of values where ab lies
(-16.87331, -16.83079)
(iii) the absolute error
1
Absolute error = [−16.83079 − −16.87331]= 0.02126
2

Absolute error n division

Given two numbers a and b with errors ∆a + ∆b


𝑎 𝑎𝑚𝑎𝑥 (𝑎+ ∆𝑎)
( ) = = (𝑏− ∆𝑏)
𝑏 𝑚𝑎𝑥 𝑏𝑚𝑖𝑛
𝑎 𝑎𝑚𝑖𝑛 (𝑎− ∆𝑎)
( ) = = (𝑏+ ∆𝑏)
𝑏 𝑚𝑖𝑛 𝑏𝑚𝑎𝑥

Example 10

Given a = 1.26, b = 0.435. Find the absolute error of


𝑎
(i) Range of value where lies
𝑏
𝑎 𝑎𝑚𝑎𝑥 (1.25+0.005)
( ) = = = 2.91139
𝑏 𝑚𝑎𝑥 𝑏𝑚𝑖𝑛 (0.435−0.0005)
𝑎 𝑎𝑚𝑖𝑛 (1.25−0.005)
( ) = = = 2.88175
𝑏 𝑚𝑖𝑛 𝑏𝑚𝑎𝑥 (0.435+0.0005)
Range of values is (2.88175, 2.91139)

(ii) Absolute error


1
= (2.91139 − 2.88175) = 0.01482
2

Example 11

(a) Given that y = 𝑒 𝑥 and x = 0.62correct to two decimal places, find the interval within which the
exact value of y lies. (05marks)

𝑒𝑥 = 0.005
𝑦𝑚𝑎𝑥 = 𝑒 0.625 = 1.8682
𝑦𝑚𝑖𝑛 = 𝑒 0.615 = 1.8497
The interval = (1.8497, 1.8682)

(b) Show that the maximum possible relative error in y𝑠𝑖𝑛2 𝑥 is


∆𝑦
| | + 2 cot 𝑥 |∆𝑥|, where∆𝑥 and ∆𝑦 are errors in x and y respectively
𝑦
𝜋 𝜋
Hence find the percentage error in calculating y𝑠𝑖𝑛2 𝑥 if y = 5.2 ± 0.05 and x = ±
6 360
(07 marks)
z = ysin2x
𝑒𝑧 = ∆𝑦𝑠𝑖𝑛2 𝑥 + 2y∆𝑥𝑐𝑜𝑠 𝑥𝑠𝑖𝑛𝑥
𝑒𝑧 ∆𝑦𝑠𝑖𝑛2 𝑥 2y∆𝑥𝑐𝑜𝑠 𝑥𝑠𝑖𝑛𝑥
= +
𝑧 ysin2 x ysin2 x
𝑒𝑧 ∆𝑦
| |= | + 2𝑐𝑜𝑡𝑥. ∆𝑥|
𝑧 𝑦
∆𝑦
≤| | + 2𝑐𝑜𝑡𝑥. |∆𝑥|
𝑦
∆𝑦
∴ Maximum possible error is | | + 2𝑐𝑜𝑡𝑥. |∆𝑥|
𝑦
0.05 𝜋 𝜋
𝑝𝑒𝑟𝑐𝑒𝑛𝑡𝑎𝑔𝑒 𝑒𝑟𝑟𝑜𝑟 = [ + 2 cot . | |] 𝑥 100% = 3.9845%
5.2 6 360

Example 12

Two numbers A and B have maximum possible error ea and eb respectively.

(a) Write an expression for the maximum possible error in their sum
Maximum possible error = |𝑒𝑎 ||𝑒𝑏 |

(b) If A = 2.03 and B = 1.547, find the maximum possible error in A + B (05marks)
𝑒𝑎 = 0.005, 𝑒𝑏 = 0.0005
|𝑒(𝐴+𝐵) | = |0.005| + |0.0005|
= 0.0055

Example 13
1
Given that y = + 𝑥 and x = 2.4 correct to one decimal place, find the limits within which y lies.
𝑥
(05marks)
1 1
Error in 2.4 = 𝑥 = 0.05
2 10
1
ymax = 2.45 + = 2.8755
2.35
1
ymin = 2.35 + = 2.7582
2.45
∴ the limits are [2.7582, 2.8755]

Example 14

The numbers X = 1.2, Y = 1.33 and Z = 2.245 have been rounded off to the given decimal places. find
𝑌
the maximum possible value of correct to 3 decimal places
𝑍−𝑋

(𝑌+∆𝑌) (1.33+0.005)
Maximum value = (𝑍−∆𝑍)−(𝑋+∆𝑋) = (2.245−0.0005)−( 1.2+0.05)
= 1.342

Revision exercise 1
1. Given the numbers x = 2.678 and y = 0.8765 measured the nearest possible decimal places
indicated.
(i) state the maximum possible error in x ans y [∆x = 0.0005, ∆y = 0.00005]
(ii) find the limits within which the product xy lie [2.3467, 2.3478]
(iii) determine the maximum possible error in xy [0.000572]

2. The length, width and height of water all rounded off to 3.65m, 2.14m and 2.5m respectively.
Determine the least and greatest amount of water the tank can contain [19.066, 19.992]
3. Given that the values x = 4, y = 6 and z = 8 each has been approximate to the nearest integer.
find the maximum and minimum values of
𝑦
(i) [1.85714, 1.22222]
𝑥
𝑧−𝑥
(ii) [0.90909, 0.46154]
𝑦
(iii) (𝑥 + 𝑦)𝑧 [93.5, 67.5]

Error propagation
Triangular inequality state that |𝑎 ± 𝑏| ≤ |∆𝑎| + |∆𝑏|

Addition
Consider two numbers X and Y are approximated by x and y with errors ∆x and ∆y.

|𝑒𝑥+𝑦 | = |(𝑥 + ∆𝑥) + (𝑦 + ∆𝑦) − (𝑥 + 𝑦)|

=|∆𝑥 + ∆𝑦|= |∆𝑥| + |∆𝑦|


∆𝑥 ∆𝑦
R.Emax = [| |+| |]
𝑋+𝑌 𝑋+𝑌

Alternatively
1
absolute error = [max − 𝑚𝑖𝑛]
2
1
= [(𝑥 + ∆𝑥) + (𝑦 + ∆𝑦)] − [(𝑥 − ∆𝑥) + (𝑦 − ∆𝑦)]
2
|𝑒𝑥+𝑦 |=|∆𝑥 + ∆𝑦|= |∆𝑥| + |∆𝑦|
∆𝑥 ∆𝑦
R.Emax = [| |+| |]
𝑋+𝑌 𝑋+𝑌

Example 15

Given numbers x = 7.824 and y = 3.36 rounded to the given number of decimal places. Find the limits
within which (x + y) lies

Solution

∆x = 0.0005, ∆y = 0.005
Alternatively
|𝑒𝑥+𝑦 |=|∆𝑥| + |∆𝑦|= 0.0005 + 0.005 = 0.0055
(x + y)max = 7.8245 + 3.365 =10.1895
working value (x + y) = 7.824 + 3.36 = 10.184
(x + y)min = 7.8235 + 3.355 = 10.1785
Upper limit = 10.184 + 0.0055 = 10.1895

Lower limit = 10.184 - 0.0055 = 10.1785

Example 16

If x = 4.95 and y = 2.2 are each rounded off to the given number of decimal places. Calculate

(i) The percentage error in x + y

Solution

∆x = 0.005, ∆y = 0.05
∆𝑥 ∆𝑦 0.005 0.05
%error = [| |+| |] 𝑥 100% = [| |+| |] 𝑥 100% = 0.769
𝑋+𝑌 𝑋+𝑌 4.95+2.2 4.95+2.2

Alternatively

Working value x + y = 4.95 + 2.2 = 7.15

|𝑒𝑥+𝑦 |=|∆𝑥| + |∆𝑦|= 0.005 + 0.05 = 0.055


0.055
% error = 𝑥 100% = 0.769
7.15

(ii) Find the limit within which (x + y) is expected to lie. Give your answer to two decimal places.
Upper limit = 7.15 + 0.055 = 7.21; lower limit = 7.15 – 0.055 = 7.10
Alternatively
Upper limit = 4.955 + 2.25 = 7.21; lower limit = 4.945 + 2.15 = 7.10

Subtraction
Consider two numbers X and Y are approximated by x and y with errors ∆x and ∆y.

|𝑒𝑥−𝑦 | = |(𝑥 + ∆𝑥) − (𝑦 + ∆𝑦) − (𝑥 − 𝑦)|

=|∆𝑥 − ∆𝑦|= |∆𝑥| + |∆𝑦|


∆𝑥 ∆𝑦
R.Emax = [| |+| |]
𝑋−𝑌 𝑋−𝑌
Alternatively
1
absolute error = [max − 𝑚𝑖𝑛]
2
1
= {[(𝑥 + ∆𝑥) − (𝑦 − ∆𝑦)] − [(𝑥 + ∆𝑥) − (𝑦 + ∆𝑦)]}
2

|𝑒𝑥−𝑦 | = |∆𝑥 + ∆𝑦|= |∆𝑥| + |∆𝑦|


∆𝑥 ∆𝑦
R.Emax = [| |+| |]
𝑋−𝑌 𝑋−𝑌

Example 17

Given number x = 6.375 and y 4.46 rounded off to the given number of decimal places. Find the limit
within which (x – y) lies

Solution

∆x = 0.0005, ∆y = 0.005

|𝑒𝑥−𝑦 | = |∆𝑥| + |∆𝑦| = |0.0005| + |0.005| = 0.0055

working value = 6.375 – 4.46 = 1.915

Upper limit = 1.915 + 0.0055 = 1.9205; Lower limit = 1.915 – 0.0055 = 1.9095

Alternatively

(x – y)max = 6.3755 – 4.455 = 1.9205

(x – y)min = 6.3745 – 4.465 = 1.9095

Example 18

If x = 1.563 and y = 9.87 are each rounded off to the given number of decimal places. Calculate

(i) the percentage error in (x – y)


∆𝑥 ∆𝑦 0.0005 0.005
% error = [| |+| |] 𝑥 100% = [| |+| |] 𝑥100% = 0.0662
𝑋−𝑌 𝑋−𝑌 1.563−9.87 1.563−9.87
Alternatively
Working value = x – y = 1.563 9.87 = -8.307
|𝑒𝑥−𝑦 | = |∆𝑥| + |∆𝑦| = |0.0005| + |0.005| =0.0055
0.0055
% 𝑒𝑟𝑟𝑜𝑟 = 𝑥 100% = 0.0662
−8.307
(ii) the limit within which (x – y) is expected to lie. Give your answer to three decimal places
Upper limit = -8.307 + 0.0055 = -8.302
Lower limit = -8.307 - 0.0055 = -8.313
Alternatively
(x – y)max = 1.5635 – 9.865 =-8.302
(x – y)min =1.5625 – 9.875 =-8.313
Multiplication
Consider two numbers X and Y are approximated by x and y with errors ∆x and ∆y.

|𝑒𝑥𝑦 | = |(𝑥 + ∆𝑥)(𝑦 + ∆𝑦) − (𝑥𝑦)|

= |𝑥𝑦 + 𝑦∆𝑥 + 𝑥∆𝑦 + ∆𝑥∆𝑦 − 𝑥𝑦|

Since ∆x and ∆y are very small, ∆𝑥∆𝑦 ≈ 0

|𝑒𝑥𝑦 | = |𝑦∆𝑥 + 𝑥∆𝑦| = |𝑦∆𝑥| + |𝑥∆𝑦|


𝑦∆𝑥 𝑥∆𝑦
R.Emax = | |+| |
𝑥𝑦 𝑥𝑦

∆𝑥 ∆𝑦
=| | + | |
𝑥 𝑦

Alternatively
1
absolute error = |max − 𝑚𝑖𝑛|
2
1
= [(𝑥 + ∆𝑥)(𝑦 + ∆𝑦) − [(𝑥 − ∆𝑥)(𝑦 − ∆𝑦)]]
2

|𝑒𝑥𝑦 | =|𝑦∆𝑥 + 𝑥∆𝑦| = |𝑦∆𝑥| + |𝑥∆𝑦|


𝑦∆𝑥 𝑥∆𝑦
R.Emax = | |+| |
𝑥𝑦 𝑥𝑦

Example 19

Given numbers x = 6.375 and y = 4.46 rounded off to eh given number of decimal places. Find the
limit within which (xy) lies

Solution

∆x =0.0005 ∆y = 0.005

|𝑒𝑥𝑦 | =|𝑦∆𝑥| + |𝑥∆𝑦| = |6.375 𝑥 0.005| + |4.46 𝑥 0.0005| = 0.0341

working value = xy = 6.375 x 4.46 = 28.4325

Upper limit = 28.4325 + 0.0341 =28.4666

Lower limit = 28.4325 - 0.0341 = 28.3984

Alternatively

(xy)max = 6.3755 x 4.465 =28.4666

(xy)min = 6.3745 x 4.455 = 28.3984

Example 20

If x = 1.563 and y = 9.87 are each rounded off to the given number of decimal places. Calculate

(i) Percentage error in (xy)


∆x = 0.0005, ∆y = 0.005
∆𝑥 ∆𝑦 0.0006 0.005
% error ={| | + | |} 𝑥 100% = {| |+| |} 𝑥 100% = 0.0826
𝑥 𝑦 1.563 9.87

Alternatively

Working value = 1.563 x 9.87 = 15.4268

|𝑒𝑥𝑦 | = |𝑦∆𝑥| + |𝑥∆𝑦| = 9.87 x 0.0005 + 1.563 x 0.005 =0.0128


0.0128
% 𝑒𝑟𝑟𝑜𝑟 = 𝑥 100% = 0.0826
15.4268
(ii) the limit within which (xy) is expected to lie. Give your answer to three decimal places.

Upper limit = 15.4268 + 0.0128 = 15.440

Lower limit =15.4268 - 0.0128 =15.414

Alternatively

Upper limit = 1.5635 x 9.875 = 15.440

Lower limit = 1.5625 x 9.865 =15.414

Division
Consider two numbers X and Y are approximated by x and y with errors ∆x and ∆y.
𝑥+ ∆𝑥 𝑥 𝑥𝑦+𝑦∆𝑥−𝑥∆𝑦−𝑥𝑦
|𝑒𝑥⁄𝑦 | = | − | =| | Alternatively
𝑦+ ∆𝑦 𝑦 𝑦 2 +𝑦∆𝑦
1
𝑦∆𝑥−𝑥 ∆𝑦
absolute error = |max − 𝑚𝑖𝑛|
2
=| ∆𝑦 |
𝑦 2 (1+ ) 1 (𝑥+ ∆𝑥) (𝑥− ∆𝑥)
𝑦
= |(𝑦− − (𝑦+ |
2 ∆𝑦) ∆𝑦)
∆𝑦
Since ∆𝑥 and ∆𝑦 are very small, then ≈0 𝑥∆𝑦+𝑦∆𝑥
𝑦
𝑒𝑥⁄𝑦 = | |
𝑦 2 −∆𝑦 2
𝑦∆𝑥−𝑥 ∆𝑦
|𝑒𝑥⁄𝑦 | = | |
𝑦2 Since ∆𝑥 and ∆𝑦 are very small, then∆𝑦 2 ≈ 0
|𝑦∆𝑥|+|𝑥 ∆𝑦|
|𝑒𝑥⁄𝑦 | ≤ |𝑦 2 | |𝑒𝑥⁄𝑦 | = |
𝑦∆𝑥−𝑥 ∆𝑦
|
𝑦2
|𝑦∆𝑥|+|𝑥 ∆𝑦|
emax = |𝑦∆𝑥|+|𝑥 ∆𝑦|
|𝑦 2 | |𝑒𝑥⁄𝑦 | ≤ |𝑦 2 |
|𝑦∆𝑥|−|𝑥 ∆𝑦| 𝑥
R.Emax = ÷ |𝑦∆𝑥|+|𝑥 ∆𝑦|
|𝑦 2 | 𝑦 emax = |𝑦 2 |
∆𝑥 ∆𝑦
R.Emax =| | + | | R.Emax =
|𝑦∆𝑥|−|𝑥 ∆𝑦|
÷
𝑥
𝑥 𝑦
|𝑦 2 | 𝑦

∆𝑥 ∆𝑦
R.Emax =| | + | |
𝑥 𝑦
Example 21

Given numbers x = 5.794 and y = 0.28 rounded off to the given number of decimal places. Find limit
𝑥
within which lies
𝑦

Solution
Upper limit = 20.6929 + 0.3713 =21.0642
∆x = 0.0005, ∆y = 0.005
Lower limit =20.6929 - 0.3713 = 20 .3198
|𝑦∆𝑥|+|𝑥 ∆𝑦|
|𝑒𝑥⁄𝑦 | = |𝑦 2 | Alternatively
|0.28 𝑥 0.0005|+ |5.794 𝑥 0.005| 5.7945
= Upper limit = =21.079
|0.282 | 0.275
5.7935
= 0.3713 Lower limit = = 20.3281
0.285
𝑥 5.794
Working value = = = 20.6929
𝑦 0.28

Example 22

If x = 7.37 and y = 2.00 are each rounded off to the given number of decimal places. Calculate

(i) Percentage error

∆𝑥 ∆𝑦 0.005 0.005
% error = ={| | + | |} 𝑥100% = {| |+| |} 𝑥100% = 0.318
𝑥 𝑦 7.37 2.00
Alternatively
|𝑦∆𝑥|+|𝑥 ∆𝑦| |2.00 𝑥 0.005|+ |7.37 𝑥 0.005|
|𝑒𝑥⁄𝑦 | = |𝑦 2 |
= |2.002 |
= 0.0117

𝑥 7.37
Working value = = = 3.685
𝑦 2.00

0.0117
% error = 𝑥 100 = 0.318
3.685

𝑥
(ii) the limit within which ( ) is expected to lie. Give your answer to three decimal places.
𝑦

Upper limit = 3.685 + 0.318 = 3.697

Lower limit = 3.685 − 0.318 = 3.673

Alternatively
7.375
Upper limit = = 3.697
1.995
7.365
Lower limit = = 3.673
2.005

Error in functions
Given a function f(x) with a maximum possible error ∆x.

Absolute error, |𝑒| = |∆𝑥|𝑓 1 (𝑥)


|∆𝑥|𝑓 1 (𝑥)
Maximum possible relative error, R.E =
𝑓(𝑥)

Example 23

Find the absolute error and maximum relative error in each of the following functions

(i) y = x4
|𝑒|=|∆𝑥|𝑓 1 (𝑥) = 4x3|∆𝑥|
|∆𝑥|𝑓 1 (𝑥) 4x3 |∆𝑥| 4|∆𝑥|
R.E = = =
𝑓(𝑥) 𝑥4 𝑥

3
(ii) y = 𝑥2
1
3
|𝑒|=|∆𝑥|𝑓 1 (𝑥) = 𝑥 2 |∆𝑥|
2
1
3
|∆𝑥|𝑓 1 (𝑥) 𝑥 2 |∆𝑥| 3 |∆𝑥|
2
R.E = = 3 =
𝑓(𝑥) 2 𝑥
𝑥2
(iii) y = sin x
|𝑒|=|∆𝑥|𝑓 1 (𝑥) =cos 𝑥|∆𝑥|
|∆𝑥|𝑓 1 (𝑥) cos 𝑥|∆𝑥|
R.E = = = |∆𝑥||cot 𝑥|
𝑓(𝑥) 𝑠𝑖𝑛 𝑥

Example 24

Given that the error in measuring an angle is 0.40. find the maximum possible error and relative
error in tanx if x = 600.

Solution
0.0280
|𝑒|=|∆𝑥|𝑓 1 (𝑥) = (1 + tan2x)|∆𝑥| R.E = = 0.0162
tan 60
0.4
|𝑒|=(1 + tan260)| 𝜋| = 0.0280
180

Error in a function that has more variables


Given a function f(x, y) with a maximum possible error ∆x and ∆y respectively

Absolute error, |𝑒| =|∆𝑥|𝑓 1 (𝑥) + |∆𝑦|𝑓 1 (𝑦)


|∆𝑥|𝑓 1 (𝑥)+|∆𝑦|𝑓 1 (𝑦)
Maximum possible relative error =
𝑓(𝑥,𝑦)

Example 25

Given that X and Y are rounded off to give x and y with error ∆x and ∆y respectively. Show that the
∆𝑥 ∆𝑦
maximum relative error recorded in x4y is given by 4 | | + | |
𝑥 𝑦

Solution

|𝑒| =|∆𝑥|𝑓 1 (𝑥) + |∆𝑦|𝑓 1 (𝑦) =|∆𝑥4𝑥 3 𝑦| + ∆𝑦𝑥 4 4|𝑥 3 𝑦||∆𝑥|+ |𝑥 4 ||∆𝑦|
R.E =
𝑥4𝑦
3
|𝑒| ≤ 4|𝑥 𝑦||∆𝑥| + |𝑥 4 ||∆𝑦| ∆𝑥 ∆𝑦
=4 | | + | |
𝑥 𝑦
|𝑒𝑚𝑎𝑥 | = 4|𝑥 3 𝑦||∆𝑥| + |𝑥 4 ||∆𝑦|
Example 26

Show that the maximum possible relative error in y𝑠𝑖𝑛2 𝑥 is


∆𝑦
| | + 2 cot 𝑥 |∆𝑥|, where∆𝑥 and ∆𝑦 are errors in x and y respectively
𝑦
𝜋 𝜋
Hence find the percentage error in calculating y𝑠𝑖𝑛2 𝑥 if y = 5.2 ± 0.05 and x = ± (07 marks)
6 360
z = ysin2x
𝑒𝑧 = ∆𝑦𝑠𝑖𝑛2 𝑥 + 2y∆𝑥𝑐𝑜𝑠 𝑥𝑠𝑖𝑛𝑥
𝑒𝑧 ∆𝑦𝑠𝑖𝑛2 𝑥 2y∆𝑥𝑐𝑜𝑠 𝑥𝑠𝑖𝑛𝑥
= +
𝑧 ysin2 x ysin2 x
𝑒𝑧 ∆𝑦
| |= | + 2𝑐𝑜𝑡𝑥. ∆𝑥|
𝑧 𝑦
∆𝑦
≤| | + 2𝑐𝑜𝑡𝑥. |∆𝑥|
𝑦
∆𝑦
∴ Maximum possible error is | | + 2𝑐𝑜𝑡𝑥. |∆𝑥|
𝑦
0.05 𝜋 𝜋
𝑝𝑒𝑟𝑐𝑒𝑛𝑡𝑎𝑔𝑒 𝑒𝑟𝑟𝑜𝑟 = [ + 2 cot . | |] 𝑥 100% = 3.9845%
5.2 6 360

Thank you
Dr. Bbosa Science
Flowcharts in mathematics
A flow chart is a diagram comprising of systematic steps followed in order to solve a problem.

Shapes used

1. Start/stop

START STOP START STOP

2. OPERATIONASSIGNMENT

N = N +1 N=N+1

This indicates that the new number is obtained by adding one to the previous N

READ, x0 READ, x0

3. Decision box

Note: all other shapes can be interchanged except for the decision box

Dry run or trace


This is the method of predicting the outcome of a given flow chart using a table
Example 1

Perform a dry run and state the purpose of the flowchart

Solution

Dry run

x y
0 1
1 1
2 2
3 6
4 24
5 120
6 720
Purpose is to compute and print 6!

Relationship is y = x!

Example 2
Study the flow chart below and perform dry run of the flowchart

START Solution

Dry run
S=0
N S Is N = 15?
N=1 1 1 NO
3 4 NO
5 9 NO
S=S+N 7 16 NO
N=N+2
9 25 NO
NO 11 36 NO
Is N = 15?
6? 13 49 NO
YES 15 64 YES
PRINT, S Purpose is to compute and print the first 8 square
numbers
STOP
Example 3

Perform a dry run and state the purpose of the flowchart

START Solution

Dry run
R =1
C R Is C = 8?
0 1 NO
C= 0
1 2 NO
2 4 NO
R=Rx2 3 8 NO
C=C+1 5 32 NO
NO 6 64 NO
Is C = 8?
7 128 NO
YES 8 256 YES
PRINT, C, R Purpose is to compute and print 28

STOP

Example 4

The flowchart below is used to read the root of the equation 2x3 + 5x – 8 = 0

START
Carry out a dry run of the flow chart and obtain
n=0 the root of 2x3 + 5x – 8 = 0 with an error less than
0.005

READ, x0 = 1.2 N 𝑋𝑛 𝑋𝑛+1 𝑋𝑛+1 − 𝑋𝑛


0 1.2 1.0933 0.1067
1 1.0933 1.0867 0.0066
3 +8
4𝑥𝑛
𝑋𝑛 = 𝑋𝑛+1
𝑥𝑛+1 = 2 1.0867 1.0866 0.001
6𝑥 2 +5

𝑛 =𝑛+1
Root is 1.087
Is
NO
𝑋𝑛+1 − 𝑋𝑛 < 0.005
< 0.005
YES
PRINT, Xn+1

STOP
Example 5

Study the flowchart below (i) Carry out a dry run of the flowchart, taking
N = 20, X0 = 4 and obtain the root of correct
START
to 3dp.
(ii) State its purpose
n=0
Solution
READ: N = X0 N 𝑋𝑛 𝑋𝑛+1 𝑋𝑛+1 − 𝑋𝑛
0 4.0 4.5 0.5
𝑋𝑛 = 𝑋𝑛+1 1 4.5 4.4722 0.0278
𝑋𝑛2 +𝑁
𝑥𝑛+1 = 2 4.4722 4.4721 0.0001
2(𝑋𝑛 )

𝑛 =𝑛+1
Root is 4.472
Is
NO
𝑋𝑛+1 − 𝑋𝑛 < 0.0005 (ii) to print the square root of a number N
< 0.005
YES
PRINT, Xn+1

STOP
Constructing flowcharts
1. Draw a flow chart that reads and prints 2. Draw a flowchart for computing and printing
the mean of the first ten counting the mean of the square roots of the first 20
numbers natural numbers

Solution

Let S be sum and m the mean

3. Draw a flowchart that computes and prints


4. Draw a flowchart that computes the root
the sum of the cubes of the first 30 natural of the equation ax2 + bx + c = 0
numbers Solution
BEGIN BEGIN

N=0 Read a, b, c

S=0
IS NO COMPLEX
2
b -4ac ≥ 0
ROOT
N= N+1 YES
−𝑏 ± √𝑏 2 − 4𝑎𝑐 STOP
N=N+1 𝑥=
S=S+N 3 2𝑎

NO PRINT, x1, x2
Is N =30?

YES STOP
PRINT, S

END
Newton Raphson’s method and Flowcharts

Example 6

(a) Show that the iterative formula based on Newton Raphson’s method for approximating the root
of the equation 2Inx – x + 1 = 0 is given by
2𝐼𝑛𝑥𝑛 −1
𝑥𝑛+1 = 𝑥𝑛 ( ), n = 0, 1,2 ……………………….. (03marks)
𝑥𝑛 −2
f(x) = 2Inx – x + 1
2 𝑥𝑛 (2− 𝑥𝑛 )− 𝑥𝑛 (2𝐼𝑛𝑥𝑛 − 𝑥𝑛 +1)
f’(x) = − 1 = (2− 𝑥𝑛 )
𝑥
also 𝑥𝑛 (2− 𝑥𝑛 − 2𝐼𝑛𝑥𝑛 + 𝑥𝑛 −1)
= (2− 𝑥𝑛 )
f(xn) = 2Inxn – xn + 1 𝑥𝑛 (1− 2𝐼𝑛𝑥𝑛 )
f’(xn)=
2
−1 = (2− 𝑥𝑛 )
𝑥𝑛
−𝑥𝑛 ( 2𝐼𝑛𝑥𝑛 −1)
Using 𝑥𝑛+1 = 𝑥𝑛 −
𝑓(𝑥𝑛 ) =
−( 𝑥𝑛 −2)
𝑓′(𝑥𝑛 )
𝑥𝑛 ( 2𝐼𝑛𝑥𝑛 −1)
By substitution, we get = ( 𝑥𝑛 −2)
2𝐼𝑛𝑥𝑛 − 𝑥𝑛 +1
𝑥𝑛+1 = 𝑥𝑛 − 2
( −1)
𝑥𝑛
2
𝑥𝑛 ( −1)− 2𝐼𝑛𝑥𝑛 − 𝑥𝑛 +1
𝑥𝑛
= 2
( −1)
𝑥𝑛
(b) Draw a flow chart that:

(i) reads the initial approximation x0 of the root

(ii) computes and prints the root correct to two decimal places, using the formula in (a)
(05marks)

(ii) Taking x0 = 3.4, perform a dry run to find


the root of the equation (04marks)

Dry run

n xn xn+1 𝑥𝑛+1 − 𝑥𝑛
0 3.4 3.51548 0.11548
1 3.51548 3.51286 0.00262
2 3.51286 3.51286 0.0000
Example 7
1
(a) Show that the Newton-Raphson formula for finding the root of the equation x = 𝑁 5 is given by
4𝑋𝑛5 +𝑁
𝑋𝑛+1 = , n = 0, 1, 2, … (04marks)
5𝑋𝑛4

1
𝑥 = 𝑁5
𝑥5 = 𝑁
𝑥5 − 𝑁 = 0
Let f(x) = 𝑥 5 − 𝑁
𝑓(𝑥𝑛 ) = 𝑥𝑛5 − 𝑁
𝑓′(𝑥𝑛 ) = 5𝑥𝑛4

Using
𝑓(𝑥𝑛 ) 5 −𝑁
𝑥𝑛 5 − 𝑥 5 −𝑁
5𝑥𝑛 𝑛
𝑥𝑛+1 = 𝑥𝑛 − = 𝑥𝑛 − 4 = 4
𝑓 ′(𝑥𝑛 ) 5𝑥𝑛 5𝑥𝑛

5 +𝑁
4𝑥𝑛
𝑥𝑛+1 = 4 , n = 0, 1, 2 …
5𝑥𝑛

(b) Construct a flow chart that


(i) reads N and the first approximation x0.
(ii) computes the root to three decimal places
(iii) Prints the root (xn) and the number of iteration (n) (05marks)

(c) Taking N = 50, x0 = 2.2, perform a dry


run for the flow chart. Give your root
correct to three decimal
places.(03marks)

N = 50, xo = 2.2

n xn xn+1 𝑥𝑛+1 − 𝑥𝑛
0 2.2 2.18688 0.01312
1 218688 2.18672 0.00016
Root = 2.187(3D)
Example 8

(a) Show that the iterative formula based on Newton Raphson’s method for solving the equation

Inx + x – 2 = 0 is given by
𝑥𝑛 (3−𝐼𝑛𝑥𝑛 )
𝑋𝑛+1 = , 𝑛 = 0, 1,2 …. (04marks)
1+ 𝑥𝑛

𝑙𝑒𝑡 𝑓(𝑥) = 𝐼𝑛𝑥 + 𝑥 −2


𝐼𝑛𝑥𝑛 + 𝑥𝑛 −2
𝑓(𝑥𝑛 ) = 𝐼𝑛𝑥𝑛 + 𝑥𝑛 − 2 =𝑥𝑛 − ( 1+ 𝑥𝑛 )
1 1+ 𝑥𝑛 𝑥𝑛
𝑓 ′ (𝑥) = +1=
𝑥𝑛 𝑥𝑛
𝑥𝑛 𝑥𝑛 (𝐼𝑛𝑥𝑛 + 𝑥𝑛 −2)
= −
1 1+ 𝑥𝑛
Using N.R.M
𝑥𝑛 (1+ 𝑥𝑛 −𝐼𝑛𝑥𝑛 − 𝑥𝑛 +2)
𝑥𝑛+1 = 𝑥𝑛 −
𝑓(𝑥𝑛 ) =
1+ 𝑥𝑛
𝑓′(𝑥𝑛 )
𝑥𝑛 (3−𝐼𝑛𝑥𝑛 )
(b)(i) Construct a flow chart that ; = ; n = 0, 1, 2,
1+ 𝑥𝑛

- reads the initial approximation as r ……………

- computes using the interactive formula in (a), and prints the root of equation Inx + x -2 = 0,
when the error is less than 1.0 x 10-4.

(ii) Perform a dry run of the flow chart when r = 1.6. (08marks)

n xn Xn+1 𝑥𝑛+1 − 𝑥𝑛
0 1.6 1.5569 0.0431
1 1.5569 1.5571 0.0002
2 1.5571 1.5571 0.0000
Hence the root = 1.557(3D)
Example 9

(a) Show that iterative formula based on Newton Raphson’s method for approximating the sixth
1 𝑁
root of a number N is given by 𝑥𝑛+1 = (5𝑥𝑛 + )
6 𝑥𝑛 5
(b) Draw a flowchart that
(i) Reads N and the initial approximation x0 of the root
(ii) computes and prints the root to three decimal places
(c) Taking N = 60, x0 = 1.9, perform a dry run for the flow chart, give your root correct to three
decimal places.

Solution
1 Using NRM
𝑥 = 𝑁6
𝑓(𝑥𝑛 ) 6 −𝑁
𝑥6 = 𝑁 𝑥𝑛+1 = 𝑥𝑛 − = 𝑥𝑛 −
𝑥𝑛
5
𝑓 ′(𝑥𝑛 ) 6𝑥𝑛
𝑥6 − 𝑁 = 0
Let f(x) = 𝑥 6 − 𝑁 5 )− (𝑥 6 −𝑁)
𝑥𝑛 (6𝑥𝑛 𝑛
= 5
𝑓(𝑥𝑛 ) = 𝑥𝑛6 − 𝑁 6𝑥𝑛

𝑓′(𝑥𝑛 ) = 6𝑥𝑛5 6 +𝑁
5𝑥𝑛
𝑥𝑛+1 = 5 , n = 0, 1, 2 …
6𝑥𝑛

(b) Flowchart (c) Dry run

n 𝒙𝒏 𝒙𝒏+𝟏 𝒙𝒏+𝟏
− 𝒙𝒏
0 1.9 1.9872 0.082
1 1.9872 1.9787 0.0085
2 1.9787 1.9786 0.0001
Example 10

(a) Show that the iterative formula based on Newton Raphson’s method for approximating the
3 𝑁
fourth root of a number N is given by 𝑥𝑛+1 = (𝑥𝑛 + ).
4 3𝑥𝑛 3
(b) Draw a flowchart that
(i) Records N and initial approximation x0 of the root
(ii) computes and prints the root after four iterations.
(c) Taking N = 39.0, x0 = 2.0, perform a dry run for the flowchart, give your root correct to three
decimal places

Solution
1 Using NRM
𝑥 = 𝑁4
𝑓(𝑥𝑛 ) 4 −𝑁
𝑥4 = 𝑁 𝑥𝑛+1 = 𝑥𝑛 − = 𝑥𝑛 −
𝑥𝑛
3
𝑓 ′(𝑥𝑛 ) 4𝑥𝑛
𝑥4 − 𝑁 = 0
Let f(x) = 𝑥 4 − 𝑁 3 )− (𝑥 3 −𝑁)
𝑥𝑛 (4𝑥𝑛 𝑛
= 3
𝑓(𝑥𝑛 ) = 𝑥𝑛4 − 𝑁 4𝑥𝑛

𝑓′(𝑥𝑛 ) = 4𝑥𝑛3 3 +𝑁
3𝑥𝑛 3 𝑁
𝑥𝑛+1 = = (𝑥𝑛 + ), n = 0, 1, 2 …
4
4𝑥𝑛 4 3𝑥𝑛 3

(b) Flowchart
(c) Dry run

n 𝒙𝒏 𝒙𝒏+𝟏 𝒙𝒏+𝟏 − 𝒙𝒏
0 2.0 2.7188 0.7188
1 2.7188 2.5242 0.1945
2 2.5242 2.4994 0.0249
3 2.4994 4.4990 0.0004
Example 11
(a) Show that the iterative formula based on Newton’s Raphson’s method for finding the natural
𝑒 𝑥𝑛 (𝑥𝑛 −1)+𝑁
logarithm of a number N is given by 𝑥𝑛+1 = , n = 0, 1, 2, …
𝑒 𝑥𝑛
(b) Draw a flowchart that
(i) Records N and initial approximation x0 of the root
(ii) computes and prints the natural logarithm after four iteration and gives natural logarithm
to 3 decimal places.
(c) Taking N = 10, x0 = 2, perform a dry run for the flowchart, give your root correct to three
decimal places
Solution
(a) x = InN; ex = N => ex – N = 0 𝑥𝑛 𝑒 𝑥 −(𝑒 𝑥𝑛 −𝑁)
=
f(x) = ex – N; f1(x) = ex 𝑒𝑥
𝑒 𝑥𝑛 −𝑁 𝑒 𝑥 (𝑥𝑛 −1)+𝑁
𝑥𝑛+1 = 𝑥𝑛 − ( ) =
𝑒𝑥 𝑒𝑥

(b) Flowchart

(c) Dry run

n 𝒙𝒏 𝒙𝒏+𝟏 𝒙𝒏+𝟏 − 𝒙𝒏
0 2.0 2.3533 0.3533
1 2.3533 2.3039 0.0494
2 2.3039 2.3026 0.0013
3 2.3026 2.3026 0.0000

Example 12

A shop offers a 25% discount on all items in their store and a second discount of 5% for paying cash.

(a) Construct a flowchart for the above information


(b) perform a dry run for (i) a shoe of 75,000/= cash and (ii) a shirt of 45,000/= credit
(a) Flowchart (c) dry run

MP D= Pay Cash Credit


0.75MP = 0.95D =D
75,000 56,250 cash 53437.50 --------
45,000 33,750 credit ----------- 33750

Example 13

Given that a man deposited 100,000/= to a bank which gives a compound interest of 5%. Draw a
flowchart to compute the amount of money accumulated after 5 years and perform a dry run for the
flowchart.

Flowchart Dry run

n P A
0 100,000 100,000
1 100,000 105,000
2 105,000 110,250
3 110,250 115,762.0
4 115,762,5 121,550.625
5 121,550.625 127,628.1563
Revision Exercise
1. (a) show that the iterative formula based on Newton Raphson’s method for approximating the
1 𝑁
cube root of a number N is given by 𝑥𝑛+1 = (2𝑥𝑛 + 2 ) , 𝑛 = 0, 1, 2, …
3 𝑥𝑛
(b) Draw a flowchart that
(i) reads N and the initial approximation x0 of the root
(ii) computes and prints the root to three decimal places.
(c) Taking N = 54, x0 = 3.7, perform a dry run for the flowchart, give your root to three decimal
places [3.780]
2. (a) show that the iterative formula based on Newton Raphson’s method for approximating the
3 𝑁
fourth root of a number N is given by 𝑥𝑛+1 = (𝑥𝑛 + 3 ) , 𝑛 = 0, 1, 2, …
4 3𝑥𝑛
(b) Draw a flowchart that
(i) reads N and the initial approximation x0 of the root
(ii) computes and prints the root to two decimal places.
(c) Taking N = 35, x0 = 2.3, perform a dry run for the flowchart, give your root to two decimal
places. [2.43]
3. (a) show that the iterative formula based on Newton Raphson’s method for finding the root of a
1 5 +𝑁
4𝑥𝑛
number 𝑁 5 is given by 𝑥𝑛+1 = ( ) , 𝑛 = 0, 1, 2, …
5𝑁𝑛4
(c) Draw a flowchart that
(i) reads N and the initial approximation x0 of the root
(ii) computes and prints the root to three decimal places.
(d) Taking N = 50, x0 = 2.2, perform a dry run for the flowchart, give your root to three decimal
places [2.187]
4. (a) show that the iterative formula based on Newton Raphson’s method for approximating the
2𝐼𝑛𝑥𝑛 −1
cube root of 2Inx – x + 1 = 0 is given by 𝑥𝑛+1 = 𝑥𝑛 ( ) , 𝑛 = 0, 1, 2, …
𝑥𝑛 −2
(b) Draw a flowchart that
(i) reads N and the initial approximation x0 of the root
(ii) computes and prints the root
(c) Taking x0 = 3.4, perform a dry run for the flowchart, give your root to three decimal places
5. (a) show that the iterative formula based on Newton Raphson’s method for approximating the
3−𝐼𝑛𝑥𝑛
cube root of Inx + x – 2 = is given by 𝑥𝑛+1 = 𝑥𝑛 ( ) , 𝑛 = 0, 1, 2, …
1+𝑥𝑛
(b) Draw a flowchart that
(iii) reads N and the initial approximation r of the root
(iv) computes and prints the root of the equation, when the error is less than 10. x 10-4.
(c) Taking r = 1.6, perform a dry run for the flowchart, give your root to three decimal places
6. (a) show that the iterative formula based on Newton Raphson’s method for approximating the
𝑒 𝑥𝑛 (𝑥𝑛 −1)+2
cube root of x = In(x +2) is given by 𝑥𝑛+1 = 𝑛 = 0, 1, 2, …
𝑒 𝑥𝑛 −1
(b) Draw a flowchart that
(i) reads the initial approximation x0 of the root
(ii) computes and prints the root to three decimal places
(c) Taking x0 = 1.2, perform a dry run for the flowchart, give your root to three decimal places
7. (a) show that the iterative formula based on Newton Raphson’s method for finding the natural
𝑒 𝑥𝑛 (𝑥𝑛 −1)+𝑁
logarithm of number N is given by 𝑥𝑛+1 = , 𝑛 = 0, 1, 2, …
𝑒 𝑥𝑛
(b) Draw a flowchart that
(i) reads N and the initial approximation x0 of the root
(ii) computes and prints the root to two decimal places.
(c) Taking N = 45, x0 = 3.5, perform a dry run for the flowchart, give your root to two decimal
places [3.81]
8. (a) show that the iterative formula based on Newton Raphson’s method for finding the root of
3+ 8
4𝑥𝑛
the 2x3 + 5x -8 is given by 𝑥𝑛+1 = ( 2 +5 ) , 𝑛 = 0, 1, 2, …
6𝑥𝑛
(b) Draw a flowchart that
(i) reads N and the initial approximation 𝛼 of the root
(ii) computes and prints the root when the error is less than 0.001.
(c) Taking 𝛼= 1.1, perform a dry run for the flowchart, give your root to three decimal places
[1.087]
9. A shop offers a 25% discount on all items in their store and a second discount of 5% for paying
cash.
(a) Construct a flowchart for the above information
(b) perform a dry run for (i) a radio of 125,000/= cash and (ii) a T.V of 340,000/= credit
[89.062.50, 255,0000]
10. Given that a man deposited 120,000/= to a bank which gives a compound interest of 15%. Draw
a flowchart to compute the amount of money accumulated after 4 years and perform a dry run
for the flowchart. [209,880.75/=]

Thank you
Dr. Bbosa Science
Resultant of forces
A force is anything which change a body’s state of rest or uniform motion in a straight line. Examples
are weight, tension, reaction, friction, resistance force.

Resultant of two forces

Consider two forces A and B inclined to each other at an angle θ

(i) θ is right angle (θ =900)


Resultant, R = √𝐴2 + 𝐵2
𝐴
Direction of resultant, β =tan−1 ( )
𝐵

(ii) θ is acute (00 ≤ θ ≤ 900)


sin 𝛼 sin(180−𝜃)
Direction of resultant, =
𝐴 𝑅

Asin(180−𝜃)
𝛼 = sin−1 ( )
𝑅

Resultant, R =√[𝐴2 + 𝐵2 − 2𝐴𝐵𝑐𝑜𝑠 (180 − 𝜃)]

(iii) θ is obtuse (900 ≤ θ ≤ 1800)

sin 𝛼 sin(180−𝜃)
Direction of resultant, =
𝐴 𝑅

Asin(180−𝜃)
𝛼 = sin−1 ( )
𝑅

Resultant, R =√[𝐴2 + 𝐵2 − 2𝐴𝐵𝑐𝑜𝑠 (180 − 𝜃)]


Example 1

Two forces of magnitude 5N and 12N act on a particle with their direction inclined at 900. Find the
magnitude and direction of the resultant
5
𝑅 = √52 + 122 = 13N α =tan−1 ( ) = 22.60
12

The resultant = 13N at 22.60 to 12N force

Example 2

Forces of magnitude 7N and 9N act on a particle at an angle of 600 between them. Find the
magnitude and direction of the resultant.

sin 𝛼 sin(180−𝜃)
Direction of resultant, =
9 13.89

9sin(180−60)
𝛼 = sin−1 ( ) = 34.130
13.89

Resultant, R =√[𝐴2 + 𝐵2 − 2𝐴𝐵𝑐𝑜𝑠 (180 − 𝜃)]

=√[72 + 92 − 2𝑥7 𝑥 9𝑐𝑜𝑠 (180 − 60)]

= 13.89N

Example 3

Find the angle between a force of 7N and 4N their resultant has a magnitude of 9N

Solution
2
𝛼 = cos −1 (− ) = 106.60
7

the angle θ between the forces = 180 – 106.6

= 73.40

92 = 72 + 42 – 2 x 7 x 4 x cosα

Example 4

Forces of 3N and 2N act on a particle at an angle of 1500 between them. Find the magnitude and
direction of the resultant.

sin 𝛼 sin(30)
Direction of resultant, =
2 1.61

2sin(180−60)
𝛼 = sin−1 ( ) = 38.30
1.61

R2 = 22 + 32 – 2 x 2 x 2 x cos(30)

R = 1.61N
Revision exercise

1. Two forces of magnitude 7N and 24N act on a particle with their direction at 900. Find the
magnitude and direction of the resultant. [25N, 16.260 with 24N force]
2. Forces of 5N and 8N act on a particle at an angle of 500 between them. Find the magnitude and
direction of the resultant. [11.9N at 190 with 8N force]
3. Forces of 4N and 6N act on a particle at angle 600 between them. Find the magnitude and the
direction of the resultant. [5.29N, at 40.90 with 6N force]
4. Forces of 9N and 10N act on a particle at angle 400 between them. Find the magnitude and the
direction of the resultant. [17.9N, at 18.90 with 10N force]
5. Forces of 12N and 10N act on a particle at angle 1050 between them. Find the magnitude and
the direction of the resultant. [13.5N, at 45.70 with 12N force]
6. Forces of 8N and 3N act on a particle at angle 1600 between them. Find the magnitude and the
direction of the resultant. [5.28N, at 11.20 with 8N force]
7. Find the angle between a force of 10N and 4N their resultant has a magnitude of 8N. [130.50]
8. The angle between a force αN and a force of 3N is 1200. If the resultant of the two forces has
magnitude 7N, find the value of α. [8N]
9. The angle between a force βN and a force of 8N is 450. If the resultant of the two forces has a
magnitude 15N, find the value of β. [8.24N]

Thank you
Dr. Bbosa Science
Resolutions of forces acting on a polygon
For any regular polygon

- all sides are equal


- all angles are equal
360
- an exterior angle = where n is the number of sides
𝑛

Example 1

ABCD is a rectangle with AB= 4cm and BC = 3cm. Forces of magnitude 2N, 1N, 5N,6N and 7N act
along AB, BC, CD, AD and AC respectively. In each case the direction of the force being given by the
order of the letters. Given that AB is horizontal determine

(i) the magnitude of the resultant force (ii) direction of the resultant with AB

2.6
Direction,𝛼 = tan−1 ( )
11.2

3 =13.0690
𝜃 = tan−1 ( ) = 36.87
4
Direction is 13.0690 above AB
2 0 −5 0 7𝑐𝑜𝑠36.87 2.6
R=( )+( )+( )+( )+( )=( )
0 1 0 6 7𝑠𝑖𝑛36.87 11.2
𝑅 = √2.62 + 11.22 = 11.498N

Example 2

ABC is an equilateral triangle. Forces of magnitude 12N, 10N and 10N act along AB, BC and CA
respectively, in each case the direction of the force being given by the order of the letters. Given
that AB is horizontal determine

(i) the magnitude of the resultant force


12 −10𝑐𝑜𝑠60 −10𝑐𝑜𝑠60 2
𝑅=( )+( )+( ) =( )
0 10𝑠𝑖𝑛60 −10𝑠𝑖𝑛60 0
𝑅 = √22 + 02 = 2N

(ii) Direction of the resultant with AB

0
Direction 𝛼 = tan−1 ( ) = 02
2

Example 3

ABCDEF is a regular hexagon. Force of magnitude 2N, 5N, 3N, 4N, 3N and 1N act along the line AB,
BC, CD, DE, EF and FA respectively, in each case the direction of the force being given by the order of
the letters. Given that AB is horizontal, determine

(i) the magnitude of the resultant force and

2 5𝑐𝑜𝑠60 −3𝑐𝑜𝑠60 −4 −3𝑐𝑜𝑠60 1 cos 60 −2


𝑅 = ( )+( )+( )+( )+( )+ ( )=( )
0 5𝑠𝑖𝑛60 3𝑠𝑖𝑛60 0 −3𝑠𝑖𝑛60 −1𝑠𝑖𝑛60 3.4641

𝑅 = √(−2)2 + 3.46412 = 4𝑁

(ii) direction of the resultant with AB.

3.461
𝛼 = tan−1 ( ) = 600 to AB
2

Example 4

ABCDEF is a regular hexagon. Forces of magnitude 3N, 4N, 2N and 6N act along the line AB, AC, EA
and AF respectively, in each case the direction of the force being given by the order of the letters.
Given that AB is horizontal, determine

(i) the magnitude of the resultant force


3 4𝑐𝑜𝑠30 −6𝑐𝑜𝑠60 0 2√3
𝑅 =( )+( )+( )+( )=( )
0 4𝑠𝑖𝑛30 6𝑠𝑖𝑛60 −2 3√3

2 2
𝑅 = √(2√3) + (3√3) =6.245N

(ii) direction of the resultant force

3√3
𝜃 = tan−1 ( )=56.30
2√3

1. ABCD is a square. Forces of magnitude 6N, 4N and 2√2𝑁 act along AD, AB and AC
respectively in each case the direction of force being the order of the letters. Given that AB
is horizontal, determine the magnitude and direction of the resultant force.
[10N at 53.10 with AB]
2. ABCD is a square. Forces of magnitude 2N, 1N, √2𝑁 and 4N act along AB, BC and AC and DA
respectively in each case the direction of force being the order of the letters. Given that AB
is horizontal, determine the magnitude and direction of the resultant force.
[5.13N at 33.70 with AB]
3. ABCD is a square. Three forces of magnitude 4N, 10N and 7𝑁 act along AB, AD and CA
respectively in each case the direction of force being the order of the letters. Given that AB
is horizontal, determine the magnitude [5.1388N]
4. In equilateral triangle PQR, three forces of magnitude 5N, 10N and 8N act along the side PQ,
QR and PR respectively. Their direction are the order the letters. Find the magnitude of the
resultant force. [16.1N]
5. ABCD is a square. Forces of magnitude 6√3𝑁 N, 2N and 4√3𝑁 act along AB, CB and CD
respectively in each case the direction of force being the order of the letters. Given that AB
is horizontal, determine the magnitude and direction of the resultant force.
[4N at 300 to AB]
6. ABCD is a rectangle with AB= 4cm and BC = 3cm. Forces of magnitude 3N, 1N, and 10N act
along AB, DC and AC respectively. In each case the direction of the force being given by the
order of the letters. Given that AB is horizontal determine the magnitude and direction of
the resultant force. [13.4N at 26.60 with AB]
7. ABCD is a rectangle. Forces of magnitude 8N, 4N, 10N and 2N act along AB, CB, CD and AD
respectively. In each case the direction of the force being given by the order of the letters.
Given that AB is horizontal determine the magnitude and direction of the resultant force.
[283N at 450 at AB]
8. In equilateral triangle ABC, forces of magnitude 10N each act along the side AB, BC and AC
respectively. Their direction are the order the letters. Find the magnitude of the resultant
force and the angle it makes with AB. [20N at 600to AB]
9. In equilateral triangle ABC, forces of magnitude 5N, 9N and 7N act along the side AB, BC and
CA respectively. Their direction are the order the letters. Find the magnitude of the resultant
force and the angle it makes with AB. [2√3N at 300to AB]
10. In equilateral triangle ABC, forces of magnitude 4N, 4N and 6N act along the side AB, BC and
AC respectively. Their direction are the order the letters. Find the magnitude of the resultant
force and the angle it makes with AB. [10N at 600 to AB]
11. ABCDEF is a regular hexagon. Forces of magnitude 2N, 5N, 3N, 4N, 3N and 1N act along the
line AB, BC, CD,DE, EF and AF respectively, in each case the direction of the force being given
by the order of the letters. Given that AB is horizontal, determine the magnitude and
direction of the resultant force. [6N at 600 to AB]
12. ABCDEF is a regular hexagon. Forces of magnitude 8N, 7N, 6N, 4N, 7N, and 6N act along the
line AB, BC, CD, DE, EF and FA respectively, in each case the direction of the force being
given by the order of the letters. Given that AB is horizontal, determine the magnitude and
direction of the resultant force. [12.5N at 760 to AB]
13. PQRSTU is a regular hexagon. Forces of magnitude 4N, 5N, 2N, and 6N act along the line
PQ, PR, PT and PU respectively, in each case the direction of the force being given by the
order of the letters. Given that PQ is horizontal, determine the magnitude and direction of
the resultant force. [11.065N at 61.20 to PQ]
14. ABCD is a square. Forces of magnitude 10𝑁 N, 9N, 8N and 5𝑁 act along AB, BC, CD and AD
respectively in each case the direction of force being the order of the letters. Given that AB
is horizontal, determine the magnitude and direction of the resultant force.
[2√5N at 63.430 to AB]
15. ABCD is a rectangle with AB= 4cm and BC = 3cm. Forces of magnitude 3N, 10N, 4N, 6N and
5N act along AB, BC, CD, DA, and AC respectively. In each case the direction of the force
being given by the order of the letters. Given that AB is horizontal determine the magnitude
and direction of the resultant force. [7.62N at 66.80 with AB]

Thank you
Dr. Bbosa Science
Moment of a force acting on a polygon
Example 1

ABCD is a square of side 4m. Forces of magnitude 4N, 3N, 2N and 5N act along AB, BC, CD and AD
respectively in each case the direction of force being given by the order of the letters. Given that AB
is horizontal, find the moment of force about

(i) center of the square


(ii) point A

Solution

O G = 4 x 2 + 3 x 2+ 2 x 2 – (5 x 2)

=8Nm anticlockwise

A G = (4 x 0) + (3 x 4) + (2 x 2) – (5 x 0)

= 20Nm anticlockwise

Example 2

ABCD is a rectangle where AB = 6m and BC = 4m. Forces of magnitude 3N, 4N, 5N, 2N and 6N act
along line AB, BC, CD, AD, and BD respectively. In each case the direction of force being given by
the order of the letters. Find the moment of forces about

(i) center of the rectangle (ii) point A

Solution O G = 3x3 + 4 x 3+ 5 x 2 – (3 x 2) + (6sin33.70 x 3) -

(6sin33.70 x 3) = 22Nm anticlockwise

A G = (3 x 0) + (4 x 6) + (5 x 4) – (2 x 0) +

(6sin33.70 x 0) - (6cos33.70 x 0)

= 63.97Nm anticlockwise
Example 3

ABCD is a rectangle where AB = 4m and BC = 3m. Forces of magnitude 4N, 3N, 5N, 6N and 5N act
along line AB, BC, CD, DA, and Ac respectively. In each case the direction of force being given by
the order of the letters. Find the moment of forces about

(ii) center of the rectangle (ii) point A

O G = 4 x 1.5 + 3 x 2 + 5 x 1.5 + (6 x 2) -

(5sin36.90 x 2) + (5sin36.90 x 1.5)

= 31.49Nm anticlockwise

A G = (3 x 4) + (5 x 3)

=27Nm anticlockwise

Example 4

ABCDEF is a regular hexagon of side 5m. Forces of magnitude 2N, 5N, 3N, 4N, 3N and 4N act
along the lines AB, BC, CD, DE, EF and FA respectively. In each case the direction of force being
given by the order of the letters. Given that AB is horizontal, find the sum of moments of forces
about

(i) center O of the hexagon

OM = √52 − (2.5)2 = 4.33m

O G = (2 + 5 + 3 + 4 + 3 + 4) x OM = 21 x 4.33 = 90.93Nm anticlockwise

(ii) Point A
A G = (5sin600 x 5) + (3sin600 x 10) + (4 x 10sin600) + (3sin600 x 5)

= 95.26 Nm anticlockwise
Example 5

Forces of magnitude 3N, 4N, 5N and 6N act on a rectangle along lines AB, BC, CD and DA of a
rectangle. Their direction is the order of the letters. BC is horizontal. Find the resultant force and the
couple at the centre of a rectangle of sides 2m and 4m.

Solution

→, X = 4 - 6 = -2

↑, Y = 5 – 3 = 2

The resultant force, R = √(−2)2 + 22

=2.83N

For a couple

O G = 4 x 1+ 5 x 2 +6 x 1 + 3 x 2

= 26Nm anticlockwise

For the system of forces to reduce to couple, an additional force equal in magnitude to the resultant
but acting in opposite direction must be added. Hence the moment of the couple about the center is
26Nm in the sense BADC or clockwise.
Revision exercise
1. Forces of 2N, 3N, 4N and 5N act along the sides of a square ABCD of side 4m in direction AB,
BC, CD and AD respectively. Find the sum of moments of the forces about
(i) the center of square [8Nm]
(ii) Point A [28Nm]
2. Forces of 5N, 6N, 4N, 7N, 6N and 8N act in direction AB, BC, CD, DA, AC, and DB respectively
of a square ABCD of side 6m. Find the sum of moments of forces about
(i) centre of the square [66Nm]
(ii) point A [26Nm]
3. ABCD is a rectangle with AB = 8cm and BC = 6 cm. Forces of 4N, 5N, 3N, 6N and 8N act in the
direction AB, BC, CD, AD and BD respectively of the rectangle ABCD, find the sum of
moments of the forces about
(i) the center of the rectangle [17Nm]
(ii) point A [96.4Nm]
4. ABCDEF is a regular hexagon 2m. Forces of magnitude 5N, 2N, 6N, 4N, 8N and 3N acting
along the line AB, BC, CD, DE, EF and FA respectively, in each case the direction of the force
being given by the order of the letters. Given that AB is horizontal. Find the sum of moments
of the force about point A. [50.23Nm]
5. ABCDEF is a regular hexagon with side 3m. Forces of magnitude 4N, 5N, 1N, 3N, 7N and 2N
act along line AB, BC, CD, DE, EF and FA respectively, in each case the direction of the force
being given by the order of the letters. Find the sum of moments of the forces about point A
[30√3Nm]
6. ABDCEF is a regular hexagon of 4m. Forces of magnitude 8N, 4N, 7N, 4N, 6N and 5N act
along AB, BC, CD, DE, EF and FA respectively, in each case the direction of the force being
given by the order of the letters. Given that AB is horizontal, find the sum of moments of the
forces about point A [64√3Nm]
7. ABDCEF is a regular hexagon of 4m. Forces of magnitude 5N, 6N, 7N, 4N, 5N and 8N act
along AB, BC, CD, DE, EF and FA respectively, in each case the direction of the force being
given by the order of the letters. Given that AB is horizontal, find the sum of moments of the
forces about
(i) centre of hexagon [70√3Nm]
(ii) point A [66√3Nm]
8. ABDCEF is a regular hexagon of 3m. Forces of magnitude 3N, 1N, 2N, 5N, 6N and 4N act
along AB, BC, CD, ED, EF and AF respectively, in each case the direction of the force being
given by the order of the letters. Given that AB is horizontal, find the sum of moments of the
forces about
(iii) centre of hexagon [4.5√3Nm]
(iv) point A

Thank You

Dr. Bbosa Science


Motion in straight line
Distance and displacement
Distance is a length between 2 fixe points

Displacement is the distance covered in a specific direction

Speed and velocity


Speed is the rate of change of distance with time

Velocity is the rate of change of displacement with time


𝑡𝑜𝑡𝑎𝑙 𝑑𝑖𝑠𝑡𝑎𝑛𝑐𝑒
Average speed =
𝑡𝑜𝑡𝑎𝑙 𝑡𝑖𝑚𝑒 𝑡𝑎𝑘𝑒𝑛

𝑡𝑜𝑡𝑎𝑙 𝑑𝑖𝑠𝑝𝑙𝑎𝑐𝑒𝑚𝑒𝑛𝑡
Average velocity =
𝑡𝑜𝑡𝑎𝑙 𝑡𝑖𝑚𝑒 𝑡𝑎𝑘𝑒𝑛

Example 1

Find the distance travelled in 5s by a body moving with a constant speed of 3.2ms-1

Solution
𝑡𝑜𝑡𝑎𝑙 𝑑𝑖𝑠𝑡𝑎𝑛𝑐𝑒 distance = 16m
𝑡𝑜𝑡𝑎𝑙 𝑑𝑖𝑠𝑡𝑎𝑛𝑐𝑒 3.2 =
Average speed = 5
𝑡𝑜𝑡𝑎𝑙 𝑡𝑖𝑚𝑒 𝑡𝑎𝑘𝑒𝑛

Example 2

John ran 1500m in 3minutes and 33s, find his average speed.
𝑡𝑜𝑡𝑎𝑙 𝑑𝑖𝑠𝑡𝑎𝑛𝑐𝑒 1500
Average speed = speed = = 7.04ms-1
𝑡𝑜𝑡𝑎𝑙 𝑡𝑖𝑚𝑒 𝑡𝑎𝑘𝑒𝑛 (3 𝑥 60+33)

Acceleration
It is the rate of change of velocity
𝑐ℎ𝑎𝑛𝑔𝑒 𝑖𝑛 𝑣𝑒𝑙𝑜𝑐𝑖𝑡𝑦 𝑣−𝑢
Acceleration = a= where v = final velocity, u = initial velocity, t = time
𝑡𝑜𝑡𝑎𝑙 𝑡𝑖𝑚𝑒 𝑡𝑎𝑘𝑒𝑛 𝑡

Uniform acceleration

This is the constant rate of change of velocity with time

digitalteachers.co.ug
Equations of uniform acceleration
1st equation

Suppose a body moving in a straight line with uniform acceleration a, increases its velocity from u to
v in time t, then from the definition of acceleration
𝑣−𝑢 at = v -u v = u + at ………..1
a=
𝑡

2nd equation

Suppose an object with velocity u moves with uniform acceleration a time t and attains a velocity v,
the distance s travelled by the object is given by: s = average velocity x time
𝑣+𝑢 2𝑢𝑡+𝑎𝑡 2
s=( ) 𝑡 but v = u + at s=( )
2 2

𝑢+𝑢+𝑎𝑡 𝟏
s=( )𝑡 s = ut + 𝒂𝒕𝟐 …………2
2 𝟐

3rd equation

s = average velocity x time


𝑣+𝑢 𝑣−𝑢 𝑣 2 − 𝑢2
s=( ) 𝑡 but t = s=( )
2 𝑎 2𝑎

𝑣+𝑢 𝑣−𝑢 𝒗𝟐 = 𝒖𝟐 − 𝟐𝒂𝒔 …………3


s=( )( )=
2 𝑎

Example 3

A car is initially at rest at a point O. The car moves from O in a straight line with an acceleration of
4ms-2. find how far the car

(i) is from O after 2s


𝟏 1
From s = ut + 𝒂𝒕𝟐 ; s = 0 x 2 + 𝑥 4 𝑥 22 = 8m
𝟐 2

(ii) is from O after 3s


1
s = 0 x 2 + 𝑥 4 𝑥 22 = 18m
2

(iii) distance travelled in the third second = 18 – 8 = 10m

Example 4

A body at O moving with a velocity 10ms-2 decelerates at 2ms-2.

(a) find the displacement of the body from O after 7s


1
From s = ut + 𝑎𝑡 2
2
1
s = 10 x 7 + 𝑥 − 2 𝑥 72 = 21m
2
(b) how far from O does the body come to rest and how long does it take
𝑣 2 − 𝑢2 02 − 102
s=( )= = 25m
2𝑎 2 𝑥−2
𝑣−𝑢 0− 10
t= = = 5s
𝑎 −2

digitalteachers.co.ug
Example 5

A taxi approaching a stage runs two successive half kilometres in 16s and 20s respectively. Assuming
the retardation is uniform, find

(i) Initial speed of the taxi


1
s = ut + 𝑎𝑡 2
2
For the first half kilometre or 500m
1
500 =16u + 𝑎 (16)2 ……… (i)
2
for the kilometre or 1000m
1
1000 = 36u + 𝑎 (36)2 …….. (ii)
2
from eqn. (i) and eqn. (ii)
25
a= and u = 34.028ms-1
72

(ii) the further distance, the taxi runs before stopping


𝑣 2 − 𝑢2 02 − (34.028)2
s=( )= s = ( 25 ) = 1667.3m
2𝑎 2( )
72
Extra distance = 1667.3 – 1000 = 667.3m

Example 6

An overloaded taxi travelling at constant velocity of 90km/h overtakes a stationary traffic police car.
2s later, the police car sets in pursuit, accelerating at a uniform rate of 6ms-2. How far does the
traffic car travel before catching up with the taxi?

Solution
For the car to catch taxi; sT = sC
t1 = time taken by the taxi
25t1 =3 𝑡2 2
t2 = time taken by the police car

t1 = 2 + t2 25(2 + t2) =3 𝑡2 2
4
speed of the taxi in m/s t = 10s or t = s
3
90 𝑥 1000 the car leaves 2s later then 10s is the correct time
90km/h = = 25ms-1
3600
since it gives positive distance
1
s = ut + 𝑎𝑡 2
2 sC = 3 𝑡2 2 =3 𝑥 102 = 300m
sT = 25t1
1
sC = 0 x t2 + 𝑥 6 𝑥 𝑡2 2 = 3 𝑡2 2
2

Example 7

A lorry starts from a point A and moves along a straight horizontal road with a constant acceleration
of 2ms-2. At the same time a car moving with a speed of 20ms-1 and a constant acceleration of 3ms-1
is 400m behind the point A and moving in the same direction as the lorry. find:

(a) how far from A the car overtakes the lorry.


a car over takes the lorry; both move in the same time, t

digitalteachers.co.ug
1
s = ut + 𝑎𝑡 2
2

distance moved by the car = 400 + distance moved by the lorry


1 1
20t + 𝑥 3𝑥 𝑡 2 = 400 + 𝑥 2 𝑥 𝑡2
2 2

t2 + 40t – 800 = 0; t = 14.64s or t = -54.64s

Hence t = 14.64s
1
sL = 𝑥 2 𝑥 (14.64)2 = 214.33m
2

(b) the speed of the lorry when it is being overtaken

v = u + at

= 0 + 2 x 14.64 = 29.28ms-1

Example 8

The seed of a taxi decreases from 90kmh-1 to 18kmh-1 in a distance of 120 metres. Find the speed of
the taxi when it had covered a distance of 50metres. (05marks)

Given u = 90kmh-1, v = 18kmh-1, s = 120m = 0.12km


Using v2 = u2 + 2as
182 = 902 + 2a(0.12)
a = -32400kmh-2
When s = 50m = 0.05km, u = 90kmh-1, a = -32400kmh-2
Using v2 = u2 + 2as
v2 = 902 – 2 x 32400 x 0.05 = 4860
v = √4860 = 69.71𝑘𝑚ℎ−1

Example 9

(a) Show that the final velocity v of a body which starts with an initial velocity u and moves with
1
uniform acceleration a consequently covering a distance x, is given by v = [𝑢2 + 2𝑎𝑥 ]2

x = average velocity x time


𝒗𝟐 = 𝒖𝟐 + 𝟐𝒂𝒙
𝑣+𝑢 𝑣−𝑢
x=( ) 𝑡 but t = 1
2 𝑎
v = [𝑢2 + 2𝑎𝑥 ]2
𝑣+𝑢 𝑣−𝑢 𝑣 2 − 𝑢2
x=( )( )=( )
2 𝑎 2𝑎

(b) Find the value of x in (a) if v = 300m/s, u = 10m/s and a = 5m/s


1
30 = [102 + 2𝑥 5𝑥 ]2
900 = 100 + 10x
x =80m

digitalteachers.co.ug
Velocity-time graphs
Example 10

A car started from rest and attained a velocity of 20m/s in 40s. It then maintained the velocity
attained for 50s. After that it was brought to rest by a constant breaking force in 20s.

(i) Draw a velocity-time graph for the motion

(ii) using the graph, find the total distance travelled by the car
Total distance = total area under the graph
1 1
= 𝑏ℎ + 𝑙𝑤 + 𝑏ℎ
2 2
1 1
= 𝑥 40 𝑥 20 + 50𝑥 20 + 𝑥 20 𝑥 20 = 1600m
2 2

Method II (area of a trapezium)


1 1
A = ℎ(𝑎 + 𝑏) = 𝑥 20(50 + 110) = 1600m
2 2

(iii) what is the acceleration of the car?


𝑣−𝑢 20−0
a= = = 0.5𝑚𝑠 −2
𝑡 40

Example 11

A car from rest accelerates steadily to 10s up to a velocity f 20ms. It continues with uniform velocity
for further 20s and then decelerates so that it stops in 20s.

(a) Draw a velocity-time graph to represent the motion

(i) Distance = area under the graph


1 1
A= 𝑥 10 𝑥 20 + 20𝑥 20 + 𝑥 20 𝑥 20
2 2

= 700m

Method II (area of a trapezium)

(b) Calculate 1
A= 𝑥 20(50 + 20)= 700m
2
(i) acceleration
𝑣−𝑢 20−0 𝑑𝑖𝑠𝑡𝑎𝑛𝑐𝑒
a= = = 2𝑚𝑠 −2 Average speed =
𝑡𝑖𝑚𝑒
𝑡 10
(ii) deceleration 700
𝑣−𝑢 0−20 = = 14𝑚/𝑠
a= = = −1𝑚𝑠 −2 50
𝑡 20

digitalteachers.co.ug
Example 12

The graph below shows the motion in the body.

(a) Describe the motion of the body


A body with initial velocity of 15m/s accelerates steadily to a velocity of 20m/s in 4s, it then
continues with a uniform velocity for 6s and brought to rest in 2s.
(b) Calculate the total distance travelled
1 1
Distance = 4 x 15 + 𝑥 4 𝑥 5 + 20 𝑥 6 + 𝑥 20 𝑥 2 = 210m
2 2

Revision exercise
1. P, Q and R are points on a straight road such that PQ = 20m and QR = 55m. A cyclist moving with
uniform acceleration passes O and then notices that it takes him 10s and 15s to travel between
2
P and Q and Q and R respectively. find the acceleration [ a = 𝑚𝑠 −2 ]
15
2. A car travels from Kampala to Jinja and back. It takes average speed on the return journey is
4km/h greater than that on the outward journey and it takes 12 minutes less. Given that
Kampala and Jinja are 80km apart, find the average speed on the outward journey.[30.05kmh]
3. Car A traveling at 35ms-1 along a straight horizontal road, accelerates uniformly at 0, 4ms-2. At
the same time, another car B moving at 44ms-1 and accelerating uniformly at 0.5ms-2 is 200m
behind A
(i) Find the time taken before car B over takes car A. [20s]
(ii) speed with which B over takes A. [55m/s]
4. A car is being driven along a road at 72kmh-1 notices a fallen tree on the road 800m ahead and
suddenly reduces the speed to 36kh-1 by applying brakes. For how long were the brakes applied
[53.33s]
5. A train starts from station a with a uniform acceleration of 0.2ms-2 for 2 minutes and attains a
maximum speed and moves uniformly for 15 minutes. it is then brought to rest at constant
retardation of 5/3ms-2 at station B. find the distance between A and B. [23212.8m]
6. A motorcycle decelerated uniformly from 20kmh-1 to 8kmh-1 in travelling 896m. find the rate of
deceleration in ms2 [0.0145ms-2]
7. A body moves with a uniform acceleration and covers a distance of 27m in 3s; it then moves
with a uniform velocity and covers a distance of 60m in 5s. Find the initial velocity and
acceleration of the body. [6ms-1, 2ms-2]
8. A particle is projected away from an origin O with initial velocity of 0.25ms-1. The particle travels
in a straight line and accelerates at 1.5ms-2. find
(i) how far the particle is from O after 4s [7.5m]
(ii) the distance travelled by the particle during the fourth second after projection. [5.5m]
9. A taxi which is moving with a uniform acceleration is observed to take 20s and 30s to travel
successive 400m. find
68
(i) initial speed of the taxi. [ 𝑚𝑠 −1 ]
3

digitalteachers.co.ug
(ii) the further distance it covers before stopping [163.3m]
10. Two cyclist A and B are 36m apart on a straight road. Cyclist B starts from rest with an
acceleration of 6ms-2 while A is in pursuit of B with velocity of 20ms-1 and acceleration of 4ms-1.
Find the time taken when A overtakes B [13466s]

Thank You

Dr. Bbosa Science

digitalteachers.co.ug

You might also like